You are on page 1of 1286

Define anisocytosis.

varying cell sizes

Define poikilocytosis varying cell shapes

From which cells do B cells


stem cells in bone marrow
arise?

From which cells do plasma


B cells
cells differentiate?
Microglia are not discernable
How can a Nissl stain be used in a Nissl stain while
to differentiate microglia oligodendroglia appear as
from oligodendroglia? small dark nuclei with dark
chromatin

In what type of CNS tissue


(white or grey) are white matter
oligodendroglia predominant?

Into what cell type does a


monocyte differentiate in Macrophages
tissues?

Name 2 substances produced


histiminase and arylsulfatase
by an eosinophil.
Name the three types of basophils, eosinophils, and
leukocytic granulocytes. neutrophils

Name the two types of


lymphocytes and monocytes
mononuclear leukocytes.

- cellular immune response -


What are 2 functions of T cell
regulation of B lymphocytes
lymphocytes?
and macrophages

What are 2 morphological - small irregular nuclei - and


features of microglia? relatively little cytoplasm
- follicles of lymph nodes -
What are 3 examples of white pulp of spleen -
peripheral lymphoid tissue? unencapsulated lymphoid
tissue

- pagocytosis of bacteria,
cell debris, and senescent red
What are 3 functions of a cells - scavenges damaged
macrophage? cells and tissues - can
function as an antigen
presenting cell

- Large - Kidney-shaped
What are 3 morphological
nucleus - Extensive 'frosted
characteristics of monocytes?
glass' cytoplasm

- Off center nucleus - Clock


What are 4 characteristics of face chromatin distribution -
the plasma cell morphology? Abundant RER - Well
developed Golgi apparatus
What are 4 morphologic - Round - Small - Densely
characteristics of staining nucleus - Small
lymphocytes? amount of pale cytoplasm

What are 4 substances - hydrolytic enzymes -


contained within the lysozyme - myeloperoxidase
lysosomes of neutrophils? - lactoferrin

- cytotoxic T cells (MHC I,


What are 4 types of cells into CD8) - helper T cells (MHCII,
which T cells differentiate? CD4) - suppressor T cells -
delayed hypersensitivity T
cells

Neoplastic, Asthma, Allergic


What are the 5 important
process, Collagen vascular
causes for eosinophilia in
disease, and Parasites
humans?
(pneumonic NAACP)
What are the blood cell
diffenentiation names of the - Pluripotent hematopoietic
ACTIVE T CELL line beginning stem cell - Lymphoblast - T
with the pluripotent cell - Active T cell
hematopoietic stem cell? (4)

What are the blood cell


differentiation names of the - Pluripotent hematopoietic
ERYTHROCYTE cell line stem cell - Proerythroblast -
beginning with pluripotent Reticulocyte - Erythrocyte
hematopoietic stem cell? (4)

What are the blood cell


differentiation names of the
- Pluripotent hematopoietic
MONOCYTE cell lines
stem cell - Mono blast -
beginning with the
Monocyte
pluripotent hematopoietic
stem cell? (3)

What are the blood cell


differentiation names of the - Myeloblast - Promyelocyte
NEUTROPHIL, EOSINOPHIL, - Myelocyte - Metamyelocyte
and BASOPHIL cell lines - Stab cell - Neutrophil,
beginning with the eosinophil or basophil
myeloblast stage? (6)
What are the blood cell
differentiation names of the - Pluripotent hematopoietic
PLASMA CELL line beginning stem cell - Lymphoblast - B
with the pluripotent cell - Plasma cell
hematopoietic stem cell? (4)

What are the blood cell


differentiation names of the - Pluripotento hematopoietic
PLATELET CELL line beginning stem cell - Megakaryoblast -
with the hematopoietic stem Megakaryocyte - Platelets
cell? (4)

What are the components of - Type I pneumocyte - tight


the air-blood barrier? junction - endothelial cell

What are the steps of - maturation in the marrow -


maturation of a B cell? (2 migration to peripheral
points) lymphoid tissue
- Heparin (anticoagulant) -
What are the substances
histamine (vasodilator) -
contained within the densly
vasoactive amines - Slow
basophilic granules of the
reacting substance of
basophil? (4)
anaphylaxis

What are two basic - multilobed nucleus - large,


morphological characteristics spherical azurophilic primary
of neutrophils? granules (lysosomes)

- acute inflammmatory
What are two important
response of a cell -
functions of a neutrophil?
phagocytosis

What are two names for an


Erythrocytosis and
increased number of red
polycythemia
cells?
What cell type closely
basophil
resembles a mast cell?

What cranial nerves are


CN VII, VIII (association with
commonly involved in an
internal acoustic meatus)
acoustic neuroma?

What disease is characterized


by destruction of Multiple sclerosis
oligodendroglia?

What does CD stand for? cluster of differentiation


What drug prevents mast cell
Cromolyn sodium
degranulation?

What immunoglobulin can


bind to the membrane of a IgE
mast cell?

a baby (developing)
What is a reticulocyte?
erythrocyte

What is an important example


Acoustic neuroma
of a Schwannoma?
DPPC
What is another name for
(dipalmitoylphosphatidylcholi
pulmonary sufractant?
ne)

What is percentage of
leukocytes in the blood exist less than 1%
as basophils?

What is the 'gap' between the


myelination segment of 2 Node of Ranvier
Schwann cells called?

What is the advantage of the


easy gas exchange (Oxygen
large surface area:volume
and Carbon dioxide)
ratio in erythrocytes?
What is the basic morphologic
Anucleate, biconcave
structure of an erythrocyte?

- bilobate nucleus - packed


What is the basic morphology
with large eosinophilic
of an eosinophil? (2 things)
granules of uniform size

What is the embryologic


mesoderm
origin of microglia?

What is the function of


Interferon gamma with macrophage activation
relation to macrophages?
What is the function of
phagocytosis in CNS
microglia?

What is the function of myelination of multiple CNS


oligodendroglia? axons

lowers alveolar surface


What is the function of
tension and prevents
pulmonary surfactant?
atelectasis

myelination of PNS (a
What is the function of
Schwann cell myelinates only
Schwann cells?
one PNS axon)
> 2.0 in fetal lung is
What is the importance of the
indicative of fetal lung
lecithin:sphingomyelin ratio?
maturity

Membranes contain the


chloride bicarbonate antiport
What is the importance of the
allowing the RBC to transport
physiologic chloride shift in
carbon dioxide from the the
erythrocytes?
lung periphery for
elimination.

What is the last segment of


lung tissue in which ciliated respiratory bronchioles
cells are found?

terminal broncioles
What is the last segment of (remember ciliated cells
lung tissue in which goblet sweep away mucous
cells are found? produced by goblet cells and
therefore run deeper)
What is the primary function
Mediates allergic reactions
of a basophil?

What is the primary function


Defense against infections
of a leukocyte?

What is the primary function


Mediates allergic reactions
of a mast cell?

production of large amounts


What is the primary function
of a specific antibody to a
of a plasma cell?
particular antigen
glucose (90% anaerobically
What is the primary source of
degraded to lactate, 10% by
energy for erythrocytes?
HMP shunt)

release of histamine, heparin,


What is the process of
and eosinophil chemotactic
degranulation in mast cells?
factors

What is the range of


4,000 - 10,000 cells per
concentration for leukocytes
microliter
in the blood?

What is the response of an


eosiniphil to antigen antibody high degree of phagocytosis
complexes?
What is the response of transformation into large
microglia to tissue dammage? ameboid phagocytic cells

What is the response to fusion to form multinucleated


microglia infected with HIV? giant cells in CNS

What is the survival time for


120 days
an erythrocyte?

What pathognomonic change


is seen in neutrophils of a
hypersegmented polys
person who is folate/vitamin
B12 deficient?
What percentage of
leukocytes exist as 1 - 6%
eosinophils in the blood?

What percentage of
leukocytes exist as 40 - 75%
neutrophils in the blood?

What percentage of
leukocytes in blood are 2 - 10%
monocytes?

What process occurs when


Type II pneumocytes develop
type I pneumocytes are
into type I
damaged?
What substance in
eosinophilic granules is
primarily responsible for major basic protein
defense against helminths
and protozoan infections?

Where is the site of


Thymus
maturation of T lymphocytes?

Which cell type constituitively


secretes pulmonary Type II pneumocyte
surfactant?

Which cell type lines the


Type I pneumocyte
alveoli?
Which leukemia is the result
Multiple myeloma
of plasma cell neoplasm?

Which type of hypersensitivity


Type I hypersensitivity
reaction is a mast cell
reaction
involved in?

Which type of immunity do B


humoral immunity
cells exhibit?

After arising from the floor of


the primitive pharynx, where It descends down into the
does the thryoid diverticulum neck
go?
After the first breath at birth,
what causes closure of the An increase in oxygen
ductus arteriosus?

A decrease resistance in
After the first breath at birth,
pulmonary vasculature causes
what causes the closure of
increased left atrial pressure
the foramen ovale?
vs. right atrial pressure

Although the diaphragm


descends during
C3-C5
development, it maintains
innervation from ____?

An easy pneumonic to
Young Liver Synthesizes
remember fetal erythropoiesis
Blood
is?
At what time in the course of
development is the fetus
Weeks 3-8
most susceptible to
teratogens?

Deoxygenated blood from the


SVC is expelled into the
pulmonary artery and ____ ductus arteriosus
____ to the lower body of the
fetus.

Do the cardiovascular
structures arise from neural
Mesoderm
crest (ectoderm), mesoderm,
or endoderm?

Do the chromaffin cells of the


adrenal medulla arise from
Neural Crest (Ectoderm)
neural crest (ectoderm),
mesoderm, or endoderm?
Do the enterochromaffin cells
arise from neural crest
Neural Crest (Ectoderm)
(ectoderm), mesoderm, or
endoderm?

Do the lungs arise from


neural crest (ectoderm), Endoderm
mesoderm, or endoderm?

Do the lymphatics arise from


neural crest (ectoderm), Mesoderm
mesoderm, or endoderm?

Do the melanocytes arise


from neural crest (ectoderm), Neural Crest (Ectoderm)
mesoderm, or endoderm?
Do the neural crest cells arise
from mesoderm, ectoderm, or Ectoderm
endoderm?

Do the odontoblasts arise


from neural crest (ectoderm), Neural Crest (Ectoderm)
mesoderm, or endoderm?

Do the parafollicular (C) cells


of the thyroid arise from
Neural Crest (Ectoderm)
neural crest (ectoderm),
mesoderm, or endoderm?

Do the Schwann cells arise


from neural crest (ectoderm), Neural Crest (Ectoderm)
mesoderm, or endoderm?
Do the urogenital structures
arise from neural crest
Mesoderm
(ectoderm), mesoderm, or
endoderm?

Does blood arise from neural


crest (ectoderm), mesoderm, Mesoderm
or endoderm?

Does bone arise from neural


crest (ectoderm), mesoderm, Mesoderm
or endoderm?

Does muscle arise from


neural crest (ectoderm), Mesoderm
mesoderm, or endoderm?
Does the thyroid arise from
neural crest (ectoderm), Endoderm
mesoderm, or endoderm?

Does the adrenal cortex arise


from neural crest (ectoderm), Mesoderm
mesoderm, or endoderm?

Does the ANS arise from


neural crest (ectoderm), Neural Crest (Ectoderm)
mesoderm, or endoderm?

Does the celiac ganglion arise


from neural crest (ectoderm), Neural Crest (Ectoderm)
mesoderm, or endoderm?
Does the dorsal root ganglion
arise from neural crest
Neural Crest (Ectoderm)
(ectoderm), mesoderm, or
endoderm?

Does the dura connective


tissue arise from neural crest
Mesoderm
(ectoderm), mesoderm, or
endoderm?

Does the gut tube epithelium


arise from neural crest
Endoderm
(ectoderm), mesoderm, or
endoderm?

Does the liver arise from


neural crest (ectoderm), Endoderm
mesoderm, or endoderm?
Does the pancreas arise from
neural crest (ectoderm), Endoderm
mesoderm, or endoderm?

Does the parathyroid arise


from neural crest (ectoderm), Endoderm
mesoderm, or endoderm?

Does the pia arise from


neural crest (ectoderm), Neural Crest (Ectoderm)
mesoderm, or endoderm?

Does the serous linings of


body cavities arise from
Mesoderm
neural crest (ectoderm),
mesoderm, or endoderm?
Does the spleen arise from
neural crest (ectoderm), Mesoderm
mesoderm, or endoderm?

Does the thymus arise from


neural crest (ectoderm), Endoderm
mesoderm, or endoderm?

From what does the


ligamentum teres hepatis Umbilical vein
arise?

Results from incomplete


How does a bicornate uterus
fusion of the
form?
paramesonephric ducts
Failure of fusion of the
How does a cleft lip form? maxillary and medial nasal
processes

Failure of fusion of the lateral


palatine processes, the nasal
How does a cleft palate form?
septum, and/or the median
palatine process

Inferior poles of both kidneys


fuse, as they ascend from the
How does a horseshoe kidney pelvis during development
form? they get trapped under the
inferior mesenteric artery,
and remain low in the
abdomen

How is meckel's diverticulum Omphalomesenteric cyst is a


different than an cystic dilatation of the
omphalomesenteric cyst? vitelline duct
How long does full
development of 2 months
spermatogenesis take?

- 2 umbilical arteries (carries


How many arteries and veins
deoxygenated blood away
does the umbilical cord
from fetus) - 1 umbilical vein
contain?
(oxygenated blood to fetus)

Is a primary spermatocyte 2N
4N
or 4N?

Is a primary spermatocyte
Diploid, 4N
haploid or diploid?
Is a secondary spermatocyte
Haploid, 2N
haploid or diploid?

Is a secondary spermatocyte
2N
N or 2N?

Is a speratogonium haploid or
Diploid, 2N
diploid?

Is a spermatid haploid or
Haploid, N
diploid?
Meiosis I is arrested in which
Prophase
phase until ovulation?

Meiosis II is arrested in which Metaphase (an egg MET a


phase until fertilization? sperm)

Most oxygenated blood


reaching the heart via IVC is
diverted through the ____ ____ foramen ovale
and pumped out the aorta to
the head.

The right common cardinal


vein and right anterior
Superior vena cava
cardinal vein give rise to what
adult heart structure?
The stapedius muscle of the
ear is formed by which 2nd
branchial arch?

This type of bone formation


consists of ossification of
cartilaginous molds and Endochondral
forms long bones at primary
and secondary centers.

True or False, blood in the


umbilical vein is 100% False, it is 80% saturated
saturated with oxygen?

True or False, there are two


True, type A & type B
types of spermatogonia?
What are the 1st branchial
arch derivatives innervated CN V2 and V3
by?

What are the 2nd branchial


arch derivatives innervated CN VII
by?

What are the 3rd branchial


arch derivatives innervated CN IX
by?

What are the 4th and 6th


branchial arch derivatives CN X
innervated by?
What are the cartilage - Thyroid - Cricoid -
derivatives (5) of the 4th and Arytenoids - Corniculate -
6th branchial arches? Cuneiform

- 2 inches long - 2 feet from


the ileocecal valve - 2% of the
What are the five 2's
population - Commonly
associated with meckel's
presents in the first 2 years of
diverticulum?
life - May have 2 types of
epithelia

- 2 germ layers: epiblast


& hypoblast - 2 cavities:
What are the rule of 2's for
amniotic cavity & yolk
the 2nd week of
sac - 2 components to the
development?
placenta: cytotrophoblast
& syncytiotrophoblast

3 germ layers (gastrula):


What are the rule of 3's for
ectoderm, mesoderm,
the 3rd week of development?
endoderm
What can a persistent cervical
A branchial cyst in the neck
sinus lead to?

What can be found in the It is dense with immature T


cortex of the thymus? cells

It is pale with mature T cells,


What can be found in the
epithelial reticular cells, and
medulla of the thymus?
Hassall's corpuscles

What connects the thyroid


The thyroglossal duct
diverticulum to the tongue?
What devlopmental
contributions does the 5th None
branchial arch make?

What do the 2nd - 4th


branchial clefts form, which
are obliterated by Temporary cervical sinuses
proliferation of the 2nd arch
mesenchyme?

What does aberrant


development of the 3rd and DiGeorge's syndrome
4th pouches cause?

What does the 1st aortic arch


Part of the maxillary artery
give rise to?
What does the 2nd
Epithelial lining of the
pharyngeal pouch develop
palantine tonsils
into?

Common carotid artery and


What does the 3rd aortic arch
proximal part of the internal
give rise to?
carotid artery

What does the 4th pharyngeal


Superior parathyroids
pouch develop into?

What does the 5th aortic arch


Nothing
give rise to?
What does the 5th pharyngeal
C cells of the thyroid
pouch develop into?

The proximal part of the


What does the 6th aortic arch
pulmonary arteries and (on
give rise to?
left only) ductus arteriosus

What does the ductus


Ligamentum arteriosum
arteriosus give rise to?

What does the ductus


venosus shunt blood away Liver
from?
What does the first branchial
The external auditory meatus
cleft develop into?

What does the foramen ovale


Fossa ovalis
give rise to?

What does the left 4th aortic


Aortic arch
arch give rise to?

What does the ligamentum


Ductus venosus
venosum come from?
What does the notochord give
Nucleus Pulposus
rise to?

What does the primitive atria Trabeculated left and right


give rise to? atrium

What does the primitive Trabeculated parts of the left


ventricle give rise to? and right ventricle

What does the right 4th aortic Proximal part of the right
arch give rise to? subclavian artery
What does the right horn of
Smooth part of the right
the sinus venosus give rise
atrium
to?

Dorsal mesentery, but is


What does the spleen arise
supplied by the artery of the
from?
foregut

What does the thymus arise Epithelium of the 3rd


from? branchial pouch

What does the thyroid The floor of the primitive


diverticulum arise from? pharynx
What does the truncus The ascending aorta and
arteriosus give rise to? pulmonary trunk

What does the umbilical


Medial umbilical ligaments
arteries give rise to?

What ear muscle does the 1st


Tensor tympani
branchial arch form?

What effect does 13-cis-


Extremely high risk for birth
retinoic acid have on the
defects
fetus?
What effect does ACE
Renal Damage
inhibitors have on the fetus?

What effect does iodide have Congenital goiter or


on the fetus? hypothyroidism

What effect does warfarin and


Multiple anomalies
x-rays have on the fetus?

What effects does cocaine Abnormal fetal development


have on the fetus? and fetal addiction
What embryonic structure are
the smooth parts of the left
Bulbus cordis
and right ventricle derived
from?

What embryonic structure


Left horn of the sinus
does the coronary sinus come
venosus
from?

What embryonic structure


does the median umbilical Allantois (urachus)
ligament come from?

What fetal landmark has


developed within week 2 of Bilaminar disk
fertilization?
What fetal landmark has
occurred within week 1 of Implantation
fertilization?

What fetal landmark has


occurred within week 3 of Gastrulation
fertilization?

What fetal landmarks (2) have


Primitive streak and neural
developed within week 3 of
plate begin to form
fertilization?

- Neurohypophysis - CNS
What five things arise from
neurons - Oligodendrocytes -
neuroectoderm?
Astrocytes - Pineal gland
- Septum transversum -
What four structures make up pleuroperitoneal folds - body
the diaphragm? wall - dorsal mesentery of
esophagus

- Adenohypophysis - Lens of
What four things arise from
eye - Epithelial linings -
surface ectoderm?
Epidermis

What four things does


- Mandible - Malleus - Incus
Meckel's cartilage (from the
- Sphenomandibular ligament
1st arch) develop into?

What four things does - Stapes - Styloid process -


Reichert's cartilage (from the Lesser horn of hyoid -
2nd arch) develop into? Stylohyoid ligament
What four things does the
Body, tail, isthmus, and
dorsal pancreatic bud
accessory pancreatic duct
become?

What four things does the - Seminal vesicles -


mesonephric (wolffian) duct Epididymis - Ejaculatory duct
develop into? - Ductus deferens

What induces the ectoderm to


form the neuroectoderm Notochord
(neural plate)?

Abdominal contents herniate


into the thorax due to
What is a hiatal hernia?
incomplete development of
the diaphragm
Abnormal opening of penile
urethra on inferior side of
What is a hypospadias?
penis due to failure of
urethral folds to close

What is a single umbilical Congenital and chromosomal


artery associated with? anomalies

What is a urachal cyst or


The allantois
sinus a remnant of?

What is an abnormal opening


of penile urethra on superior
side of penis due to faulty Epispadias
positioning of the genital
tubercle?
What is associated with an
Exstrophy of the bladder
epispadias?

What is Meckel's Persistence of the vitelline


diverticulum? duct or yolk sac

Bilateral renal agenesis or


What is oligohydramnios
posterior urethral valves (in
associated with?
males)

What is oligohydramnios? < 0.5 L of amniotic fluid


What is polyhydramnios Esophageal/duodenal atresia,
associated with? anencephaly

What is polyhydramnios? > 1.5-2 L of amniotic fluid

Bilateral renal agenesis, that


results in ologohydramnios
What is Potter's syndrome? causing limb and facial
deformities and pulmonary
hypoplasia (Babies with
Potter's can’t pee in utero)

What is the acrosome of


Golgi apparatus
sperm derived from?
What is the female
homologue to the corpus Vestibular bulbs
spongiosum in the male?

What is the female


Urethral and paraurethral
homologue to the prostate
glands (of Skene)
gland in the male?

What is the female


homologue to the scrotum in Labia majora
the male?

What is the female


homologue to the ventral
Labia minora
shaft of the penis in the
male?
What is the flagellum (tail)
One of the centrioles
derived from?

What is the food supply of


Fructose
sperm?

What is the male homologue


to the glans clitoris in the Glans penis
female?

What is the male homologue


to the greater vestibular Bulbourethral glands (of
glands (of Bartholin) in the Cowper)
female?
What is the most common
congenital anomaly of the GI Meckel's diverticulum
tract?

What is the most common


The tongue
ectopic thyroid tissue site?

What is the normal remnant


Foramen cecum
of the thyroglossal duct?

What is the postnatal The nucleus pulposus of the


derivative of the notochord? intervertebral disc
What is the site of T-cell
Thymus
maturation?

What part of the gut is the


Foregut
pancreas derived?

What suppresses the


development of the Mullerian inhibiting substance
paramesonephric ducts in (secreted by the testes)
males?

What teratogenic agent


causes limb defects ('flipper' Thalidomide
limbs)?
What three structures does - Thymus - Left inferior
the 3rd pharyngeal pouch parathyroid - Right inferior
develop into? parathyroid

What three things does the - Middle ear cavity -


1st pharyngeal pouch develop Eustachian tube - Mastoid air
into? cells

What three things does the


- Fallopian tube - Uterus -
paramesonephric (mullerian)
Part of the vagina
duct develop into?

What three things does the - Pancreatic head - uncinate


ventral pancreatic bud process - main pancreatic
become? duct
Heart begins to beat, upper
What two things occur during
and lower limb buds begin to
week 4 of fetal development?
form

What type of bone formation


is spontaneous without Intramembranous
preexisting cartilage?

What type of twins would


have 1 placenta, 2 amniotic Monozygotic twins
sacs, and 1 chorion?

What type of twins would


Monozygotic or dizygotic
have 2 amniotic sacs and 2
twins
placentas?
What will DiGeorge's T cell deficiency &
syndrome lead to? hypocalcemia

When do primary oocytes


During fetal life
begin meiosis I?

When do primary oocytes


Just prior to ovulation
complete meiosis I?

When does fetal


erythropoiesis occur in the Week 28 and onward
bone marrow?
When does fetal
erythropoiesis occur in the Weeks 6-30
liver?

When does fetal


erythropoiesis occur in the Weeks 9-28
spleen?

When does organogenesis


Weeks 3-8
occur in the fetus?

Where does positive and


At the corticomedullary
negative selection occur in
junction
the thymus?
Where does spermatogenesis
Seminferous tubules
take place?

Where is the first place fetal


erythropoiesis occurs and Yolk sac (3-8 wk)
when does this take place?

Which aortic arch does the


stapedial artery and the hyoid 2nd aortic arch
artery come from?

Which branchial arch are the


greater horn of hyoid and the
3rd branchial arch
stylopharyngeus muscle
derived from?
Which branchial arch does
Meckel's cartilage develop 1st arch
from?

Which branchial arch forms


the anterior 2/3 of the 1st arch
tongue?

Which branchial arch forms


the incus and malleus of the 1st arch
ear?

Which ear bone(s) does the


Stapes
2nd branchial arch form?
Which embryonic tissue are
Ectoderm
branchial clefts derived from?

Which embryonic tissue are


branchial pouches derived Endoderm
from?

Which is more common a


Hypospadias
hypospadias or epispadias?

Which muscles (3) are - Most pharyngeal


derivatives of the 4th constrictors - Cricothyroid -
branchial arch? Levator veli palatini
Which muscles (4) are - Muscles of facial expression
derivatives of the 2nd - Stapedius - Stylohyoid -
branchial arch? Posterior belly of digastric

- Temporalis - Masseter -
Lateral pterygoid - Medial
Which muscles (8) are
pterygoid - Mylohyoid -
derivatives of the 1st
Anterior belly of digastric -
branchial arch?
Tensor tympani - Tensor veli
palatini

All intrinsic muscles of the


Which muscles are derivatives
larynx, except the
of the 6th branchial arch?
cricothyroid

Which pharyngeal arch does


Reichert's cartilage develop 2nd arch
from?
Which teratogenic agent
causes vaginal clear cell DES
adenocarcinoma?

Which two branchial arches


form the posterior 1/3 of the 3rd and 4th arches
tongue?

Which two embryonic tissues


are branchial arches derived Mesoderm and neural crests
from?

Which week of fetal


development have the
Week 10
genitalia taken on male/
female characteristics?
A common football injury
--Medial collateral ligament
caused by clipping from the
--Medial meniscus --Anterior
lateral side will damage what
cruciate ligament
structures (3 answers)?

A lumbar puncture is
Iliac crest
performed at what landmark/

A positive anterior drawer


Anterior cruciate ligament
sign indicates damage to
(ACL)
what structure?

A pudendal nerve block is


Ischial spine
performed at what landmark?
Abnormal passive abduction
Medial collateral ligament
of the knee indicates damage
(MCL)
to what structure?

Anterior' in ACL refers to


Tibial
what attachment?

At what level is a lumbar


Between L3-L4 or L4-L5
puncture performed?

Common peroneal nerve


Loss of dorsiflexion(Foot
damage manifests what
Drop)
deficit?
Common peroneal, Tibial,
--'L4-S2 (common peroneal)
Femoral, and Obturator
--L4-S3 (tibial) --L2-L4
nerves arise from what spinal
(femoral) and (obturator)
cord segments (4 answers)?

Coronary artery occlusion Left anterior descending


usually occurs where? artery (LAD)

Do the coronary arteries fill


Diastole
during systole or diastole?

Erection and sensation of the


S2-S4
penis is in what dermatomes?
Femoral nerve damage
Loss of knee jerk
manifests what deficit?

The left wraps around the


How does the course of the arch of the aorta and the
left recurrent laryngeal nerve ligamentum arteriosum while
differ from that of the right? the right wraps around the
subclavian artery.

2/3 of the way from the


How is the appendix located? umbilicus to the anterior
superior iliac spine

--Right has three


How many lobes are in the
(superior,middle,inferior) --
right and left lungs and what
Left has two (superior and
are their names?
inferior) and the lingula
1.Left gastric-azygous vv.
2.Superior-Middle/Inferior
Name five portal-systemic rectal vv. 3.Paraumbilical-
anastomoses. inferior epigastric
4.Retroperitoneal-renal vv.
5.Retroperitoneal-
paravertebral vv.

--Suspensory ligament of
ovaries --Transverse cervical
Name the 4 ligaments of the
(cardinal) ligament --Round
uterus.
ligament of uterus --Broad
ligament

--Opponens digiti minimi --


Name the hypothenar
Abductor digiti minimi --
muscles.
Flexor digiti minimi

1.Duodenum(2nd-4th parts)
2.Descending colon
Name the retroperitoneal 3.Ascending colon 4.Kidney
structures (9). & ureters 5.Pancreas
6.Aorta 7.Inferior vena cava
8.Adrenal glands 9.Rectum
--Supraspinatus --
Name the rotator cuff
Infraspinatus --teres minor
muscles.
--Subscapularis

--Opponens pollicis --
Name the thenar muscles Abductor pollicis brevis --
Flexor pollicis brevis

Obturator nerve damage


Loss of hip adduction
manifests what deficit?

Pain from the diaphragm is


Shoulder
usually referred where?
Subarachnoid space extends
S2
to what spinal level?

The area of the body that


contains the appendix is McBurney's point
known as what?

The femoral triangle contains --Femoral nerve --Femoral


what structures from lateral artery --Femoral vein --
to medial? Femoral Canal (lymphatics)

The inguinal ligament exists


L1
in what dermatome?
The kneecaps exist in what
L4
dermatome?

The male sexual response of


ejaculation is mediated by
Visceral and somatic nerves
what part of the nervous
system?

The male sexual response of


emission is mediated by what Sympathetic nervous system
part of the nervous system?

The male sexual response of


Parasympathetic nervous
erection is mediated by what
system
part of the nervous system?
The nipple exists in what
T4
dermatome?

The recurrent laryngeal nerve 1.CN X 2.All intrinsic muscles


arises from what cranial nerve of the larynx except the
and supplies what muscles? cricothyroid muscle.

The SA and AV nodes are


usually supplied by what Right Coronary Artery (RCA)
artery?

The spinal cord ends at what


L1-L2
level in adults?
The umbilicus exists in what
T10
dermatome?

The xiphoid process exists in


T7
what dermatome?

Tibial nerve damage


Loss of plantar flexion
manifests what deficit?

Protrusions of peritoneum
What are hernias? through an opening, usually
sites of weakness.
Modified smooth muscle of
afferent arteriole in the
What are JG cells?
juxtaglomerular apparatus of
the kidney

--Inferior epigastric artery --


What are the boundaries of
Lateral border of the rectus
the inguinal (Hesselbach)
abdominus --Inguinal
triangle?
ligament

--Skin --Connective tissue


--Aponeurosis --Loose
What are the layers connective tissue --
encountered from the Pericranium --Dura mater --
outsided down to the brain? Subdural space --Arachnoid
--Subarachnoid space --Pia
mater --Brain

--Esophageal varices --
What are the manifestations
Hemorrhoids --Caput
of portal hypertension?
medusae
What condition is usually
associated with portal Alcoholic cirrhosis
hypertension?

What defect may predispose


Defective development of the
an infant for a diaphragmatic
pleuroperitoneal membrane
hernia?

What gut regions and 1.Foregut 2.--Stomach to


structures does the celiac duodenum --liver --
artery supply? gallbladder --pancreas

What gut regions and 1.Hindgut 2.--Distal 1/3 of


structures does the IMA transverse colon to upper
supply? portion of rectum
What gut regions and 1.Midgut 2.--Duodenum to
structures does the SMA proximal 2/3 of transverse
supply? colon

What is a diaphragmatic Abdominal retroperitoneal


hernia? structures enter the thorax

entrance of abdominal
What is a femoral hernia? contents through the femoral
canal.

Stomach contents herniate


upward through the
What is a hiatal hernia?
esophageal hiatus of the
diaphragm
What is the arterial blood
--Superior rectal a. (Above)
supply difference above and
--Inferior rectal a. (Below)
below the pectinate line?

Through weak abdominal


wall, into the inguinal
What is the course of a direct
triangle, medial to the inferior
inguinal hernia?
epigastric artery, through the
external inguinal ring only.

Through the internal (deep)


inguinal ring and the external
What is the course of an (superficial) inguinal ring
indirect inguinal hernia? lateral to the inferior
epigastric artery and into the
scrotum

What is the course of the Pass under uterine artery and


ureters? under the ductus deferens
1.Coordinates motility along
What is the function of
entire gut wall 2.Regulates
Myenteric plexus?
local secretions, blood flow,
Submucosal plexus?
and absorption

What is the function of the JG --secrete renin and


cells? erythropoietin

What is the innervation --Visceral innervation


difference above and below (Above) --Somatic
the pectinate line? innervation (Below)

What is the innervation of the


Phrenic nerve (C3,4,5)
diaphram?
Sodium sensor in part of the
distal convoluted distal
What is the macula densa?
tubule in the juxtaglomerular
apparatus of the kidney

What is the Myenteric plexus


1. Auerbach's plexus 2.
also known as? Submucosal
Meissner's plexus
plexus?

What is the pectinate line of Where the hindgut meets


the rectum? ectoderm in the rectum

What is the relationship of the


two pulmonary arteries in the Right anterior Left superior
lung hilus?
What is the usual pathology
Internal hemorrhoids (not
above the pectinate line of
painful) Adenocarcinoma
the rectum?

What is the usual pathology External hemorrhoids


below the pectinate line of (painful) Squamous cell
the rectum? carcinoma

--Superior rectal v. to IMV to


What is the venous drainage portal system (Above) --
difference above and below Inferior rectal v. to internal
the pectinate line? pudendal v. to internal iliac v.
to IVC (Below)

--Muscularis mucosae --
What layers of the gut wall Inner circular muscle layer --
contribute to motility (4)? Myenteric plexus --Outer
longitudinal muscle layer
What layers of the gut wall --Serosa --Lamina propria --
contribute to support (3)? Submucosa

What muscle opens the jaw? Lateral pterygoid

What nerve innervates most 1.Vagus Nerve (CNX)


of the 'glossus' muscles and 2.Palatoglossus (innervated
which is the exception? by hypoglossal n.)

1.Trigeminal Nerve,
What nerve innervates most
Mandibular branch 2.Tensor
of the 'palat' muscles and
veli palatini (innervated by
which is the exception?
vagus n)
What nerve innervates the
muscles that close and open Trigeminal Nerve (V3)
the jaw?

Cell bodies of
What neurons do the GI
parasympathetic terminal
enteric plexus contain?
effector neurons

What part of the heart does anterior interventricular


the LAD supply? septum

What spinal cord levels are


vertebral disk herniation most Between L5 and S1
likely to occur?
What structure is in the
femoral triangle but not in --Femoral nerve
the femoral sheath?

1.Internal Jugular Vein


What structures are in the (lateral) --2.Common Carotid
carotid sheath? Artery (medial) --3.Vagus
Nerve (posterior)

1.Skin/superficial fascia
2.Ligaments
(supraspinatous,interspinous,
What structures are pierced ligamentum flavum)
when doing an LP? 3.Epidural space 4.Dura
mater 5.Subdural space
6.Arachnoid 7.Subarachnoid
space--CSF

--Round ligaments of the


What structures do the broad
uterus --Ovaries --Uterine
ligament contain (4)?
tubules --Uterine vessels
--Tertiary bronchus --
What structures make up the
Bronchial artery --Pulmonary
bronchopulmonary segment?
artery

--IVC at T8 --esophagus,
What structures perforate the
vagal trunks at T10 --aorta,
diaphragm at what vertebral
thoracic duct, axygous vein at
levels?
T12

What three muscles close the --Masseter --Temporalis --


jaw? Medial pterygoid

What usually provides the


Posterior descending artery
blood supply for the inferior
(PD) of the RCA
left ventricle?
in response to decreased
When do the JG cells secrete renal BP, decreased sodium
renin? delivery to distal tubule, and
increased sympathetic tone

When is damage to the


recurrent laryngeal nerve 1.Thyroid surgery
most likely to happen and 2.Hoarseness
what are its results(2
answers)?

Where is the CSF found? Subarachnoid space

1.Between the inner and outer


Where is the Myenteric plexus layers of smooth muscle in GI
located? Submucosal plexus? tract wall 2.Between mucosa
and inner layer of smooth
muscle in GI tract wall.
Which ligament contains the Suspensory ligament of the
ovarian vessels? ovary

Which ligament contains the Transverse cervical (cardinal)


uterine vessels? ligament

Which lung is the usual site of


Right lung
an inhaled foreign body?

Which lung provides a space Left lung (in the place of the
for the heart to occupy? middle lobe)
Which meningeal layer is not
Pia mater
pierced during an LP?

Who usually gets a direct 1.Older men 2.Infants (failure


inguinal hernia? indirect of processus vaginalis to
hernia (and why)? close)

What are the 3 layers of


1) Endoneurium 2)
peripheral nerves? (inner to
Perineurium 3) Epineurium
outer)

1. bone 2. tendon 3. skin 4.


Where is type I collagen
dentin 5. fascia 6. cornea 7.
found?(7)
late wound repair
1. cartilage (including hyaline)
Where is type II collagen
2. vitreous body 3. nucleus
found? (3)
pulposus.

1. Cochlea- hearing 2.
What are the functions of the
vestibule- linear acceleration
major structures of the inner
3. semicircular canals-
ear bony labyrinth?
angular acceleration.

What are the major structures


1. Cochlea 2. vestibule 3.
of the inner ear bony
semicircular canals
labyrinth?

What are the major structures 1. Cochlear duct 2. utricle. 3.


of the inner ear membranous saccule 4. semicircular
labyrinth? canals.
Name two proteins involved
in the structure of macula 1. Desmoplakin 2.Keratin
adherens.

1. Distribution center of proteins and


lipids from ER to plasma membrane,
lysosomes, secretory vessicles 2.
Modifies N-oligosaccharides on
Name 6 functions of Golgi asparagine 3. Adds O-oligosaccharides
apparatus. to Ser and Thr residues 4. Proteoglycan
assembly from proteoglycan core
proteins 5. Sulfation of sugars in
proteoglycans and of selected tyrosine
on proteins

6. Addition of mannose-6-
phosphate to specific
Name 6 functions of Golgi
lysosomal proteins, which
apparatus (continued answer)
targets the protein to the
lysosome

Name two proteins involved


1. E-cadherins 2. actin
in the structure of zona
filaments
adherens?
1. liver hepatocytes, 2.
Which cells are rich in smooth
steroid hormone-producing
ER?
cells of adrenal cortex.

1. M cells take up antigen. 2.


stimulated B cells leave Peyer's patch
and travel through lymph and blood to
Describe the immune lamina propria of intestine. 3. In
response stimulated via lamina propria B cells differentiate into
IgA-secreting plasma cells. 4. IgA
Peyer's patches. receives protective secretory
component. 5. IgA is transported
across epithelium to gut to deal with
intraluminal Ag.

1. Mucus-secreting goblet
Which cells are rich in rough cells of small intestine, 2.
ER? antibody-secreting plasma
cells.

1. Nonspecific filtration by
What are the functions of the macrophages. 2. storage/
lymph node? proliferation of B and T cells
3. Ab production.
1. skin 2.blood vessels
Where is type III collagen
3.uterus 4.fetal tissue
found? (5)
5.granulation tissue

1. zona occludens 2.zona


Name five types of epithelial adherens 3.macula adherens
cell junctions. 4.gap junction
5.hemidesmosome

Describe microtubule 9+2 arrangement of


arrangement of cilia. microtubules.

A Peyer's patch is 'covered' by


Describe the outer structure single layer of cuboidal
enterocytes, interspersed with
of a Peyer's patch.
specialized M cells (no goblet
cells).
A secondary lymphoid organ.
Has many afferents, one or
What is a lymph node?
more efferents. With
Include information on
trabeculae. Major histological
structural components.
regions = Follicle, Medulla,
Paracortex

What is the primary


regulatory control of zona ACTH, hypothalamic CRH
fasciculata secretion?

What is the primary


regulatory control of zona ACTH, hypothalamic CRH
reticularis secretion?

What are/is the primary


secretory product of the zona aldosterone
glomerulosa?
What do Brunner's glands
alkaline mucus
secrete?

Allow macromolecules of
What is the function of liver plasma full access to surface
sinusoids? of liver cells through space of
Disse.

Allows adjacent cells to


What is the function of a gap
communicate for electric and
junction?
metabolic functions.

What is produced by alpha


cells of the Islets of alpha cells produce glucagon
Langerhans?
What three cell types are
alpha, beta, and gamma cells
found in Islets of Langerhans?

What type of cells are Nissl Are found in neurons. Are not
bodies found? In what parts found in axon or axon
of the cell? hillock.

IN what area of the spleen are B cells are found within the
B cells found? white pulp of the spleen.

What is type IV collagen basement membrane or basal


found? (1) lamina
What is produced by beta
cells of the Islets of beta cells produce insulin
Langerhans?

What is the only GI


Brunner's glands
submucosal gland?

Describe the histological Capsule, Zona glomerulosa,


layers of the adrenal glands Zona fasciculata, Zona
(outside to in) reticularis, Medulla.

What are/is the primary


secretory product of the Catecholamines (Epi, NE)
adrenal medulla?
Check out the picture in the
Memo to you.
book.

Check out the picture in the


Memo to you.
book. p. 105

What do the medullary cords Closely packed lymphocytes


consist of? and plasma cells.

What is the most common


Collagen Type I - 90%
type of collagen?
What is the most abundant
Collagen.
protein in the human body?

Define Islets of Langerhans. Collections of endocrine cells.

What is the function of Connect cells to underlying


hemidesmosomes? extracellular matrix.

What are/is the primary


secretory product of the zona cortisol, sex hormones.
fasciculata?
What is another name for
Desmosome
macula adherens?

What is the effect of duodenal Duodenal ulcers cause


ulcers on Brunner's gland hypertrophy of Brunner's
histology? glands.

Dynein causes the bending of


How does dynein function in
cilium by differential sliding
cilia function?
of doublets.

What kind of protein is


Dynein is an ATPase.
dynein?
Describe the role of dynein in Dynein links peripheral 9
cilia structure. doublets of microtubules.

Endolymph is made by the


What makes endolymph?
stria vascularis.

Endoneurium invests single


What is Endoneurium? nerve fiber of the peripheral
nerve.

Epineurium (dense connective


What is Epineurium? tissue) surrounds entire never
(fascicles and blood vessels)
What is type X collagen
epiphyseal plate
found? (1)

Filtration of plasma occurs


Plasma is filtered on the basis
according to net charge and
of what properties?
size.

From the fusion of


How is the glomerular
endothelial and podocyte
basement membrane formed?
basement membranes.

What is produced by gamma


gamma cells produce
cells of the Islets of
somatostatin.
Langerhans?
GFR (Glomerulosa,
Fasciculata, Reticularis)
What is the mnemonic to
corresponds to Salt (Na+),
remember layers and
Sugar (glucocorticoids) and
products of adrenal cortex?
Sex (androgens) The deeper
you go, the sweeter it gets.

Hair cells are the sensory


What is the function of hair
elements in both the cochlear
cells?
and vestibular apparatus.

Name a protein involved in


the structure of Integrin.
hemidesmosomes.

What is another name for


Intermediate junction.
zona adherens?
Irregular 'capillaries' with
Describe the histological
round pores 100-200 nm in
structure of sinusoids of the
diameter and no basement
liver.
membrane.

Is the site of steroid synthesis


What is the function of
and detoxification of drugs
smooth ER?
and poisons

Is the site of synthesis of


What is the function of rough secretory (exported proteins
ER? and of N-linked
oligosaccharide addition to
many proteins.

What part of pancreas are the Islets of Langerhans are most


Islets of Langerhans numerous in the tail of
concentrated? pancreas.
What structural defect causes Kartagener's syndrome is due
Kartagener's syndrome? What to dynein arm defect. Results
is the consequence? in immotile cilia.

Large, encapsulated sensory


receptors found in deeper
Define Pacinian corpuscles. layers of skin at ligaments,
joint capsules, serous
membranes, mesenteries.

Where are Brunner's glands Located in submucosa of


located? duodenum

Long, vascular channels in


Describe the histologic
red pulp. With fenestrated
structure of sinusoids of the
'barrel hoop' basement
spleen.
membrane.
What is the histologic change Lymph node paracortex
in lymph nodes during an becomes enlarged during
extreme cellular immune extreme cellular immune
response? response.

What is the histologic Lymph node paracortex is not


presentation of DiGeorge's well developed in patients
syndrome? with DiGeorge's syndrome.

What kind of cells are found


nearby the sinusoids of the Macrophages
spleen?

What are the major structures Medulla consists of medullary


of the lymph node medulla? cords and medullary sinuses.
Medullary sinuses
What do medullary sinuses
communicate with efferent
communicate with?
lymphatics.

Medullary sinuses contain


What do medullary sinuses
reticular cells and
consist of?
macrophages.

Meissner's corpuscles are


What is the function of involved in light
Meissner's corpuscles? discriminatory touch of
glabrous skin.

What is the histologic change


in nephrotic syndrome? What Negative charge is lost.
is the consequence of this Plasma protein is lost in urine
change?
What is the glomerular
basement membrane coated Negatively charged heparan
with? (provides negative sulfate.
charge to filter).

What is the most common


tumor the adrenal medulla in Neuroblastoma
children?

Pacinian corpuscles are


What is the function of
involved in pressure, coarse
Pacinian corpuscles?
touch, vibration, and tension.

What do the Islets of


Pancreatic buds.
Langerhans arise from?
What specialized vascular
Paracortex contains high
structure is found in the
endothelial venules (HEV). T
lymph node paracortex?
and B cells enter from the
What is the function of this
blood through the HEV.
structure?

What cells are found in the


Paracortex houses T cells.
lymph node paracortex?

Perineurium (permeability
What is Perineurium? barrier) surrounds a fascicle
of nerve fibers.

What is the most common


tumor the adrenal medulla in Pheochromocytoma
adults?
Compare the consequences Pheochromocytoma causes
of pheochromocytoma vs. episodic hypertension
neuroblastoma on blood Neuroblastoma does NOT
pressure cause episodic hypertension

Pores in liver sinusoids


allowing plasma
What is the space of Disse?
macromolecules access to
liver cell surfaces.

What is the primary


Preganglionic sympathetic
regulatory control of adrenal
fibers
medulla secretion?

What is the function of zona Prevents diffusion across


occludens? intracellular space.
Primary follicles are dense
Describe the appearance and and dormant. Secondary
status of primary vs. follicles have pale central
secondary follicles. germinal centers and are
active.

Describe the location of the Region of cortex between


lymph node paracortex. follicles and medulla.

What is the primary


regulatory control of zona Renin-angiotensin
glomerulosa secretion?

What is the glomerular


Responsible for the actual
basement membrane
filtration of plasma.
responsible for?
What is another name for
reticulin
type III collagen?

What are Nissl bodies? rough ER

Semicircular canals contain


Where in the inner ear are the
ampullae Functions in
ampullae found? What is the
detecting angular
function of this structure?
acceleration.

What are/is the primary


sex hormones (e.g.
secretory product of the zona
androgens)
reticularis?
What is the function of lymph Site of B-cell localization and
node follicles? proliferation.

Small, discrete sites of


Define macula adherens.
attachment of epithelial cells.

Small, encapsulated sensory


Define Meissner's corpuscles. receptors found in dermis of
palm, soles and digits of skin.

Specialized cell interspersed


What is an M cell? What is it's between the cuboidal
function. enterocytes covering a Peyer's
patch. M cells take up
antigens.
stratum Corneum, stratum
Name the layers of epidermis Lucidum, stratum
from surface to base. Granulosum, stratum
Spinosum, stratum Basalis.

What is the location of zona Surrounds the perimeter just


adherens? below zona occludens.

Synthesize enzymes (e.g.


What is the function of Nissl
ChAT) and peptide
bodies?
neurotransmitters.

T cells are found in the PALS


In what area of the spleen are
and the red pulp of the
T cells found?
spleen.
Which part of the cochlea The base of the cochlea picks
picks up high frequency up high frequency sound the
sound? Which picks up low apex picks up low frequency
frequency? sound

What is the bony labyrinth The bony labyrinth is filled


filled with? Describe its with perilymph. Perilymph is
composition. Na+ rich, similar to ECF

The failure of addition of


mannose-6-phosphate to
What is the cause of I cell
lysosome proteins. These
disease? What is the
enzymes are secreted outside
consequence?
the cell instead of being
targeted to the lysosome.

The membranous labyrinth is


What is the membranous
filled with endolymph.
labyrinth filled with? Describe
Endolymph is K+ rich, similar
its composition.
to ICF.
What layer of the peripheral
The perineurium must be
nerve must be rejoined in
rejoined in microsurgery for
microsurgery for limb
limb reattachment.
reattachment?

Where in the inner ear are the The utricle and saccule
maculae found? What is the contain maculae Functions in
function of this structure? detecting linear acceleration.

How is the function of gap Through a connection with


junctions accomplished? central channels.

What is another name for


Tight junction.
zona occludens?
Type ONE: bONE Type TWO:
What are mnemonics for
carTWOlage Type FOUR:
remembering locations for
under the FLOOR (basement
type I, II and IV collagen?
membrane)

Unencapsulated lymphoid
tissue found in lamina propria
What is a Peyer's patch?
and submucosa of small
intestine.

What type of infection may


induce an extreme cellular
immune response? What Viral response is an example.
happens to the lymph node The paracortex enlarges.
during such an immune
response?

According to the
Homunculus man, place the foot, trunk, hand, face,
following in order (from tongue
medial to lateral). hand, foot,
tongue, face, trunk
(T or F) Can Bell's palsy occur
true
idiopathically?

(T or F) Can fasiculations be
True
present in a LMN lesion?

(T or F) Is the anterior nucleus


of the thalamus part of the True
limbic system?

(T or F) Is the cingulate gyrus


True
part of the limbic system?
(T or F) Is the Entrorhinal
cortex part of the limbic True
system?

(T or F) Is the hippocampal
formation part of the limbic True
system?

(T or F) Is the mammillary
body part of the limbic True
system?

(T or F) Is the septal area part


True
of the limbic system?
(T or F) Thoracic outlet
syndrome results in atrophy True
of the interosseous muscles?

(T or F) Thoracic outlet
syndrome results in atrophy
True
of the thenar and hypothenar
eminences?

(T or F) Thoracic outlet
syndrome results in
disappearance of the radial True
pulse upon moving the head
to the opposite side?

(T or F) Thoracic outlet
syndrome results in sensory
True
deficits on the medial side of
the forearm and hand?
A lesion of the globus
Wilson's disease
pallidus causes what disease?

Wernicke-Korsakoff's
A lesion of the mammillary
encephalopathy
bodies (bilateraly) produces
(confabulations, anterograde
what?
amnesia)

A lesion of the optic chiasm


bitemporal hemianopsia
produces?

A lesion of the right dorsal


left lower quadrantic anopsia
optic radiation (parietal
(a temporal lesion)
lesion) produces?
A lesion of the right Meyer's
left upper quadrantic anopsia
loop (temporal lobe)
(a temporal lesion)
produces?

A lesion of the right optic


right anopsia
nerve produces?

A lesion of the right optic left homonymous


tract produces? hemianopsia

A lesion of the right visual


left hemianopsia with macular
fibers just prior to the visual
sparing
cortex produces?
A lesion of the Striatum can Huntington's and Wilson's
cause which 2 diseases? disease

A positive Babinski is an
indicator for a (UMN or LMN) UMN lesion
lesion?

A rupture of the middle


menigeal artery causes what
epidural hematoma
type of hematoma? (epidural
or subdural)

A rupture of the superior


cerebral veins causes what
subdural hematoma
type of hematoma? (epidural
or subdural)
An aneurysm of the anterior
communicating artery may visual defects
cause what type of defects?

An aneurysm of what artery posterior communicating


may cause CN III palsy? artery

Are D1 neurons in the basal


ganglia inhibitory or Excitatory
excitatory?

Are D2 neurons in the basal


ganglia inhibitory or Inhibitory
excitatory?
Beginning with anterior ant. comm. - ACA - ICA -
communicating artery post. comm. - PCA - PCA -
describe the path around the post. comm. - ICA - ACA -
circle of Willis. ant. comm.

AIDS, Lyme disease,


Bell's Palsy is seen as a
Sarcoidosis, Tumors,
complication in what 5
Diabetes (ALexander Bell with
things?
STD)

Brodmann's area 17 is? principal visual cortex

Wernicke's area (associative


Brodmann's area 22 is?
auditory cortex)
Brodmann's area 3,1,2 is? principal sensory area

Brodmann's area 4 is? principal motor area

Brodmann's area 41, 42 is? primary auditory cortex

Brodmann's area 44, 45 is? Broca's area (motor speech)


Brodmann's area 6 is? premotor area

frontal eye movement and


Brodmann's area 8 is?
pupilary change area

CN I has what function? smell

CN I passes through what


cribriform plate
'hole'?
CN II has what function? sight

CN II passes through what


optic canal
'hole'?

eye movement, pupil


CN III has what 4 functions? constriction, accommodation,
eyelid opening

medial rectus, superior


CN III inervates what 5 rectus, inferior rectus, inferior
muscles. oblique, levator palpebrae
superioris
CN III passes through what
superior orbital fissure
'hole'?

CN IV has what function? eye movement

CN IV inervates what muscle. superior oblique

CN IV passes through what


superior orbital fissure
'hole'?
posterior 1/3 taste,
swallowing, salivation
CN IX has what 4 functions?
(parotid), monitoring carotid
body and sinus

CN IX passes through what


jugular foramen
'hole'?

CN V has what 2 functions? mastication, facial sensation

CN V1 passes through what


superior orbital fissure
'hole'?
CN V2 passes through what
foramen rotundum
'hole'?

CN V3 passes through what


foramen ovale
'hole'?

CN VI has what function? eye movement

CN VI inervates what muscle. lateral rectus


CN VI passes through what
superior orbital fissure
'hole'?

facial movement, anterior 2/3


CN VII has what 4 functions? taste, lacrimation, salivation
(SL, SM glands)

CN VII passes through what


internal auditory meatus
'hole'?

CN VIII has what 2 functions? hearing, balance


CN VIII passes through what
internal auditory meatus
'hole'?

taste, swallowing, palate


CN X has what 5 functions? elevation, talking,
thoracoabdominal viscera

CN X passes through what


jugular foramen
'hole'?

head turning, shoulder


CN XI has what 2 functions?
shrugging
jugular foramen (descending)
CN XI passes through what
-- foramen magnum
'hole'?
(ascending)

CN XII has what function? tounge movements

CN XII passes through what


hypoglossal canal
'hole'?

Complete the muscle spindle


reflex arc by placing the muscle stretch - intrafusal
following in order: alpha stretch - Ia afferent - alpha
motor, Ia afferent, muscle motor - extrafusal
stretch, extrafusal contraction
contraction, intrafusal
stretch.
Extrafusal fibers are
innervated by what motor alpha motor neuron
neuron?

From which 3 spinal roots


does long thoracic nerve C5, C6, C7
arises?

General sensory/motor
dysfunction and aphasia are
anterior circle
caused by stroke of the? (ant.
circle or post. circle)

Give 3 characteristics of a atrophy, flaccid paralysis,


LMN lesion. absent deep tendon reflexes
Give 3 charateristics of medial rectus palsy on lateral
internuclear ophthalmoplegia gaze, nystagmus in abducted
(INO) eye, normal convergence.

spastic paralysis, increased


Give 4 characteristics of an
deep tendon reflexes, +
UMN lesion.
Babinski, minor to no atrophy

Golgi tendon organs send


group Ib afferents
their signal via what nerve?

Horner's Syndrome is present


if the lesion in Brown- T1
Sequard is above what level?
How are the fibers of the
corticospinal tract laminated? arms- medial, legs-lateral
(legs/arms medial or lateral?)

How are the fibers of the


dorsal column laminated? legs-medial, arms-lateral
(legs/arms medial or lateral?)

How are the fibers of the


spinothalmic tract laminated?
cervical-medial, sacral-lateral
(sacral/cervical medial or
lateral?)

How do glucose and amino


carrier-mediated transport
acids cross the blood-brain
mechanism
barrier?
How does the hypothalamus via releasing factors (ie. TRH,
control the adenohypophysis? CRF, GnRF, etc.)

Huntington's patients
typically have what type of Chorea
movements?

If the radial nerve is lesioned, triceps reflex and


what 2 reflexes are lost? brachioradialis reflex

If you break your humerus


mid-shaft, which nerve would radial nerve
likely injure?
If you break your medial
epicondyle of the humerus,
ulnar nerve
which nerve would likely
injure?

If you break your


supracondyle of the humerus,
median nerve
which nerve would likely
injure?

If you break your surgical


neck of the humerus, which axillary nerve
nerve would likely injure?

In a lesion of the radial nerve,


what muscle is associated extensor carpi radialis longus
with wrist drop?
Intrafusal fibers are
encapsulated and make up
group Ia afferents
muscle spindles that send
their signal via what nerve?

Intrafusal fibers are


innervated by what motor gamma motor neuron
neuron?

Is Bell's palsy an UMN or a


LMN
LMN lesion?

Is the Babinski reflex (positive


or negative) when the big toe
positive (pathologic)
dorsiflexes and the other toes
fan-out?
Name 2 locations for lesions ventral white commissure and
in Syringomyelia? ventral horns

Name 3 locations for lesions dorsal columns, lateral


in Vit.B12 neuropathy corticospinal tracts, and
(Friedreich's ataxia)? spinocerebellar tracts

Thirst/waterbalance,
Adenohypophysis control,
Name 7 functions of the Neurohypophysis control,
hypothalamus? Hunger/satiety, Autonomic
regulation, Temperature
regulation, Sexual emotions.
TAN HATS

internal auditory meatus,


Name the 4 foramina that are
jugular foramen, hypoglossal
in the posterior cranial fossa?
canal, and foramen magnum.
optic canal, superior orbital
Name the 5 foramina that are fissure, foramen rotundum,
in the middle cranial fossa? foramen ovale, and foramen
spinosum.

Feeding, Fighting, Feeling,


Name the 5 functions of the
Flight, sex (F--K) [the
Limbic system?
famous 5 F's]

Name the 5 segments of the


roots - trunks - divisions -
brachial plexus in order from
cords - branches
proximal to distal.

Name the type of movement


with slow writhing Athetosis
movements (esp. the fingers)?
Name the type of movement
with sudden, jerky, Chorea
purposeless movements?

Name the type of movement


with sudden, wild flailing of Hemiballismus
one arm?

Neurons from the globus


pallidus have what action on Inhibitory
the ventral anterior nucleus?

Neurons from the striatum


have what action on the Inhibitory
globus pallidus?
Place the following in order
(from light entering the eye to retina, CN II, pretectal nuclei,
reflex). Pretectal nuclei, Edinger-Westphal nuclei, CN
pupillary constrictor muscle, III, ciliary ganglion, pupillary
retina, ciliary ganglion, constrictor muscle
Edinger-Westphal nuclei, CN
II, CN III.

Stimulation from the


paraventricular nucleus cause oxytocin
the release of what hormone?

Stimulation from the


supraoptic nucleus cause the ADH (vasopressin)
release of what hormone?

choriod plexus epithelium,


intracerebral capillary
The Blood-Brain Barrier is
endothelium, astrocytes.
formed by what 3 structures?
(First Aid says Arachnoid but
the brains say that’s a typo)
The central retinal artery is a
ophthalmic artery
branch off what larger artery?

The embryologic defect of


subclavian artery and inferior
having a cervical rib can
trunk of the brachial plexus
compress what 2 structures?

The fasciculus cuneatus


contains fibers from the upper extremities
upper or lower body?

The fasciculus gracilis


contains fibers from the lower extremities
upper or lower body?
The hippocampal formation is
connected to the mammillary
fornix
body and septal area via what
structure?

The hippocampus has input


entorhinal cortex, septal area
from what two areas?

The hippocampus has output


mammillary body, septal area
to what two areas?

The infraorbital nerve is a


CN V2
branch off what larger nerve?
The Nucleus Ambiguus has
CN IX, X, XII
fibers from what 3 CNs?

The Nucleus Solitarius has


CN VII, IX, X
fibers from what 3 CNs?

Traction or tear of the


superior trunk of the brachial Erb-Duchenne palsy (waiter's
plexus causes what tip)
syndrome?

Vertigo, ataxia, visual deficits,


and coma are caused by
posterior circle
stroke of the? (ant. circle or
post. circle)
Visual fibers from the lateral
geniculate body terminate on
Calcarine fissure
the upper and lower banks of
what fissure?

What 1 nerve root is assoc.


S1
with the achilles reflex?

What 1 nerve root is assoc.


C5
with the biceps reflex?

What 1 nerve root is assoc.


L4
with the patella reflex?
What 1 nerve root is assoc.
C7
with the triceps reflex?

What 2 areas have sensation


lateral palm/thumb and the
deficit in a lesion of the
radial 2 1/2 fingers
median nerve?

What 2 areas have sensation


medial palm and the ulnar 1
deficit in a lesion of the ulnar
1/2 fingers
nerve?

What 2 cutaneus nerves are posterior brachial cutaneous


lost in a lesion of the radial and posterior antebrachial
nerve? cutaneous
What 2 spinal roots make up
the inferior trunk of the C8, T1
brachial plexus?

What 2 spinal roots make up


the superior trunk of the C5, C6
brachial plexus?

What 2 structures pass


through the internal auditory CN VII, VIII
meatus?

What 2 symptoms are seen


difficulty flexing the arm,
with a lesion of the
variable sensory loss
musculocutaneus nerve?
What 2 symptoms are seen
weak intrinsic muscles of the
with a lesion of the ulnar
hand, Pope's blessing
nerve?

What 3 blood barriers does blood-brain, blood-gas,


the body have? blood-testis

What 3 muscles are lost in a


coracobrachialis, biceps
lesion of the
brachii, and brachialis
musculocutaneous nerve?

triceps brachii,
What 3 muscles are lost in a
brachioradialis, and extensor
lesion of the radial nerve?
carpi radialis longus
What 3 structures pass spinal roots of CN XI
through the foramen (ascending), brainstem,
magnum? vertebral arteries

What 3 structures pass CN II, ophthalmic artery,


through the optic canal? central retinal vein

Brachioradialis, Extensors of
What 4 'muscles' does the
the wrist and fingers,
radial nerve innervate?
Supinator, Triceps. (BEST)

substantia nigra pars


What 4 areas is there
compacta, globus pallidus,
decreased output in
ventral anterior nucleus,
Parkinson's?
cortex
wrist flextion, wrist
What 4 movements are
addduction, thumb
limpaired in a lesion of the
adduction, and adductiont of
ulnar nerve?
the 2 ulnar fingers

forearm pronation, wrist


What 4 movements are lost in
flexion, finger flexion, and
a lesion of the median nerve?
several thumb movements

What 4 structures pass CN IX, X, XI(descending),


through the jugular foramen? jugular vein

What 4 things do the lateral internal capsule, caudate,


striate arteries supply? putamen, globus pallidus
What 5 spinal nerves that
C5, C6, C7, C8, T1
make up the brachial plexus?

What 5 structures pass


CN III, IV, V1, VI, ophthalmic
through the supperior orbital
vein
fissure?

What 5 types of cells make up Astrocytes, Microglia,


the suportive cells of the Oligodendroglia, Schwann
CNS/PNS? cells, Ependymal cells.

What are 2 characteristics of impaired proprioception and


Tabes Dorsalis? locomotor ataxia
What are 3 clinical findings of arm hangs by the side,
the arm in Erb-Duchenne medially rotated, forearm is
palsy? pronated

What are the 2 classic causes blow to the shoulder and


of Erb-Duchenne palsy? trauma during birth

What are the 3 classic


symptoms of Horner's ptosis, miosis, anhydrosis
syndrome?

ipsi motor paralysis(spastic),


What are the 4 classic ipsi loss of dorsal column,
findings of Brown-Sequard contra loss of spinothalamic,
syndrome? ipsi loss of ALL sensation at
the level of the lesion
What are the input and output
input - mammillary body,
of the anterior nucleus of the
output - cingulate gyrus
thalamus?

input - anterior nucleus of


What are the input and output
the thalamus, output -
of the cingulate gyrus?
entorhinal cortex

input - cingulate gyrus,


What are the input and output
output - hippocampal
of the entorhinal cortex?
formation

input - hippocampal
What are the input and output
formation, output - anterior
of the mammillary body?
nucleus of the thalamus
input - hippocampal
What are the input and output
formation, output -
of the septal area?
hippocampal formation

What artery do the lateral


internal carotid artery
striate branch off of?

What artery does the anterior


inferior cerebellar artery basilar artery
branch off of?

What artery does the anterior


vertebral artery
spinal artery branch off of?
What artery does the
posterior inferior cerebellar vertebral artery
artery branch off of?

What artery does the superior


cerebellar artery branch off basilar artery
of?

What artery supplies Broca's


middle cerebral artery
and Wernicke's speech areas?

What artery supplies the


medial surface of the brain anterior cerebral artery
(foot-leg area)?
What bone do all the
foramina of the middle sphenoid bone
cranial fossa pass through?

What CN arises dorsally? CN IV trochlear

What CN is the afferent limb


CN II
of the pupillary light reflex?

What CN is the efferent limb


CN III
of the pupillary light reflex?
What CNs lie medially at the
CN III, VI, XIII (3 - 6 - 12)
brain stem?

What CNS/ PNS supportive


cell has the following
Oligodendroglia
functions: central myelin
production?

What CNS/ PNS supportive


cell has the following
Ependymal cells
functions: inner lining of the
ventricles?

What CNS/ PNS supportive


cell has the following
Schwann cells
functions: peripheral myelin
production?
What CNS/ PNS supportive
cell has the following Microglia
functions: phagocytosis?

What CNS/ PNS supportive


cell has the following
Astrocytes
functions: physical support,
repair, K+ metabolism?

What disease does Tabes


tertiary syphilis
Dorsalis result from?

What disorder results from a


Internuclear ophthalmoplegia
lesion in the medial
(INO)
longitudinal fasciculus (MLF).
What embryologic defect is
thoracic outlet syndrome by having a cervical rib.
caused by

What happens if a swinging results in pupil dialation of


light test is performed on a the defective eye as the light
Marcus Gann pupil (afferent is swung from the normal eye
pupil defect)? to the defective eye

What happens if you


both eyes constrict
illuminate one pupil in a
(consensual reflex)
normal patient?

What hypo/hyper-kinetic
disorder is marked by
decreased serum ceruloplasm Wilson's disease
and Keyser-Fleischer rings in
the eyes.
the eyes DO NOT constrict to
What is a Argyll Robertson
light, but DO accommodate
pupil?
to near objects

What is affected in a central


paralysis of the contralateral
VII lesion (lesion above the
lower quadrant
facial nucleus - UMN)?

What is affected in a
peripheral VII lesion (lesion at paralysis of the ipsilateral
or below the facial nucleus - face both upper and lower.
LMN)?

What is the common name for


Bell's palsy
a peripheral VII lesion?
What is the consequence
when your CNS stimulates the increased sensitivity of the
gamma motor neuron and the reflex arc
intrafusal fibers contract?

The striatum to the


What is the direct pathway substantia nigra pars
from the striatum to the reticularis /medial globus
cortex? pallidus to the thalamus to
the cortex (excitatory path)

What is the embryologic


tissue origin of Microglia Mesoderm
(ecto/meso/edo)?

The striatum to the lateral


globus pallidus to the
What is the indirect pathway subthalamic nucleus to the
from the striatum to the substantia nigra/medial
cortex? globus pallidus to the
thalamus to the cortex
(inhibitory pathway but still
increases the thalamic drive)
What is the lesion in Brown- hemisection of the spinal
Sequard syndrome? cord

What is the most common anterior communicating


circle of Willis aneurysm? artery

What is the name for the


small muscle fiber type that Intrafusal fibers
regulates muscle length?

What lesion produces coma? reticular activating system


What lesion produces
conduction aphasia, poor
repetition w/ poor Arcuate fasiculus
comprehension, and fluent
speech?

What lesion produces Kluver-


Bucy syndrome (hyperorality,
Amygdala (bilateral)
hypersexuality, disinhibited
behavior)?

What lesion produces motor


(expressive) aphasia with Broca's area (motor speech)
good comprehension?

What lesion produces


personality changes and frontal lobe - these are
deficits in concentration, frontal release signs
orientation, judgement?
What lesion produces sensory
Wernicke's area (associative
(fluent/receptive) aphasia
auditory cortex)
with poor comprehension?

What lesion produces spatial right parietal lobe --


neglect syndrome? contralateral neglect.

What lobe of the brain is the


frontal
Broca's area in?

What lobe of the brain is the


frontal eye movement and frontal
pupillary change area in?
What lobe of the brain is the
frontal
premotor area in?

What lobe of the brain is the


primary auditory cortex area temporal
in?

What lobe of the brain is the


frontal
principal motor area in?

What lobe of the brain is the


parietal
principal sensory area in?
What lobe of the brain is the
principal visual cortex area occipital
in?

What lobe of the brain is the


temporal
Wernicke's area in?

What midbrain structure is


important in mitigating
Basal Ganglia
voluntary movements and
making postural adjustments?

What mineral causes the


copper
pathology of Wilson's disease
What muscle depresses and
inferior rectus
extorts the eye?

What muscle elevates and


superior rectus
intorts the eye?

What muscle extorts,


elevates, and adducts the inferior oblique
eye?

What muscle fiber type makes


up the muscle bulk and
Extrafusal fibers
provides the force for
contraction?
What muscle intorts,
depresses, and abducts the superior oblique
eye?

What muscle sensor senses


tension and provides
golgi tendon organs
inhibitory feedback to alpha
motor neurons?

What muscular disorder is a


medial longitudinal fasciculus Multiple Sclerosis (MLF=MS)
syndrome associated with?

What nerve is known as the


radial nerve
great extensor nerve?
What neurotransmitter is
decrease in Parkinson's dopamine
disease

What nucleus if typically contralateral subthalamic


lesioned in hemiballismus? nucleus

What nucleus of the


hypothalamus controls suprachiasmatic nucleus
circadian rhythms?

What nucleus of the


hypothalamus controls lateral nucleus
hunger?
What nucleus of the
hypothalamus controls ventromedial nucleus
satiety?

What nucleus of the


hypothalamus controls sexual septate nucleus
emotions?

What nucleus of the


hypothalamus controls thirst supraoptic nucleus
and water balance?

What part of the


hypothalamus (ant./post.)
anterior hypothalamus
controls autonomic
regulation?
What part of the
hypothalamus (ant./post.) anterior hypothalamus
controls cooling when hot?

What part of the


hypothalamus (ant./post.)
posterior hypothalamus
controls heat conservation
when cold?

What part of the ventral


spinal cord is spared with
dorsal columns
complete occlusion of the
ventral artery?

What passes through the CN III, IV, V1, V2, VI, post-
cavernous sinus? (nerves and ganglionic SNS and the
artery Internal carotid artery
What reflex is lost in a lesion
of the musculocutaneous biceps reflex
nerve?

What structure passes


CN V3
through the foramen ovale?

What structure passes


through the foramen CN V2
rotundum?

What structure passes


through the foramen middle meningeal artery
spinosum?
What structure passes
through the hypoglossal CN XII
canal?

What symptom is seen with a


Deltoid paralysis
lesion of the axillary nerve?

What symptom is seen with a


decreased thumb function
lesion of the median nerve?

What syndrome is seen with a


lesion of the long thoracic Winged scapula
nerve?
What syndrome is seen with a
lesion of the lower trunk of Claw hand
the brachial plexus?

What syndrome is seen with a


lesion of the posterior cord of Wrist drop
the brachial plexus?

What syndrome is seen with a


Saturday night palsy
lesion of the radial nerve?

What syndrome is seen with a


Waiter's tip (Erb-Duchenne
lesion of the upper trunk of
palsy)
the brachial plexus?
What two bones do all the
foramina of the posterior temporal and occipital bones
cranial fossa pass through?

What two hypothalamic nuclei


does the posterior pituitary supraoptic nucleus and
(neurohypophysis) receive paraventricular nucleus.
neuronal projections from?

What type of fibers do the


motor
corticospinal tracts carry?

What type of fibers do the sensory - pressure, vibration,


dorsal columns carry? touch, proprioception
What type of fibers do the sensory - pain and
spinothalmic tracts carry? temperature

What type of function does


CN I have? (sensory, motor, or sensory
both)

What type of function does


CN II have? (sensory, motor, sensory
or both)

What type of function does


CN III have? (sensory, motor, motor
or both)
What type of function does
CN IV have? (sensory, motor, motor
or both)

What type of function does


CN IX have? (sensory, motor, both
or both)

What type of function does


CN V have? (sensory, motor, both
or both)

What type of function does


CN VI have? (sensory, motor, motor
or both)
What type of function does
CN VII have? (sensory, motor, both
or both)

What type of function does


CN VIII have? (sensory, motor, sensory
or both)

What type of function does


CN X have? (sensory, motor, both
or both)

What type of function does


CN XI have? (sensory, motor, motor
or both)
What type of function does
CN XII have? (sensory, motor, motor
or both)

What type of lesion is seen in combo of UMN and LMN


Amyotrophic Lateral lesions with no sensory
Sclerosis? deficit

random asymmetric lesions in


What type of lesion is seen in
mostly white matter of the
Multiple Sclerosis?
cervical region

What type of lesion is seen in


acquired LMN lesion causing
Poliomyelitis and is it genetic
flaccid paralysis
or acquired?
What type of lesion is seen in genetic LMN lesion causing
Werdnig-Hoffmann disease flaccid paralysis (aka. Floppy
and is it genetic or acquired? infant disease)

What type of molecule can


cross the blood-brain barrier Lipid-soluable/nonpolar
most easily? (lipid/nonlipid, molecules
polar/nonpolar)

What vagal nuclei controls


motor innervation to the Nucleus Ambiguus
pharynx, larynx, and upper (Motor=aMbiguus)
esophagus?

What vagal nuclei controls


Nucleus Solitarius
visceral sensory in formation
(Sensory=Solitarius)
like taste and gut distention?
What vagal nuclei sends
parasympathetic fibers to the dorsal motor nucleus of CN X
heart, lungs, and upper GI?

What would happen


temperature regulation if you lose the ability to conserve
lesioned your posterior heat
hypothalamus?

What would happen


temperature regulation if you have hyperphagia and
lesioned your ventromedial become obese
nucleus of the hypothalamus?

When is a positive Babinski a


during the first year of life
normal reflex?
Where is the lesion in a
Subthalamic nucleus
patient with hemiballismus?

Where is the lesion in Substantia nigra pars


Parkinson's? compacta

Which CN is the only nerve


that does not abut the wall in CN VI (abducens)
the cavernous sinus?

Which CNs pass through the


CN II - VI
middle cranial fossa?
Which CNs pass through the
CN VII - XII
posterior cranial fossa?

Which division of the facial


motor nucleus has duel upper division
innervation? (upper or lower)

Which thalamic nucleus has a Lateral Geniculate Nucleus


visual function? (LGB)

Which thalamic nucleus has Medial Geniculate Nucleus


an auditory function? (MGB)
Which thalamic nucleus has
Ventral Anterior Nucleus (VA)
pre-motor function?

Which thalamic nucleus has


the function of body senses Ventral Posterior Lateral
(proprioception, pressure, Nucleus (VPL)
pain, touch, vibration)?

Which thalamic nucleus has


Ventral Posterior Medial
the function of facial
Nucleus (VPM)
sensation and pain?

Which thalamic nucleus is the


Ventral Lateral Nucleus (VL)
primary motor cortex?
Which way does the head
toward the lesion -- note:
deviate in a unilateral lesion
First-Aid is wrong in the
(LMN) of CN XI? (toward or
book)
away)

Which way does the jaw


deviate in a unilateral lesion
toward the lesion
(LMN) of CN V? (toward or
away)

Which way does the patient


tend to fall in a unilateral
toward the lesion
lesion (LMN) of the
cerebellum? (toward or away)

Which way does the tongue


deviate in a unilateral lesion
toward the lesion
(LMN) of CN XII? (toward or
away)
Which way does the uvula
deviate in a unilateral lesion
away from the lesion
(LMN) of CN X? (toward or
away)

Why does the arm hang by


paralysis of shoulder
the side in Erb-Duchenne
abductors
palsy?

Why is L-dopa use for L-dopa crosses the blood-


parkinsonism instead of brain barrier while dopamine
dopamine? does not.

Why is the arm medially


paralysis of the lateral
rotated in Erb-Duchenne
rotators
palsy?
Why is the forearm pronated
loss of the biceps brachii
in Erb-Duchenne palsy?
Yes. Both hypnagogic (just
before sleep) and
Are hallucinations common in
hypnopompic (with
narcolepsy patients?
awakening) are both
common.

Sudden collapse (falls asleep)


Define cataplexy.
while awake.

Person stops breathing for at


Define sleep apnea. least 10 seconds during
sleep.

"In central sleep apnea, there


Distinguish between central is no respiratory effort. In
and obstructive sleep apnea. Obstructive sleep apnea,
there is respiratory effort
against airway obstruction."
Yes. Studies have shown a
Does narcolepsy have a
strong genetic component of
genetic component?
narcolepsy.

Does REM sleep increase or


Decreases
decrease with age?

Extraocular movements Parapontine Reticular


during REM sleep are due to Formation/Conjugate Gaze
what portion of the brain? Center

Every 90 minutes (duration


How often does REM sleep
may increase during the
occur?
night)
Name 1 neurotransmitter
change associated with Decreased Ach
Alzheimer's disease.

Name 1 neurotransmitter
change associated with Decreased Dopamine
Parkinson's disease.

Name 1 neurotransmitter
change associated with Increased Dopamine
Schizophrenia

"1. Induces production of


FFA, 17-OH corticosteroids,
Name 2 effects of stress on lipids, cholesterol, and
the body. catecholamines 2. Affects
water reabsorption, muscular
tonicity, gastrocolic reflex,
and mucosal circulation."
Name 2 neurotransmitter
Decreased NE and serotonin
changes associated with
(5-HT)
depression.

Name 2 neurotransmitter
changes associated with Decreased GABA and Ach
Huntington's disease.

1. Reduced slow-wave sleep


Name 3 changes in sleep 2. Decreased REM latency 3.
stages often found in people Early morning awakening
with depression. (important screening
question)

Name 3 possible findings in "Sleepwalking, night terrors,


non-REM sleep. and bedwetting"
"Increased/variable pulse,
rapid eye movements,
Name 4 physiological actions
increased/variable blood
found in REM sleep.
pressure, and penile/clitoral
tumescence"

"Dreaming, loss of motor


Name 5 possible findings in tone, possible memory
REM sleep. processing function, erection,
and increased brain 02 use"

"Alpha, Beta (highest


frequency, lowest amplitude),
Name 5 possible waveform
Theta, Delta (lowest
patterns seen in the various
frequency, highest
sleep/wake stages.
amplitude), Sleep spindles
with K-complexes"

"Concentration, Orientation,
Name 7 functions of the Language, Abstraction,
frontal lobe. Judgment, Motor regulation,
Mood"
Name a possible chronic
Chronic fatigue
outcome of sleep apnea.

"Obesity, loud snoring,


Name five findings associated systemic/pulmonary HTN,
with sleep apnea. arrhythmias, and possibly
sudden death."

"1. Drugs (e.g. antiHTN,


neuroleptics, SSRIs, and
What 3 things does the
ethanol) 2. Diseases (e.g.
differential diagnosis for
depression and diabetes) 3.
sexual dysfunction include?
Psychological (e.g.
performance anxiety)"

What is a common treatment Stimulants (e.g.


for narcolepsy? amphetamines)
What is a helpful mnemonic
for the order of the
corresponding waveform "At night, BATS Drink Blood."
patterns in each stage of
sleep?

What is considered the key to Serotonergic predominance of


initiating sleep? the raphe nucleus

What is narcolepsy? Person falls asleep suddenly

What is the most notable


change in function in a Lack of social judgment
frontal lobe lesion?
What is the principal
neurotransmitter involved in Ach
REM sleep?

What medication shortens


stage 4 sleep and can be Imipramine
used to treat enuresis?

What medication shortens


stage 4 sleep and is useful in
Benzodiazepines
the treatment of night terrors
and sleepwalking?

What neurotransmitter can


NE
reduce REM sleep?
What percentage of time is
0.25
spent in REM sleep?

What percentage of time is


0.05
spent in stage 1 sleep?

What percentage of time is


0.45
spent in stage 2 sleep?

What percentage of time is


0.25
spent in stage 3-4 sleep?
What phenomenon caused The EEG pattern during REM
REM sleep to be known as sleep is the same as the EEG
'paradoxical' or of a person that is awake and
'desynchronized' sleep? alert.

"What waveform pattern is


seen in a young adult who is
awake (eyes open), alert, and Beta waves
has active mental
concentration?"

What waveform pattern is


seen in a young adult who is
Alpha waves
awake but has his/her eyes
closed?

What waveform pattern is


Sleep spindles and K-
seen in a young adult who is
complexes
in deeper (stage 2) sleep?
What waveform pattern is
seen in a young adult who is Theta waves
in light (stage 1) sleep?

What waveform pattern is


seen in a young adult who is Beta waves
in REM sleep?

"What waveform pattern is


seen in a young adult who is
Delta waves
in the deepest, Non-REM
(stage 3-4) sleep? "

A person who is unable to


remember things that
Anterograde Amnesia
occurred after a CNS insult
has…?
Anterograde amnesia caused
Korsakoff's amnesia
by thiamine deficiency?

Are DT's life threatening? Yes

Are the illness production and


motivation in somatoform No
disorders consicous drives?

Bipolar I describes? manic


Bipolar II describes? hypomanic

"Distinct period of abnormally


and persistently elevated,
Define a Manic episode.
expansive, or irritable mood
lasting at least 1 week"

Discrete period of intense


fear and discomfort peaking
Define a panic disorder.
in 10 minutes with 4/5
characteristics

Define Anosognosia. being unaware that one is ill


Being unable to locate one's
Define Autotopagnosia.
own body parts

"symptoms suggest motor or


sensory neurologic or
Define Conversion disorder.
physical disorder, but tests
and PE are negative"

body seems unreal or


Define depersonalization.
dissociated

"when patterns become


inflexible and maladaptive,
Define Personality disorder causing impairment in social
or occupational functioning
or subjective distress"
"an enduring pattern of
perceiving, relating to, and
thinking about the
Define Personality trait. environment and oneself that
is exhibited in a wide range
of important social and
personal contexts"

false beliefs not shared by


other memebers of culture/
Delusions are…. subculture that are firmly
maintained in spite of
obvious proof to the contrary

distrustful and suspicious;


Describe a Paranoid
projection is main defense
Personality
mech

voluntary social withdrawl; no


Describe a Schizoid
psychosis; limited emotional
Personality.
expression
"interpersonal awkwardness,
Describe a Schizotypal
odd thought patterns and
Personality.
appearance"

Does the person who has the


"yes, they are exhibiting
phobia recognize their fear as
insight"
excessive?

Does the phobic fear interfere


yes
with normal routine?

perceptions in the absence of


Hallucinations are….
external stimuli
as odd or ecentric; cannot
How are Cluster A
develop meaningful social
personalities described?
relationships; Weird

How are Cluster B "Dramatic, emotional, or


personalities described? erratic; Wild"

How are Cluster C "Anxious and fearful,


personalities described? 'Worried'"

Recurrent-requires 2 or more
How is a major depressive depressive episodes with a
disorder characterized? symptom free interval of 2
months
"5 of the following for 2 weeks,
including (1) depressed mood or
(2) anhedonia: Sleep
How is a major depressive disturbances, Loss of Interest,
Guilt, Loss of Energy, Loss of
episode characterized?
Concentration, Change in
Appetite, Psychomotor
retardation, Suicidal ideation,
Depressed mood"

How is maladaptive pattern of 3 or more of the above signs


substance abuse defined? in 1 year

"periods of psychosis and


How is Schizophrenia
disturbed behavior lasting
described?
>6months, "

> 1 month and causes


How long does the
distress or social/
disturbance due to PSSD last?
occupational impairment
"6 separate criteria exist for
bipolar disorders with
How many criteria sets exist
combinations of manic,
for bipolar disorder?
hypomanic, and depressed
episodes"

"7; Visual, Auditory,


How many hallucination types
Olfactory, Gustatory, Tactile,
are there? Name them.
Hypnagogic, Hypnopompic"

How many heroin addicts are


"~500,000"
there in the US?

"submissive and clinging,


How would you decribe a
excessive need to be taken
dependent personality?
care of, low self-confidence"
How would you decribe an "preoccupation with order,
obsessive-compulsive? perfectionism and control"

"unstable mood and behavior;


How would you describe a
impulsive, sense of
Borderline personality?
emptiness"

"excessive emotionally,
How would you describe a
somatization, attention
histrionic personality?
seeking, sexually provocative"

"as having a disregard for and


How would you describe an
violation of rights of others,
Antisocial?
criminality"
"sensitive to rejection,
How would you describe an
socially inhibited, timid,
avoidant personality?
feelings of inadequacy"

"grandiosity; sense of
How would you desribe a entitlement, many demand
Narcissistic personality? 'top' physician/best health
care"

"If a patient consciously fakes


or claims to have a disorder
in order to attain a specific Malingering
gain, how is this behavior
described?"

"misinterpretation of actual
Illusions are…. external stimuli, ex.
Mistaking coat rack for man"
In what kind of disorder does
a person consciously create
symptoms in order to assume Factitious Disorder
a sick role and get medical
attention?

"NO, it is schedule I (ie. not


Is Heroin prescribable?
prescribable)"

Is the motivation concious in


NO
Munchausen's by proxy?

"Disorganized, Catatonic,
Name the 5 subtypes of
paranoid, Undifferentiated,
schizophrenia.
Residual"
Name the types of Cluster B "Antisocial, Borederline,
personalities. Histrionic, Narcissistic"

"3 or more of the following:


Distractibility, Insomnia,
To be a manic episode what
Grandiosity, Flight of Ideas,
characteristic behaviors must
Inc in Activity/pyschomotor
be present?
agitation, Pressured Speech,
Thoughtlessness"

Whar are the signs of "Anxiety, seizures, delirium,


barbituate withdrawl? life-threatening CV collapse"

"Irritablility, headache,
Whar are the signs of nicotine
anxiety, weight gain, craving,
withdrawl?
tachycardia"
"Ambivalence(uncertainty),
Autism(self-preoccupation
Whare are the 4 A's of
and lack of communication),
schizophrenia?
Affect(blunted), Associations
(loose)"

"development of mulitple
cognitive deficits: memory,
Whare are the characteristics apahasia, apraxia, agnosia,
of Dementia? loss of abstract thought,
behavioral/personality
changes, impaired
judgement"

"Amnesia, ataxia,
Whare are the signs of
somnolence, minor resp
Benzodiazepine intox?
depression"

"restlessness, insomina,
Whare are the signs of increased diuresis, muscle
caffeine intox? twitching, cardiac
arrhythmias"
"Euphoria, anxiety, paranoid
delusions, perception of
Whare are the signs of slowed time, impaired
Marijuana intox? jugdment, social withdrawl,
increased appetite, dry mouth
and hallucinations"

"Belligerence, impulsiveness,
fever, psychomotor agitation,
Whare are the signs of PCP vertical and horizontal
intoxication? nystagmus, tachycardia,
ataxia, homicidality,
psychosis and delirium"

What are 2 opioid


Naloxane and Naltrexone
comptetitive inhibitors?

What are some common substance use/abuse or


causes of delirium? medical illness
"Hepatitis, abscesses, OD,
What are some related hemorrhoids(reason enough
diagnoses of Heroin addicts? to shy away), AIDS, and right-
sided endocarditis"

"Palipitations, Abdominal
What are the 5 characteristics distress, Nausea, Increased
of panic? perspiration, Chest pains,
chills and choking"

"conversion, somatoform pain


What are the 6 somatoform disorder, hypochondriasis,
disorders? somatization disorder, body
dysmorphic disorder,
pseudocyesis"

"Decreased attention span


and level of arousal,
What are the characteristics disorganized thinking,
of Delirium? hallucinations, illusions,
misperceptions, disturbance
of sleep-wake cycle, cognitive
dysfxn"
What are the etiologic factors "genetics and environment,
for schizophrenia? genetics outweigh env"

"Tolerance, Withdrawl, Substance


taken in larger amounts than
intended, Persistent desire or
What are the maladaptive attempst to cut down, lots of
energy spent trying to obtain
signs of substance use?
substance, withdrawl from
responsibility, used continued in
spite of knowing the problems
that it cause"

"flat affect, social withdrawl,


What are the negative
thought blocking, lack of
symptoms of schizophrenia?
emotion"

"Hallucinations, delusions,
What are the Positive
strange behavior and loose
symptoms of Schizophrenia?
associations"
"Disinhibition, emotional
What are the signs of alcohol
lability, slurred speech,
use?
ataxia, coma, blackouts"

"Tremor, Tachycardia, HTN,


malaise, nausea, seizures,
What are the signs of alcohole
delirium tremens,
withdrawl?
tremulousness, agitation,
hallucinations"

"Psychomotor agitation,
impaired judgement,
pupillary dilation, HTN,
What are the signs of tachycardia, euphoria,
Amphetamine intoxication? prolonged wakefulness and
attention, cardiac
arrhythmias, delusions,
hallucinations, fever"

"Post-use crash, including


anxiety, lethargy, headache,
What are the signs of stomach cramps, hunger,
Amphetamine withdrawl? severe depression, dysphoric
mood, fatigue, insomnia/
hypersomnia"
What are the signs of
respiratory depression
Barbiutate intox?

What are the signs of benzo "Rebound anxiety, seizures,


withdrawl? tremor, insomnia"

What are the signs of caffeine "Headache, lethargy,


withdrawl? depression, weight gain"

"Euphoria, psychomotor
agitation, impaired judgment,
tachycardia, pupillary
What are the signs of cocaine dilation, HTN, hallucinations
intoxication? (including tactile:bugs on
skin), paranoid ideations,
angina, and sudden cardiac
death"
"Hypersomnolence, fatigue,
What are the signs of cocaine
depression, malaise, severe
withdrawl?
craving, suicidality"

"Marked anxiety and


What are the signs of LSD depression, delusions, visual
intoxication? hallucinations and
flashbacks"

"dilated pupils, lacrimation,


What are the signs of narcotic rhinorrhea, sweating,
abstinence syndrome? yawning, irritability, and
muscle aches"

"CNS depression, nausea and


What are the signs of opioid vomiting, constipation,
intoxication? pupillary constriction,
seizures"
"Anxiety, insomnia, anorexia,
sweating/piloerection(cold
What are the signs of opioid turkey), fever, rhinorrhea,
withdrawl? nausea, stomach cramps,
diarrhea, flu-like symptoms,
yawning"

"Recurrence of intoxication
symptoms due to
What are the signs of PCP
reabsorption in GI tract,
withdrawl?
sudden onset of severe,
random, homicidal violence"

"ANS Hyperactivity
What are the symptoms of (tachycardia, tremors, and
DT's(in order of appearance)? anxiety), Psychotic symptoms
(hallucinations, delusions),
confusion"

What are the symptoms of "Restlessness, insomnia,


nicotine intoxication? anxiety, arrhythmias"
What are the types of Cluster "Paranoid, Schizoid,
A personalities? Schizotypal"

What are the types of Cluster "Avoidant, Obsessive-


C personalities? Compulsive, Dependent"

What are treatment options of


systematic desensitization
phobias?

What can be confused with


depression
dementia in elderly?
complications associated with
What complication result from
anesthesia and retrograde
ECT?
amnesia

What does exposure to object


an anxiety response
of phobia evoke?

What drug is used for long


term maintenance of heroin Methadone
detox?

What else is classically "Confabulations, ie. Making it


associated with Korsakoff's? up as you go along"
What is a hallmark sign of
track marks
heroin addiction?

it is like a manic episode


except mood disturbance is
not severe enough to cause
What is a Hypomanic Episode? marked impairment in social
and/or occupational
functioning or to necessitate
hospitalization; no psychotic
features

What is a pain somatoform pain that is not explained


disorder? completely by illness

"fear that is excessive or


unreasonable, cued by the
What is a phobia?
presence or anticipation of a
specific object or entity"
What is a schizoaffective a combo of schizophrenia
disorder? and a mood disorder

What is a somatization Varitey of complaints in


disorder? multiple organ systems

milder form of bipolar lasting


What is Cyclothymic disorder?
at least 2 years

What is dementia commonly irreversible


characterized by? memory loss
What is destroyed in
Mamillary bodies(bilaterally)
Korsakoff's Amnesia?

What is drug of choice for


Lithium
bipolar?

milder form of depression


What is Dysthymia?
lasting at least 2 years

a tx option for major


depressive disorder refractory
What is Electroconvulsive to other tx. It is painless and
Therapy(ECT)? produces a seizure with
transient memory loss and
disorientation.
"misinterpretation of normal
physical findins, leading to
preoccupation with and fear
What is Hypochondriasis?
of having a serious medical
illness in spite of medical
reassurance"

What is it called when a


parent causes their child to
Munchausen's by proxy
become ill in order to receive
attention?

when a person experienced or


witnessed an event that involoved
actual or threatened death or
What is post-traumatic stress serious injury. The traumatic
event is reexperienced; person
disorder?
persistently avoids stimuli
associated with the trauma and
experiences persistent symptoms
of increas

what the symptom does for


What is primary gain? the patient's internal psychic
economy
What is retrograde amnesia a ECT-electroconvulsive
complication of? therapy(shock)

What the symptoms gets the


What is secondary gain? patient(sympathy or
attention)

What is tertiary gain? what the caretaker gets

"One or more of the following


in 1 year: Recurrent use
resulting in failure to complete
What is the criterion for dx of responsiblities, recurrent use in
substance abuse? physically hazardous situations,
recurrent legal problems,
continued use in spite of
persistent problems of use"
"maladaptive pattern of use
leading to clinically
What is the definition of
significant impairment or
substance abuse?
distress, symptoms have not
met criteria for dependence"

delusion is a disorder in the


content of thought(the actual
What is the difference
idea) where a loose
between delusions and loos
association is a disorder in
associations?
the form of thought(the way
the idea is tied together)

what is the fear of heights? acrophobia

what is the fear of marriage? gamophobia


what is the fear of open
agoraphobia
places?

what is the fear of pain? algophobia

What is the Fifth A? Auditory hallucinations

What is the inability to


remember things that Retrograde Amnesia
happened before CNS insult?
Waxing and waning level of
What is the key to delirium
conciousness that develops
diagnosis?
rapidly

"rule out delirium-patient is


What is the key to dementia alert, no change in level of
diagnosis? conciousness. More often
gradual onset. "

What is the lifetime


prevalence for Major "13% for males, and 21% for
Depressive Disorder in Males females"
and Females:

"1.5%-(males/females,
What is the lifetime
blacks/whites) presents
prevalence for schizophrenia?
earlier in men"
What is the most common
psych illness on medical and "Delirium, often reversible"
surgical floors?

What is the order of loss or


"Time, place, and Person"
orientation?

What is the response to the "intense fear, helplessness or


traumatic event? horror"

What is the trigger for DT's? alcohols withdrawl


What patient population will
Alcoholics
you see Korsakoff's?

What questions do you have Is the patient aware of him/


to answer when assessing an herself as a person? Does the
patient's orientation? patient know his/her name?

What syndrome is manifested


by a chronic history of
multiple hospital admissions Munchausen's
and willingness to receive
invasive procedures?

patient is convinced that part


Whate is body dysmorpic
of one's own anatomy is
disorder?
malformed
false belief of being pregnant
Whate is pseudocyesis? associated with objective
signs of pregnancy

"Visual(acute organic brain


syndrom), Auditory
(Schizophrenia), Olfactory(aura of
When are the halluinations psychomotor epilepsy), Gustatory
common? (rare), Tactile(DT's and Cocaine
abusers), Hypnagogic(while going
to sleep), Hypnopmpic(while
waking from sleep)"

When do DT's peak? 2-5 days after last drink

When must a painc disorder in the context of the


be dx? occurrence
"Who are more likely to be
male
antisocial, male or female?"

"Who are more likely to be


female
borderline, male or female?"

Who do you need to see to


witness caffeine withdrawl Blake Williams
approx every six weeks?

Obligation to respect pts as


Define Autonomy. individuals and to honor their
preferences in medical care
"- discussion of pertinent
information- obtaining the
"Legally, what does informed
patient's agreement to the
consent require?"
plan of care- freedom from
coercion"

Pt autonomy vs. beneficence: "If pt makes an informed


when does autonomy win decision, ultimately, the pt.
out? has the right to decide."

"- Dr. breach of duty to


patient- pt. suffers harm-
What 3 proofs are required
breach of duty causes harm
for a sucessful malpractice
Note--beyond reasonable
civil suit for neglegence?
doubt not needed, just more
likely than not"

"- potential harm to others is


serious- likelihood of harm is
What are the 4 exceptions to great- no alternative means
confidentiality? exist to warn or to protect
those at risk- Drs. Can take
steps to prevent harm"
"- pt. lacks decision-making
capacity (not letally competent)-
implied consent in an emergency-
What are the 4 exceptions to therapeutic privelege--
withholding information when
informed consent?
disclosure would severly harm the
pt or undermine informed
decision-making capacity-
waver--pt. waves "

"- pt. makes and


communicates a choice- pt. is
informed- decision remains
What are the 5 signs of a pt's stable over time- decision
decision-making capacity? consistent w/ pt's values and
goals- decision not a result
of delusions or
hallucinations"
"- LIVING WILLS--pt. directs Dr.
to withhold/withdraw life-
sustaining tx if the pt develops
What are the types of written terminal disease or enters a
persisitent vegative state-
advance directives?
DURABLE POWER OF ATTORNEY--
pt designates a surrohate to
make medical decisions in an
event pt. loses deci"

Dr. have special ethical


responsibility to act in the
What is beneficence? pt's best interest. Pt.
autonomy may conflict with
beneficience
Confidentiality respects pt.
privacy and autonomy.
Disclosure to family and
What is confidentiality? friends should be guided by
what pt. would want. Pt can
waive right to confidentiality
(i.e. to insurance co.)

"Do no harm. But, if benefits


of intervention outweigh
What is nonmaleficence? risks, pt may make an
informed decision to
proceed."

What must patients "- risks- benefits-


understand in informed alternatives, which includes
consent? no intervention"

"Incapacitated patient's prior oral


statements commonly used as
guide, but problems arise from
When is an oral directive variance in interpretation. if pt
was INFORMED, directive is
valid?
SPECIFIC, pt. MAKES A CHOICE
and decision is REPEATED over
time, the oral directive is more
valid."
At what ages does sexual
9 to 12 years of age
abuse in children peak?

At what time (minutes after "after 1 min. and 5 min.,


birth) do you run an APGAR score 0-2 in 5 categories (10
score? is perfect score)"

Define low birth weight less than 2500g

Irreversible changes of long


term deprivation of affection 6 months
occurs after how long?
Physical abuse in children
leads to how many deaths in ~3000 deaths/yr.
the U.S.?

"- healed fractures on x-ray-


cigarette burns- subdural
What are evidence of physical
hematomas- multiple
abuse in children?
bruises- retinal hemorrhage
or detachment"

What are evidence of sexual "genital/anal trauma, STDs,


abuse in children? UTIs"

"1. sexual changes: sexual


interest does NOT decrease (men:
slower erection/ejaculation,
What are the 5 changes in the longer refractory period; women:
vaginal shortening, thinning,
elderly?
dryness) 2. sleep patterns:
decreased REM, decreased slow-
wave sleep, increased sleep
latency, increase"
"- A= Apperance (color)- P=
Pulse- G= Grimace (reflex
What are the 5 components of
irritability)- A= Activity
the APGAR score at birth?
(muscle tone)- R=
Respiration"

"1. Weak 2. Wordless 3.


Wanting (socially) 4. Wary
What are the 7 effects of
(lack trust) 5. Weight loss 6.
long-term deprivation of
anaclitic depression 7.
affection?
physical illness [Hint: 5 W's
and 2 more]"

What are the development


"- group play- rides tricycle-
milestones at about 3 years
copies line or circle drawing"
old in preschool?

What are the development "- cooperative play- simple


milestones at about 4 years drawings (stick figure)- hops
old in preschool? on 1 foot"
What are the developmental
milestones at about 12-14 #NAME?
months old in infancy?

What are the developmental


"- walking- few words-
milestones at about 15
separation anxiety"
months old in infancy?

What are the developmental "- holds head up- social


milestones at about 3 months smile- Moro reflex
old in an infant? disappears"

What are the developmental


"- rolls on back- sits when
milestones at about 4-5
propped"
months old in an infant?
What are the developmental
"- stranger anxiety- sits
milestones at about 7-9
alone- orients to voice"
months old in infancy?

What are the developmental "- abstract reasoning (formal


milestones during operations)- formation of
adolescence? personality"

"- development of conscience


What are the developmental
(superego)- same-sex
milestones during the
friends- identification with
schoolage years (6-11y/o)?
same-sex parent"

"Denial-Anger-Barganing-
What are the Kubler-Ross Grieving-Acceptance, don't
dying stages? occur necessarily in this order
[Hint: Death Arrives Brining
Grave Adjustments]"
"assoc. w/ gtr. Incidence of
physical and emotional
problems. Complications
What are the risks of low birth include:- infections-
weight? respiratory distress
syndrome- necrotizing
entercolitis- persistent fetal
circulation"

"Stress:- physical illness-


What can cause regression to
punishment- birth of new
younger behavior in children?
sibling- tiredness"

prematurity or intrauterine
What causes low birth weight?
growth retardation

depression in an infant owing


to continued separation from
What is anaclitic depression? caregiver--can result in
failure to thrive. Infant
becomes withdrawn and
unresponsive
"normal bereavement
characterized by shock,
What is grief? denial, guilt, and somatic
symptoms, Typically lasts
6mo. to 1yr."

"includes excessively intense


or prolonged grief or grief
What is pathologic grief?
that's delayed, inhibited, or
denied"

What is the result of severe


long-term deprivation of death
affection?

When can a child parallel


"Toddler, 24-48 months old"
play?
When does a child achieve
"Toddler, 24-36 months old"
core gender identity?

When does a child achieve


"Toddler, 12-24 months old"
object permanence?

When does a child achieve


"Toddler, 18-24 months old"
rapprochement?

When does a child become "Preschool, 30-36 months


toilet trained? old"
When is adolescence for boys "Boys: 13 years old Girls: 11
and for girls? years old"

Who is usually the abuser in


female primary caregiver
physical abuse in children?

Who is usually the abuser in "known to victim, usually


sexual abuse in children? male"

Case-control studies are


Retrospective (case control)
often?
Characteristics of a normal Gaussian = Bell Shaped
statistical distribution? ( mean=median=mode )

Country with highest divorce


U.S.
rate

Peaks on either side of the


Define a bimodal distribution
median

Pooling data from several


Define a Meta-analysis studies to achieve greater
statistical power
Asymmetry with the tail on
Define a negative skew the left
(mean<median<mode)

Asymmetry with the tail on


Define a positive skew the right
(mean>median>mode)

The trueness of test


Define Accuracy
measurements

Hypothesis that there is some


Define Alternate Hypothesis
difference
Define Coefficient of r^2 (Correlation coefficient
Determination squared)

Always between -1 and 1.


Define Correlation coefficient
Absolute value indicates the
(r )
strength of correlation.

Number of true negatives /


Define Negative Predictive
number that tested neg. for
Value
disease

"Number of true positives /


Define Positive Predictive number that tested pos. for
disease or the prob. Of
Value
having a condition, given a
pos. test"
"The consistency of a test
Define Precision (reliability), absence of
random error"

Define Primary Disease "Prevent occurrence, e.g.,


Prevention vaccination"

Disease risk in exposed


group / disease risk in
Define Relative Risk
unexposed group; a/a+b / c/
c+d

Reproducibility of a test;
Define Reliability repeat measurements are the
same
Define Secondary Disease "Early detection of disease,
Prevention e.g., Pap smear"

Standard Error of the Mean;


Define SEM standard deviation / square
root of n (sample size)

Number of true positives / all


Define sensitivity
people with disease

Number of true negatives /


Define specificity number of all people w/o the
disease
"Reduce disability form
Define Tertiary Disease
disease, e.g. insulin for
Prevention
diabetics"

Probability of rejecting a null


Define the Power of a study
hypothesis when it is false

Whether a test truly measures


Define Validity what it purports to measure;
the appropriateness of a test

Do divorcees remarry
Yes
frequently?
How do disease prevalence
Higher prevalence = Higher
and positive predictive value
Positive Predictive Value
relate?

How do prevalence and


P>I for chronic diseases; P=I
incidence relate to disease
for acute diseases
length?

How do SEM and Standard "SD > SEM; as n increases,


Deviation relate? SEM decreases"

How do you measure the


'power' of a study or the
1-beta
probability that the study will
see a difference if one exists?
"It is the probability of
How does alpha relate to the
making a Type I error, is
Type I error?
equal to p (p is usually <.05)"

How does beta relate to the Beta is the probability of


Type II error? making a Type II error

How many people >65 "35,000,000 (approx. 13%)"

"If the 95% confidence interval


for OR of RR includes 1, what That the study is inconclusive
does this mean?"
In what age group will the
greatest population increase Those >85
be seen in?

Increasing sample size will


affect the Power of a study By increasing the Power
how?

Is divorce related to
No
industrialization?

Is HIV positivity a reportable


No
disease?
"Teenage marriages, Mixed
Marriages at high risk religions, Low socio-
economic status"

Memory aid for Medicare/ MedicarE=Elderly;


Medicaid MedicaiD=Destitute

Preventive services needed "Influenza, pneumococcal


for Alcoholism immunizations; TB test"

Preventive services needed


"Eye, foot exams; Urine test"
for Diabetes
Preventive services needed "HIV, TB tests; hepatitis
for Drug Abuse immunization"

Preventive services needed "HIV, Hep B, syphilis,


for High-risk sexual behavior gonorrhea, chlamydia tests"

"Preventive services needed


for Homeless, Refugee, or TB test
Immigrant"

Preventive services needed


Blood glucose test
for Obesity
Random error yields poor? Precision

Relative Risk is used for what


Cohort
kind of study?

Systematic error yields poor? Accuracy

U.S. population in 2000 "300,000,000"


"Unlike specificity and
sensitivity, what are Prevalence of disease in the
predictive values dependent population
on?"

"White, male, alone, prior


attempts, presence and
What are risk factors for lethality of plan, medical
suicide? illness, alcohol or drug use,
on 3 or more prescription
meds."

"Injuries, cancer, congenital


What are the leading causes
anomalies, homicide, heart
of death in AGE 1-14?
disease"

What are the leading causes "Injuries, homicide, suicide,


of death in AGE 15-24? cancer, heart disease"
What are the leading causes "Cancer, heart disease,
of death in AGE 25-64? injuries, stroke, suicide"

What are the leading causes "Heart disease, cancer,


of death in AGE 65+? stroke, COPD, pneumonia"

"Congenital anomalies, SIDS,


What are the leading causes short gestation, respiratory
of death in INFANTS? distress syndrome, maternal
complications during
pregnancy"

"Dilation and curettage,


What are the most common hysterectomy, tonsillectomy,
sterilization, hernia repair,
surgeries?
oophorectomy, cesarean
section, cholecystectomy"
"AIDS, chickenpox,
gonorrhea, hepatitis A and B,
What are the reportable
measles, mumps, rubella,
diseases?
salmonella, shigella, syphilis,
and tuberculosis"

The difference between two


What does a t-test check?
means (Mr. T is mean)

Variance of 3 or more
What does an ANOVA
variables (Analysis of
analyze?
Variance)

"Sex (male), Age, Depression,


Previous attempt, Ethanol,
What does SAD PERSONS
represent? Rational thought, Sickness,
Organized plan, No spouse,
Social support lacking"
1)The total # of endpoints
What factors influence the experienced by a population;
Power of a study? 2) Difference in compliance
between treatment groups

Observational study. Sample


chosen based on presence or
What is a case-control study?
absence of disease. Info
collected about risk factors.

Experimental study.
What is a Clinical trial? Compares benefit of 2 or
more treatments.

Observational study. Sample


chosen based on presence or
What is a Cohort study? absence of Risk Factors.
Subjects followed over time
for disease development.
(Framingham study)
What is a memory key for
SAD PERSONS
suicide risk factors?

"Hypothesis of no difference,
What is a Null Hypothesis? e.g., no assoc. between
disease and risk factor"

Cannot overcome limitations


What is a pitfall of Meta-
of individual studies or bias
analysis?
in study selection

Stating that an effect of


What is a Type I error (alpha)? difference exists when one
really does not
Stating that there is not an
What is a Type II error (beta) effect or difference when
actually there is

Approximates relative risk


What is an Odds Ratio used
when prevalence is not too
for?
high; OR = ad/bc

To check differences between


What is Chi-Square test used 2 or more percentages or
for? proportions of categorical
outcomes

What is desirable for


High specificity
confirmatory tests?
What is desirable for High sensitivity is desirable
screening tests? for a screening test

Number of new cases in a


What is incidence?
population per unit time

Fed. And State assistance for


What is Medicaid? those on welfare or who are
indigent

What is Medicare Part A? Hospital related


What is Medicare Part B? Supplemental

What is Medicare? Fed. Program for the Elderly

Total number of cases in a


What is prevalence? population at a given time
(incidence x disease duration)

What is the False Negative


1-sensitivity
Ratio?
What is the False Positive
1-specificity
Ratio?

What is the highest quality


Clinical Trial
study?

The probability of making a


What is the p value?
Type I error.

What percent of medical costs


30%
will those >35 incur?
What type of studies are Odds
Retrospective (case control)
Ratios used for?

When do divorces peak? During the 2nd to 3rd year.

Which sex has the most


Females
surgeries?

Are intelligence tests


Objective
objective or projective tests?
Unacceptable feelings and
Define acting out. thoughts are expressed
through actions

Guilty feelings alleviated by


Define altruism. unsolicited generosity toward
others

Learning in which a natural


response is elicited by a
Define classical conditioning. conditioned stimulus that
previously was presented in
conjunction with an
unconditioned stimulus

Avoidance of awareness of
Define denial.
some painful reality
Process whereby avoided
ideas and feelings are
Define displacement.
transferred to some neutral
person or object

"Temporary, drastic change in


personality, memory,
Define dissociation. consciousness, or motor
behavior to avoid emotional
stress"

automatic and unconscious


Define ego defenses. reactions to phychological
stress

Partially remaining at a more


Define fixation.
childish level of development
Appreciating the amusing
nature of an anxiety-
Define humor.
provoking or adverse
situation

Modeling behavior after


Define identification.
another person

Separation of feelings from


Define isolation.
ideas and events

Learning in which a particular


Define operant conditioning. action is elicited because it
produces a reward
An unacceptable internal
Define projection. impulse that is attributed to
an external source

"Proclaiming logical reasons


for actions actually performed
Define rationalization.
for other reasons, usually to
avoid self-blame"

Process whereby a warded-


off idea or feeling is replaced
Define reaction formation.
by an (unconsciously derived)
emphasis on its opposite

Turning back the


maturational clock and going
Define regression.
back to earlier modes of
dealing with the world
Involuntary withholding of an
Define repression. idea or feeling from
conscious awareness.

Process whereby one replaces


an unacceptable wish with a
course of action that is
Define sublimation.
similar to the wish but does
not conflict with one's value
system.

Voluntary (unlike other


defenses) withholding of an
Define suppression.
idea of feeling from
conscious awareness

Pavlov's dogs (ringing of a


Give an example of classical
bell provoked salivation in
conditioning.
dogs)
Give an example of A person gets upset when a
continuous reinforcement vending machine doesn't
schedule. work

Give an example of negative a mouse presses a button to


reinforcement of opperative avoid shock (do not confuse
conditioining. with punishment)

Give an example of positive


a mouse presses a button to
reinforcement of opperative
get food
conditioning.

Give an example of remembering your phone


preconscious topography. number
Give an example of variable
A person continuing to play a
ratio reinforcement
slot machine at a casino
schedules.

Is IQ testing more highly


correlated with genetics or School achievement
school achievement?

"-costly-lengthy-intensive-
Name 4 characteristics of
places great demands on the
psychoanalysis.
patient"

What are Freud's three


"-Id-Superego-Ego"
structures of the mind
"-Sense of Impotence
(powerlessness)-Sense of
What are the four factors in Guilt-Sense of Anger-Sense
hopelessness? of loss/Deprivation leading to
depression (Mnemonic
IGAD!)"

"Acting out, Disassociation,


Denial, Displacement,
What are the immature ego Fixation, Identification,
defenses? (12) Isolation, Projection,
Rationalization, Reaction
formation, Regression,
Repression"

"-Sublimation-Altruism-
What are the MATURE ego Suppression-Humor
defenses? (4) (Mneumonic: Mature women
wear a SASH)"

What are the two most


famous forms of intelligence Stanford-Binet and Wechsler
testing?
What are two factors with
Genetics and school
which IQ scores are
achievement
correlated?

What is an example of acting


Temper tantrums
out?

What is an example of Mafia boss makes large


altruism? donation to charity

A common reaction in newly


What is an example of denial. diagnosed AIDS and cancer
patients
What is an example of Mother yells at child because
displacement? she is angry at her husband

Extreme forms can result in


What is an example of multiple personalities
dissociation? (dissociative identity
disorder).

What is an example of
Men fixating on sports games
fixation?

What is an example of Nervous medical student


humor? jokes about the boards
What is an example of Abused child becomes an
identification? abuser

Describing murder in graphic


What is an example of
detail with no emotional
isolation?
response

A man who wants another


What is an example of
woman thinks his wife is
projection?
cheating on him

"Saying the job was not


What is an example of
important anyway, after
rationalization?
getting fired"
What is an example of A patient with libidinous
reaction formation? thoughts enters a monastery

"Seen in children under stress


What is an example of (eg., bedwetting) and in
regression? patients on dialysis (eg.,
crying)"

What is an example of Aggressive impulses used to


sublimation? succeed in business ventures

Choosing not to think about


What is an example of
the USMLE until the week of
suppression?
the exam
the removal of an aversive
What is negative
stimulus so as to increase
reinforcement?
behavior

What is positive the desired reward which


reinforcement? produces an action

What is the basic mechanism


Repression
underlying all ego defenses?

To make the patient aware of


What is the central goal of
what is hidden in his/her
Freudian psychoanalysis?
unconscious
What is the IQ criteria for IQ lower than 70 (or 2
diagnosis of mental standard deviations below the
retardation? mean)

What is the topographical


term used in psychoanalysis
Unconscious
to describe what you are not
aware of?

What is the topographical


term used in psychoanalysis
used to describe what you are Preconscious
able to make conscious with
effort?

What is the topographical


term used in psychoanalysis
Conscious
used to describe what you are
aware of?
What number is defined as
100 (with a standard
the mean for standard IQ
deviation of 15)
testing?

"What term fist described by


Freud is used to refer to
repressed sexual feelings of a
child for the opposite-sex Oedipus complex
parent, accompanied by
rivalry with same-sex
parent?"

What term is used to describe


the form of insight therapy
developed by Freud which is Psychoanalysis
often used for changing
chronic personality problems?

Bridge and mediator between


What thought structures is the unconscious mind and
the Ego responsible for? the world (Think-Deals with
conflict)
What thought structures is
" - Primal urges-sex-
the Id responsible for? (3
agression (Think-'I want it')"
things)

What thought structures is " - Moral values-conscience


the Superego responsible for? (Think-'You know you cant
(2 things) have it')"

What type of behavior behavior which shows the


requires a continuous most rapid extinction when
reinforcement schedule? not rewarded

What type of behavior behavior which shows the


requires a variable ratio slowest extinction when not
reinforcement schedule? rewarded
Which is the conditioned
(learned) stimulus in Pavlov's ringing bell
experiment?

Which is the natural response


salivation
in Pavlov's experiment?

Which is the unconditioned


stimulus in Pavlov's food
experiment?
The amino acid is covalently
At which end of the tRNA is
bound to the 3' end of the
the aa bound?
tRNA.

Can RNA polymerase initiate


Yes.
chains?

Substituting purine for purine


Define transition.
or pyrimidine for pyrimidine.

Substituting purine for


Define transversion.
pyrimidine or vice versa.
Accurate base pairing is
required only in the first 2
nucleotide positions of an
Define tRNA wobble. mRNA codon, so codons
differing in the 3rd 'wobble'
position may code for the
same tRNA/amino acid.

Origin of replication: continuous DNA


synthesis on leading strand and
discontinuous (Okazaki fragments) on
lagging strand. Primase makes an RNA
Describe DNA replication. primer on which DNA polymerase can
initiate replication. DNA polymerase
reaches primer of preceding fragment;
5'®3' exonuclease activity of DNA
polymerase I degrades RNA primer;
DNA ligase seals;

3'->5' exonuclease activity of


DNA polymerase 'proofreads'
Describe DNA replication each added nucleotide. DNA
(continued answer) topoisomerases create a nick
in the helix to relieve
supercoils

Less condensed (vs.


Describe Euchromatin Heterochromatin),
transcriptionally active
Condensed, transcriptionally
Describe Heterochromatin
inactive

1) Purines (A,G) have 2 rings.


Describe key structural
2) Pyrimidines (C,T,U) have 1
differences between
ring 3) Guanine has a ketone.
nucleotides.
4) Thymine has a methyl

Excision repair-specific
glycosylase recognizes and
removes damaged base.
Describe single-strand, Endonuclease makes a break
several bases to the 5' side.
excision repair.
Exonuclease removes short
stretch of nucleotides. DNA
polymerase fills gap. DNA ligase
seals.

Eukaryotic genome has


Describe the difference
multiple origins of
between Eukaryotic Vs.
replication. Bacteria, viruses
Bacterial, viral and plasmid
and plasmids have only one
origin of replications
origin of replication.
Eukaryotes have 3 different
RNA polymerases ('I, II, III
Describe the main difference
synthesize RMT') and
in eukaryotic and prokaryotic
prokaryotes have 1 RNA
synthesis of RNA.
polymerase (which makes all
3 kinds of RNA).

Introns are precisely spliced out


of primary mRNA transcripts. A
Describe the method by lariat-shaped intermediate is
which introns are removed formed. Small nuclear
ribonucleoprotein particles
from primary mRNA (snRNP) facilitate splicing by
transcript. binding to primary mRNA
transcripts and forming
spliceosomes.

Describe the number of G-C bond (3 H-bonds) A-T


bonds per purine-pyrimidine bond (2 H-bonds G-C bond is
pair. Which is stronger? stronger

Condensed by (-) charged


DNA looped around (+)
Describe the structure of charged histones
Chromatin. (nucleosome bead). H1 ties
the nucleosome together in a
string (30 nm fiber)
75-90 nucleotides, cloverleaf
form, anticodon end is opposite
3' aminoacyl end. All tRNAs both
eukaryotic and prokaryotic, have
Describe tRNA structure. CCA at 3' end along with a high
percentage of chemically
modified bases. The amino acid
is covalently bound to the 3' end
of the tRNA.

Does RNA polymerase have a


No.
proof reading function?

How do purines and


Purines and pyrimidines pair
pyrimidines interact,
(A-T, G-C) via H-bonds
molecularly?

RNA polymerase II opens DNA


How does RNA polymerase II at promoter site (A-T rich
open DNA? upstream sequence- TATA
and CAAT)
1) Capping on 5' end (7-
How is the original RNA
methyl G) 2)Polyadenylation
transcript processed in
on 3' end ( =200 As) 3)
eukaryotes? (3)
Splicing out of introns

In eukaryotes, what must


Only processed RNA is
occur before an newly
transported out the nucleus
synthesized RNA transcript
of eukaryotes.
leaves the nucleus?

Name 3 types of RNA 1) mRNA 2) rRNA 3) tRNA

Name the charged histones


around which (-) charged
H2A, H2B, H3, H4 histones
DNA loops (nucleosome
core).
Name the enzyme responsible
for the synthesis of RNA in RNA polymerase
prokaryotes.

1) specific glycosylase. 2)
Name the enzymes involved endonuclease. 3)
in ss-DNA repair. (5) exonuclease. 4) Dan
polymerase. 5) DNA ligase.

Name the enzymes RNA polymerase I RNA


responsible for the synthesis polymerase II RNA
of eukaryotic RNA. polymerase III

Exons contain the actual


What are exons? genetic information coding
for a protein
What are four types of
1) Silent 2) Missense 3)
mutations that can occur in
Nonsense 4) Frame shift
DNA?

Introns are intervening


What are introns?
noncoding segments of DNA

1) Unambiguous 2)
What are the four features of
Degenerate 3) Commaless,
the Genetic Code?
non-overlapping 4)Universal

What atoms link aa in a


Amino acids are linked N to C
protein chain?
What changes occur in DNA In mitosis, DNA condenses to
structure during mitosis? form mitotic chromosomes

What codon sequence is All tRNAs, both eukaryotic


found at the 3' end of all and prokaryotic, have CCA at
tRNAs? 3' end.

5' &gt; 3'. Remember that the


5' of the incoming nucleotide
What direction is DNA bears the triphosphate
synthesized in? (energy source for the bond).
The 3' hydroxyl of the
nascent chain is the target.

Protein synthesis also


What direction is protein
proceed in the 5' to 3' (5' &gt;
synthesized in?
3')
5' &gt; 3'. Remember that the
5' of the incoming nucleotide
What direction is RNA bears the triphosphate
synthesized in? (energy source for the bond).
The 3' hydroxyl of the
nascent chain is the target.

AUG codes for methionin,


which may be removed before
What does the AUG mRNA translation is completed. In
sequence code for? prokaryotes the initial AUG
codes for a formyl-methionin
(f-met).

What does the P in P-site


P-site: peptidyl; A-site:
stand for? What does the A in
aminoacyl;
A-site stand for?

What does the statement, 'the


The code is non-overlapping.
genetic code is commaless'
The exception are some
mean? What is the exception
viruses
to this rule?
What does the statement, 'the
More than one codon may
genetic code is degenerate'
code for the same amino acid
mean?

What does the statement, 'the


Each codon specifies only one
genetic code is unambiguous'
amino acid
mean?

The same code is used in all


What does the statement, 'the
lifeforms. The exceptions are
genetic code is universal'
1. mitochondria, 2.
mean? What are the
archaeobacteria, 3.
exceptions (4)?
Mycoplasma, 4. some yeasts

Aminoacyl-tRNA synthetase.
This enzyme (one per aa, uses
What enzyme is responsible ATP) scrutinizes aa before and
for 'charging' tRNA? How does after it binds to tRNA. If
it work? incorrect, bond is hydrolyzed
by synthetase. The aa-tRNA
bond has energy for
formation of peptide bond.
What inhibits RNA polymerase alpha-amanitin inhibits RNA
II? polymerase II

Mutation results in a different


What is a conservative aa encoded, but that new aa
missense mutation? is similar in chemical
structure to the original code

What is a frameshift A change in DNA resulting in


mutation? What is usually the misreading of all nucleotides
effect on the encoded downstream. Usually results
protein? in a truncated protein.

Mutation results in a different


What is a missense mutation?
aa encoded.
A change in DNA resulting in
What is a nonsense mutation?
an early stop codon.

Site where RNA polymerase


and multiple other
What is a promoter? transcription factors bind to
DNA upstream from gene
locus.

Mutation results in the same


What is a silent mutation?
aa encoded. Often the base
What usually causes a silent
change is in the 3rd position
mutation?
of the codon

snRNP = small nuclear


ribonucleoprotein. snRNPs
What is a snRNP? What is its
function? facilitate splicing by binding
to primary mRNA transcripts
and forming spliceosomes.
Stretch of Dan that alters
gene expression by binding
transcription facts. May be
What is an enhancer? located close to, far from, or
even within (an intron) the
gene whose expression it
regulates.

The discontinuous DNA


What is an Okazaki fragment? synthesized on the lagging
strand during DNA replication

hnRNA = heterogeneous
What is hnRNA? nuclear RNA The initial RNA
transcript is called hnRNA

Aminoacyl-tRNA synthetase
What is responsible for the
and binding of charged tRNA
accuracy of amino acid
to the codon are responsible
selection during peptide
for accuracy of amino acid
synthesis?
selection.
What is the broad
Purines (A, G) and Pyrimidines
classification of nucleotides?
(C, T, U)
(2)

hnRNA = the initial RNA


What is the difference
transcript mRNA = capped
between hnRNA and mRNA?
and tailed transcript

What is the difference Uracil found in RNA Thymine


between thymine and uracil? found in DNA

What is the function of DNA


DNA ligase seals synthesized
ligase during DNA
DNA into a continuous strand
Replication?
5'®3' exonuclease activity of
DNA polymerase I degrades
What is the function of DNA
RNA primer; 3'®5'
polymerase during DNA
exonuclease activity of DNA
Replication? (2)
polymerase 'proofreads' each
added nucleotide.

What is the function of DNA DNA topoisomerases create a


topoisomerase during DNA nick in the helix to relieve
Replication? supercoils

Primase makes an RNA


What is the function of primer on which DNA
primase in DNA Replication? polymerase can initiate
replication.

What is the function of RNA RNA polymerase I makes


polymerase I? rRNA
What is the function of RNA RNA polymerase II makes
polymerase II? mRNA

What is the function of RNA RNA polymerase III makes


polymerase III? tRNA

What is the mRNA initiation


AUG, or rarely GUG
codon?

What is the mRNA stop UGA (U Go Away) UAA (U Are


codons? (3) Away) UAG (U Are Gone)
A mischarged tRNA (bound to
What is the result of
wrong aa) reads usual codon
'mischarged' tRNA?
but inserts wrong amino acid.

What is the role of


Endonuclease makes a break
endonuclease in ss-DNA
several bases to the 5' side.
repair?

What is the role of excision


Recognizes and removes
repair-specific glycosylase in
damaged base.
ss-DNA repair

What is the role of


Exonuclease removes short
exonuclease in ss-DNA
stretch of nucleotides.
repair?
If incorrect, the aa-tRNA
What prevents an incorrect
bond is hydrolyzed by
aa-tRNA pairing?
aminoacyl-tRNA synthetase.

H1 ties the nucleosome


What role does histone H1
together in a string (30nm
play in chromatin structure?
fiber)

The aa-tRNA bond has energy


What supplies the energy for
for formation of peptide
formation of peptide bond?
bond.

Promoter mutation commonly


What would most likely be the
results in dramatic decrease
result of a mutation of the
in amount of gene
promoter sequence?
transcribed.
ATP is used in tRNA charging,
When is ATP used in protein
whereas GTP is used in
synthesis? When is GTP used
binding of tRNA ribosome
in protein synthesis?
and for translocations.

If both strands are damaged,


repair may proceed via
When is recombination
recombination with
involved in DNA repair?
undamaged homologous
chromosome.

Where does RNA processing RNA processing occurs in the


occur in eukaryotes? nucleus.

Which is the largest type of


mRNA (massive)
RNA?
Which is the most abundant
rRNA (rampant)
type of RNA?

Which is the smallest type of


tRNA (tiny)
RNA?

Codons differing in the 3rd


Which nucleotide position in
'wobble' position may code
the codon has room for
for the same tRNA/amino
'wobble'?
acid

Electrophorese RNA on a gel


transfer to a filter expose
How do you do a Northern
Blot? filter to a labeled DNA probe
visualize the DNA probe
annealed to the desired RNA
Electrophorese DNA on a gel
transfer to a filter and
How do you do a Southern denature the DNA expose to
Blot? a labeled DNA probe
visualize probe annealed to
desired DNA fragment

Separate protein by
electrophoresis transfer to a
How do you do a
filter expose to a labeled DNA
Southwestern blot?
probe visualize DNA bound to
desired protein

Separate protein by
How do you do a Western electrophoresis transfer to a
Blot? filter expose to a labeled
antibody visualize Ab bound
to desired protein

1. Heat DNA to denature. 2.


Cool DNA and let the primers
aneal. 3. Heat-stable
How do you do PCR? (4 steps)
polymerase replicates DNA
following each premer 4.
Repeat
SCID, Lesh-Nyhan, CF,
familial hypercholesterolemia
What are some genetic retinoblastoma, sickle cell, B-
diseases detectable by PCR? thalassemia, hemophilia A
(11) and B, von Willebrand's dz,
lysosomal dz, and glycogen
stroage dz

What gene is involved in


CFTR
cystic fibrosis?

What gene is involved in


familial LDL-R
hypercholesterolemia?

What gene is involved in


HGPRT
Lesh-Nyhan syndrome?
What gene is involved in
Rb
retinoblastoma

What gene is involved in


adenosine deaminase
SCID?

What gene is involved in


? globin gene
Sickle cell and ?-thal?

Rapid lab test in which an


antibody or an antigen
(usually collected from a
What is an ELISA (enzyme patient) is exposed to an Ag
linke immunosorbant assay)? or Ab liked to to an enzyme.
A positive test results in a
Ag-Ab match and is usually
indicated by a color change
Lab procedure used to
What is PCR? synthsize many copies of a
desired fragment of DNA

Glucose-6-phosphatase
Von Geirke's disease is a deficiency; also known as
result of? Type I Glycogen Storage
disease

A build up of sphingomyelin
and cholesterol in
reticuloendothelial and Niemann-Pick disease
parenchymal cells and tissues
is found in what disease

Osteogenesis
imperfecta;disease of
A child is born with multiple abnormal collagen synthesis
fractures and blue sclera what resulting in fractures and
is the diagnosis translucent Conn tiss over
chorioid causing the blue
sclera
A congenital deficiency of Albinism, can't synthesize
tyrosinase would lead to melanin from tyrosine

A patient presents with


cataracts,
hepatosplenomegaly, and Galactosemia
mental retardation, what is
the Dx?

A patient presents with


corneal clouding and mental
retardation that is, based on
family history, inherited in an Hurler's syndrome
Autsomal recessive pattern,
you impress your intern with
a Dx of

A patient presents with 1.


Hyperextensible skin 2.
Tendency to bleed 3. Ehlers-Danlos syndrome
Hypermobile joints you
astutely Dx them with
Absence of
Galactosylceramide Beta- accumulation of
galactosidase leads to galactocerebroside in the
thebuild up of what brain; Krabbe's disease
compound in what disease

Absence of hexosaminidase A
GM2-ganglioside
results in the acumulation of
accumulation; Tay-Sachs
what molecule that is
disease
characteristic of what disease

albinism increase risk of


skin cancer
developing what

Autosomal dominant defects


will effect what members of a male and female
family
Autosomal recessive
disorders often result in what enzyme deficiencies
kind of defect/deficiencie?

Autosomal recessive
disorders usually effect how usually only one generation
many generations in a family?

Bloom's syndrome is
sensitivity to radiation as a
characterized by sensitivity to
result of a DNA repair defect
what as a result of

Creatine and Urea are both


Arginine
made from?
Defects in structural genes
often follow what pattern of Autosomal dominant
inheritance?

sulfatide in the brain, kidney,


Defiency of arylsulfatase A liver, and peripherla nerves.
results in the accumulation of Characteristic of
what molecule where Metachromatic
Leukodystrophy

when differences in
phenotype depend on
define genetic imprinting
whether the mutation is of
paternal or maternal origin

when not all individuals with


define incomplete penetrance a mutant genotype show the
mutant phenotype
the tendency for certain
alleles at two linked loci to
define Linkage Disequilibrium occur together more often
thatn expected by chance, as
measured in a population

one gene has more than one


effect on an individual's
define pleiotropy phenotype, autosomal
dominant defects are oftne
pleiotropic

nature and severity of the


Define variable expression phenotype varies from one
individual to another

Fanconi's anemia is caused by


cross-linking agents
what typr of agents
Ganglioside is made up of Ceramide + oligosacharide +
what sialic acid

Gaucher's disease is caused


Beta-glucocerebrosidase
by a deficiency of

Glucocerebroside
accumulation in the brain,
Gaucher's disease
liver, spleen, and bone
marrow are characteristic of

Hglycine is used to make Porphyrin which is then used


what important compound to make Heme
Histamine is synthesized
Histidine
form what compound

Purine salvage pathway. ADA


normal converts adenosine to
How does adenosine inosine without it ATP &amp;
deaminase defiency cause dATP build up inhibiting
SCID ribonucleotide reductase
which prevents DNA
synthesis lowering
lymphocyte production

X-linked recessive; increase


How is Lesch-Nyhan in uric acid production.
syndrome inheritied andwhat Retardation, self mutalation,
is the result and symptoms aggression, hyperuricemia,
gout, and choreathetosis

Hunter's syndrome is
deficiency of iduronate
characterised by what
sulfatase; X-linked recessive
biochemical problem and how
mild form of Hurler's
is it inherited
In ataxia-telangiectasia DNA
damage caused by what X-rays
source cannot be repaired

hemolytic anemia;oxidizing
in G6PD deficiency the
agents( fava beans,
decrease in NADPH can lead
sulfonamides, primaquine)
to _____ if exposed to _____
and antituberculosis.

In PKU what builds up and phenyalanine builds upe


what can be found in the leading to phenylketones in
urine the urine

In PKU, what amino acid


tyrosine.
becomes essential
in the Hardy-Weinberg
p and q are each separate
equation, what are the p and
alleles; 2pq = heterozygote
q and what is 2pq

no male to male transmission


is characteristic of what type X-linked recessive
of genetic disorders?

Pompe's disease is caused by lysosomal alpha-1,4-


a defect in? glucosidase defiency

Sickle cell anemia is caused AR single missense mutation


by what defect and what is in the beta globin; 1:400
it's prevalence blacks
recurrent painful crisis and
Siclkle cell anemia patients
increased susceptibility to
often present with
infections

Skin sensitivity to UV light


secondary to a DNA repair
xeroderma pigmentosum
defect is characteristic of
what?

Sphingosine + fatty acid


ceramide
yields

the COL1A gene mutation is


Osteogenesis
associated with what disease
imperfecta;dominant negitive
and what type of mutation is
mutation
this
The main defect in Ataxia-
DNA repair defect
telangiectasi is a ___

The most common form of


abnormal Collagen Type I
Osteogenesis imperfecta has
synthesis;Autosomal
what genetic problem and
dominant
inheritance

the transporter for what


COLA: Cys,Ornithine, Lysine
amino acids is defective in
and Arginine
cystinuria

Thymidine dimers are formed


by exposure of DNA to UV
light, are the dimers formed dimers are on same side
on the opposing strands of a
DNA molecule or on the same
side?
Tryptophan can be used to
Niacin, 'Serotonin, melatonin
make what three chemicals

deficiency of debranching
Type III Glycogen storage
enzyme alpha-1,6-
disease is a defiency of ?
glucosidase

Lactase defiency, avoid diary


What is the cause Tx and
products or add lactse pills to
symptoms of Lactase
diet symptoms bloating,
intolerance?
cramps, osmotic diarrhea

none, mitochondrial
What % of kids born to father myopathies are inherited
with mitochondrial myopathie from mitochondria which is
will be effected only inherited from the
mother
What are the clinical signs of optic atrophy, spasticity, early
Krabbe's disease death

What are the components of Ceramide + glucose/


Cerebroside galactose

What are the components of Ceramide +


Sphingomyelin phosphorylcholine

What are the components of


serine + palmitate
sphingosine
What are the findings and
neurologic defects; increase
treatment of pyruvate
intake of ketogenic nutrients
dehydrogenase

increased glycogen in skeletal


muscle due to a Glycogen
What are the findings in
phosphorylase defiency
McArdles's disease and what
strenuous exercise cause
is the problem
myoglobinuria and painful
cramps

Mental retardation, fair skin,


What are the findings in PKU eczema, musty body odor Tx.
and what is the treatment Decreasee phenylalanin
(nutrasweet) and increase
tyrosine

Cardiomegaly and systemic


findings, leading to early
What are the findings in
death. Pompe's trashes the
Pompe's disease and what is
Pump. (Heart, Liver and
it alternate name
muscle) Type II Glycogen
storage disease
severe fasting hypoglycemia,
What are the findings in Von
increased glycogen in the
Gierke's disease
liver

1.There is no mutation
occuring at the locus 2. There
What are the four
is no selection for any of the
assumptions of the Hardy-
geno types at the locus 3.
Weinberg equilibrium
Random mating 4. no
migration in or out

What are the purely ketogneic


lysine and leucine
amino acids

What are the signs and asymptomatic, benign,


symptoms of essential Fructose appears in blood
fructosuria and urine
Homocysteine accumulates in
urine and cystine becomes
what are the signs and essential Methionine and it's
symptoms of Homocystinuria metabolites build up in blood
Mental retardation,
osteoporosis, dislocation of
the lens

Dark Urine from alkapton


What are the symptoms of bodies; also connective tissue
Alkaptonuria is dark, may have arthralgias.
Bengin disease

What are the symptoms of hypoglycmeia, jaundice,


Fructose intolerance cirrhosis

CNS defects, mental


What are the symptoms of
retardation and death. Urine
Maple syrup Urine disease
smells like maple syrup
tyrosine, thyroxine Dopa
What biological chemicals are
dopamine NE, Epinephrine
derived form phenylalanine
and Melanin

What causes Essential


defect in fructokinase
Fructosuria

Absence of HGPRTase,
(normally converts
What causes Lesch-Nyhan hypoxanthine to IMP and
syndrome guanine to GMP) Lacks
Nucleotide Salvage (LNS)-
purines

What do melanin and both derived directly from


Norepinephrine have in dopamine Phenylalanine to
common tyrosine to Dopa to Dopamin
when one allele of an allele pair is
lost. An example is when a
patient inherits or develops a
What does the term 'loss of mutation in a tumor suppressor
gene and the complimentary
heterozygosity' mean
allele is then lost to deletion/
mutation. The patient would not
develop the cancer until the loss
of the normal allele.

Glycolytic enzyme
deficiencies 1. Hexokinase
What enzyme defiencies are 2.glucose-phosphate
associated with hemolytic isomerase 3.aldolase
anemia 4.triose-phosphate isomerase
5. phosphate-glycerate
kinase enolase pyruvate
kinase

lack of migration of neural


What event in embryology can
crest cells to skin (form
cause albinism
melanocytes)

What genetic error can cause


Adenosine deaminase
Severe Combined
deficiency
Immnuodeficiency (SCID)?
what group of people can be
alcoholics due to B1
seen with pyruvate
defiecincy
dehydrognease deficiency

What is a complication of
cystine kidney stones
cystinuria

a mutation that exerts a


dominant effect because the
What is a dominant negitive body cannot produce enough
mutation? of the normal gene product
with only one allele
functioning normally

What is a good pnuemonic for Very-Von Gierke's Poor-


the four glycogen storage Pompes Carbohydrate-Cori's
diseases Metabolism-McArdles
What is commonly associated dry skin, melanoma and other
with xeroderma pigementosa? cancers

What is crucail to the


diagnosis of an Autosomal Family history
dominant disease?

the severity of the diseas


worsens or age of onset of
What is genetic anticipation?
disease is earlier in
succeeding generations

What is inheritance of G6PD


dfiency and what population X-linked recessive; blacks
is effected more often
What is NAD/NADP made
Niacin, 'Serotonin, melatonin
form

What is osteogenesis
imperfecta often confused child abuse
with

What is the biochemical


defect in Metachromatic deficiency of arylsulfatase A;
leukodystrophy and what is Autosomal recessive
the inheritance pattern

What is the biochemical


deficiency of Beta-
defect in Nieman-Pick
glucocerebrosidase;
disease and how is it
autosomal recessive
inherited
decrease in NADPH which is
necessary to reduce
What is the biochemical effect
glutathione which in turn
of G6PD defiency
detoxifies free radicals and
peroxides

inheritied defect o the tubular


What is the cause of amino acid transporter for
Cystinuria and what are the Cystine, ornithine, Lysine and
signs/symptoms Arginine in kidneys excess
cystine in urine

Alpha-galactosidase A
What is the cause of Fabry's deficiency; gives
disease and what is the accumulation of ceramide
common clinical problem trihexoside causing renal
failure

Absence of galactose-1-
What is the cause of phosphate uridyltransferase;
galactosemia? accumualtion of toxic
substances (galactitol)
Alpha-L-iduronidase defiency
What is the characteristic
leads to corneal clouding and
defect in Hurler's syndrome
mental retardation

Multiple café-au-lait spots,


What is the characteristic
neurofibromas increased
findings in Neurofibromatosis
tumor susceptibilty

pulmonary infections,
What is the clinical picture of exocrine pancreatic
a patient with cystic fibrosis insufficiency, infertility in
men

What is the clinical picture of


muscular weakness and
Duchenne's muscular
degeneration
dystrophy
decreased Alpha-ketoacid
What is the defect in Maple dehydrogenase. blocks
Syrup Urine disease degradation of branched
amino acids Ile. Val. Leu

What is the epidemiology of Age-dependnet and/or


lactose intolerance hereditary (blacks and Asians)

defect in cystathionine
synthase. Two forms: 1.
What is the etiology of deficiency 2. decreased
Homocystinuria affinity of synthase for
pyridoxal phosphate
(cofactor)

What is the finding of


mild mental retardation but
Hunter's syndrome on
no corneal clouding
H&amp;P
What is the formula for p^2 +2pq + q^2 =1 and p
Hardy-Weinberg equilibrium? +q=1

What is the genetic mech. Of AR; multiple loss-of-function


Cystic fibrosis and it's mutations in a chloride
inheritance channel

What is the genetic X-linked recessive;caused by


mechanism of Duchenne's multiple loss-of-funtion
muscular dystrophy mutations in a muscle protein

X-linked; progressive
What is the genetic expansion of unstable DNA
causes failure to express
mechanism of Fragile X MR
gene-encoding RNA-binding
protein
what is the genetic AD, multiple loss-of function
mechanism of mutations in a signaling
Neurofibromatosis molecule

What is the genetic problem trisomy 21 chromosomal


in Down's Syndrome imbalance

What is the inheritance of


Autosomal recessive
Krabbe's disease

Autosomal recessive; 1:30 in


what is the inheritance
Jews of European descent and
pattern and carrier frequency
1:300 in the general
in Tay-Sach's
populaition
10 types of this syndrome
What is the inheritance
Type IV-Auto Dominant Type
pattern of Ehlers-Danlos
VI-Auto Recessive Type IX-
syndrome
X-linked recessive

What is the inheritance


Autosomal recessive
pattern of Gaucher's disease

What is the inheritance


pattern of xeroderma autosomal recessive
pigmentosa

What is the inheritiance


X-linked recessive
pattern of Fabry's disease
Heinz Bodies: altered
What is the lab symptoms of
hemoglobin precipitate in
G6PD defiency
RBC

What is the most


distinguishing finding in Tay- cherry red Macula; these
Sach's disease on Physical patients die by 3
exam

Gaucher's cells with the


What is the pathogneumonic
characteristic 'crinkled paper'
cell type founde in Gaucher's
appearance of enlarged
disease
cytoplasm

What is the pattern of


inheritance of Leber's mitochondrial inheritance
hereditary optic neuropathy?
increased susceptibility to
What is the phenotype in
fractures;connective tissue
Osteogenesis imperfecta
fragility

Mental and growth


retardation, dysmorphic
What is the phenotype of
features, internal organ
Down's syndrome
anomalies especially heart
problems

mental retardation,
What is the phenotype of
characteristic facial features,
Fragile X
large testes

What is the predominant


problem in Ehlers-Danlos Faulty collagen synthesis
syndrome
What is the prevalance of 1:2000 whites; very rare
cystic fibrosis among Asians

What is the prevalandce of 1:3000 with 50% being new


Neurofibromatosis mutations

What is the prevalence of


1:800; increased risk with
Down's syndrome and what
advanced maternal age
are the risk factors

What is the prevalence of


1:300; 33% new mutations
Duchenne's musc. dys.
What is the prevalence of 1:1500 males: can be in
Fragile X- associated mental feamales is a multi-step
retardation process

What is the prevalence of


1:10000,
osteogenesis imperfecta

What is the prevalence of


1:10000
Phenylketonuria

What is the priamry defect in


defiency of aldolase B,
Fructose intolerance and how
autosomal recessive
is it inherited?
congenital defiency of
What is the primary defect in homogentisic acid oxidase in
Alkaptonuria the degradative pathway of
tyrosine

either 1.decreased
What is the primary defect(s) phenylalanie hydroxylase or
found in Phenylketoneuria 2. decreased
tetrahydrobiopterin cofactor

What is the rate-limiting


Glucose-6-phosphate
enzyme in the Hexose-
dehydrogenase
Monophosphate shunt?

backup of substrate (pyruvate


What is the result of pyruvate
and alanine) resulting in lactic
dehydrognease deficiency
acidosis
defective excision repair such
What is the specific defect in as uvr ABC exonuclease; have
Xeroderma pigmentosa inability to repair thymidine
dimer formed by UV light

What is the treament of


Acetazolide to alkinlize the
Cystinuria and what is a
urine cystine kidney stones
possible consequence of not
due to excess cysteine
treating

decrease intake of both


What is the treatment of
fructose and sucrose (glucose
fructose intolerance?
+ fructose)

1. For a defiency in
cystathionine synthase tx by
What is the treatment of decrease Met and increase
Homocystinuria Cys in diet 2. for decreased
affinity of synthase Tx by
decrease vitamin B6 in diet
Exclude galactose and lactose
What is the Tx of
(galactose +glucose) form
galactosemia?
diet

what offspring of females


affected with a mitochondrial
all offspring can be effected
inherited disease will be
effected?

what percent of offspring


from two autosomal recessive
25%
carrier parents will be
effected?

what percent of sons of a


heterazygous mother carrying
50%
an x-linked disease will be
effected?
What period of life do
often present clinically after
autosomal dominant defects
puberty
present in?

What three phenyl ketones


phenylacetate, phenyllactate,
build up in the urine of PKU
phenylpyruvate
patients

What to thyroxine and Dopa


both derived form tyrosine
have in common

What type of genetic error is


usually more severe AR disorders are often more
autosomal recessive or severe
dominant?
What type of inheritance is
transmitted only through mitochondrial
mothers?

When do patients usually


present with autosomal present in childhood
recessive disorders?

RBC's metabolize glucose


Why are RBC so susceptible to anaerobically (no
Glycolytic enzyme def. mitochondria) and depends
on glycolysis

deficent aldolase B causes the


accumulation of Fructose 1-
Why do people with fructose phosphate which acts as a
intolerance become phosphate sink and traps the
hypoglycemic? phosphate. Decreased
phosphate availability inhibits
glycogenolysis and
gluconeogenesis
X-linked recessive disease is
males
aften more severe in

2,3-BPG via
1,3-BPG
bisphosphoglycerate mutase

Acyl coenzyme A, lipoamide

aldehydes TPP
prostaglandins, -
Arachidonate
thromboxanes, -leukotrienes

Associate the following signal


j
molecule precursors.

At body pH, what AA are Arg and Lys His is neutral at


negatively charged? pH 7.4

At body pH, what AA are


Asp and Glu
positvely charged?
ATP cAMP via adenylate cyclase

By what rxn order kinetics


does alcohol dehydrogenase zero order kinetics
operate?

CH(3) groups SAM

Choline CDP-choline
ACh via choline
Choline
acetyltransferase

CO(2) biotin

glucagon phosphorylates
Contrast glucagon and stuff, -turns glycogen
insulin. synthase off and
phosphorylase on

hexokinase throughout the


Contrast hexokinase and body, -GK in liver and has
glucokinase. lower affinity but higher
capacity for glucose
Contrast hexokinase and only HK is feedback inhibited
glucokinase. by G6P

Does insulin affect glucose


uptake of brain, RBC's and No
liver?

Does insulin inhibit glucagon


release by alpha cells of yes
pancreas?

electrons NADH, NADPH, FADH(2)


fructose-1,6-bis-P via PFK
Fructose-6-phosphate (rate limiting step of
glycolysis)

Glucose UDP-Glucose

GABA via glutamate


glutamate decarboxylase (requires vit.
B6)

GTP cGMP via guanylate cyclase


How are ketone bodies
in urine
excreted?

FA and AA converted to
How are ketone bodies
acetoacetate and b-
formed?
hydroxybutyrate

How do the statin drugs they inhibit HMG-CoA


work? reductase

inhibits acetylaldehyde
How does disulfiram work?
dehydrogenase
How does FA enter the
via citrate shuttle
cytosol?

How does FA enter the


via the carnitine shuttle
mitochondria?

inhibits ALA dehydratase and


How does lead affect heme
ferrochelatase prevents
synthesis?
incorporation of Fe

How does the brain


to 2 molecules of acetyl coA
metabolize ketone bodies?
collected by liver, conjugated
How is bilirubin removed
with glucuronate excreted in
from the body?
bile

ethanol oxidized to
acetylaldehyde by alcohol
How is ethanol metabolized? dehyd and NAD+
acetalaldehyde ox to acetate
by acetylaldehyde and NAD+

How is FA entering the


by cytoplasmic malonyl-CoA
mitochondria inhibited?

How is glutamate converted By the loss of amonium and


to a-ketogluturate reduction of NADP
How is glutamine converted
By the loss of amonium
to glutamate?

scavenged from RBC's and Fe


How is heme catabolized?
+2 is reused

by target cells through


How is LDL uptake
receptor-mediated
undergone?
endocytosis

How is most plasma LCAT(lecethin-cholesterol


cholesterol esterfied? acyltransferase)
How is NAD+ generally used
catabolic processes
metabolically?

anabolic processes (steroid


How is NADPH generally used
and FA synthesis), repiratory
metabolically?
burst, P-450

by need for ATP and supply of


How is TCA regulated?
NAD+

How many ATP's per acetyl


12
CoA?
How many ATP equivalants
are needed to generate 6
glucose from pyruvate?

How many enzyme activities


does pyruvate dehydrognase 3
possess

How many moles of ATP are


generated aerobically through 36 ATP
G3P shuttle?

How many moles of ATP are


generated aerobically through 38 ATP
malate shuttle?
How many moles of ATP are
2 ATP
generated anaerobically?

In what tissue does heme


liver and bone marrow
synthesis occur (2)?

insulin moves glucose into


Insulin mneumonic
cells

Is Serum C peptide present


with exogenous insulin No
intake?
Kwashikor results from a
Malabsorption, Edema,
protein deficient MEAL
Anemia, Liver (fatty)
(mneumonic)

Mnemonic for
Pathway Produces Fresh
gluconeogenesis irreversible
Glucose
enzymes?

Mnemonic for SAM. SAM the methyl donor man

Name 6 common products of glucose, lactate, Acetyl CoA


pyruvate metabolism? +CO2, OAA, Alanine
Name the activated carriers
with associated moleclues tetrahydrofolates
(one carbon units).

phosphoryl ATP

T/F. Uncouplers stop ATP


FALSE
production?

Underproduction of heme microcytic hypochromic


causes what anemia? anemia
What AA are required during
Arg and His
periods of growth?

What are clincial syndromes xanthomas, atherosclerosis


of this disorder? homozygotes MI by age 30

What are major pathways most including


occur in the liver (8)? gluconeogenesis, etc.

What are the activators of


cAMP, and calcium ion
glycgenolysis?
What are the cofactors of pyrophosphate, lipoic acid,
pyruvate dehydrogenase (5)? CoA, FAD and NAD

What are the components of a


TG, FFA and apo E
cholymicron remnant?

What are the components of a TG, apo C-II, apo E, B-48,


cholymicron? apo A

What are the components of


less TG, CE, B-100 and E
IDL?
What are the components of
CE and B-100
LDL?

What are the components of TG, Cholesterol ester, B-100,


VLDL? CII and E

What are the degradation


heme to biliverdin to bilirubin
product steps (3)?

What are the effector


insulin increases, glucagon
hormones of cholesterol
decreases
synthesis?
What are the effector decreased glucagon and
hormones of glyc and pyr ox? increased insulin

What are the effector insulin decreases, epi and


hormones of glycogenolysis? glucagon increases

What are the effector insulin increases, glucagon


hormones of lipogenesis (2)? decreases

What are the electron rotenone, antimycin A, CN-,


transport chain inhibitors? CO
What are the essential
Ile, Phe, Try
glucogenic/ketogenic AA?

What are the essential


Met, Thr, Val, Arg, His
gluconeogenic AA?

What are the essential


Leu and Lys
ketogenic AA?

-pyruvate carboxylase, -PEP


What are the irreversible
carboxykinase,-fructose-1,6-
enzymes of gluconeogenesis
bisphosphotase, -glu-6-
(4)?
phosphotase
-glucokinase/hexokinase,-
What are the irreversible
PFK,-pyruvate kinase,-
enzymes of glycolysis (4)?
pyruvate dehdrogenase

What are the main substrates -glucose, -lipoprotein


used by adipose tissue (2)? triacylglycerol

-glucose, -aa and ketone


What are the main substrates
bodies when starved, -
used by brain?
polyunsat FA in neonates

FFA, -some glucose, -lactate,


What are the main substrates
-ketone bodies, VLDL and
used by heart?
cholymicrom triacylglycerol
What are the main substrates FFA, -glucose, -lactate, -
used by liver? glycerol, fructose, -AA

Acetyl CoA for pyruvate


What are the major activators carboxylase and cAMP for PEP
of gluconeogenesis? carboxykinase and F-1,6-
bis-P

What are the major activators AMP, fructose2,6-bis-P,


of glycolysis and pyruvate fructose 1,6-bis-P in muscle,
oxidation? CoA, NAD, ADP and pyruvate

What are the major metabolic


- esterfication of FA's -
pathways of the adipose
lipolysis
tissue (2)?
What are the major metabolic
-glycolysis,-aa metabolism
pathways of the brain (2)?

What are the major metabolic Aerobic pathways like B


pathways of the heart? oxidation and TCA cycle

What are the major products


-FFA, -glycerol
of the adipose tissue (2)?

What are the major products


lactate
of the brain?
glucose,-VLDL,-HDL,-ketone
What are the major products
bodies,-urea,-uric acid, -bile
of the liver (10)?
acids, -plasma proteins

What are the major regulatory pyruvate carboxylase, PEP


enzymes of gluconeogenesis carboxykinase and F-1,6-
(3)? bis-P

What are the major regulatory


PFK and pyruvate
enzymes of glycolysis and
dehydrogenase
pyruvate oxidation?

What are the major regulatory


enzymes of cholesterol HMG-CoA reductase
synthesis?
What are the products of the
glucose and ketone bodies
liver in the fasting state?

What are the products of the


glycogen and fats/VLDL
liver in the fed state?

What are the products of the 3NADH, 1FADH2, 2CO2,


TCA cycle? 1GTP per Acetyl CoA

massive hemolysis, -block in


What are the some causes of catabolism, -diplaced from
hyperbilirubinemia (4)? binding sites on albumin,
decreased excretion
conjugated (direct/
What are the sources of glucuronidated) and
hyperbilirubinemia (2)? unconjugated(indirect/
insoluble)

What are the specialist -lipoprotein lipase, and well


enzymes of muscle (2)? developed resp chain

What are the specialist


-lipoprotein lipase,-hormone
enzymes of the adipose
sensitive lipase
tissue (2)?

-lipoprotein lipase, -
What are the specialist
respiratory chain well-
enzymes of the heart (2)?
developed
-glucokinase,-glu-6-
phosphotase,-glycerol
What are the specialist kinase, -PEP carboxykinase, -
enzymes of the liver? fructokinase,-arginase,-HMG
coA synthase and lyase, -7a-
hydroxylase

What are the three sites in the NADH dehydrogenase, Cyt b/


electron transport chain for c1, and cytochrome oxidase
active proton transfer? aa3

What can occur as an excess pancreatitis, lipemia retinalis


of cholymicrons (3)? and eruptive xanthomas

What can occur as an excess


pancreatitis
of VLDL?
What catalyzes cholymicron
lipoprotein lipase
to cholymicron remnant?

What catalyzes IDL to LDL? hepatic TG lipase

What catalyzes VLDL to IDL? lipoprotein lipase

What causes a hangover? acetylaldehyde accumulates


What color is bilirubin and
what is the condition of yellow, jaundice
excess?

What complex is pyruvate a-ketogluturate


dehydrogenase similar to? dehydrogenase complex

bronchconstriction,
What do LT c4, D4 and E4 do vasoconstriction, contract
(4)? smooth muscle, increase
vascular permeability

What does breath smell like


fruity(acetone)
during ketoacidosis?
What does excess LDL cause atherosclerosis, xanthomas,
(3)? and arcus corneae

ATPase inhibitor that


What does oligomycin do to
increases proton gradient but
ETC?
not ATP production

What does PGI stand for? platelet gathering inhibitor

thromboxanes,
What does the COX pathway
prostaglandins and
yield?
prostacyclin
What does the lipooxygenase
leukotrienes
pathway yield?

Ornithine, citrulline,
carbamoyl-p, aspartate,
What does this stand for?
arginosuccinate, fumurate,
arginine, urea

What enzyme catalyzes the


rate limiting step of HMG-CoA reductase
cholesterol syn.?

What induces the PPP? insulin


What is a major component of
modified LDL
atherosclerotic plaque?

What is an uncoupling agent


2,4 DNP (dynamite)
to the ETC?

What is cofactor required for


methionine (SAM) vitamin B12
regeneration?

platelet aggregation and


What is does PGI 2 inhibit (2)?
vasodilation
What is familial hyper- AD genetic defect in LDL
cholesteremia? receptor

What is Kwashikor? protein malnutrition

What is LT B4? neutrophil chemotactic agent

Protein-calorie malnutrition
What is Marasmus?
resulting in tissue wasting
What is the activator of
Citrate
lipogenesis?

What is the activator of PPP? NADP+

What is the clinical picture of


small child with swollen belly
Kwashikor?

What is the committed step of glycine+succ CoA to delta-


heme synthesis? aminolevulinate
What is the composition of Base(adenine), ribose, 3
ATP? phosphoryls

What is the consequence of


accumulated intermediates of porphyrias
heme synthesis?

What is the easy way to


First 4 B vitamins + lipoic
remember the cofactors of
acid
PDH complex?

What is the effector hormone


Glucagon
for glycogenesis?
What is the energy content of
the 2 phosphoanhydride 7 kcal/mol each
bonds?

transfers cholesterol from


What is the functiion of HDL
periphery to liver acts as a
(2)?
repository for apoC and apoE

What is the functioin of FA uptake to cells from


lipoprotein lipase? choly's and VLDL's

What is the function of rate limiting step of heme


aminolevulinate (ALA) synthesis converts succinyl
synthase ? CoA and glycine to ALA
delivers dietary TG to
What is the function of
peripheral tissues delivers
cholymicrons (2)?
dietary cholesterol to liver

What is the function of


degradation of stored TG's
hormone sensitive lipase?

delivers hepatic cholesterol to


What is the function of LDL?
peripheral tissues

What is the function of liberates arachidonic acid


phospholipase A2? from cell membrane
-produces ribose-5-P from
What is the function of PPP G6P for nucleotide
(3)? synthsesis,-produces
NADPH,-part of HMP shunt

transfers methyl units to wide


What is the function of SAM?
variety of receptors

transfers excess reducing


What is the function of the equivalants from RBC's and
Cori Cycle? muscle to liver to allow
muscle to function
anaerobically

What is the function of Tx A2 platelet aggregation and


(2)? vasoconstriction
Delivers hepatic TG to
What is the function of VLDL?
peripheral tisssue

What is the inhibitor of


long-chain acyl-CoA, -cAMP
lipogenesis (2)?

What is the inhibitor of PPP? NADPH

What is the limiting reagent


NAD+
of ethanol metabolism?
What is the main substrate of
glucose
fast twitch muscle?

What is the main substrate of ketone bodies, -FFA, -


slow twitch muscle? triacylglycerol

What is the major function of


rapid movement
fast twitch muscle?

What is the major function of


sustained movement
slow twitch muscle?
What is the major function of increase vascular smooth
the a1 receptor? muscle contraction

-decrease sympathetic
What is the major function of
outflow, -decrease insulin
the a2 receptor (2)?
release

increase HR, -inc.


What is the major function of conntractility, -inc. renin
the B1 receptor (5)? release,-inc. lipolysis, -inc.
aq. Humor formation

What is the major function of vasodilation, bronchodilation,


the B2 receptor (3)? inc. glucagon release
What is the major function of relax renal vascular smooth
the D1 receptor? muscle

What is the major function of modulate transmitter release,


the D2 receptor? esp. in brain

increase mucous production,


What is the major function of
-contract bronchioles, -
the H1 receptor (4)?
pruritis, -pain

What is the major function of


increase gastric acid secretion
the H2 receptor (4)?
What is the major function of service for the other organs
the liver? and tissues

What is the major function of


CNS
the M1 receptor?

What is the major function of


decrease heart rate
the M2 receptor?

What is the major function of increase exocrine gland


the M3 receptor? secretions
What is the major function of increase vascular smooth
the V1 receptor? muscle contraction

increase water permeability


What is the major function of
and reabsorption in the renal
the V2 receptor?
collecting tubules

citrate (FA and ketone bodies)


What is the major inhibitor of
ATP and cAMP, -acetyl CoA,
glyc and pyr ox?
NADH, ATP

What is the major inhibitor of


ATP, long-chain acyl-coA
TCA?
What is the major inhibitor of
cholesterol and cAMP
cholesterol synthesis (2)?

What is the major inhibitor of ADP and AMP and F-2,6-bis-


glycogenesis? P

What is the major metabolic


pathway of fast twitch glycolysis
muscle?

What is the major metabolic


Aerobic pathways like B
pathway of slow twitch
oxidation and TCA cycle
muscle?
What is the major product of
lactate
fast twitch muscle?

What is the major product of


lactate
slow twitch muscle?

What is the major regulatory


glycogen synthase
enzyme of glycogenolysis?

What is the major regulatory


acetyl CoA carboxylase
enzyme of lipogenesis?
What is the major regulatory
enzyme of pentose glucose-6-P dehydrogenase
phosphate pathway (PPP)?

What is the major regulatory


citrate synthase
enzyme of TCA?

What is the mechanism for directly block ETC, cause


the ETC inhibitors? decreased proton gradient

inc. membrane permeability,


What is the mechanism for
decreases proton gradient
the ETC uncoupler?
and increases O2 consmption
NADH/NAD increases in liver
causes diversion of pyruvate
What is the mechanism of
to lactate and OAA to malate
ethanol hypoglycemia?
inhibits gluconeogenesis and
thus leads to hypoglycemia

What is the mnemonic for the


Cindy Is Kinky So She
various substrates of the TCA
Fornicates More Often
cycle?

What is the mneumonic for


PriVaTe TIM HALL
essential AA?

A-1 Activates LCAT B-100


What is the mneumonic for Binds to receptor C-II is a
major apolipoproteins? Cofactor for LPL E mediates
Extra (remnant) uptake
Ordinarily, Careless Crappers
What is the pneumonic for
Are Also Frivolous About
the Urea cylce substrates?
Urination

an intestinal intermediate
What is urobilinogen? reabsorbed to blood and
excreted in urine as urobilin

What other physiological side


fatty liver seen in chronic
affect occurs by this
alcoholics
hypoglycemia?

What rxn does pyruvate pyruvate+NAD+CoA goes to


dehydrogenase catalyze? acetyl-CoA +CO2+NADH
What second messenger adenylcyclase reduces cAMP
system does Gi work levels and protein kinase A is
through? reduced

phospholipase C, PIP2 to IP3


What second messenger
and DAG DAG works through
system does Gq work
protein kinase C IP3 increases
through?
IC calcium ion

What second messenger adenylcyclase converts ATP to


system does Gs work cAMP to phosphorylate
through? protein kinase A

sparingly water soluble toxic


What some properties of
to CNS transported by
bilirubin (3)?
albumin
What step does this enzyme
HMG-CoA to mevalonate
work on?

What suppresses
insulin
glycogenesis?

What tissues require insulin


adipose and skeletal muscle
for glucose uptake?

What type of metabolism -gluconeogenesis, -urea


occurs in the both (3)? cycle, heme synthesis
glycolysis, -fatty acid
What type of metabolism synthesis, -HMP shunt, -
occurs in the cytoplasm (5)? protein synthesis (RER), -
steroid synthesis (SER)

What type of metabolism


Beta oxidation, -Acetyl-CoA
occurs in the mitochondria
production, -Kreb's cycle
(3)?

When are ketone bodies during prolonged starvation


formed (2)? diabetic ketoacidosis

Where are basic AA found in in Histones that bind to


high amounts? negative DNA
Where are cholymicrons
small intestine
made?

Where are ketone bodies


liver
made?

Where does FA degradation mitochondria, where it will be


occur? used

Where does FA synthesis


cytosol
occur?
Where is ALA synthase found
mitochondria, heme
and what inhibits it?

Where is HDL secreted from


liver and small intestine
(2)?

Where is insulin made? B cells of pancreas

is a product of HMP shunt


Where is NADPH generated? and the malate
dehydrogenase rxn
Where is SAM generated? From ATP and methionine

Where is VLDL made? liver

Which ketone body is


detected in urine test (1 acetoacetate
only)?

Carbon monoxide has a CO has 200x greater affinity


greater affinity for what for hemoglobin than for
molecule? oxygen
hemoglobin is composed of 4
Describe the subunits for
polypeptide subunits (2 alpha
hemoglobin?
and 2 beta)

During the cycle of the


phosphorylated
sodium pump, it is __.

1. enzyme concentration
alteration (syntesis and/or
destruction) 2. covalent
modification (eg.
How are enzymes regulated? phosphorylation) 3. proteolytic
modification (zymogen) 4.
allosteric regulation (eg. feedback
inhibition) 5. transcriptional
regulation (eg. steroid hormones)

Ca2+ -&gt; activates troponin


-&gt; moves tropomyosin -
How does calcium cause
&gt; exposes actin-bining
skeletal muscle contraction?
site -&gt; allows actin-
myosin interaction
Ca2+ -&gt; binds to
How does calcium cause calmodulin because smooth
smooth muscle contraction? muscle doesn't have
troponins!

procollagen molecules cleaved


at terminal regions by
How does procollagen peptidases to become insoluble
tropocollagen, which aggregates
molecules become collagen
to form fibrils procollagen
fibrils?
molecules are exocytosed into
extracellular space, where this
process occurs

- binds to amino acids in


How is CO2 transported from globin chain (at N-terminus),
not to heme - favors T form
tissue to lungs?
of Hb (thus promotes O2
unloading)

by the formation of covalent


How is collagen fibillar lysine-hydroxylysine
structure reinforced? crosslinks between
tropocollagen molecules
increased Cl-, H+, CO2, DPG,
and temperature favor T form
How is hemoglobin structure (low affinity of O2)--shifting
regulated? the dissociation curve to the
right, leading to increased O2
unloading

Most cells are in which cell


G0
cycle phase?

On what cellular stuctures are


flagella, cilia, mitotic spindles
microtubules found?

- RBC membranes - myelin -


bile - surfactant (DPPC--
On which cells is
dipalmitoyl
phosphotidylcholine (lecithin)
phosphatidylcholine) - also
a major component?
used in the esterification of
cholesterol
Only the cytoplasmic side of
glycosylated lipids or proteins
membrane contains what?

The lower the Km, the


(higher/lower/remains same) higher
the affinity

- T (taut) form has low affininty for


oxygen - R (relaxed) form has 300x
higher affinity for oxygen Hb exerts
What are the 2 forms of positive cooperativity and negative
allostery, accounting for the
hemoglobin?
sigmoid-shaped O2 disassociation
curve (which myoglobin doesn't have)
[Hint: When you're RELAXED, you do
your job better (carry more O2)]

Mitosis (Prophase-
Metaphase-Anaphase-
What are the cell cycle Telophase) G1 (Gap or
phases? Growth) S (Synthesis of DNA)
G2 (Gap or Growth) G0
(quiescent G1 phase)
- cylindrical structure 24nm in
dia and variable length - helical
array of polymerized dimers of
What are the characteristics alpha- and beta-tubulin (13 per
circumference) - each dimer has
of a microtubule?
2 GTP bound - grows slowly,
collapses quickly - involved in
slow axoplasmic transport in
neurons

- resemble substrates - bind


reversibly to active sites of
enzymes - high substrate
What are the characteristics concentrations overcomes
of competitive inhibitors? effect of inhibitors - Vmax
remains unchanges - Km
increases compared to
unhibited
- doesn't resemble substrate -
bind to enzyme but not
necessarily at active site -
What are the characteristics inhibition can't be overcome by
of noncompetitive inhibitors? high substrate concentration -
Vmax decreases - Km remains
unchanged compared to
uninhibited

What cell cycle phases are


G1 and G0
variable in duration?
- mebendazole/
thiabendazole (antihelmintic)
What drugs act on
-taxol (anti-breast cancer) -
microtubules?
griseofulvin (antifungal) -
cholchicine (anti-gout)

- Ouabain inhibits the pump


by binding to the K+ site -
What drugs inhibits the cardiac glycosides (digoxin,
sodium pump? digitoxin) also inhibit the
pump, causing increased
cardiac contractility

blood disorder where


methemoglobin, an oxidized
form of hemoglobin (ferric,
What is methemoglobinemia? Fe3+) that does not bind O2
as readily. Iron in Hb is
normally in a reduced state
(ferrous, Fe2+)

fibril is made of molecules


collagen fibril--many
What is the difference staggered collagen molecules
between collagen fibril and linked by lysyl oxidase
collagen molecule? collagen molecule--3
collagen alpha chains, usually
of Gly-x-y (x and y = pro,
OH-pro, or OH-lys)
What is the first step in
forming collagen from prolyl hydroxylation endoplasmic
and lysyl residues? Where reticulum vitamin C
does it occur? What nutrient
does it require?

- cholesterol (~50%,
promotes membrane stability)
What is the plasma
- phospholipids (~50%) -
membrane composition?
sphingolipids -glycolipids -
proteins

What is the sodium pump? Na-K ATPase

What molecules, how many of


them, and in what direction
are moved across the 3 Na go out and 2 K go in
membrane by the sodium
pump?
on the membrane, with the
Where is the sodium pump? ATP site on the cytoplasmic
side of the pump

Which cell cycle phase is


rapidly dividing cells have a
shorter in rapidly dividing
shorter G1 phase
cells?

Which cell cycle phase is


mitosis
usually shortest?

Arthralgia's, fatigue,
headaches, skin changes,
sore throat, alopecia are Vit A (Retinal)
symptoms foundin what
vitamin deficiency
in sarcoid, epitheliod
macrophages convert vitamin
How does sarcoidosis cause
D into it's active form leading
hypercalcemia?
to increased absorption of
calcium

How many ATP are made Niacin=NAD niacin is B3=3


from Niacin ATP

Riboflavin is B2 2
What are the characteristic
C's=cheilosis and Corneal
findings seen in Riboflavin
vascularization and also
deficiencies
Angular stomatitis

What are the fat soluble


D,A,K,E
vitamins
What are the findings in Vit D Hypercalcemia, loss of
excess? appetite, stupor

1. Vit D intoxication 2.
Malignancy 3.
What are the possible causes Hyperparathyroidism 4. Milk-
of hypercalcemia? alkali syndrome 5.
Sarcoidosis 6. Paget's disease
of bone

1. Macrocytic megaloblastic
What are the signs and anemia 2. Neurologic
symptoms of vit B12 symptoms-optic neuropathy,
deficiency? subacute combined
degeneration 3. glossitis

What are the signs of Biotin dermatitis, enteritis


deficiency and what are antiobiotic use and ingestion
possilble causes? of raw eggs
What are the signs of Folate Macrocytic megaloblastic
deficiency? anemia sprue

What are the symptoms in Vit dermatitis, enterititis,


B5 deficiency alopecia, adrenal insufficiency

3 D's Diarrhea, Dermatitis,


What are the symptoms of
Dementia, and also Beefy
Pellagra
glossitis

1. Malabsorption- Sprue,
enteritis, Diphyllobthrium
latum (Dr. Lohr's fish
What are the usual causes of tapeworm) 2.lack of intrinsic
Vit B12 deficiency? factor (pernicious anemia) 3.
Absence of the terminal
ileum- Crohn's disease or
surgery
B complex vitamins
What are the water soluble
(B1,2,3,5,6,12), Vit C, Biotin,
vitamins
Folate

What can induce pyridoxine INH and oral contraceptives


deficiency? Symptoms? convulsion, hyperirritability

What clotting factors are Vit K


dependent and what anti- Factors II, VII, IX, X and
clotting drug acts as a Vit K Protein C and S Warfarin
antagonist?

what disease is characterized


by polyneuritis, cardiac Beriberi (Ber1BerI), due to Vit
pathology and edema and B1 deficiency
what is the cause
What diseases conditions are
Rickets in kids Osteomalacia
caused by Vitamin D
in adults hypocalcemic tetany
deficiency?

What diseases is caused by Scurvy swollen gums,


Vit C deficiency and what are bruising, anemia, poor wound
the findings? healing

Cofactor for homocysteine


What is B12 used for in the
methylation and
body?
methylmalonyl-CoA handling

What is Niacin's function in


part of NAD, NADP and is
the body and from what can it
derived from tryptophan
be derived
What is the active form of
1,25 (OH)2 D3 = active form
Vitamin D?

What is the alternate name of


niacin Pellagra Hartnup
Vit B3, problems in
disease, malignant carcinoid
deficiency, common cause of
syndrome, and INH
defieciency

What is the alternate name of


vitamin B1 and what are the Beriberi and Wernicke-
characteristic diseases of it's Korsakoff syndrome
deficiency

What is the folic acid


PABA is the precursor sulfa
precursor in bacteria and
drugs and dapsone are PABA
what antibiotics exploit this
analogs
fact?
What is the form of vitamin D
D2 = ergocalciferol
consumed in milk?

What is the form of Vitamin D


D3 = cholecalciferol
found in sun-exposed skin?

cofactor in oxidation and


What is the function and
reduction (FAD,FMN)
alternate name of Vit. B2
RiboFlavin

Cofactor for carboxylations 1.


Pyruvate to oxaloacetate 2.
What is the function of Biotin? Acetyl-CoA to malonyl Co-A
3. Prprionyl-CoA to
methylmalonyl-CoA
coenzyme for 1-carbon
transfer;methylation reactions
What is the function of folic
important for the synthesis of
acid?
nitrogenous bases in DNA
and RNA

What is the function of Vit A, visual pigments (retinal)


it's alternate name, and retinol night blindness and
symptoms in deficiency dry skin

it becomes Thiamine
Pyrophosphate (TPP) and is
used in: oxidative
What is the function of Vit B1 decarboxylation of Alpha-
keto acids (pyruvate, Alpha-
ketoglutarate cofactor for
tranketolase in the HMP shunt

B6 (pyridoxine) is converted
to pyridoxal phosphate a
What is the function of Vit B6 cofactore in transamination
reactions (ALT &amp; AST),
decarboxylation, and trans-
sulfuration
antioxidant; especially in
Erythrocytes where it protects
What is the function of Vit E?
them from hemolysis Vit E is
for Erythrocytes

1. Cross linking of collagen-


hydroxylation of proline and
lysine in collagen synthesis
What is the function of keeping iron in the Fe2+
vitamin C? reduced state making it more
absorbable 3. Cofactor for
coverting dopamine to
norepinephrine

What is the function of increase intestinal absorption


Vitamin D? of Calcium and phosphate

catalyzes gamma-
carboxylation of glutamic
What is the function of
acid residues on various
Vitamin K?
proteins concerned with
clotting.
What is the most common
vitamin deficiency in the Foilic Acid
United States?

2 F's, 2C's and 2ATP 2


F's=FAD&amp;FMN 2 C's =
What is the rule of 2's for Vit signs of deficiency cheilosis
B2 and corneal vascularization
oxidation of FADH2 leads to 2
ATP

What is the sotrage form of


25-OH D3
Vitamin D?

What is the source of Vit B12 found only in animal products


and what is B12's other name cobalamin
constituent of CoA, part of
fatty acid synthase. Cofactor
What is vit B5's function and
for acyl transfers
alternate name
Pantothenate (Pantothen-A is
in Co-A)

What manifestation is specific high output cardiac failure


to wet beriberi? (dilated cardiomyopathy)

What molecule in egg white


binds up Biotin and causes Avidin
deficiency

Neonatal hemorrhage with


increased PT increased aPTT,
but normal bleeding time in
What problems do you see in general, mild vitamin k
Vit K deficiency? deficiency will prolong PT and
have normal PTT severe
deficiency will prolong PT and
PTT
What scenerios are vit B
alcholism and malnutrition
defieciencies often seen in

What test is used to detect


Schilling test
B12 deficiency

What two general types of Malabsorption syndromes


things will cause fat soluble ( cystic fibrosis and sprue)
vitamin deficiency and mineral oil intake

What vitamins more


fat soluble vitamins b/c these
commonly cause toxicity and
accumulate in fat
why
What water soluble vitamin
B12 which is stored in the
does not wash out easily from
liver
the body

What will a defieciency in Vit E Increased fragility of


lead to? erythrocytes

synthesized only in
Where is B12 synthesized and
microorganisms stored
stored
primarily in liver

synthesized by ntestinal flora


Where is Vit K synthesized
prolonged broad spectrum
and what is one cause of Vit K
antibiotic use can kill off the
deficiency?
flora can cause a deficiency
(T/F) The outer membrane FALSE: they DO act a major
for G+ and the cell surface antigens BUT the
membrane for G- act as outer mb for G- and the cell
major surface antigens. membrane for G+

Are endotoxins heat stable? yes, stable at 100C for 1 hr.

Are endotoxins secreted from


NO
cells?

Are endotoxins used as no, they don't produce


antigens in vaccines? protective immune response
no, destroyed rapidly at 60C
Are exotoxins heat stable? (exception: Staphylococcal
enterotoxin)

Are exotoxins secreted from


YES
cells?

Are exotoxins used as Yes, TOXOIDS are used as


antigens in vaccines? vaccines

Describe the chemical


Sugar backbone with cross-
composition of
linked peptide side chains.
peptidoglycan.
inner and outer lipid bilayer
membranes - thin layer of
Describe the major peptidoglycan - periplasmic
components of a G- cell wall. space - contains
lipopolysaccharide,
lipoprotein and phospholipid

one lipid bilayer membrane -


Describe the major
thick layer of peptidoglycan -
components of a G+ cell wall.
contains teichoic acid

Describe the process of DNA transfer from one


conjugation. bacterium to another.

Describe the process of DNA transfer by a virus from


transduction. one cell to another
Describe the process of purified DNA is taken up by a
transformation. cell

Does endotoxin induce and


no, not well
antigenic response?

Does exotoxin induce and Yes, induces high-titer


antigenic response? antibodies called antitoxins

Give two general functions of Gives rigid support - protects


peptidoglycan against osmotic pressure
Group A are Bacitracin
How are Group A and Group B
sensitive - Group B are
Strep primarily differentiated?
Bacitracin resistant

How are the pathogenic


on the basis of sugar
Neisseria species
fermentation
differentiated?

How are the species of


on the basis of their
Streptococcus primarily
HEMOLYTIC capabilities
differentiated?

Lag phase - log (exponential)


List the four phases of the
phase - stationary phase -
bacterial growth curve.
death phase
Name 2 G- rods that are
considered slow lactose Citrobacter and Serratia
fermenters.

Name 3 G- rods that are


1) Klebsiella 2) E. coli 3)
considered fast lactose
Enterobacter
fermenters.

Name 3 G- rods which are


lactose nonfermenters and Shigella, Salmonella, Proteus
Oxidase(-)?

Name 4 bacteria that use IgA 1) Strep. pneumoniae 2)


protease to colonize mucosal Neisseria meningitidis 3)
surfaces. Neisseria gonorrhea 4) H. flu
1) H. flu 2) Pasteruella 3)
Name 4 genus of bacteria
Brucella 4) Bordetella
that are G- 'coccoid' rods.
pertussis

Treponema - Rickettsia -
Name 6 bacteria that don't Mycobacteria - Mycoplasma
Gram's stain well? - Legionella pneumophila -
Chlamydia

1) Clostridium (an anaerobe)


Name four genus of bacteria
2) Coynebacterium 3) Listeria
that are G+ rods.
4) Bacillus

1) Corynebacterium
diphtheriae 2) Clostridium
Name seven G+ bacteria tetani 3) Clostridium
species that make exotoxins. botulinum 4) Clostridium
perfringens 5) Bacillus
anthracis 6) Staph. aureus 7)
Strep. pyogenes
Name three diseases caused Tetanus - botulism -
by exotoxins. diptheria

Name three G- bacteria 1) E. coli 2) Vibrio cholerae


species that make exotoxins. 3) Bordetella pertussis

Name three Lactose- Eschericia, Klebsiella,


fermenting enterics. Enterobacter

Name two diseases caused by Meningococcemia - sepsis by


endotoxins. G(-) rods
Name two type of Strep that S. pneumoniae - Viridans
exhibit alpha hemolysis? strep. (e.g. S. mutans)

Name two types of Strep. that Enterococcus (E. faecalis) and


are non-hemolytic (gamma Peptostreptococcus
hemolysis). (anaerobe)

Name two types of Strep. that Group A Strep. (GAS) and


exhibit beta hemolysis. Group B Strep. (GBS)

Teichoic acid induces what


TNF and IL-1
two cytokines?
What are the effects of it is a superantigen - it
erythrogenic toxin? causes rash of Scarlet fever

it is a hemolysin - it is the
What are the effects of
antigen for ASO-antibody
streptolysin O?
found in rheumatic fever

What are the effects of the one toxin in the toxin


exotoxin secreted by Bacillus complex is an adenylate
anthracis? (1) cyclase

Stimulates adenylate cyclase


What are the effects of the by ADP ribosylation - causes
exotoxin secreted by whooping cough - inhibits
Bordetella pertussis? (3) chemokine receptor, causing
lymphocytosis
blocks release of
What are the effects of the acetylcholine: causes
exotoxin secreted by anticholenergic symptoms,
Clostridium botulinum? CNS paralysis; can cause
'floppy baby'

alpha toxin is a lecithinase -


What are the effects of the
causes gas gangrene - get a
exotoxin secreted by
double zone of hemolysis on
Clostridium perfringens?
blood agar

What are the effects of the blocks release of the


exotoxin secreted by inhibitory NT glycine; causes
Clostridium tetani? 'lockjaw'

What are the effects of the


1) inactivates EF-2 by ADP
exotoxin secreted by
ribosylation 2) pharyngitis 3)
Corynebacterium diphtheria?
'pseudomembrane' in throat
(3)
this heat labile toxin
What are the effects of the stimulates adenylate cyclase
exotoxin secreted by E. coli? by ADP ribosylation of G
(2) protein - causes watery
diarrhea

superantigen; induces IL-1


What are the effects of the
and IL-2 synthesis in Toxic
exotoxin secreted by Staph.
Shock Syndrome; also causes
aureus?
food poisoning

Stimulates adenylate cyclase


by ADP ribosylation of G
What are the effects of the
protein - increases pumping
exotoxin secreted by Vibro
of Cl- and H2O into gut -
cholerae? (3)
causes voluminous rice-water
diarrhea

What are the general clinical


fever, shock
effects of endotoxin?(2)
1) Acivates macrophages 2)
What are three primary/
Activates completment (alt.
general effects of endotoxin
pathway) 3) Activates
(especially lipid A)?
Hageman factor

What are two exotoxins Erythrogenic toxin and


secreted by Strep. pyogenes? streptolysin O

Mediate adherence of bacteria


What are two functions of the to the cell surface - sex pilus
pilus/fimbrae? forms attachment b/t 2
bacteria during conjugation

1) Neisseria memingitidis:
What are two species of Gram
maltose fermenter 2)
(-) cocci and how are they
Neisseria gonorrhoeae:
differentiated?
maltose NONfementer
What bacteria produces a
Pseudomonas aeruginosa
blue-green pigment?

What bacteria produces a red Serratia marcescens


pigment? ('maraschino cherries are red')

What bacteria produces a Staph. aureus (Aureus= gold


yellow pigment? in Latin)

What culture requirements do


Sabouraud's agar
Fungi have?
What culture requirements do
MacConkey's agar (make pink
Lactose-fermenting enterics
colonies)
have?

What culture requirements


Bordet-Gengou (potato) agar
does B. pertussis have?

What culture requirements


Tellurite agar
does C. diphtheriae have?

What culture requirements chocolate agar with factors V


does H. flu have? (NAD) and X (hematin)
What culture requirements Charcol yeast extract agar
does Legionella pneumophia buffered with increased iron
have? and cysteine

What culture requirements


Thayer-Martin (VCN) media
does N. gonorrhea have?

What G- rod is a lactose


nonfermenter and is Oxidase Pseudomonas
+?

What is a function of the


site of oxidative and
plasma membrane in
transport enzymes
bacterial cells.
exotoxin treated with
formaldehyde (or acid or
What is a toxoid?
heat); retains antigeniciy but
looses toxicity

What is an acronym for


TRMMLC: These Rascals May
remembering 6 bacteria that
Microscopically Lack Color
don't Gram's stain well?

On a Blood agar plate:


What is meant by alpha, beta, alpha= complete; clear -
and gamma hemolysis? beta= partial; green -
gamma= no hemolysis; red

What is the chemical


polysaccharide
composition of a glycocalix?
What is the chemical
RNA and protein in 30S and
composition of bacterial
50S subunits
ribosomes?

What is the chemical


Lipopolysaccharide
composition of endotoxin?

What is the chemical


polypeptide
composition of exotoxin?

What is the chemical keratin-like coat - dipicolinic


composition of spores? acid
What is the funciton and
chemical composition of the for motility - made of protein
flagellum?

mediates adherence to
What is the function of a
surfaces, especially foreign
glycocalix?
surfaces (i.e. catheters)

provides resistance to
What is the function of
dehydration, heat, and
spores?
chemicals

Polysaccharide (*except
What is the major chemical
Bacillus anthracis, which
composition of the capsule?
contains D-Glutamate)
What is the major function of
antiphagocytic
the capsule?

What is the mode of action of


includes TNF and IL-1
endotoxin?

What is the nature of the DNA


Chromosomal or plasmid
transferred in conjugation?

Any gene in generalized


What is the nature of the DNA transduction; only certain
transferred in transduction? genes in specialized
transduction
What is the nature of the DNA
Any DNA
transferred in transformation?

the space between the inner


What is the periplasm? Where
and outer cell membranes
is it found?
found in G(-) bacteria.

What is the primary test to


are they Lactose Fermenters?
subcatergorize G- rods?

cell wall of most G- bacteria


What is the source of
(think N-dotoxin=gram
endotoxins?
Negative)
What is the source of certain species of some G+
exotoxins? and G- bacteria

What is the unique chemical


component of Gram (-) cell Lipopolysaccharide
membranes?

What is the unique chemical


component of Gram + cell Teichoic acid
membranes?

What is used to stain


Use silver stain.
Legionella?
What species is Group A
S. pyogenes
Strep?

What species is Group B


S. agalactiae
Strep?

What stain is amyloid and


gives an apple-green
Congo red
birefringence in polarized
light?

What stain is used for acid


Ziehl-Neelsen
fast bacteria?
What stain is used for
Borrelia, Plasmodium,
Giemsa's
trypanosomes, and
Chlamydia?

What stain is used for


India ink
Cryptococcus neoformans?

What stains gylcogen,


mucopolysaccharides and is
PAS (periodic acid Schiff)
used to diagnose Whipple's
disease?

Staph. are Catalase (+) and


What test distinguishes Staph.
are in clusters - Strep. are
and Strep?
Catalase (-) and are in chains
What test distinguishes Staph. S. aureus is Coagulase (+) -
aureus from Staph. S. epidermidis and S.
epidermidis and Staph. saprophyticus are Coagulase
saprophyticus? (-)

S. pneumoniae: have Capsule;


What two things distinguish
Optochin Sensitive - Viridans
S. pneumoniae from Viridans
strep: No capsule; Optochin
Strep.?
Resistant

What type of enzymes allows


certain bacteria to colonize IgA proteases
mucosal surfaces?

When endotoxin activates


C3a: hypotension, edema -
complement, what are the
C5a: neutrophil chemotaxis
secondary effects?
When endotoxin activates
Hageman, what are the coagulation cascade: DIC
secondary effects?

When endotoxin activates


IL-1--fever - TNF--fever,
macrophages, what 3
hemmoragic tissue necrosis
cytokines are released and
- Nitic oxide--hypotension,
what are the secondary
shock
effects?

Where are the genes for


on the bacterial chromosome
endotoxin located?

Where are the genes for on a plasmid or in a


exotoxin located? bacteriophage
Where are the spores of
canned food - honey
Clostridium botulinum found?

in the outer membrane of G


Where is LPS found?
(-) cell walls

EXOTOXIN: fatal dose is ~1ug!


Which has a higher toxicity:
(for endotoxin, fatal dose is
exotoxin or endotoxin?
hundreds of micrograms)

Which type of Neisseria Gonococci (Glucose=


ferment Glucose only? Gonococci)
Meningococci
Which type of Neisseria
(MaltoseGlucose=
ferment maltose and glucose?
MeninGococci)

Which types of transfer can


only transformation
eukaryotic cells do?

all 3: conjugation,
Which types of transfer can
transduction, and
prokaryotic cells do?
transformation

Why don't Mycobacteria high lipid content cell wall


Gram's stain well? requires acid-fast stain
Why don't Mycoplasma
no cell wall
Gram's stain well?

Why don't Rickettsia, they are intracellular


Chlamydia, and Legionella (Legionella is Mainly
Gram's stain well? intracellular)

too thin to be visualized (use


Why don't Treponema Gram's
darkfield microscopy and
stain well?
antibody staining)

Name 5 species of bacteria


Borrelia burgdorferi -
that are transmitted to
Brucella spp. - Francisella
humans from animals.
tularensis - Yersinia pestis -
(Acronym: BBugs From Your
Pasteurella multocida
Pet.)
All Rickettsiae (except one arthropod (Coxiella is
genus) are transmitted by atypical: transmitted by
what type of vector? aeresol)

G- bugs are resistant to PenG


but may be susceptible to
Are G(-) bugs resistant to Pen
pen. derivative like ampicillin.
G? to ampicillin? to
The G- outer mb inhibits
vancomycin?
entry of PenG and
vancomycin.

Strep. pneumoniae is
Are Strep. pneumoniae
optochin-Sensitive - Viridans
sensitve to optochin? Are
streptococci is optochin-
Viridans strep.?
Resistant

Are Strep. pyogenes


YES. both are catalase +
Bacitracin-sensitive?&gt;
Are Viridans strep. alpha,
alpha
beta, or non-hemolytic?

Because of drug resistance,


what in an alternate rifampin with dapsone and
treatment combination for clofazimine
leprosy?

Besides the rash, what other


body systems are affected by joints -CNS -heart
Lyme disease? (3)

Aerobic, G(-) rod. - Non-


Describe lab-findings for lactose fermenting - Oxidase
positive - Produces
Pseudomonas aeruginosa.
pyocyanin (blue-green
pigment)
Describe the disease often resistant to multiple
associated with M. avium- drugs; causes disseminated
intracellulare. disease in AIDS.

contains type b capsulare


polysaccharide conjugated to
Describe the H. flu vaccine.
diphtheria toxoid or other
When is it given?
protein. -Given b/t 2m and
18m.

Describe the typical findings 1) Ferments lactose 2) watery


with diarrhea caused by diarrhea 3) no fever/
enterotoxigenic E. coli. (3) leukocytosis

1) Comma-shaped organisms
Describe the typical findings
2) rice-water stools 3) no
with Vibro cholerae. (3)
fever/leukocytosis
Do Streptococcus pneumonia
have catalase? Do Viridans YES. both are catalase +
Strep. have catalase?

Enterococci are hardier than


nonenterococcal group D
6.5% NaCl (used as lab test)
bacteria. What lab conditions
can they grow in?

Following primary infection


-dormant tubercle bacilli
with TB, if preallergic
form in several organs -
lymphatic or hematogenous
REACTIVATION can occur in
dissemination occurs, what
adult life
follows?

Following primary infection


Miliary tuberculosis and
with TB, if severe bacteremia
possibly death
occurs, what follows?
Following primary infection Immunity and
with TB, if the lesion heals by hypersensitivity---&gt;
fibrosis, what is the result? tuberculin positive

Following primary infection


with TB, under what HIV, malnutrition. This
conditions would the lesion progressive lung disease can
likely progress to lung rarely lead to death.
disease?

1) Heals by fibrosis 2)
Following primary infection
Progressive lung disease 3)
with TB, what are 4 possible
Severe bacteremia 4)
courses the disease could
Preallergic lymphatic or
take?
hematogenous dissemination

Give 3 examples of obligate Clostridium - Bacteroides -


anaerobes. Actinomyces
1) burn wound infection 2)
Give 3 types of infection
nosocomial pneumonia 3)
Pseudomonas aeruginosa is
pneumonia with cystic
commonly responsible for.
fibrosis

1) Strep. pneumoniae 2)
Haemophilus influenza
Give 4 examples of
(especially b) 3) Neisseria
encapsulated bacteria.
memingitidis 4) Klebsiella
pneumoniae

Epiglottitis -Meningitis -
H. flu causes what? (4) Otitis media -Pneumonia
(haEMOPhilus)

using aniline dyes (Wright's or


How are Borrelia visualized? Giemsa stain) in light
microscopy
How are Mycobacteria acid-fast stain =Ziehl-
visualized in the lab? Neelson

How are Treponema


by dark-field microscopy
visualized?

1) Reinfection of partially
immune hypersensitized hosts
How can secondary (usu. adults) =exogenous
tuberculosis in the lung source 2) Reactivation of
occur?(2) dormant tubercle bacilli in
immunocompromised or
debilitated hosts
=endogenous source

How can you remember that


they live in the mouth and are
Viridans strep are resistant to
not afraid of the (opto-)CHIN
optochin?
How does primary syphilis with a painless chancre
present? (localized disease; 2-10 wks).

disseminated disease (1-3m


later) with constitutional
How does secondary syphilis
symptoms, maculopapular
present?
rash, condylomata lata
(genital lesions)

gummas (granulomas),
How does tertiary syphilis aortitis, neurosyphilis (tabes
present? dorsalis), Argyll-Robertson
pupil

via exotoxin encoded by


How does the bacterium beta-prophage; exotoxin
cause the disease? inhibits protein synthesis via
ADP-ribosylation of EF-2
typhus: maculopapillary rash
BEGINS ON TRUNCK, moves
How does the rash with
peripherally -RMSF: macules
typhus differ from the rash
progressing to petichiae
with RMSF?
BEGIN ON HANDS &amp;FFET
and move inward.

How is Brucellosis/Undulant dairy products, contact with


fever transmitted? animals

How is Cellulitis transmitted? Animal bite; cats, dogs

How is H. flu transmitted? aeresol


use silver stain (doesn't Gram
How is Legionnaires' disease stain well) -culture with
diagnosed in lab? charcoal yeast extract with
iron and cysteine.

aeresol transmission from


How is Legionnaires' disease envirnomental water source
transmitted? habitat (NO human-to-
human transmission).

How is Lyme disease Tick bite; Ixodes ticks that


transmitted? live of deer and mice

How is Shigella spread? food, fingers, feces, and flies'


How is the Plague Flea bite; rodents, especially
transmitted? prairie dogs

How is Tuleremia
Tick bite; rabbits, deer
transmitted?

Is Bacillus anthracis G+ or
It is a G+, spore-forming rod
G-? What is its morphology?

Is there an animal reservoir


Yes, armadillos in the US
for leprosy?
Polyarthritis - Erythema
List 5 findings associated
marginatum -Chorea -
with rheumatic fever. (Hint:
Carditis - Subcutaneous
PECCS)
nodules

ADP ribosylation -Beta-


List the 'ABCDEFG' of prophage -Corynebacterium
diphtheria. - Diphtheria - Elongation
Factor 2 - Granules

Name 2 alpha-hemolytic Strep. pneumoniae - Viridans


bacteria. streptococci

Name 2 bugs that cause


diarrhea but NOT fever and E. coli and Vibro cholerae
leukocytosis?
Name 2 disease processes
1) UTI 2) subacute
that can be caused by
endocarditis
enterococci.

Name 2 species of Enterococcus faecalis -


enterococci. Enterococcus faecium

pseudomembraneous
Name 2 symptoms of pharyngitis (grayish white
diphtheria. membrane) -
lymphadenopathy

Bacillus anthracis -
Name 3 spore forming
Clostridium perfringens - C.
bacteria.
tetani
1) Staph. aureus 2) Strep.
Name 4 beta-hemolytic pyogenes (GAS) 3) Strep.
bacteria. agalactiae (GBS) 4) Listeria
monocytogenes

Klebsiella -E. coli -


Name 4 lactose-fermenting Enterobacter Citrobacter
enteric bacteria. (think Lactose is KEE for first
three listed)

Norcardia - Pserudomonas
Name 4 obligate aerobic
aeruginosa - Mycobacterium
bacteria.
tuberculosis - Bacillus

1) Vibrio cholerae 2)
enterotoxigenic E. coli 3)
Name 5 bugs that cause
watery diarrhea. viruses (rotavirus) 4)
protozoa (Cryptosporidium
and (5) Giardia)
1) Salmonella 2) Shigella 3)
Campylobacter jejuni 4)
Name 6 bugs that cause enterohemorrhagic/
bloody diarrhea. enteroinvasive E.coli 5)
Yersinia enterocilitica 6)
Entamoeba histolytica (a
protozoan)

1) Mycobacterium 2) Brucella
Name 7 faculatative 3) Francisella 4) Listeria 5)
intracellular bacteria. Yersinia 6) Legionella 7)
Salmonella

Borrelia (big size) -


Name three genera of
Leptospira -Treponema
spirochetes.
(think: BLT; B is big)

Name two lab tests used to


VDRL and FTA-ABS
detect syphilis?
Name two non-lactose
fermenting bacteria that
Salmonella and Shigella
invade intestinal mucosa and
can cause bloody diarrhea.

Rickettsia and Chlamydia


Name two obligate
(Hint: 'stay inside when its
intracellular bacteria.
Really Cold.')

RMSF is endemic to what part the East Coast (in spite of the
of the US? name)

Spore are formed by certain Gram+ rods, usually in soil;


species of what type of form spores only when
bacteria? nutrients are limited
T/F Chlamydia are obligate
intracellular parasites that TRUE
cause mucosal infections.

T/F Chlamys means cloak. TRUE (intracellular)

T/F Enterobacteriaceae are


oxidase negative and are TRUE
glucose fermenters.

T/F H. pylori infection is a


risk factor for peptic ulcer TRUE
and gastric carcinoma.
T/F Penicillin is not an TRUE Mycoplama are
effective treatment against naturally resistant b/c they
Mycoplasma pneumoniae. have no cell wall.

TRUE: endotoxin---&gt;
T/F Pseudomonas produces
fever, shock -exotoxin---
both endotoxin and exotoxin.
&gt; inactivates EF-2

T/F Rickettsiae are obligate


intracellular parasites and TRUE
need CoA and NAD.

FALSE: ALL enterococci are


T/F Some enterococci are
naturally resistant to Pen/
resistant to PenG.
cephlosporins.
T/F Spores have no metabolic
TRUE
activity.

T/F: S. aureus food


FALSE: rapid onset of S.
poisoning is due to the
aureus food poisoning is due
ingestion of bacteria that
to injestion of PREFORMED
rapidly secrete toxin once
toxin
they enter the GI tract.

The Weil-Felix reaction


Positive: typhus and RMSF -
usually tests positive for what
Negative: Q fever -Cross-
two diseases? Negative for
reacts: with Proteus antigen
what? Cross reacts with what?

Think COFFEe for Capsular -O-antigen -


Enterobacteriaceae. What Flagellar antigen -Ferment
does that stand for? glucose -Enterobacteriaceae
Nagging Pests Must Breath
What's a pneumonic for
(=Norcardia - Pserudomonas
remembering 4 obligate
aeruginosa - Mycobacterium
aerobes?
tuberculosis - Bacillus

burn-wound infections -
Pneumonia (esp. in cystic
What (6) infections can
fibrosis) -Sepsis (black skin
Pseudomonas aeruginosa
lesions) -External Otitis
cause?
(swimmer's ear) - UTI -hot
tub folliculitis

What 2 bugs can cause


bloody diarrhea, fever, and
Salmonella and Shigella
leukocytosis, but do not
ferment lactose?

The Ixodes tick transmits it.


What animals carry Lyme - Deer are required for tick
disease? life cycle. - Mice are
important resservoirs.
What anitbody class is
necessary for an immune
IgG2.
response to encapsulated
bacteria?

What are 2 disease processes 1) dental caries: Strep.


caused by Viridans strep and mutans 2) bacterial
what species are responsible? endocarditis: Strep. sanguis

(1) bismuth (Pepto-Bismal),


metronidazole, and
What are 2 options for triple tetracyclin or amoxicillin. OR
thearpy treatment of H. (2) metronidazole,
pylori? omeprazole, and
clarithromycin (#2 is more
expensive)

FTA-ABS is 1) more specific


What are 3 advantages/
2) positive earlier in disease
differences between VDRL
3) remains positive longer
and FTA-ABS?
than VDRL
1) Pyogenic--pharyngitis,
cellulitis, skin infection 2)
What are 3 disease processes Toxigenic--scarlet fever, TSS
caused by Strep. pyogenes? 3) Immunologic--rheumatic
fever, acute
glomerulonephritis

1) Viruses (mono, hepatitis)


What are 4 biological false 2) Drugs 3) Rheumatic fever
positives for VDRL? and rheumatic arthritis 4)
Lupus and leprosy (=VDRL)

1) insidious onset 2)
What are 4 clinical symptoms headache 3) nonproductive
of 'walking' pneumonia? cough 4) diffuse interstitial
infiltrate

What are 4 clinical symptoms 1) fever 2) night sweats 3)


of TB? weight loss 4) hemoptysis
1) CNS (parenchmal
What are 5 areas that can be tuberculoma or meningitis)
affected by extrapulmonary 2) Vertebral body (Pott's
TB? disease) 3) Lymphadenitis 4)
Renal 5) GI

culture on chocolate agar


with factor V (NAD) and X
What are the culture (hematin). [Think: 'Child has
requirement for H. flu? 'flu'; mom goes to five (V) and
dime (X) store to buy
chocolate.']

cytoplasmic inclusions on
What are the lab findings with
Giemsa fluorescent antibody-
Chlamydia?
stains smear

1) rash on palms and soles


What are the symptoms of (migrating to wrists, ankles,
RMSF? (3) then trunck) 2) headache 3)
fever
1) erythema chronicum
migrans, flu-like symptoms
What are the three stages of 2) neurologic and cardiac
Lyme disease? manefestations 3)
autoimmune migratory
polyarthritis

1) Elementary body (small,


dense): Enters cell via
What are the two forms of
endocytosis 2) Initial or
chlamydia?
Reticulate body: Replicates in
the cell by fission

1) lepromatous- failed cell-


What are the two forms of
mediated immunity, worse 2)
leprosy (or Hansen's disease)?
tuberculoid- self-limited.

What are two drugs that could


be used to treat 'walking' tetracycline or erythromycin
pneumonia?
What are two drugs that could
erythromycin or tetracycline
treat Chlmydia?

What are two lab findings 1) X-ray looks worse than


associated with 'walking' patient 2)High titer of cold
pneumonia? agglutinins (IgM)

Clostridium difficile; it kills


What are usually associated enterocytes, usu. is
with pseudomembraneous overgrowth secondary to
colitis? antibiotic use (esp.
clindamycin or ampicillin)

What bacteria are G+, spore-


Clostridia
forming, anaerobic bacilli?
What bacteria causes a
malignant pustule (painless
ulcer); black skin lesions that Bacillus anthracis
are vesicular papules covered
by a blak eschar?

What bacteria exhibits a


'tumbling' motility, is found in
unpasteurized milk, and Listeria monocytogenes
causes meningitis in
newborns?

What bacteria is catalase(-)


Strep. agalactiae
and bacitracin-resistant?

What bacteria is catalase(-)


Strep. pyogenes
and bacitracin-sensitive?
What bacteria is catalase+
Staph. aureus
and coagulase+?

What bacteria produces


alpha-toxin, a hemolytic
lecithinase that causes Clostridium perfringens
myonecrosis or gas
gangrene?

What bacterium causes


Pasteurella multocida
Cellulitis?

What bacterium causes


Mycobacterium leprae
leprosy?
What bacterium causes Lyme
Borrelia burgdorferi
disease?

What bacterium causes the


Yersinia pestis
Plague?

What bacterium causes


Francisella tularensis
Tularemia?

What bacterium causes Brucella spp. (a.k.a.


Undulant fever? Brucellosis)
What bug causes atypical
Mycoplama pneumoniae
'walking' pneumonia?

What bug causes


gastroenteritis and up to 90% Helicobacter pylori
of duodenal ulcers?

What bug causes


Legionella pneumophila
Legionnaire's disease?

What bug is associated with


Pseudomonas aeruginosa
burn wound infections?
What bug is comma- or S-
shaped and grows at 42C,
Campylobacter jejuni
and causes bloody diarrhea
with fever and leukocytosis?

What bug that causes


diarrhea is usually
Yersinia enterocolitica
transmitted from pet feces
(e.g. puppies)?

Clostridium tetani: exotoxin


What causes tetanus? (give produced blocks glycine
bacteria and disease process) release (inhibitory NT) from
Renshaw cells in spinal cord

NOT H. flu -it is caused by


What causes the flu?
influenza virus
What chemical is found in the
dipicolinic acid
core of spores?

What coccobacillus causes


vaginosis: greenish vaginal
Gardnerella vaginalis
discharge with a fishy smell;
nonpainful?

Whooping cough: toxin


permanently disables G-
What disease does Bordetella protein in respiratory mucosa
perussis cause? How? (turns the 'off' off);ciliated
epithelial cells are killed;
mucosal cells are overactive.

Cholera: toxin permanently


activates G-protein in
What disease does Vibrio
cholerae cause? How? intestinal mucosa (turns the
'on' on) causing rice-water
diarrhea
What disease is caused by
Lyme Disease
Borrelia?

Botulism: associated with


contaminated canned food,
What disease is caused by
produces a preformed, heat-
Clostridium botulinum? What
labile toxin that inhibits ACh
pathophys. does it cause?
release---&gt; flaccid
paralysis.

What diseases (2) are caused Syphilis (T. pallidum) -yaws


by Treponema? (T. pertenue; not and STD)

Inflammatory disease: skin


infections, organ abcess,
pneumonia - Toxin-
What diseases can be caused mediated disease: Toxic
by Staph. aureus? Shock Syn., scalded skin
syndrome (exfoliative toxin),
rapid onset food poisoning
(enterotoxins)
What do Chlamydia chronic infection, cause
trachomatis serotypes A, B, blindness in Africa (ABC=
and C cause? Africa / Blindness / Chronic

What do Chlamydia urethritis/ PID - neonatal


trachomatis serotypes D-K pneumonia -neonatal
cause? (3) conjuctivitis

What do Chlamydia lymphogranuloma venereum


trachomatis serotypes L1,L2, (acute lymphadentis: positive
and L3 cause? Frei test)

rash on palm and sole is seen


What do RMSF, syphilis, and
in each (coxasackievirus A
coxsackievirus A infection
=hand, foot, and mouth
have in common?
disease)
it degrades H2O2, an
What does catalase do? Which antimicrobial product of
bacteria have it? PMNs. - Staphlococci make
catalase; Strep. do NOT.

What does the H-antigen H: flagellar antigen, found on


represent? motile species

What does the K-antigen K: capsular, relates to


represent? virulence

O-antigen is the
What does the O-antigen
polysaccharide of endotoxin
represent?
(found on all species)
What does VDRL detect? (It
detects antibody that reacts
detects non-specific antibody
with beef cardiolipin
that reacts with what?)

What drug of choice is used


to treat Norcardia? Sulfa for Norcarida,
Actinomyces? (Acronym: Actinomyces gets Penicillin
SNAP)

What enteric bacterial


infection may be prolonged Salmonellosis
with antibiotic treatment?

What enzyme allows H. pylori urease (cleaves urea to


to creat an alkaline ammonia); used in urease
environment? breath test
What family includes E. coli,
Salmonella, Klebsiella,
Enterobacteriaceae
Enterobacter, Serratia, and
Proteus?

What family of bacteria uses


the O-, K-, and H-antigen Enterobacteriaceae
nomenclature?

1) antiphagocytic 2) antigen
What function does the
in vaccines (Pneumovax, H.
capsule serve? (2: one for the
flu b, meningococcal
bacterium, one other)
vaccines)

What G+ anaerobe causes


oral/facial abscesses with
'sulfur granules' that may Acinomyces israelii
drain through sinus tracts in
skin?
What G+ and also weakly acid
fast aorobe found in soil
causes pulmonary infections Norcardia asteroides
in immunocompromised
patients?

What general type of bacteria


are normal flora in GI tract Anaerobes
but pathogenic elsewhere?

What general type of bacteria


Lactose-fermenting enteric
grow pink colonies on
bacteria
MacConkey's agar?

Occurs in Primary TB (usually


a child) -Ghon complex=
What is a Ghon complex and draining Hilar nodes and
in whom does it occur? Ghon focus, exudative
parenchymal lesion (usu. in
LOWER lobes of lung)
G+ rods with metachromatic
What is a lab diagnosis of
granules; grows on tellurite
diphtheria based on?
agar. (Coryne=club shaped)

What is a major difference


Salmonella are motile;
between Salmonella and
Shigella are nonmotile
Shigella observable in the lab?

if encapsulated bug is
What is a positive Quellung present, capsule SWELLS
reaction? when specific anticapsular
antisera are added.

What is notable about


has an avian reservoir
Chrmydia psittaci?
What is one reason M. leparae
infects skin and superficial It likes cool temperatures
nerves?

erythema chronicum migrans,


What is the classic symptom
an expanding 'bull's eys' red
of Lyme Disease?
rash with central clearing.

What is the classic triad of


1) headache 2) fever 3) rash
symptoms associated with
(vasiculitis)
Rickettsiae?

What is the common


Fibrocaseous cavitary lesion
manifestation of secondary
usu. in APICIES of lung
TB?
What is the common site of
the apicies of the lung (which
infection for Mycobacterium
have the highest PO2)
tuberculosis?

What is the D.O.C. to treat


Metroidazole
Gardnerella vaginalis?

What is the DOC for treating


tetracycline
rickettsial infections?

What is the DOC for


treatment of most rickettsial tetracycline
infections?
What is the DOC to treat
tetracycline
Lyme Disease?

What is the DOC to treat


Penicillin G
syphilis?

What is the drug of choice for


Treat meningitis with
H. flu meningitis? What DOC
CEFTRIAXONE; Rifampin for
for prophylaxis in close-
prophylaxis.
contacts?

What is the drug of choice for


Erythromycin
Legionaires' disease?
What is the morphology of H.
Small G(-) (coccobacillary) rod
flu?

What is the morphology of H.


Gram (-) rod
pylori?

What is the primary drug dapsone (toxicity is hemolysis


used to treat leprosy? and methemoglobinemia)

What is the recommended aminoglycoside plus


treatment for Pseudomonas extended-spectrum penicillin
aeruginosa infection? (e.g. piperacillin or ticarcillin)
What is the source of
infection and the bacterium R. typhi; from fleas
that causes endemic typhus?

What is the source of


R. prowazekii; from human
infection and the bacterium
body louse
that causes epidemic typhus?

What is the source of


Coxiella burnetii; from
infection and the bacterium
inhaled aersols
that causes Q fever?

What is the source of


infection and the bacterium Rickettsia rickettsii; from tick
that causes Rocky Mountain bite
Spotted Fever?
TSST-1; it is a superantigen
What is the toxin responsible that binds to class II MHC and
for TSS is Staph. aureus? T-cell receptors---&gt;
polyclonal T-cell activation

What is the unique


component found in
cholesterol
Mycoplamsa bacterial
membranes?

What is the unique feature of its peptidoglycan wall lacks


Chlamydiae cell walls? muramic acid

inhalation anthrax; can cause


What is woolsorter's disease?
life-threatening pneumonia
What lab test assays for
Weil-Felix reaction
antirickettsial antibodies?

What Lancefield Antigen


Group D
Group are enterococci in?

They are non-typealbe. They


What Lancefield Antigen do not have a C-carbohydrate
Group are Viridans strep in? on their cell wall to be
classified by.

autoclaving; they are highly


What level of disinfection is
resistant to destruction by
required to kill spores?
heat and chemicals
What populations are most
patients younger than age 30
likely to get Mycoplama
- military recruits - prisons
pneumoniae infection?

What rickettsial disease is


atypical in that it has no rash,
no vector, negative Weil-Felix Q fever (Coxiella burnetii)
reaction, and its causative
organism can survive outside
for a long time?

What species are associated


Vibrio parahaemolytica and
with food poisoning in
Virbrio vulnificus
contaminated seafood?

What species causes


Corynebacterium diptheriae
diphtheria?
What species is associated
with food poisoning in
Clostridium botulinum
improperly canned foods
(bulging cans)?

What species is associated Staphylococcus aureus (this


with food poisoning in meats, food poisoining usu. starts
mayonnaise, and custard? quickly and ends quickly)

What species is associated


with food poisoning in Salmonella
poultry, meat, and eggs?

What species is associated


with food poisoning in Clostridium perfringens
reheated meat dishes?
What species is associated Bacillus cereus ('Food
with food poisoning in poisoning from reheated rice?
reheated rice? Be serious!')

What species is associated


with food poisoning in
E. coli 0157-H7
undercooked meat and
unpasteurized juices?

What species of Mycobacteria


causes pulmonary, TB-like M. kansasii
symptoms?

What strain of Haemophilus


influenza causes most capsular type b
invasive disease?
What symptoms are
associated with M. cervical lymphadenitis in kids
scrofulaceum

Viridans are resistant to


What test differentiates
optochin; S. pneu. are
Viridans from S. pneumoniae?
sensitive to optochin

What two bugs secrete


exotoxins that act via ADP
ribosylation of G-proteins, Vibrio cholerae - Bordetella
permanently activating adenyl pertussis
cyclase (resulting in increased
cAMP)?

What two genera of G+ rods


form long branching Acinomyces and Nocardia
filaments resembling fungi?
What type of bacteria are
difficult to culture, produce
gas in tissue (CO2 and H2), Anaerobes
and are generally foul-
smelling?

What type of bacteria is


associated with rusty sputum,
Pneumoccocus
sepsis in sickle cell, and
splenectomy?

What type of E. coli are


enterohemmoragic/
associated with bloody
enteroinvasive E. coli
diarrhea?

What type of immunologic


response is elicited by a monocyte response
Salmonella infection?
arthritis - conjunctivitis -
What types of infection can
pneumonia - nongonococcal
chlamydia cause? (4)
urethritis

What virulence factor of


Staph. aureus binds Fc-IgG,
Protein A
inhibiting complement
fixation and phagocytosis?

What virulence factor of


Strep. pyogenes also serves
M-protein
as an antigen to which the
host makes antibodies?

What will likely be visible


under the microscope in the Clue cell, or vaginal epithelial
case of Gardnerella vaginallis cells covered with bacteria
infection?
Where are Viridans strep.
normal flora of oropharynx
found (reservoir)?

Where are when is Lyme common in northeast US in


disease common? summer months

Which disease/toxin causes


Pertussis toxin: by inhibiting
lymphocytosis? (Cholera or
chemokine receptors
Pertussis)

Which has an animal


Salmonella: poultry, meat,
reservoir? (Salmonella or
eggs
Shigella)
Which is more specific for
FTA-ABS is more specific
syphilis: VDRL or FTA-ABS?

Which is more virulent? Shigella (10^1 organisms vs.


(Salmonella or Shigella) Salmonella 10^5 organisms)

Which is motile? (Salmonella Salmonella (think: salmon


or Shigella) swim)

Which species of chlamydia


causes and atypical C. pneumonia -transmitted
pneumonia? How is it via aeresol
transmitted?
Which two species of
C. trachomatis -C.
chlamydia infect only
pneumoniae
humans?

they lack catalase and/or


Why are anaerobes
oxidase and are susceptible
susceptible to oxygen?
to oxidative damage

M.tuberculosis is an aerobe;
Why does TB usually infect
there is more oxygen at the
the upper lobes of the lung?
apicies

Why must rickettsia and


they can't make their own
chlamydia always be
ATP
intracellular?
Are most fungal spores
yes
asexual?

Are most P. Carinii infections


no, most of are asymptomatic
symptomatic?

Are the above mentioned yes, except coccioidomycosis


systemic mycoses dimorphic? which is a spherule in tissue

How do the S. Schenckii yeast


Cigar-shaped budding yeast
appear in the pus?
How do you diagnose
cysts on acid fast stain
cryptosporidium?

How do you diagnose


Trophozoites or cysts in stool
giardiasis?

How do you get P. Carinii? Inhalation

fluconazole or ketoconazole
How do you treat systemic for local infection,
mycoses? amphotericin B for systemic
infection
Itraconazole or Potassium
How do you Tx S. Schenckii?
Iodide

Mold with septate hyphae


How does Aspergillus appear that branch at a V-shaped (45
microscopically? degree angle) , they are NOT
dimorphic

It is a mold with irregular


How does Mucor species
nonseptate hyphae branching
appear microscopically?
at wide angles&gt;90 degrees

How does
Captain's wheel' appearance
Paracocciodioidomycosis
(like on a sailboat)
appear histologically?
How is Clonorchis sinensis undercooked fish; causes
transmitted and what disease inflammation of the biliary
results? tract

snails are host; cercariae


How is Schistosoma penetrate skin of humans;
transmitted and what disease causes granulomas, fibrosis,
results? and inflammation of the
spleen and liver

How is Ancylostoma Larvae penetrate skin of feet;


Duodenale transmitted and intestinal infection can cause
what disease results? anemia

How is Ascaris Lumbricoides


Eggs are visible in feces;
transmitted and what disease
intestinal infection
results?
How is cryptosporidium
Cysts in Water
transmitted?

How is Dracunculus
In drinking water; sink
medinensis transmitted and
inflammation and ulceration
what disease results?

Eggs in dog feces cause cysts


How is E. granulosis
in liver; causes anaphylaxis if
transmitted and what disease
echinococcal antigens
results?
released from cysts

How is E. Histolytica
Cysts in Water
transmitted?
How is Enterobius food contaminated with eggs;
Vermicularis transmitted and intestinal infections; causes
what disease results? anal pruritus

How is giardia transmitted? Cysts in Water

Transmitted by deer fly;


How is Loa loa transmitted causes swelling the in the
and what disease results? skin (can see worm crawling
in conjunctiva)

How is malaria dx? Blood smear


How is malaria transmitted? mosquito (Anopheles)

How is Onchocerca volvulus transmitted by female


transmitted and what disease blackflies; causes river
results? blindness

Undercooked crab meat;


How is Paragonimus
causes inflammation and
Westermani transmitted and
secondary bacterial infection
what disease results?
of the lung

How is Sporothrix schenckii Dimorphic fugus that lives on


appear under the scope? vegetation
How is Strongyloides
larvae in soil penetrate the
Stercoralis transmitted and
skin; intestinal infection
what disease results?

food contaminated with eggs;


How is T. Canis transmitted
causes granulomas (if in
and what disease results?
retina=blindness)

undercooked pork tapeworm;


How is T. Solium transmitted
causes mass lesions in the
and what disease results?
brain, cysticercosis

How is T. Vaginalis
sexually
transmitted?
How is Toxo transmitted? cysts in meat or cat feces

How is Trichinella Spiralis undercooked meat, usually


transmitted and what disease pork; inflammation of muscle,
results? periorbital edema

female mosquito; causes


How is Wucheria transmitted
blockage of lymphatic vessels
and what disease results?
(elephantiasis)

In what cells do you find


macrophages
histoplasmosis?
Is Pneumocystis Carinii a Yes, but originally classified
yeast? as a Protozoa

budding yeast with


Microscopically how does
pseudohyphae, germ tube
Candida appear?
formation at 37 degrees C)

Schistosoma, Clonorchis
Name 3 Trematodes (Flukes) . sinensis, Paragonimus
Westermani

Candida Albicans, Aspergillus


Name 4 opportunistic fungal fumigatus, Cryptococcus
infections. Neoformans, Mucor and
Rhizopus species
Coccidiomycosis,
Histoplasmosis,
Name 4 systemic mycoses.
Paracoccidioidomycosis,
Blastomycosis

Name two asexual spores Hisoplasmosis and


transmitted by inhalation. Coccidiodomycosis

Name two Cestodes Taenia Solium, and


(Tapeworms) . Echinococcus Ganulosus

What agar is used to culture


Sabouraud's Agar
for systemic mycoses?
asexual fungal spores (ex.
What are Conidia? Blastoconidia, and
arthroconidia)

Thrush in Immunocompromised
pts (neonates, patients on
steroids, diabetics and AIDS
What are some common pts) , endocarditis in IV drug
Candida infections? users, vaginitis (high pH,
Diabetes, post-antibiotic) ,
diaper rash, disseminated
candidiasis (to any organ)

What are some infections Cryptococcal meningitis,


caused by cryptococcus? cryptococcosis

Ancylostoma duodenale
(hookworm) , Ascaris
Lumbricoides, Enterobius
What are the 10 Nematodes Vermicularis (pinworm) ,
(roundworms) we are Strongyloides stercoralis,
concerned with? Trichinella Spiralis, Dracunculus
Medinensis, Loa loa, Onchocerca
Volvulus, Toxocara Canis,
Wucheria Bancrofti
What are the 4 B's of
Big, Broad-Based, Budding
Blastomycosis?

Severe diarrhea in AIDS, Mild


What are the diseases caused
disease (watery diarrhea) in
by Cryptosporidium?
non-HIV

What are the diseases caused Brain Abscess in HIV and birth
by Toxoplasma? defects

Ear fungus, Lung cavity


What are the infections
Aspergilloma ('fungus ball') ,
caused by Aspergillus?
invasive aspergillosis.
What can systemic mycoses
TB (granuloma formation)
mimic?

Amebiasis: bloody diarrhea,


What disease are caused by
dysentery, liver abscess, RUQ
Entamoeba Histolytica?
pain

What disease does Mucor


Mucormycosis
species cause?

What disease does Pneumocystis Carinii


Pneumocystis carinii cause? Pneumonia
What disease does Sporothrix
Sprotricosis
Schenckii cause?

What disease does Chaga's Disease (heart


Trypanosoma Cruzi casue? disease)

Giardiasis: bloating,
What disease is caused by
flatulence, foul-smelling
Giardia Lamblia?
diarrhea

What disease is caused by Visceral Leishmaniasis (Kala-


Leishmanina donovani? azar)
What disease is caused by P. Diffuse interstitial pneumonia
Carinii? in HIV

What disease is caused by the Malaria: cyclic fever,


plasmodium species (vivax, headache, anemia,
ovale, malariae, falciparum) ? splenomegaly

What disease is caused by


Trypanosma Gambiense and African Sleeping sickness
Rhodesiense?

Vaginitis: foul-smeilling,
What disesase does
greenish discharge; itching
Trichomonas Vaginalis cause?
and burning
What do you tx P. Carinii TMP-SMZ, or pentamidine, or
with? dapsone

What do you use to culture


Asabouraud's Agar
cryptococcus?

What do you use to Diagnose Serology and/or trophozoites


E. Histolytica? or cysts in stool

What do you use to stain


India Ink
Cryptococcus?
What do you use to tx
nothing
cryptosporidium?

What do you use to Tx


Metronidazole
Giardiasis?

What do you use to tx T.


Metronidazole
Vaginalis?

What do you used to dx P. Lung biopsy or lavage,


Carinii? methenamine silver stain
Nystatin for superficial
What do you used to Tx
infection, Amphotericin B for
Candida Albicans?
systemic

What does Alba mean? white

fungi that are mold in the soil


What does dimorphic mean? (low temp) and yeast in tissue
(higher/body temp 37 C)

What is diagnositic for L. Macrophages containg


donovani? amastigotes
What is diagnositic of T.
Trophozoites on wet mount
Vaginalis?

What is histoplasmosis
bird or bat droppings
associated with?

traumatic introduction into


the skin, typically by a thorn
('rose gardner's' disease) ,
What is the progression of S. causes local pustule or ulcer
Schenckii infection? with nodules along draining
lymphatics (ascending
lymphangitis) . Little systemic
illness.

What is the vector for L.


Sandfly
donovani?
What is the vector for T.
Reduviid Bug
Cruzi?

What is the vector for T.


Tstese fly
Gambiense and Rhodesiense?

What is used to dx African


Blood smear
sleeping sickness?

What is used to dx T. Cruzii? blood smear


What is used to dx
serology and biopsy
toxoplasma?

What is used to treat D.


Niridazole
Medinensis?

What is used to Treat E.


Metronidazole and Iodoquinol
Histolytica?

Suramin for bloodborne


What is used to tx african
disease or melaroprol for CNS
sleeping sickness?
penetration
What is used to tx Mebendazole/pyrantel
Ancylostoma duodenale? pamoate

What is used to tx Ascaris Mebendazole/pyrantel


Lumbricoides? pamoate

What is used to tx Clonorhis


Praziquantel
sinensis?

What is used to tx E.
Albendazole
Granulosus?
What is used to tx E. Mebendazole/pyrantel
Vermicularis? pamoate

What is used to tx L.
Sodium Stibogluconate
Donovani?

What is used to tx Loa loa? diethylcarbamazine

Chloroquine ( primaquine for


vivax, ovale) , sulfadoxine +
What is used to tx malaria?
pyrimethamine, mefloquine,
quinine
What is used to tx O.
Ivermectin
Volvulus?

What is used to tx
Praziquantel
Paragonimus Wetermani?

What is used to tx S.
Ivermectin/thiabendazole
Stercoralis?

What is used to tx
Praziquantel
schistosoma?
What is used to tx T. Canis? diethylcarbamazine

What is used to tx T. Cruzii? Nifurtimox

What is used to tx T. Spiralis? Thiabendazole

What is used to tx taenia Praziquantel/niclosamide;


solium infection? albendazole for cysticercosis
What is used to tx
sulfadiazine + pyrimethamine
toxoplasma?

What is used to tx W.
diethylcarbamazine
Bancrofti?

What patient population is Ketoacidotic patients and


susceptible to Mucor disease? Leukemic patients

What stain do you use for


lung tissue when you are silver
detecting P. Carinii?
What state predisposes you to
Immunosuppression
P. Carinii infection?

What test can be used to


detect polysaccharide
latex agglutination test
capsular antigen of
Cryptococcus?

What types of infections can systemic or superficial fungal


Candida Albicans cause? in fections

When do you start when the CD4 drops below


prophylaxis in HIV patients? 200 cells/mL
Where do the mucor and in the walls of blood vessels
rhizopus species fungi and cause infarction of distal
proliferate? tissue

Where is Blastomycosis States east of the Mississippi


endemic? River and Central America

SWUS, California (San Joaquin


Where is Coccidioidomycosis
Valley or destert (desert
endemic?
bumps) 'Valley fever')

Where is Histoplasmosis Mississippi and Ohio River


endemic? valleys
Where is
Paracoccioidomycosis Rural Latin America
endemic?

Retroviruses, which have two


All viruses are haploid except
identical ssRNA molecules
_________?(1)
(diploid).

Bites from what 3 animals are


more prone to rabies
Bat, Raccoon, and Skunk
infection than a bite from a
dog?

When one of 2 viruses that


infects the cell has a mutation
that results in a nonfunctional
Define complementation? protein. The nonmutated
virus 'complements' the
mutated one by making a
functional protein that serves
both viruses.
Minor changes based on
Define genetic drift.
random mutations.

Reassorment of viral genome


(such as when human flu A
Define genetic shift.
virus recombines with swin
flu A virus.)

When virus A acquires virus B


coat proteins and acts like
Define phenotypic mixing? virus B buts its progeny will
have virus A genome and
coat.

-When viruses with


segmented genomes (eg.
Define reassortment? influenza virus) exchange
segments. -High frequency
recombination. Cause of
worldwide pandemics.
Exchange of genes between 2
chromosomes by crossing
Define recombination? over within regions of
significant base sequence
homology.

Describe its incubation period


-Short incubation period (3
and whether or not it has a
weeks) -No carriers
carrier.

Describe its incubation period


-Long incubation (3 months)
and whether or not it has a
- has carriers
carrier.

Describe the general concept A life-threatening illness


of bacterial super infection where a bacterial infection is
which can occur with superimposed on an existing
influenza infection? viral infection.
Describe the genetic and -Enveloped -ssRNA virus with
physical properties of segmented genome -prone to
influenza virus? genetic changes

It migrates in a retrograde
Describe the migration of
fashion within the CNS up n.
rabies within the CNS.
axons.

Describe the physical shape -Bullet-shaped capsid (illus.


and duration of incubation in book) -long incubation
for rabies. period (wks. - 3 months)

-assay for herpes -make a


Describe the technique and
smear of an opened skin
purpose for performing a
vesicle to detect
Tzanck test?
multinucleated giant cells
Describe whether or not it
has carriers
has a carrier.

Does HDV have carriers? Yes

Virus exists in patient for


Explain the concept of a slow
months to years before it
virus infection.
manifests as clinical disease.

From the following selection


-Infectious: dsDNA (except
which classes are considered
poxviruses and HBV) and (+)
infectious and which aren't:
ssRNA -Noninfectious:
dsDNA, ds RNA, (-)ssRNA, (+)
dsRNA and (-)ssRNA
ssRNA.
HCV is a common form of
hepatitis in what US IV drug users
population?

It detects heterophil
How does a Monospot test
antibodies by agglutination to
work?
sheep RBC's

RNA is translated into one


How is RNA translated and long polypeptide that is
processed in picornaviruses? cleaved by proteases into
many small proteins.

How many segments and


what sense is the RNA -8 segments -negative sense
genome of influenza viruses?
How may serotypes do
paramyxoviruses have except -1 -4
parainfluenza which has ___?

Into what class RNA or DNA


RNA
to all segmented viruses fall?

Humoral, with no possibility


Killed vaccines induce what
of the virus reverting to
type of immunity?
virulence

Live attenuated vaccines Humoral and Cellular -with a


induce what type of risk of the virus reverting to
immunity? virulence
Mneumonic for rotavirus
Right Out The Anus
symptoms: ROTA

Mneumonic: Hep D: Defective, Dependent on HBV

Enteric, Expectant mothers,


Mneumonic: Hep E:
Epidemics

Mneumonic: Hep A: Asymptomatic (usually)


Mneumonic: Hep B: Blood-borne

Mneumonic: picoRNAvirus pico = 'small' RNA viruses

Name 2 common bacterial TB, M. avium-intracellulare


infections in AIDS pts. complex

Name 2 common protozoan Toxoplasmosis,


infections in AIDS pts. cryptosporidiosis
Name 3 members of the Flavivirus, Togavirus, and
arborvirus family. Bunyavirus

-SSPE -encephalitis -giant


Name 3 possible sequelae of cell pneumonia (rare;found in
measles infection? immunocompromised
persons)

-Thrush (Candida ablicans) -


Name 4 common fungal cryptococcosis (cryptococcal
infections in AIDS pts. meningitis) -histoplasmosis -
Pneumocystis pneumonia

-HSV -VZV -CMV -


Name 4 common viral progressive multifocal
infections in AIDS pts. leukoencephalopathy (JC
virus)
Name 4 herpesviruses using
the mneumonic: Get herpes -CMV -HSV -EBV -VZV
in a CHEVrolet.

-Bunyaviruses -
Name 4 main segmented
Orthomyxoviruses (influenza
viruses using the mneumonic
virus) -Arenaviruses -
BOAR.
Reoviruses

Name eveloped DNA viruses


-Hepadna -Pox -Herpes
(3). HPH

-Parvo -Adeno -Papova 'You


Name naked DNA viruses (3).
need to be naked for a PAP
PAP
smear.'
Name the 3 naked RNA -Calcivivirus -Picornavirus -
viruses Naked CPR). Reovirus

Name the characteristic


cytoplasmic inclusions seen
Negri bodies
in neurons infected with
rabies.

-Herpesviruses (herpes
Name the DNA enveloped simplex virus types 1 and 2,
viruses (3). VZV, CMV, EBV) -HBV -
smallpox virus

Name the DNA nucleocapsid


Adenovirus, Papillomaviruses
viruses (2).
-Hepadnavirus -
Name the DNA viruses using
Herpesviruses -Adenovirus -
the mneumonic 'HHAPPPy
Parvovirus -Papovavirus -
viruses.'
Poxvirus

Name the illness caused by Encephalitis, fatal is not


rabies and 2 primary prevented, with seizures and
symptoms. hydrophobia.

Name the members of the


-Parainfluenza -RSV -Measles
PaRaMyxovirus using the
-Mumps
letters in bold (4 viruses).

Name the recombinant HBV (antigen = recombinant


vaccine available (1). HBsAg)
-Influenza viruses -
Name the RNA enveloped parainfluenza viruses -RSV -
viruses (9). measles -mumps -rubella -
rabies -HTLV -HIV

-Enteroviruses (poliovirus,
Name the RNA nucleocapsid coxsackievirus, echovirus,
viruses (3). hepatitis A virus) -rhinovirus
-reovirus.

Name the vaccines that are -rabies -influenza -hepatitis


killed (4). A -SalK=Killed

Name the vaccines that are -MMR -Sabin polio -VZV -


live attenuated (6). yellow fever
Of these 3 markers (HBsAg,
HBsAb, HBcAg), which ones
-HBsAg, HBcAg -HBcAg -
are positive in each of the 4
HBsAb, HBcAg -HBsAg,
phases below: (acute disease,
HBcAg
window phase, complete
recovery, chronic carrier).

On HIV, what is gp41 and


envelope protein
gp120?

On HIV, what is p24? (illus. p. rectangular nucleocapsid


205) protein

Acute: 1-3 months Latent: 3


Roughly, what are the time
months-3years
periods for acute, latent, and
Immunodefic.: 3 yrs.-death
immunodeficient stages of
(diagram p. 205 that follows
HIV?
serologic course).
Statement: HEV resembles course, severity, and
HAV in: incubation,

Use the mneumonic PERCH to -Poliovirus -Echovirus -


name members of the Rhinovirus -Coxsackievirus -
Picornavirus family. Hepatitis A

Viral nucleic acids with


(choose) same/different
nucleic acids as host are same
infective alone; others require
special enzymes (contained in
intact virion.)

What 2 antigens are used to Neuraminadase,


classify influenza? Hemagglutinin
What age group is the
primary target of children
paramyxoviruses?

What antiviral treatment is


approved for influenza A
Amantadine and Rimantadine
(especially prophylaxis) but
not for influenza B &amp; C

What antiviral treatment is


approved for influenza A and Zanamivir
B?

What are Councilman bodies acidophilic inclusions seen in


and what are they the liver of those with yellow
pathomneumonic for? fever
What are the 3 C's of -Cough -Coryza -
measles? Conjunctivitis

Chronic, Cirrhosis,
What are the 4 C's of HCV.
Carcinoma, Carriers

-Creutzfeldt-Jakob disease
What are the 4 most common (CJD: rapid progressive
diseases caused by prions? dementia) -kuru -scrapie
(sheep -'mad cow disease'

What are the causes of SSPE


-Late sequelae of measles -
and PML in
Reactivation of JC virus
immunocompromised pts.
What are the classic -high fever -black vomitous -
symptoms of yellow fever? jaundice

What are the common


-Kaposi's sarcoma (HIV pts.) -
diseases (1) and routes of
sexual contact
transmission(1) for HHV-8?

What are the common -infectious mono, Burkitt's


diseases (2) and routes of lymphoma -resp. secretions,
transmission(2) for EBV? saliva

What are the common -herpes genitalis, neonatal


diseases (2) and routes of herpes -sexual contact,
transmission(2) for HSV-2? perinatal
What are the common
-varicella zoster (shingles) -
diseases (3) and routes of
encephalitis -pneumonia
transmission(1) for VZV?

-gingivostomatitis
What are the common keratoconjunctivitis temporal
diseases (3) and routes of lobe encephalitis herpes
transmission(2) for HSV-1? labialis -respiratory
secretions and saliva

-congenital infection, mono,


What are the common
pneumonia -congenital,
diseases (3) and routes of
transfusion, sexual contact,
transmission(6) for CMV?
saliva, urine, transplant

infectious agent that does not


What are the general
contain RNA or DNA, consists
characteristics of a prion?
only of protein
What are the major viruses of -parainfluenza (croup) -RSV -
the paramyxovirus family? (4) Measles -Mumps

What are the primary -aseptic Meningitis -Orchitis


symptoms of the mumps -Parotitis (mumps give you
virus? (MOP) bumps = parotitis)

What are the primary viruses -Poliovirus -Echovirus -


of the picornavirus family? Rhinovirus -Coxsackievirus -
(PERCH) Hepatitis A

What are two classic illness -dengue fever (break-bone


caused by arborviruses? fever) -yellow fever
What general form of
encephalopathies do prions spongiform encephalopathies
present as?

-Segmentation allows
What genetic property does
reassorment to occur in RNA
segmentation afford viruses
viruses -this contributes to
and how does this play into
antigenic shifts which cause
flu epidemics?
most flu pandemics.

What group has a high


pregnant women
mortality rate from HEV?

abnormal circulating
What hematologic finding is
cytotoxic T cells (atypical
characteristic of mono?
lymphocytes)
Antibody to HBcAg; IgM
What is HBcAb, and what
HBcAb indicates recent
does it indicate?
disease

Antigen associated with core


What is HBcAg?
of HBV

What is HBeAb, and what Antibody to e antigen;


does it indicate? indicates low transmissibility

it is a 2nd different antigen


What is HBeAg, and what marker of HBV core; indicates
does it indicate? transmissibility
(HBeAg=Beware)
What is HBsAb, and what Antibody to HBsAg; provides
does it do? immunity to hepatitis B

Antigen found on surface of


What is HBsAg, and what does
HBV; continued presence
it indicate?
indicates carrier state

IgM antibody to HAV; best


What is IgM HAVAb, and what
test to detect active hepatitis
is it used to detect?
A

It is the period between


What is meant by the 'window
disappearance of HBsAg and
period' in HBV infection, and
appearance of Anti-HBs;
what is positive in this
HBcAb is pos. during this
period?
period.
What is the classic vector for Arthropods (mosquitos, ticks,
arborvirus? etc.) ARBOR=Arthropod Borne

synthesize dsDNA from RNA


What is the function of
for integration into host
reverse transcriptase in HIV?
genome.

Killed viral vaccine which is


What is the major mode of reformulated each year and is
protection from influenza given to those in high risk of
virus? infection (elderly, health-
workers, etc.)

What is the method behind


ELISA/Western blot and look for abs to viral proteins;
during what period of HIV false negatives common in
infection are they often first 1-2 months of infection
negative?
What is the mneumonic for
Tzanck heavens I don't have
remembering the Tzanck
herpes.
smear?

What is the only DNA virus


Parvoviridae (ssDNA)
that is not double stranded?

Reoviridae
What is the only RNA virus
['repeatovirus' (reovirus) is
that has dsRNA?
dsRNA]

What is the viral cause of the -Rhinovirus, 100+ serotypes


common cold? -Rhino has a Runny nose.
What neurologic infection can
picornaviruses (except
Aseptic Meningitis
rhinoviruses and hepatitis A
viruses) cause?

What physical finding is Koplik spots (bluish-gray


diagnostic for measles? spots on buccal mucosa)

Those who are


What population should not
immunocompromised and
receive a live vaccine?
their close contacts.

What reproductive
sterility; especially after
complication can mumps
puberty
cause?
What shape are all the DNA
-Icosahedral -Poxvirus
viruses? Which virus (1) is the
(complex)
exception?

ELISA (sensitive w/ high false


What test is used to make the
+ and low threshold);
presumptive dx of HIV, and
Western blot (specific, high
then, which test confirms the
false - rate with high
dx?
threshold)

What tests are gaining


popularity for monitoring PCR/viral load tests
drug tx efficacy in HIV?

What type of genome does


diploid RNA
HIV have?
What type of nucleic acid
segmented dsRNA
structure does rotavirus have?

What type of transcription -Reverse transcription -the


occurs and what type of virion contains an RNA-
polymerase does it possess? dependent DNA polymerase

What type of virus is HAV and -RNA picornavirus -fecal-oral


how is it transmitted? route

-DNA hepadnavirus -
What type of virus is HBV and
parenteral, sexual, and
how is it transmitted?
maternal-fetal routes
-RNA flavivirus -via blood
What type of virus is HCV and
and resembles HBV in its
how is it transmitted?
course and severity

What type of virus is HDV and -delta agent, it is a defective


what is special about its virus -requires HBsAg as its
envelope? evelope

-RNA calicivirus -enteric


What type of virus is HEV and
transmission; causes water-
how is it transmitted?
borne epidemics

What variant of dengue fever


hemorrhagic shock syndrome
is found in Southeast Asia?
-EBV -fever,
What virus causes and what hepatosplenomegaly,
are the classic symptoms of pharyngitis,
mononucleosis? lymphadenopathy (esp.
posterior auricular nodes)

What virus causes yellow


-flavivirus -Aedes mosquitos
(=flavi) fever, and what is its
-monkey or human reservoir
vector and reservoirs (2)?

What virus is the most


common global cause of
Rotavirus
infantile gastroenteritis and
acute diarrhea (in the US).

Where do enveloped viruses -Plasma membrane -


acquire their envelopes, and Herpesviruses which acquire
what virus is the exception to their envelope from the
this rule? nuclear membrane
Where in the cell do DNA -Nucleus -exception:
viruses replicate, and which poxvirus in cytoplasm (carries
virus is the exception to this DNA-dependent RNA
rule? polymerase)

Where in the cell do RNA


-Cytoplasm -exception:
viruses replicate, and what 2
influenza virus and
viruses are the exception to
retroviruses
this rule?

Which marker tests are


appropriate for each phase of
-HBsAg -HBsAg (Anti-HBc) -
hepatitis infection:
Anti-HBc -Anti-HBs (anti-
Incubation, Prodrome/acute
HBc)
illness, Early Convalescence,
Late Convalescence.

Which two DNA viruses don't


Papovaviruses and
have a linear genome?
Hepadnaviruses
(they're circular)
Which two hepatitis viruses A and E; 'The vowels hit your
follow the fecal-oral route? bowels.'

Which two hepatitis viruses


predispose to hepatocellular HBV and HCV
carcinoma?

-Peak incidence occurs


Why is mono called the during peak kissing years
'kissing disease?' 15-20 yo -(saliva
transmission)

- opsonization -
3 main roles of Ig binding to
neutralization - complement
bacteria
activation
A defect in phagocytosis of
neutrophils owing to lack of
NADPH oxidase activity or Chronic granulomatous
similar enzymes is indicative disease
of what immune deficiency
disease?

After exposure to what 4


Tetanus toxin, Botulinum
things are preformed
toxin, HBV, or Rabies.
(passive) antibodies given?

All nucleated cells have what


class I MHC proteins
class of MHC proteins?

Anaphylaxis, asthma, or local


wheal and flare are possible
Type I
manifestations of which type
of hypersensitivity?
Anti-gliadin autoantibodies
are associated with what Celiac disease
disease?

Anti-Scl-70 autoantibodies
are associated with what diffuse Scleroderma
disease?

Autoimmune hemolytic
anemia, Rh disease
(erythroblastosis fetalis), and type II hypersensitivity
Goodpasture's syndrome are
examples of what kind of
hypersensitivity reaction?

Class I major
1 polypeptide, with B2-
histocompatibilty complex
microglobulin
consists of …
Class II major
2 polypeptides, an a and a B
histocompatibilty complex
chain
consists of …

Cytotoxic T cells have CD(?),


which binds to class (?) MHC CD8 binds to class I MHC
on virus-infected cells.

Cell-mediated due to
Define acute transplant cytotoxic T lymphocytes
rejection. reacting against foreign
MHCs. Occurs weeks after
transplantation.

Adjuvants are nonspecific


stimulators of the immune
Define adjuvant.
response but are not
immunogenic by themselves.
Antibody-mediated vascular
Define chronic transplant damage (fibrinoid necrosis)--
rejection. irreversible. Occurs months
to years after transplantation.

Antibody-mediated due to
the presence of preormed
Define hyperacute transplant anti-donor antibodies in the
rejection. transplant recipient. Occurs
within minutes after
transplantation.

Ig epitope that differs among


Define Ig allotype. members of the same species
(on light or heavy chain)

Ig epitope determine by the


antigen-binging site (specific
Define Ig idiotype.
for a given antigen-binding
site)
Ig epitope common to a
single class of Ig (5 classes,
Define Ig isotype.
determined by the heavy
chain)

(1) Salmonella (2 flagellar


Give 3 classic examples of
variants) (2) Borrelia
bacteria with antigen
(relapsing fever) (3) Neisseria
variation.
gonorrhoeae (pilus protein)

(1) failure to synthesize class


Give 3 examples of possibly II MHC antigens (2) defective
causes for SCID? Il-2 receptors (3) adenosine
deaminase deficiency

Goodpasture's syndrome is
anti-basement membrane
associated with what kind of
antibodies.
autoantibodies?
Helper T cells have CD(?)
which binds to class (?) MHC CD4 binds to class II MHC
on antigen-presenting cells.

as bacterial infections in boys


How does Bruton's
after about 6 months of age,
agammaglobulinemia usually
when levels of maternal IgG
present?
antibody decline

Active immunity is induced


How is active immunity after exposure to foreign
acquired? antigens. There is a slow
onset with long-lasting
protection.

by receiving preformed
How is passive immunity antibodies from another host.
acquired? Antibodies have a short life
span, but the immunity has a
rapid onset.
IL-4 promotes the growth of
B cells and the synthesis of IgE and IgG
what 2 immunoglobulins?

In what immune deficiency do


neutrophils fail to respond to Job's syndrome
chemotactic stimuli?

In what T-cell deficiency do


the thymus and parathyroids
fail to develop owing to Thymic aplasia (DiGeorge
failure of development of the syndrome)
3rd and 4th pharyngeal
pouches?

Job's syndrome is associated


with high levels of what IgE
immunoglobulin?
MHC I Ag loading occurs in __
(1) in rER (viral antigens) (2)
(1?)__ while MHC II Ag loading
in acidified endosomes.
occurs in __(2?)__?

Primary biliary cirrhosis has


anti-mitochondrial antibodies
what kind of autoantibodies?

produce IL-2 (activate Tc cells


and further stimulate TH1
Role of TH1 cells?
cell) and g-interferon
(activate macrophages)

produce IL-4 and IL-5 (help B


Role of TH2 cells?
cells make Ab)
Sensitized T lymphocytes
encounter antigen and then
release lymphokines which Type IV
leads to macrophage
activation' in what
hypersensitivity reaction?

TB skin test, transplant


rejection, and contact
dermatitis are examples of Type IV
what type of hypersensitivity
reaction?

The 3 kinds of MHC class I


A, B, and C
genes are…

The 3 kinds of MHC class II


DP, DQ, DR
genes are…
The Fc portion of
immunoglobulins are at the The carboxy terminal
__?__ terminal.

What 2 cytokines are secreted


IL-1 and TNF-a
by macrophages?

What 2 kinds of
autoantibodies are specific Anti-dsDNA and anti-Smith
for systemic lupus?

What 3 cytokines are


classified as 'acute phase IL-1, IL-6, and TNF-a
cytokines'?
(1) alpha and beta interferons
induce production of a second
What 3 ways do interferons protein that degrades viral
mRNA (2) gamma interferons
interfere with viral protein
increase MHC class I expression
synthesis?
and antigen presentation in all
cells (3) activates NK cells to
kill virus-infected cells.

What affect do the acute mobilization of energy


phase cytokines have on fat reserves to raise body
and muscle? temperature

What affect do the acute Incr. Production of Colony


phase cytokines have on the stim. Factor (CS) which leads
bone marrow? to leukocytosis

What affect do the acute


phase cytokines have on the increase body temperature
hypothalamus?
What antibody isotype can
IgG
cross the placenta?

What are 3 types of antigen- macrophages, B cells, and


presenting cells? dendritic cells

What are the maim symptoms


fever, urticaria, arthralgias,
of serum sickness an at what
proteinuria, lymphadenopathy
period of time following Ag
5-10 days after Ag exposure
exposure?

maculopapular rash, jaundice,


What are the major symptoms
hepatosplenomegaly, and
of graft-vs.-host disease?
diarrhea.
What B- and T- cell
deficiency, assoc. with IgA
deficiency, presents with ataxia-telangiectasia
cerebellar problems and
spider angiomas?

formation of immune
What causes the tissue complexes of foreign
damage associated with particles and Abs that deposit
Serum sickness? in membranes where they fix
complement

What class of MHC proteins


are the main determinants of class II MHC
organ rejection?

What complement
components can cause C3a and C5a
anaphylaxis?
What components of the
alternative complement
C3b, Bb
pathway make the C3
convertase?

What components of the


alternative complement
C3b, Bb, and 3b
pathway make the C5
convertase?

What components of the


classic complement pathway C4b, C2b
make the C3 convertase?

What components of the


classic complement pathway C4b, 2b, and 3b
make the C5 convertase?
What components of the
The MAC complex--(C5b, C6,
complement pathway are
C7, C8, C9)
deficient in Neisseria sepsis?

What cytokines attract and


TNF-a and B
activate neutrophils?

What disease is associated


with a X-linked defect in a
tyrosine-kinase gene Bruton's
associated with low levels of agammaglobulinemia
all classes of
immunoglobulins?

What disease is associated


with anti-epithelial cell Pemphigus vulgaris
autoantibodies?
What disease is associated
with anti-microsomal Hashimoto's thyroiditis
autoantibodies?

What does a deficiency of C1


angioedema because of
esterase inhibitor cause (in
overactive complement
the complement cascade)?

What does a deficiency of C3 can lead to severe, recurrent


cause (in the complement pyogenic sinus and
cascade)? respiratory tract infections.

What does deficiency of


leads to paroxysmal
decay-accelerating factor
nocturnal hemoglobinuria
(DAF) in the complement
(PNH)
cascade cause?
What does Job's syndrome recurrent 'cold' (noninflamed)
classically present with? staphylococcal abscesses

TNF-a stimulates their


What does TNF-a stimulate migration to lymph nodes
dendritic cells to do during and their maturation for the
the acute phase response? initiation of the adaptive
immune response.

What elements of the


complement cascade made
C5b, C6, C7, C8, and C9
the Membrane Attack
Complex (MAC)?

What Ig is found in secretions


IgA
as a monomer or a dimer?
What Ig is found in secretions
IgM
as a monomer or a pentamer?

What immune deficiency


disease has an autosomal-
recessive defect in
phagocytosis that results Chediak-Higashi disease
from microtubular and
lysosomal defects of
phagocytic cells?

What immune deficiency is


associated with elevated IgA
Wiskott-Aldrich syndrome
levels, normal IgE levels, and
low IgM levels?

What immune deficiency


presents with tetany owing to
hypocalcemia, congenital Thymic aplasia (DiGeorge
defects of the heart and great syndrome)
vessels, and recurrent viral,
fungal, and protozoal
infections?
What immunoglobulin isotype
has the lowest concentration IgE
in serum?

What immunoglobulin isotype


is involved in type-I IgE
hypersensitivity reactions?

What immunoglobulin isotype


is produced in the primary
IgM
response to an antigen and is
on the surface of B cells?

What immunoglobulin isotype


IgE
mediates immunity to worms?
What immunoglobulin isotype
prevents the attachment of
IgA
bacteria and viruses to
mucous membranes?

What immunoglobulins bind


and activate the classic IgG and IgM (the Fc portion)
complement pathway?

What interleukin induces


naive helper T-cells to IL-12
become TH1 cells?

What interleukin induces


naive helper T-cells to IL-4
become TH2 cells?
What interleukin stimulates
the growth of both helper and IL-2
cytotoxic T-cells?

What is Chronic T-cell dysfunction specifically


mucocutaneous candidiasis? against Candida albicans.

It is a cluster of polypeptides
What is important about the associated with a T-cell
CD3 complex? receptor and is important in
signal transduction.

Ag cross-links IgE on
What is the cellular process
presensitized mast cells and
that causes type I
basophils, triggering the
hypersensitivity?
release of vasoactive amines.
IgM, IgG bind to Ag on
What is the cellular process
'enemy' cell, leading to lysis
that causes type II
(by complement) or
hypersensitivity?
phagocytosis (its cytotoxic).

What is the main antibody in


the secondary immune IgG
response?

What is the most abundant


IgG
immunoglobulin isotype?

What is the most common


selective immunoglobulin selective IgA deficiency
deficiency?
What is the rise in (1) increase specific immune
temperature during the acute response (2) increase antigen
phase response help do (3 processing (3) decrease viral
things?) and bacterial replication

What is the triad of symptoms recurrent pyogenic infections,


seen with Wiskott-Aldrich eczema, and
syndrome? thrombocytopenia

What kind of autoantibodies


are associated with CREST/ anti-centromere antibodies
Scleroderma?

What kind of autoantibodies


are known as rheumatoid anti-IgG antibodies
factor?
What kind of immunity
(antibody-mediated or cell antibody-mediated immunity
mediated) is involved in (B cells)
autoimmunity?

What kind of immunity


(antibody-mediated or cell cell mediated immunity (T
mediated) is involved in graft cells)
and tumor rejection?

What kind of transplant


rejection is reversible with
acute transplant rejection
immunosuppressants such as
cyclosporin and OKT3?

What kinds of adjuvants are


aluminum hydroxide or lipid
included in human vaccines?
antigen-presenting cells (e.g.
What kinds of cells have class
macrophages and dendritic
II MHC proteins?
cells)

What parasites have antigen trypanosomes (programmed


variation? rearrangement)

edema, necrosis, and


What symptoms characterize activation of complement due
the Arthus reaction and what to the Ag-Ab complexes that
causes them? form in the skin following
intradermal injection of Ag.

What type of cell secretes


activated T-cells
IL-3?
What type of cells does
macrophages
gamma interferon stimulate?

What type of hypersensitivity


reaction is the Arthus type III
reaction?

Where does the alternative


On microbial surfaces
complement pathway occur?

Where does the classic


antigen-antibody complexes
complement pathway occur?
the defect is in early stem-
Where is the defect in SCID? cell differentiation, leading to
B- and T-cell deficiency

Which interleukin causes


IL-1
fever?

Which interleukin enhances


IL-5
the synthesis of IgA?

Which interleukin stimulates


the production and activation IL-5
of eosinophils?
Which interleukin supports
the growth and differentiation IL-3
of bone marrow stem cells?

Which is the only type of cell-


mediated hypersensitivity
Type IV
reaction, and thus not
transferable by serum?

Wiskott-Aldrich syndrome is
a defect in the ability to an IgM response to capsular
mount what immune polysaccharides of bacteria.
response?

With what disease are anti-


histone autoantibodies drug-induced lupus
associated?
With what disease are anti-
nuclear antibodies systemic lupus
associated?

Aspiration pneumonia is
- Anaerobes
usually caused by…

Atypical pneumonia is usually - Mycoplasma - Legionella -


caused by… Chlamydia

Bug Hints: Branching rods in


Actinomyces israelii
oral infection =
Bug Hints: Currant jelly
Klebsiella
sputum =

Bug Hints: Dog or cat bite = Pasteurella multocida

Bug Hints: Pediatric Infection H. influenzae (including


= epiglottitis)

Bug Hints: Pneumonia in CS,


P. aeruginosa
burn infection =
Bug Hints: Pus, Empyema,
S. aureus
Abscess =

Bug Hints: Sepsis/Meningitis


Group B strep
in Newborn =

Bug Hints: Surgical wound = S. aureus

Bug Hints: Traumatic open


C. perfringens
wound =
Incidence of H. influenze
Incidence of what cause of meningitis has decreased
mengitis has decreased with a greatly with introduction of H.
vaccine? influenzae vaccine in the last
10-15 years

Most osteomyelitis occurs in


children
what age group?

Neonatal pneumonia is - Group B streptococci - E.


usually caused by… coli

Nosocomial pneumonia is - Staphylococcus - gram-


usually caused by… negative rods
salpingitis, endometritis,
PID includes what disorders
hydrosalpinx, tubo-ovarian
in the body?
abscess

PID is likely caused by what -Chlamydia trachomatis:


two organisms? What are the subacute, often undiagnosed
characteristics of the disease - N. gonorrhoeae:acute, high
caused by each organism? fever

- Staphylococcus - gram-
Pneumonia in
negative rods - fungi -
immunocompromised
viruses - Pnemumocystis
patients is caused by…
carinii (with HIV)

Pneumonia in the alcoholic/IV


- S. pneumoniae - Klebsiella
drug user is usually caused
- Staphylococcus
by…
Postviral pneumonia is - Staphylococcus - H.
usually caused by… influenzae

- ectopic pregnancy -
Salpingitis is a risk factor
infertility -chronic pelvic pain
for…
- adhesions

Babies with congenital


UTIs are found in men in
defects and Elderly with
which 2 age groups?
enlarged prostates

UTIs are mostly caused how? by ascending infections


What's the likely cause of
osteomyelitis in a pt where
S. aureus
you have no other
information?

What 2 pathogens likely


cause nosocomial infections
E. coli, Proteus mirabilis
associated with urinary
catherization?

What 2 pathogens likely


cause nosocomial infections CMV, RSV
in the newborn nursery?

- Serratia marcescens -
What 3 UTI-causing
Enterobacter cloacae -
organisms are often
Klebsiella pneumoniae -
nosocomial and drug-
Proteus mirabilis -
resistant?
Pseudomonas aeruginosa
- Serratia marcescens -
Staphylococcus saprophyticus
What 7 organisms are - E. coli - Enterobacter
associated with UTIs? cloacae - Klebsiella
pneumoniae - Proteus
mirabilis - Pseudomonas
aeruginosa [HINT: SEEKS PP]

What abnormal lab result is


often seen in osteomyelitis elevated ESR
patients?

What are 4 clinical findings of - fever - chills - flank pain -


Pyelonephritis? CVA tenderness [c/c with UTI]

- diysuria - frequency -
What are 4 clinical findings of
urgency - suprapubic pain [c/
UTIs?
c with pyelonephritis]
What are the 2 most common - E. coli causes UTI - S.
causes of nosocomial aureus causes wound
infections? infection

1. E. coli (50-80%) 2.
What are the 3 most common
Staphylococcus saprophyticus
causes of UTI in young
(10-30%) 3. Klebsiella
ambulatory women?
(8-10%)

What are the common causes - E. coli - Proteus - Klebsiella


of hospital-acquired UTIs? - Serratia - Pseudomonas

- N. MENINGITIDIS -
What are the common causes
Enteroviruses - S.
of meningitis in 6-60y/o?
pneumoniae - HSV
What are the common causes - S. PNEUMONIAE - Gram-
of meningitis in 60+ y/o? negative rods - Listeria

What are the common causes - S. pneumoniae - N.


of meningitis in children meningitidis - H. influenzae B
(6mo-6y/o)? - Enteroviruses

- Cryptococcus - CMV -
What are the common causes
toxoplasmosis (brain abscess)
of meningitis in HIV pts?
JC virus (PML)

What are the common causes


- GROUP B STREPTOCOCCI -
of meningitis in newborns
E. COLI - Listeria
(0-6mos)?
What are the common causes
of osteomyelitis in those with S. aureus and S. epidermidis
prosthetic replacements?

What are the common causes


- Mycoplasma - C.
of pneumonia in adults
pneumoniae - S. pneumoniae
18-40 y/o?

What are the common causes - S. pneumoniae - H.


of pneumonia in adults influenzae - Anaerobes -
40-65 y/o? Viruses - Mycoplasma

What are the common causes - Viruses (RSV) - Mycoplasma


of pneumonia in children - Chlamidia pneumonia S.
(6wk-18y)? pneumoniae
- S. pneumoniae - Anaerobes
What are the common causes
- Viruses - H. influenzae -
of pneumonia in the elderly?
Gram-neg. rods

What are the CSF finings in


- Pressure: incr. - Cell type:
bacterial meningitis
incr. PMNs - Protein: incr -
(pressure, cell type, protein
Sugar: decr
and sugar levels)?

What are the CSF finings in


- Pressure: nl or incr. - Cell
bacterial meningitis
type: incr. lymphocytes -
(pressure, cell type, protein
Protein: nl - Sugar: nl
and sugar levels)?

What are the CSF finings in


- Pressure: incr. - Cell type:
fungal/TB meningitis
incr. lymphocytes - Protein:
(pressure, cell type, protein
incr - Sugar:decr.
and sugar levels)?
What are the dominant
B fragilis &gt; E. coli
normal florae in the colon?

What are the dominant Lactobacillus, colonized by E.


normal florae in the vagina? coli and group B strep

- flow obstruction - kidney


What are the predisposing surgery - catherization -
factors of UTIs? gynecologic abnormailities -
diabetes - pregnancy

- Toxoplasma - Rubella -
What infections are
CMV - HSV/HIV -Syphilis
dangerous in pregnancy?
[HINT: ToRCHeS]
Cervical motion tenderness
What is chandelier sign?
associated with PID

What is the 2nd leading cause


of community-acquired UTI in Staphylococcus saprophyticus
sexually active women?

What is the characterisitc of


motile on wet prep
Trichomonas on a slide?

What is the common cause of


Psudomonas aeruginosa
osteomyelitis in drug addicts?
What is the common cause of
S. aureus
osteomyelitis in most people?

What is the common cause of


osteomyelitis in sickle cell Salmonella
pts?

What is the common cause of


osteomyelitis in the sexually N. gonorrhoeae
active?

What is the common cause of


M. tuberculosis
osteomyelitis in the vertebra?
What is the dominant normal
flora contributes to dental S. mutans
plaque?

What is the dominant normal


S. aureus
flora in the nose?

What is the dominant normal


viridans streptococci
flora in the oropharynx?

What is the dominant normal


S. epidermidis
flora on the skin?
Chlamydia trachomatis
What is the most common
causes 3-4 million cases per
STD in the U.S.?
year

What organism can you


presume is causing a
nosocomial infection if Psudomonas aeruginosa
respiratory equipment or
burns are involved?

What organism is the leading


cause of UTI and shows a E. coli
metallic sheen on EMB agar?

What pathogen is associated


Candida albicans
with hyperalimentation?
What pathogen is associated
Legionella
with water (ie. aerosols)?

What pathogen likely causes


nosocomial infections in the HBV
renal dialysis unit?

Neonates delivered by
What population does not csarean section have no flora
have any flora? but are rapidly colonized
after birth

10 to 1 because women have


What ratio is UTIs found more
short urethrae more likely to
in women vs. men? Why?
be colonized by fecal flora
What STD is associated with
3' Syphilis
Argyll-Robertson pupil?

What STD is associated with


Garnerella
clue cells?

What STD is associated with


- Condylomata acuminata -
Genital warts and loilocytes?
HPV 6 and 11
What is the causative agent?

What STD is associated with


jaundice? What is the - Hepatitis B - HBV
causative agent?
What STD is associated with
opportunistic infections,
- AIDS -HIV
Kaposi's sarcoma, lymphoma?
What is the causative agent?

What STD is associated with


painful penile, vulvar or
-Genital Herpies - HSV-2
cervical ulcers? What's the
causative agent?

What STD is associated with


painful ulcers, - Lymphogranuloma
lymphadenopathy, rectal venereum - Chlamydia
strictures? What is the trachomatis
causative agent?

What STD is associated with


urethritis, cervicitis, Chlamydia - Chlamydia
conjuntivitis, Reiter's trachomatis
syndrome PID? What is the
causative agent?
What STD is associated with
-Trichomoniasis -
vaginitis? What is the
Trichomonas vaginalis
causative agent?

What STD is characterized by


- chancroid - Haemophilus
painful genital ulcer? What is
ducreyi
the causative agent?

What STD is likely in a patient


- 1' Syphilis - Treponema
with a painless chancre? What
pallidum
causes it?

What STD is likely in a patient


with fever, lymphadenopathy, - 2' Syphilis - Treponema
skin rashes, condylomata pallidum
lata? What causes it?
What STD is likely in a patient
with gummas, tabes dorsalis, - 3' Syphilis - Treponema
general paresis, aortitis, pallidum
Argyll-Robertson pupil? What
causes it?

What STD is likely in a patient


with urethritis, cervicitis, PID, - Gonorrhea - Neisseria
prostatitus, epididymitis, gonorrhoeae
arthritis? What organism
causes it?

What UTI-causing organism


has a bue-green pigment,
fruity odor and is usually Pseudomonas aeruginosa
nosocomial and drug-
resistant?

What UTI-causing organism is


characterized by a large
Klebsiella pneumoniae
mucoid capsule and viscous
colonies?
What UTI-causing organism is
mobile, causing 'swarming'
on agar and also produces Proteus mirabilis
urease and is associated with
struvite stones?

What UTI-causing organism


sometimes produces a red
Serratia marcescens
pigment, is often nosocomial
and drug-resistant?
1. CF 2. albinism 3. alpha 1
antitrypsin deficiency 4.
phenylketonuria 5. thalassemias 6.
Autosomal recessive diseases sicke cell anemia 7. glycogen
storage diseases 8.
(11)
mucopolysaccharidoses (except
Hunter's) 9. sphingolipidoses (except
Fabry's) 10.infant polycystic kidney
disease 11. hemochromatosis

_____are associated with low


golic acid intake during Neural tube defects
pregnancy.

90% of adult polycystic kidney


disease cases are due to APKD1 (on chromosome 16)
mutation in _____.

meiotic nondisjunction of
95% of Down's syndrome homologous chromosomes
(4% due to Robertsonian
cases are due to what?
translocation and 1% due to
Down mosaicism)
A patent ductus arteriosus is PGE synthesis and low oxygen
maintained by what 2 things? tension

1. Skeletal: tall with long


extremities, hyperextensive
joints, long tapering fingers and
Abnormalities associated with toes 2. Cardiovascular: cystic
medial necrosis of the aorta,
Marfan's syndrome:
aortic incompetence, aortic
dissection, aortic aneurysm,
floppy mitral valve Ocular:
subluxation of lenses

Adult polycystic kidney


polycystic liver disease BERRY
disease is associated with
ANEURYSMS mitral valve
what other diseases or
prolapse
disorders?

pre and postnatal


Babies with Fetal Alcohol developmental retardation
Syndrome are at higher risk microcephaly facial
for developing what other abnormalities limb
problems? dislocation heart and lung
fistulas
dystrophin gene mutations
Becker's muscular dystrophy
(not deletions) Becker's is less
is due to____.
severe.

Besides pulmonary infections, infertility in males fat-


what are some other soluable vitamin deficiencies
consequences of CF? (A,D,E,K)

excessive and inappropriate


exposure to androgenic
Causes of female steroids during early
pseudohermaphroditism: gestation (i.e., congenital
adrenal hyperplasia or
exogenous administration of
androgens during pregnancy)

Characteristic murmur with a


continuous, 'machine-like'
patent ductus arteriosus.
always bilateral massive
enlargement of kidneys due
Characteristics of Adult to multiple large cysts
polycystic kidney disease: patients present with pain,
hematuria, HTN, and
progressive renal failure

onset before age 5 weakness


begins in the pelvic girdle
muscles and progresses
Characteristics of Duchenne's superiorly pseudohypertrophy
MD: of calf muscles due to
fibrofatty replacement of
muscle cardiac myopathy use
of Gower's maneuver

ovaries present but external


Characteristics of female
genitalia are virilized or
pseudohermaphroditism:
ambiguous

macro-orchidism (enlarged
Characteristics of Fragile X testes), long face with a large
syndrome: jaw, large everted ears, and
autism
spheroid erythrocytes
Characteristics of Hereditary hemolytic anemia increased
Sperocytosis: MCHC splenectomy is
curative

testes present, but external


Characteristics of male
genitalia are female or
pseudohermaphroditism:
ambiguous.

Children may do this to


increase venous return with squat
R-to-L shunt.

Heterozygotes (1 : 500)
Compare the cholesterol
cholest. levels around
levels of heterozygores and
300mg/dL Homozygotes
homozygotes with familial
(very rare) cholest. levels over
hyperchlosterolemia:
700 mg/dL.
severe atherosclerotic disease
Complications associated early in life tendon
with homozygous familial xanthomas (classically in the
hypercholesterolemia: Achilles tendon) Myocardial
Infarction before age 20

Congenital heart defects are


rubella
often due to which infection?

Cri-du-chat syndrome results


short arm of chromosome 5
from a congenital deletion on
46 XX or XY, 5p-
which chromosome?

meninges herniate through


Define Meningocele: spinal canal defect picture on
p. 229
meninges and spinal cord
Define Meningomyelocele: herniate through spinal canal
defect picture on p.229

disagreement between the


Define phenotypic (external
pseudohermaphroditism: genitalia) and gonadal (testes
vs. ovaries) sex.

failure of bony spinal canal to


close, but no structural
Define Spina bifida occulta: herniation. (usually seen at
lower vertebral levels) picture
on p. 299 (2002 edition)

46 XX or 47 XXY both ovary


Describe a true
and testicular tissue present;
hermaphrodite:
ambiguous genitalia
Uncorrected VSD, ASD, or
PDA leads to progressive
pulm. HTN. As pulm.
Describe Eisenmenger's resistance increases, the
syndrome: shunt changes from L to R to
R to L, which causes late
cyanosis (clubbing and
polycythemia).

Does coarctation of the aorta


affect males or females most 3:1 males to females
commonly?

Down's syndrome is
associated with increased or decreased
decreased levels of AFP?

Elevated ___ in amniotic fluid


is evidence of a neural tube AFP (alpha fetal protein)
defect.
aortic stenosis distal to
ductus arteriosus (postductal)
Explain the adult type of aDult is Distal to Ductus
coarctation of the aorta and associated with notching of
give some associated the ribs, hypertension in
symptoms. upper extremities, weak
pulses in lower extermities
(check femoral pulse)

aortic stenosis proximal to


Explain the infantile type of
insertion of ductus arteriosus
coarctation of the aorta. What
(preductal) 'INantile, IN close
is it commonly associated
to the heart.' associated with
with?
Turner's syndrome

In fetal period, shunt is R to L


(normal). In neonatal period,
Explain the pathogenesis of a lung resistance decreases and
patent ductus arteriosus: shunt becomes L to R with
subsequent RV hypertrophy
and failure (abnormal).

Aorta leaves RV (anterior) and


Explain the transposition of pulmonaryt trunk leaves LV
(posterior) this leads to
the great vessels.
separation of systemic and
pulmonary circulations.
Colon becomes covered with
adenomatous polyps after
puberty 'FAP' F= five (deletion
Familial Adenomatous on chromosome 5) A=
Polyposis features: autosomal dominant
inheritance P= positively will
get colon cancer (100%
without resection)

café-au-lait spots, neural


tumors, Lisch nodules
Findings in Von (pigmented iris hamartomas),
Recklinghausen's disease: skeletal disorders (scoliosis),
and increased tumor
susceptibility

microcephaly, severe MR,


high pitched crying/mewing -
Findings of Cri-du-chat
(Cri-du-chat is French for cry
syndrome:
of the cat), cardiac
abnormalities

depression progressive
Findings with Huntington's dementia choreiform
disease: movements caudate atrophy
dec. levels of GABA and Ach
in the brain
Frequency of L-toR shunts: VSD&gt;ASD&gt;PDA

Gender identity is based on 1. external genitalia 2. sex of


what two things? upbringing

Genetic anticipation of Fragile


X syndrome may be shown by Triplet repeat (CGG)n
what?

Highest risk of development


of fetal alcohol syndrome at 3 to 8
__ to ___ weeks.
How does CF present in
Failure to thrive
infancy?

increased concentration of
How is CF diagnosed?
Cl- ions in sweat test

muscle biopsy increased


How is MD diagnosed?
serum CPK

Huntington's disease
manifests between the ages 20 and 50
of :
1 in 3000 births short
Incidence and characteristics stature, ovarian dysgenesis,
and Turner's syndrome: webbed neck, coarction of the
aorta

1 in 1000 births
Incidence and characteristics phenotypically normal, very
of double Y males: tall, severe acne, antisocial
behavior (seen in 1-2%)

1 in 8000 births severe MR,


rocker bottom feet, low-set
ears, micrognathia,
Incidence and characteristics congenital heart disease,
of Edward's syndrome: clenched hands (flexion of
fingers), prominent occiput.
Death usually occurs within 1
year of birth.

1 in 850 births testicular


atrophy, eunuchoid body
Incidence and characteristics
shape, tall, long extremities,
of Klinefelter's syndrome:
gynecomastia, female hair
distribution
1 in 6000 births severe MR,
microphthalmia,
microcephaly, cleft lip/palate,
Incidence and characteristics abnormal forebrain
of Patau's syndrome: structures, polydactly,
congenital heart disease
Death usually occurs within 1
year of birth.

Marfan's is due a mutation in


fibrillin
which gene?

Mechanism of Fetal Alcohol


inhibition of cell migration
Syndrome may be :

testicular feminization
(androgen insensitivity)
Most common form of male
results from a mutation in the
pseudohermaphroditism is
androgen receptor gene (X
____.
linked recessive); blind-end
vagina
1. VSD (ventricular septal
Name 3 examples of L-to-R
defect) 2. ASD (atrial septal
shunts. (late cyanosis) 'blue
defect 3. PDA (patent ductus
kids'
arteriosus)

The 3 T's' 1. Tetralogy of


Name 3 examples of R-to-L
Fallot 2. Transposition of
shunts. (early cyanosis) 'blue
great vessels 3. Truncus
babies'
arteriosus

1. heart defects 2. Hypospadias


(when the urethral canal is
open on the undersurface of
Name 7 common congenital the penis or on the perineum)
malformations 3. Cleft lip w/ or w/out cleft
palate 4. congenital hip
dislocation 5. Spina Bifida 6.
Anencephaly 7. Pyloric stenosis
1. Adult polycystic kidney disease
2. Familial hyperchloresterolemia
(type IIA) 3. Marfan's syndrome 4.
Name 8 autosomal-dominant Von Recklinghausen's disease
(NFT1) 5. Von Hippel-Lindau
diseases:
disease 6. Huntington's disease
7. Familial Adenomatous
Polyposis 8. Hereditary
Sperocytosis
Name an X-linked recessive
muscular disease that leads Duchenne's Muscular
to accelerated muscle Dystrophy
breakdown.

PROVe' 1. Pulmonary Stenosis


2. RVH (right ventricular
Name the 4 components of
hypertrophy) 3. Overriding
Tetralogy of Fallot.
aorta (overrides the VSD) 4.
VSD (ventricular septal defect)

Neural tube defects (spina


bifida and anencephaly) are
associated with increased AFP (alpha fetal protein)
levels of ___ in the amniotic
fluid and maternal serum.

Newborns of mothers who


fetal alcohol syndrome (the
consumed significant
number one cause of
amounts of alcohol
congenital malformations in
(teratogen) during pregnancy
the U.S.)
are at risk for _______.
Patau's syndrome = trisomy
13 (hint: Puberty - 13)
___

defective Cl- channel --&gt; secretion


of abnormally thick mucus that plugs
lungs, pancreas, and liver --&gt;
Pathogenesis of Cystic recurrent pulmonary infections
(Pseudomonas species and Staph
Fibrosis:
aureus), chronic bronchitis,
bronchiectasis, pancreatic insufficiency
(malabsorption and steatorrhea),
meconium ileus in newborns.

Patients with Tetralogy of


cyanotic spells
Fallot often suffer _________.

Pyloric stenosis is associated polyhydramnios; projectile


with______. vomiting
True (from 1 in 1500 births in
T/F: Trisomy 21 is associated
women&lt;20 to 1 in 25 births
with advanced maternal age.
in women&gt;45)

Tetralogy of Fallot leads to


early cyanosis from a R-to-L VSD
shunt across the ____.

The defect in Von


17 (hint: 17 letters in
Recklinghausen's disease is
Recklinghausen's)
found on which chromosome?

The gene responsible for


Huntington's disease is 4 ; triplet repeat disorder
located on chromosome__.
The incidence of neural tube
defects is decreased by folate
maternal ingestion of what?

Transposition of great vessels


is a common congenital heart
diabetic
disease in offspring of _____
mothers.

Transposition of great vessels a shunt that allows adequate


is not compatible with life mixing of blood (VSD, PDA, or
unless what is present? patent foramen ovale)

Trisomy 18 is also known Edward's syndrome (hint:


as______. Election age = 18)
Turner's syndrome is the
primary amenorrhea
most common cause of what?

hemangioblastomas of
retina/cerebellum/medulla
Von Hippel-Lindau disease about half of affected
characteristics: individuals develop multiple
bilateral renal cell carcinomas
and other tumors

Von Hippel-Lindau disease is


VHL gene (tumor suppressor)
associated with the deletion
on chromosome 3 (3p)
of what gene?

mental retardation flat facial


profile prominent epicanthal
folds simian crease duodenal
What are some findings of atresia congenital heart disease
Down's syndrome? (most common malformation is
endocardial cushion defect)
Alzheimer's disease in individuals
over 35 increased risk of ALL
What does a heart with
Tetralogy of Fallot look like boot-shaped due to RVH
on x-ray?

What gender genetic disorder


has been observed with
Double Y males (XYY)
increases frequency among
inmates of penal institutions?

What is different about the


juvenile form of polycystic it is recessive
kidney disease?

requiring assistance of the


upper extermities to stand up
What is Gower's maneuver?
(indicates proximal lower
limb weakness)
What is the cause of a deleted dystrophin gene
Duchenne's MD? (hint: 'D' for deletion)

What is the cause of Tetralogy anterosuperior displacement


of Fallot? of the infundibular septum

What is the incidence of


1 in 700 births
Down's syndrome?

What is the most common


Tetralogy of Fallot
cause of early cyanosis?
What is the most common
chromosomal disorder and Down's syndrome (trisomy
cause od congenital mental 21)
retardation?

What is the most common


VSD
congenital cardiac anomaly?

What is the most common


lethal genetic disease of Cystic Fibrosis
Caucasians?

What is the second most


common cause of mental Fragile X syndrome
retardation?
Autosomal - recessive defect
What is the underlying cause
in CFTR gene on chromosome
of Cystic Fibrosis?
7

What is the underlying cause defective or absent LDL


of Familial receptor leading to elevated
hypercholesterolemia? LDL

Indomethacin PGE (may be


What is used to close a patent necessary to sustain life in
ductus arteriosus? What is conditions such as
used to keep it open? transposition of the great
vessels)

methylation and expression


Which gene is affected in
of the FMR 1 gene is affected
Fragile X?
in this X-linked disorder
Which genetic gender
disorder has an inactivated X Klinefelter's syndrome
chromosome (Barr body)?

Why are female carriers of X- because of random


linked recessive disorders inactivation of X
rarely affected? chromosomes in each cell

1. Fragile X 2. Duchenne's MD
3. hemophilia A and B 4.
Fabry's 5. G6PD deficiency 6.
X-linked recessive disorders Hunter's syndrome 7. ocular
(10) albimism 8. Lesch-Nyhan
syndrome 9. Bruton's
agammaglobulinemia 10.
Wiscott-Aldrich syndrome

Turner's syndrome (think:


XO = 'hugs and kisses' -XO- from
Tina Turner)
Klinefelter's syndrome one of
XXY = the most common causes of
hypogonadism in males

Approximately what
percentage of brain tumors 0.5
arise from metastasis?

Are basal cell carcinomas Locally invasive but rarely


invasive? metastasize

Yes. They are extremely


Are Ewing's sarcomas likely to aggressive with early
metastasize? metastasis. However, they are
responsive to chemotherapy.
Are meningiomas resectable? Yes

Are squamous cell


Locally invasive but rarely
carcinomas of the skin
metastasize
invasive?

Are the majority of adult


tumors supratentorial or Supratentorial
infratentorial?

Are the majority of childhood


tumors supratentorial or Infratentorial
infratentorial?
Common histopathology
often seen in squamous cell Keratin pearls
carcinomas of the skin?

Abnormal cells lacking


differentiation; like primitive
cells of the same tissue.
Define anaplasia Often equated with
undifferentiated malignant
neoplasms. Tumor giant cells
may be formed.

Abnormal growth with loss of


cellular orientation, shape,
Define dysplasia and size in comparison to
normal tissue maturation. It is
reversible but is often a
preneoplastic sign.

Increase in the number of


Define hyperplasia
cells (reversible)
One adult cell type is
replaced by another
(reversible). It is often
Define metaplasia secondary to irritation and/or
environmental exposure (e.g.
squamous metaplasia in the
trachea and bronchi of
smokers)

Clonal proliferation of cells


Define neoplasia that is uncontrolled and
excessive

Do oncogenes cause a gain or


Gain of function
loss of function?

Do tumor suppressor genes Loss of function. Both alleles


cause a gain or loss of must be lost for expression
function? of disease
Very significant risk! The
Does a melanoma have a depth of the tumor often
significant risk of metastasis? correlates with the risk of
metastasis.

From what cells do


Arachnoid cells external to
meningiomas most commonly
the brain
arise?

Give 2 examples of a benign


1. Adenoma 2. Papilloma
tumor of epithelial origin.

Give 2 examples of a
1. Adenocarcinoma 2.
malignant tumor of epithelial
Papillary carcinoma
origin.
Give 2 examples of malignant
tumors of blood cell 1. Leukemia 2. Lymphoma
(mesenchymal) origin.

Give an example of a benign


tumor of blood vessel Hemangioma
(mesenchymal) origin.

Give an example of a benign


tumor of bone (mesenchymal) Osteoma
origin.

Give an example of a benign


tumor of more than one cell Mature teratoma
type.
Give an example of a benign
tumor of skeletal muscle Rhabdomyoma
(mesenchymal) origin.

Give an example of a benign


tumor of smooth muscle Leiomyoma
(mesenchymal) origin.

Give an example of a
malignant tumor of blood Angiosarcoma
vessel (mesenchymal) origin.

Give an example of a
malignant tumor of bone Osteosarcoma
(mesenchymal) origin.
Give an example of a
malignant tumor of more Immature teratoma
than one cell type.

Give an example of a
malignant tumor of skeletal Rhabdomysarcoma
muscle (mesenchymal) origin.

Give an example of a
malignant tumor of smooth Leiomyosarcoma
muscle (mesenchymal) origin.

Give an example of a
Acute Lymphoblastic
neoplasm associated with
Leukemia (ALL)
Down's Syndrome.
Tumor markers are used to
confirm diagnosis, to monitor
for tumor recurrence, and to
How are tumor markers used? monitor the response to
therapy. They should not be
used as a primary tool for
diagnosis.

How is prostatic Digital rectal exam (detect


adenocarcinoma most hard nodule) or by prostate
commonly diagnosed? biopsy

How often do primary brain


Very rarely
tumors undergo metastasis?

In what population is
Usually men under the age of
osteochondroma most often
25
found?
In which age group is
prostatic adenocarcinoma Men over the age of 50
most common?

Is malignant transformation Malignant transformation to


in osteochondroma common? chondrosarcoma is rare

TNM system T= size of


Name 1 common tumor
tumor, N=node involvement,
staging system.
and M=metastases

1. Downward transtentorial
Name 3 herniation syndromes
(central) herniation 2. Uncal
that can cause either coma or
herniation 3. Cerebellar
death when the herniations
tonsillar herniation into the
compress the brainstem
foramen magnum
Name 4 factors that 1. Paget's disease of bone 2.
predispose a person to Bone infarcts 3. Radiation 4.
osteosarcoma. Familial retinoblastoma

1. Cingulate herniation under


the falx cerebri 2. Downward
Name 4 possible routes of transtentorial (central)
herniation in the brain herniation 3. Uncal herniation
4. Cerebellar tonsillar
herniation into the foramen
magnum

1. Meningioma 2.
Name 5 primary brain tumors Glioblastoma multiforme 3.
with peak incidence in Oligodendroglioma 4.
adulthood. Schwannoma 5. Pituitary
adenoma

1. Medulloblastoma 2.
Name 5 primary brain tumors Hemangioblastoma 3.
with peak incidence in Ependymomas 4. Low-grade
childhood. astrocytoma 5.
Craniopharyngioma
Name 5 sites from which 1. Lung 2. Breast 3. Skin
tumor cells metastasize to (melanoma) 4. Kidney (renal
the brain. cell carcinoma) 5. GI

Name a common The nuclei of basal cell


histopathological sign of tumors have 'palisading'
basal cell carcinoma nuclei nuclei

Fair-skinned people (blue


Name a population at a
eyes and red hair have also
greater risk for melanoma.
been considered as factors)

Name the 5 primary tumors 1. Colon 2. Stomach 3.


that metastasize to the liver Pancreas 4. Breast 5. Lung
Name two of the most
common sites of metastasis
The liver and the lung
after the regional lymph
nodes

Name two presenting 1. Bitemporal hemianopsia


sequelae of a pituitary (due to pressure on the optic
adenoma. chiasm) 2. Hypopituitarism

On which chromosome is the


17p
p53 gene located?

On which chromosome is the


13q
Rb gene located?
On which chromosomes are BRCA 1 is on 17q and BRCA 2
the BRCA genes located? is on 13q

Out of the 6 primary tumors


Metastasis from the breast
that metastasize to bone,
and prostate are the most
which two are the most
common
common?

What 2 cancers are associated 1. Burkitt's lymphoma 2.


with EBV? Nasopharyngeal carcinoma

1. Aggressive malignant
What 2 neoplasms are
lymphomas (non-Hodgkins)
associated with AIDS?
2. Kaposi's sarcoma
What 2 neoplasms are
associated with Autoimmune Benign and malignant
disease (e.g. Hashimoto's thymomas
thyroiditis, myasthenia gravis,
etc.)?

What 2 neoplasms are


1. Secondary osteosarcoma 2.
associated with Paget's
Fibrosarcoma
disease of bone?

What 2 neoplasms are


associated with Tuberous 1. Astrocytoma 2. Cardiac
sclerosis (facial angiofibroma, rhabdomyoma
seizures, and mental
retardation)?

1. Neoplastic cells have not


invaded the basement
What are 2 characteristic
membrane 2. High
findings in carcinoma in situ?
nuclear:cytoplasmic ratio and
clumped chromatin
1. Cells have invaded the
basement membrane using
What are 2 characteristic
collagenases and hydrolases
findings of an invasive
2. Able to metastasize if they
carcinoma?
reach blood or lymphatic
vessels.

What are 2 neoplasms 1. Squamous cell carcinoma


associated with Xeroderma of the skin 2. Basal cell
pigmentosum? carcinoma of the skin

What are 3 disease findings 1. Metastases to bone 2.


associated with Alkaline Obstructive biliary disease 3.
Phosphatase? Paget's disease of bone

What are 6 primary tumors 1. Kidney 2. Thyroid 3. Testes


that metastasize to bone? 4. Lung 5. Prostate 6. Breast
Psammoma bodies. These are
What are a common
spindle cells concentrically
histopathological finding of
arranged in a whorled
meningiomas?
pattern.

Ependymal cell tumors most


commonly found in the 4th
What are ependymomas?
ventricle. May cause
hydrocephalus

1. Normal 2. Hyperplasia 3.
What are the steps in the
Carcinoma In Situ/Preinvasive
progression of neoplasia?
4. Invasion

What are two signs of bone An increase in serum alkaline


metastases in prostatic phosphatase and PSA
adenocarcinoma? (prostate-specific antigen)
What are two useful tumor Prostatic acid phosphatase
markers in prostatic and prostate-specific antigen
adenocarcinoma? (PSA)

What can be associated with


Sun exposure
the risk of melanoma?

What cancer is associated


with HBV and HCV (Hep B and Hepatocellular carcinoma
C viruses)?

What cancer is associated


with HHV-8 (Kaposi's
Kaposi's carcinoma
sarcoma-associated herpes
virus)?
What cancer is associated
Adult T-cell leukemia
with the HTLV-1 virus?

What cancers are commonly


Cervical carcinoma, penile,
associated with HPV (human
and anal carcinoma
papilloma virus)?

What causes the local effect


Tissue lump or tumor
of a mass?

Destruction of epithelial
What causes the local effect
surfaces (e.g. stomach, colon,
of a nonhealing ulcer?
mouth, bronchus)
Raised intracranial pressure
What causes the local effect in brain neoplasms. Also seen
of a space-occupying lesion? with anemia due to bone
marrow replacement.

What causes the local effect Pathologic fracture or


of bone destruction? collapse of bone

What causes the local effect Venous or lymphatic


of edema? obstruction

What causes the local effect Caused by ulcerated area or


of hemorrhage? eroded vessel
What causes the local effect
Pleural effusion, pericardial
of inflammation of a serosal
effusion, or ascites
surface?

What causes the local effect


of obstruction in the biliary Jaundice
tree?

What causes the local effect


of obstruction in the Pneumonia
bronchus?

What causes the local effect


of obstruction in the left Constipation
colon?
Any site with sensory nerve
What causes the local effect endings. Remember that
of pain? tumors in the brain are
usually painless.

What causes the local effect


of perforation of an ulcer in Peritonitis or free air
the viscera?

What causes the local effect


Tumor mass in the brain.
of seizures?

Compression or destruction
What causes the localized of nerve (e.g. recurrent
loss of sensory or motor laryngeal nerve by lung or
function? thyroid cancer causes
hoarseness)
Hyperuricemia due excess
What causes the nucleic acid turnover
paraneoplastic effect gout? (secondary to cytotoxic
therapy of various neoplasms)

What causes the ACTH or ACTH-like peptide


paraneoplastic effect of (secondary to small cell lung
Cushing's disease? carcinoma)

PTH-related peptide, TGF-a,


TNF-a, IL-2 (secondary to
What causes the squamous cell lung
paraneoplastic effect of carcinoma, renal cell
hypercalcemia? carcinoma, breast carcinoma,
multiple myeloma, and bone
metastasis)

Antibodies against
What causes the
presynaptic Ca2+ channels at
paraneoplastic effect of
NMJ (Thymoma, bronchogenic
Lambert-Eaton syndrome?
carcinoma)
What causes the
Erythropoietin (secondary to
paraneoplastic effect of
renal cell carcinoma)
Polycythemia?

What causes the ADH or ANP (secondary to


paraneoplastic effect of small cell lung carcinoma and
SIADH? intracranial neoplasms)

What chemical carcinogen is


commonly associated with
CCL4
the centrilobar necrosis and
fatty acid change?

What chemical carcinogen is


commonly associated with Nitrosamines
the esophagus and stomach?
What chemical carcinogen is Asbestos (Causes
commonly associated with mesothelioma and
the lungs? bronchogenic carcinoma)

What chemical carcinogen is


commonly associated with Arsenic
the skin (squamous cell)?

What chemical carcinogen(s)


are commonly associated with Aflatoxins and vinyl chloride
the liver?

Malignant cartilaginous
What is a chondrosarcoma?
tumor.
What is a common genetic
11;22 translocation
finding in Ewing's sarcoma?

What is a common gross


Characteristic 'onion-skin'
pathological sign seen in
appearance of bone
Ewing's sarcoma?

What is a common origin of a May be of primary origin or


chondrosarcoma? from osteochondroma

What is a common sign found


Codman's triangle (from
on the x-ray of a person with
elevation of periosteum)
osteosarcoma?
Benign childhood tumor.
Often confused with pituitary
What is a craniopharyngioma? adenoma because both can
cause bitemporal
hemianopsia. Calcification of
the tumor is common.

Anaplastic small cell


What is a Ewing's sarcoma?
malignant tumor.

Locally aggressive benign


What is a giant cell tumor? tumor around the distal
femur, proximal tibial region.

What is a gross pathological


Pearly papules
sign of basal cell carcinoma?
What is a helpful mnemonic
to remember the neoplasm
We ALL go DOWN together.
associated with Down's
Syndrome?

What is a helpful mnemonic


to remember the site of Lots of Bad Stuff Kills Glia
metastasis to the brain?

What is a helpful mnemonic


to remember the types of Cancer Sometimes Penetrates
cancer that metastasize to Benign Liver
the liver?

What is a helpful mnemonic


to remember what tumors BLT with a Kosher Pickle
metastasize to bone?
Most often a cerebellar
tumor. Associated with von
What is a Hemangioblastoma? Hippel Lindau syndrome
when found with
retinoblastoma.

Diffusely infiltrating glioma.


What is a low-grade In children, it is most
astrocytoma? commonly found in the
posterior fossa.

Highly malignant cerebellar


tumor. A form of primitive
What is a medulloblastoma? neuroectodermal tumor
(PNET). Can compress 4th
ventricle causing
hydrocephalus

What is a neoplasm
Squamous cell carcinoma of
associated with actinic
the skin
keratosis?
What is a neoplasm
associated with Barrett's
Esophageal adenocarcinoma
esophagus (chronic GI
reflux)?

What is a neoplasm
commonly associated with
chronic atrophic gastritis, Gastric adenocarcinoma
pernicious anemia, and
postsurgical gastric
remnants?

What is an A relatively rare, slow


oligodendroglioma? growing, benign tumor.

Very nonspecific antigen


produced by 70% of colorectal
What is CEA
and pancreatic cancers and
(carcinoembryonic antigen)?
by gastric and breast
carcinoma
What is considered a
precursor to squamous cell Actinic keratosis
carcinoma?

What is considered to be a
precursor to malignant Dysplastic nevus
melanoma?

Histologic appearance of the


What is meant by the term tumor. Usually graded I-IV
tumor grade? based on degree of
differentiation and number of
mitoses per high-power field.

Based on site an size of


What is meant by the term primary lesion, spread to
tumor stage? regional lymph nodes, and
presence of metastases.
What is the characteristic
Characteristic 'double bubble'
appearance of a giant cell
or 'soap bubble' appearance
tumor on an x-ray?

Characteristic perivascular
What is the common rosettes. Rod-shaped
histopathology associated blepharoblasts (basal ciliary
with Ependymomas? bodies) found near the
nucleus.

Foamy cells and high


What is the common
vascularity are characteristic.
histopathology associated
Can produce EPO and lead to
with Hemangioblastoma?
polycythemia.

What is the common


Rosettes or perivascular
histopathology associated
pseudorosette pattern of cells
with medulloblastomas?
What is the common
Fried egg' appearance of cells
histopathology associated
in tumor. Often calcified.
with oligodendrogliomas?

What is the common Antoni A=compact palisading


histopathology associated nuclei; Antoni B=loose
with schwannoma? pattern

What is the common Pseudopalisading' tumor cells


histopathology found in border central areas of
Glioblastoma multiforme? necrosis and hemorrhage

What is the differentiation


Basal to apical differentiation
pattern of normal cells?
What is the histopathology
Spindle-shaped cells with
commonly associate with
multi-nucleated giant cells.
giant cell tumors?

What is the most common


Osteochondroma
benign bone tumor?

What is the most common


Usually found in sun-exposed
location of basal cell
areas of the body.
carcinoma of the skin?

What is the most common Commonly found in the


location of osteosarcoma? metaphysis of long bones
The lung is the most common
What is the most common origin of metastases. The
organ to 'send' metastases? breast and stomach are also
big sources.

Adrenal glands. This is due to


their rich blood supply. The
What is the most common
medulla usually receives
organ to receive metastases?
metastases first and then the
rest of the gland.

What is the most common


population to have Men age 30-60 years old
chondrosarcoma?

What is the most common Glioblastoma multiforme


primary brain tumor? (grade IV astrocytoma)
What is the most common
primary malignant tumor of Osteosarcoma
bone?

What is the most common


Prolactin secreting
type of pituitary adenoma?

What is the most likely


population to have Ewing's Boys under 15 years old.
sarcoma?

What is the origin of a Derived from the remnants of


craniopharyngioma? Rathke's pouch
What is the origin of a
Rathke's pouch
Pituitary adenoma?

Schwann cell origin. Often


localized to the 8th cranial
What is the origin of the
nerve (acoustic schwannoma).
Schwannoma?
Bilateral schwannoma found
in NF2.

What is the peak incidence of


20-40 years old
giant cell tumor?

What is the peak incidence of


Men 10-20 years old
osteosarcoma?
Prognosis is grave. Usually
What is the prognosis for
only have a year life
Glioblastoma multiforme?
expectancy.

What is the second most


common primary brain Meningioma
tumor?

What is the third most


common primary brain Schwannomas
tumor?

What neoplasias are Hepatocellular carcinoma and


associated with a- nonseminomatous germ cell
fetoprotein? tumors of the testis.
Hydatidiform moles,
What neoplasias are Choriocarcinomas, and
associated with B-hCG? Gestational trophoblastic
tumors.

What neoplasias are Ovarian and malignant


associated with CA-125? epithelial tumors

What neoplasias are Melanoma, neural tumors,


associated with S-100? and astrocytomas

What neoplasm is associated


with Cirrhosis (due to Hepatocellular carcinoma
alcoholism, Hep B, or Hep C)
What neoplasm is associated
Malignant melanoma
with Dysplastic nevi?

What neoplasm is associated


with Immunodeficiency Malignant lymphomas
states?

What neoplasm is associated


with Plummer-Vinson
syndrome (atrophic glossitis, Squamous cell carcinoma of
esophageal webs, and the esophagus
anemia; all due to iron
deficiency)

What neoplasm is associated


Colonic adenocarcinoma
with ulcerative colitis?
What oncogene is associated
with breast, ovarian, and erb-B2
gastric carcinomas?

What oncogene is associated


c-myc
with Burkitt's lymphoma?

What oncogene is associated


ras
with colon carcinoma?

What oncogene is associated


with Follicular and
bcl-2
undifferentiated lymphomas
(inhibits apoptosis)?
What tumor marker is
PSA (Prostatic acid
associated with Prostatic
phosphatase)
carcinoma?

What tumor suppressor gene


is associated with
Rb gene
Retinoblastoma and
osteosarcoma?

What type of metastases are


Osteoblastic metastases in
common in the late stages of
bone
prostatic adenocarcinoma?

What type of neoplasm is


Visceral malignancies
associated with Acanthosis
(stomach, lung, breast, and
nigricans (hyperpigmentation
uterus)
and epidermal thickening)
What type of skin cancer is
associated with excessive
Squamous cell carcinoma
exposure to sunlight or
arsenic exposure?

Where are chondrosarcomas Pelvis, spine, scapula,


usually located? humerus, tibia, or femur.

Where are Ewing's sarcomas Diaphysis of long bones,


most commonly found? pelvis, scapula, and ribs

Where are Glioblastoma


Cerebral hemispheres
multiformes found?
Where do giant cell tumors At epiphyseal end of long
most commonly occur? bones

Where do meningiomas most Convexities of hemispheres


commonly occur? and parasagital region

Where do oligodendrogliomas Most often found in the


most often occur? frontal lobes

Where do osteochondromas
Long metaphysis
commonly originate?
Where do squamous cell
carcinomas most commonly Hands and face
occur?

Where does prostatic From the posterior lobe


adenocarcinoma most (peripheral zone) of the
commonly arise? prostate gland

Which has more prognostic


Stage
value: tumor stage or grade?

Which is more common: Metastatic bone tumors are


metastasis to bone or primary far more common than
tumors of bone? primary tumors
Which is more common:
Metastasis to the liver is more
metastasis to the liver or
common
primary tumors of the liver?

Which tumor suppressor gene


is associated with most
p53
human cancers and the Li-
Fraumeni syndrome?

Which tumor suppressor


genes are associated with BRCA 1 and 2
breast and ovarian cancer?

___% of African-Americans
carry the HbS trait, and ___% 8%; 0.2
have the disease.
________ = defect of platelet
Glanzmann's thrombasthenia
aggregation

________ disease = defect of


Bernard-Soulier
platelet adhesion

_____= activation of
coagulation cascade leading
to microthrombi and global DIC (Disseminated
consumption of platelets, intravascular coagulation)
fibrin, and coagulation
factors.

1. Hydroxyurea (increased
2 therapies for sickle cell
HbF) 2. bone marrow
anemia:
transplantation
50% of Hodgkin's cases are
EBV
associated with which virus?

1. hemorrage 2. enzyme defects


(e.g., G6PD deficiency, PK deficiency)
3. RBC membrane defects (e.g.,
7 causes of normocytic hereditary spherocytosis) 4. Bone
Marrow disorders (e.g., aplastic
normochromic anemia:
anemia, leukemia) 5.
Hemoglobinopathies (e.g., sickle
cell) 6. Autoimmune hemolytic
anemia 7. Anemia of chronic disease

Antiplatelet antibodies and


increased megakaryocytes are ITP
seen in ____.

Auer rods, myeloblasts,


AML
adults =
bcl-2 activation is associated
t(14;18) - Follicular
with which translocation and
lymphomas
which lymphomas?

bcr-abl hybrid is associated


with which translocation and t(9;22) - CML
which leukemia?

Blood smear of a Multiple


RBCs stacked like poker chips
Myeloma patient would show
(rouleau formation)
what?

Burkitt's lymphoma: '______'


appearance associated with starry sky' EBV Africa
what virus? endemic where?
c-myc activation is associated
with which translocation and t(8;14) - Burkitt's
which lymphoma?

radiation, benzene,
chloramphenicol, alkylating
agents, antimetabolites, viral
Causes of aplastic anemia: agents (HCV,CMV,EBV, herpes
zoster-varicella), Faconi's
anemia, idiopathic (immune-
mediated, primary stem cell
defect)

Causes of platelet
ITP, TTP, drugs, DIC
abnormalities:

CFU-GM (colony forming


monocytes, neutrophils and
unit-granulocyte-monocyte)
basophils
gives rise to which cells?
CLL is very similar to what SLL (small lymphocytic
lymphoma? lymphoma)

1. Hemarthroses (bleeding
Coagulation factor defects
into joints) 2. easy bruising 3.
(macrohemorrage) cause: (3)
prolonged PT and/or PTT

Common cause of macrocytic


Vit. B12/folate deficiency
megalobalstic anemia:

common in children,
lymphoblasts, most ALL
responsive to therapy =
Waldenstrom's also has an M
Compare Multiple Myeloma
spike, but large amounts of
with Waldenstrom's
IgM are produced (not IgA or
macroglobinemia:
IgG), no lytic lesions

aplastic crisis (due to B19


parvovirus infection)
Complications often seen in autosplenectomy inc. risk of
homozygotes (sickle cell encapsulated organism
disease) include: infection Salmonella
osteomyelitis painful crisis
(vaso-occlusive) and splenic
sequestration crisis

RBCs are small, round, with


Describe the RBCs of a
no central pallor less
parient with Hereditary
membrane therefore
spherocytosis.
increased MCHC

FAB classification L1: ALL-null, ALL-common


FAB classification L2: ALL- T

FAB classification L3: ALL- B

FAB classification M1: AML (without maturation)

FAB classification M2: AML (with maturation)


FAB classification M3: acute promyelocytic leukemia

acute myelomonocytic
FAB classification M4:
leukemia

FAB classification M5: acute monocytic leukemia

FAB classification M6: acute erythroleukemia


acute megakaryocytic
FAB classification M7:
leukemia

increased number of
circulating leukocytes in
General considerations of blood bone marrow infiltrates
leukemias: of leukemic cells leukemic
cell infiltrates in liver, spleen,
and lymph nodes also
common

t(8;14) c-myc gene moves


Genetics associated with
next to heavy chain Ig gene
Burkitt's lymphoma:
(14)

Gamma 4 tetramers, lacks all


Hb Barts =
4 alpha globin genes
hydrops fetalis and
Hb Barts results in what?
intrauterine fetal death

Beta 4 tetramers, lacks 3


Hb H =
alpha globin genes

Hereditary spherocytosis
causes intrinsic, extravascular
spectrin
hemolysis due to a _____
defect.

Hereditary spherocytosis is
gallstones, splenomegaly,
associated with what other
anemia and jaundice
problems?
Hereditary spherocytosis is
Direct Coomb's test
distinguished from warm
(Hereditary spherocytosis is
antibody hemolysis by what
Coomb's negative)
test?

Hodgkin's or NHL: which is


associated with HIV &amp; NHL
immunosuppression?

Hodgkin's or NHL: which is


associated with mediastinal Hodgkin's
lymphadenopathy?

Hodgkin's or NHL: which one


displays a bimodal Hodgkin's (young and old)
distribution?
destructive bone lesions
How does Multiple Myeloma cause hypercalcemia
affect calcium levels? (punched-out lytic bone
lesions can be seen on x-ray)

In _____, the alpha globin


alpha thalassemia
chain is underproduced.

In _____, the beta chain is Beta-minor thalassemia


underproduced. (heterozygote)

In ______, the beta chain is Beta-major thalassemia


absent. (homozygote)
In anemia of chronic disease,
are these values increased or
decreased? 1. TIBC 2. ferritin 1. dec. 2. inc. 3. dec. 4. inc.
3. serum iron 4. storage iron 5. normal
in marrow macrophages 5. %
sat.

In Beta Thalassemia, cardiac secondary hemochromatosis


failure is often due to what? (due to transfusions)

In which hemorrhagic
DIC
disorder is PT increased?

In which hemorrhagic 1. Hemophilia A 2.


disorders is PTT increased? Hemophilia B 3. von
(4) Willibrand's disease 4. DIC
increased or decreased in
1. increased 2. decreased 3.
iron deficiency anemia? 1.
decreased
TIBC 2. ferritin 3. serum iron

Is Hodgkin's more common in men (except nodular


men or women? sclerosing type)

increased PT and PTT,


increased fibrin split products
Lab findings of DIC:
(D-dimers), decreased
platelet count

Lymphoblastic lymphoma
commonly presents with ALL and mediastinal mass
what?
ALL (B cell) Lymphoblastic
lymphoma, CLL (B)
Lymphocytic lymphoma,
Lymphomas derived from the Follicular center cell
B cell lineage: lymphoma, Immunoblastic
lymphoma (B), Plasmacytoid
lymphocytic lymphoma and
myeloma

ALL (T cell) Lymphoblastic


lymphoma, CLL (T)
Lymphomas derived from the Lymphocytic lymphoma,
T cell lineage: Immunoblastic lymphoma (T),
Sezary syndrome and mycosis
fungoides

Macrocytic anemia is defined


100
as MCV&gt;____.

1. anemia (dec. RBCs) 2.


Marrow failure due to infections (dec. WBCs) 3.
leukemia can lead to (3) hemorrhage (decreased
platelets)
Microcytic, hypochromic
80
anemia = MCV&lt;____.

most commonly associated


with Philadelphis
chromosome, myeloid stem CML
cell proliferation, may
accelerate to AML =

1. Hemophilia A 2.
Name 3 coagulopathies: Hemophilia B 3. von
Willibrand's disease

Name 3 etiologies of 1. iron deficiency 2.


microcytic, hypochromic Thalassemias 3. lead
anemia: poisoning
1. Qualitative platelet defects
Name 5 hemorrhagic
2. Vascular bleeding 3.
disorders that exhibit
Thrombocytopenia 4. Von
increased bleeding time.
Willibrand's disease 5. DIC

1. Small lymphocytic
lymphoma 2. Follicular
lymphoma (small cleaved cell)
Name 5 types of NHL:
3. Diffuse large cell 4.
Lymphoblastic lymphoma 5.
Burkitt's lymphoma

CML, Polycythemia rubra vera,


CML, Myelofibrosis, idiopathic
Name 9 chronic myeloid thrombocythemia, (chronic
leukemias: monocytic, chronic
myelomonocytic,
eosinophilic, chronic
erythroid) *last 4 are rare*

Hodgkin's: localized, single


group of nodes, extranodal
Nodal involvement and rare, contiguous spread NHL:
spread of Hodgkin's vs. NHL: multiple, peripheral nodes,
extranodal involvement
common, noncontiguous
spread
1. TIBC 250-400 micro
Normal values: 1. TIBC 2.
grams/ dl 2. 50-150 3.
Serum iron 3. % sat.
20-50 %

older adults,
lymphadenopathy,
hepatosplenomegaly, few
symptoms, indolent course, CLL
increased smudge cells in
peripheral blood smear, warm
Ab autoimmune hemolytic
anemia =

1. drugs that block DNA


Other causes of macrocytic synthesis (e.g., sulfa drugs,
anemia (2) AZT) 2. marked
reticulocytosis

pancytopenia with normal cell


Pathologic features of aplastic morphology, hypocellular
anemia: bone marrow with fatty
infiltration
Peak incidence of NHL
20-40
between what ages?

Philadelphia chromosome= t(9;22), bcr-abl

Plasma cells in Multiple


Myeloma produce large IgG (55%) and IgA (25%)
amounts of what?

1. mucous membrane
Platelet abnormalities bleeding 2. petechiae 3.
(microhemorrage) lead to: (4) purpura 4. prolonged
bleeding time
PMNs are hypersegmented in Vit. B 12 and folate
____ . deficiencies

PT and PTT: which one


measures the intrinsic and
PT (extrinsic) PTT (intrinsic)
which one measures the
extrinsic path?

Rank the prognoses of the NS and LP = excellent MC =


different types of Hodgkin's: intermediate LD = poor

Schistocytes are characteristic


TTP
of ___.
Sickle cell heterozygotes
malaria; (balanced
(sickle cell trait) are relatively
polymorphism)
____-resistant.

fatigue, malaise, pallor,


Symptoms of aplastic anemia: purpura, mucosal bleeding,
petechiae, infection

Mediterranean (hint:
Thalassemia is prevalent in
thalassa=sea. Think,
which populations?
thalaSEAmia)

The lymphoid stem cell gives


T cells and B cells
rise to: (2)
The majority of NHL involve T B cells (except lymphoblastic
cells or B cells? T cell origin)

withdrawal of offending
agent, allogenic bone marrow
Treatment of aplastic anemia: transplantation, RBC and
platelet transfusion, G-CSF or
GM-CSF

1. decreased serum
What are 2 indications of
haptoglobin 2. increased
hemolysis?
serum LDH

(mostly seen in Hodgkin's)


What are constitutional signs/
low grade fever, night sweats,
symptoms?
weight loss
gram-negative sepsis,
transfusion, trauma,
What are some other causes
malignancy, acute
of DIC?
pancreatitis, and nephrotic
syndrome

What genetics are involved


with follicular lymphoma t(14;18) bcl-2 expression
(small cleaved cell)?

What is another Beta chain


HbC defect. patients can be
mutation in which patients
HbC or HbSC (1 of each
have a milder disease than Hb
mutant gene)
SS patients?

pancytopenia characterized by
severe anemia, neutropenia,
and thrombocytopenia caused
What is aplastic anemia? by destruction of multipotent
myeloid stem cells, with
inadequate production or
release of differentiated cell
lines.
What is compensatorily
fetal hemoglobin (it is
increased in both forms of
inadequate, however)
Beta Thalassemia?

What is found in the urine of


Ig light chains (Bence Jones
patients with Multiple
protein)
Myeloma?

What is it called when CML --


blast crisis
&gt; AML?

Reed-Sternberg cell
What is the characteristic cell (decreased numbers of RS
of Hodgkin's lymphoma? cells indicates a better
prognosis)
the monoclonal
What is the M spike? immunoglobin spike on
serum electrophoresis

What is the most common


von Willibrand's disease
bleeding disorder?

What is the most common


obstetric complications
cause of DIC?

What is the most common


primary tumor arising within Multiple Myeloma
bone in adults?
What is the name of a chronic
Sezary syndrome
T cell leukemia?

a single AA replacement in
What mutation causes sickle
the Beta chain (normal
cell anemia?
glutamic acid with valine)

renal insufficiency, increased


What other problems result
susceptibility to infections,
from Multiple Myeloma?
anemia, and amyloidosis

What precipitates sickling of


low oxygen or dehydration
cells?
What test is used to confirm
osmotic fragility test
Hereditary spherocytosis?

What test is used to


distinguish between immune
Direct Coomb's test
vs. non-immune RBC
hemolysis?

What type of cell is cancerous


Monoclonal plasma cell, 'fried
in Multiple Myeloma and what
egg' appearance
does it resemble?

Which 2 hemorrhagic
disorders have decreased 1. thrombocytopenia 2. DIC
platelet counts?
lymphoblastic lymphoma, and
Which 2 types of NHL occur in
Burkitt's lymphoma (20% of
children?
diffuse large cell type too)

Which Beta Thalassemia Beta Thal. major, (requires


results in severe anemia? blood transfusions)

Which coagulation factors are


(extrinsic) = Factors II, V, VII,
a part of the intrinsic and
and X (intrinsic) = all factors
which are a part of the
except VII and XIII
extrinsic path?

Which coagulation factors are Hemophilia A (factor VIII


deficient in each of the deficiency) Hemophilia B
Hemophilias? (factor IX deficiency)
Which one is associated with
neurological problems, folate
Vit. B12
deficiency or Vit. B12
deficiency?

Which type of Hodgkin's LP (lymphocyte predominant)


accounts for 6% of cases and - 6% LD (lymphocyte
which type is the most rare. depleted) - rare

Which type of Hodgkin's


commonly affects males LP
under 35?

Which type of Hodgkin's


commonly affects older males
LD
and is associated with
disseminated disease?
Which type of Hodgkin's has
the most Reed Sternberg Mixed Cellularity
cells?

Which type of Hodgkin's is


characterized by collagen NS (nodular sclerosing)
banding?

Which type of Hodgkin's is


NS (nodular sclerosing)
the most common? (65-75%)

Which type of Hodgkin's is


the second most common? MC (mixed cellularity)
(25%)
Which type of Hodgkin's
primarily affects young NS
adults, women&gt;men?

Which type of NHL clinically


small lymphocytic lymphoma
presents like CLL?

Which type of NHL is difficult


follicular lymphoma
to cure?

Which type of NHL is most Lymphoblastic lymphoma


common in children? (very aggressive)
Which type of NHL is the most follicular lymphoma (small
common type in adults? cleaved cell)

Lymphoblastic lymphoma
Which types of NHL involve T (immature T cells) 20% of
cells? Diffuse large cell NHL (mature
T cells)

With iron overload 1. TIBC normal 2. Serum iron


(hemosiderosis) are values increased 3. % sat. increased
increased or decreased? 1. (100%) -see charts on p.238
TIBC 2. serum iron 3. %sat. of 2002 edition-

A patient with gallstones may


(1)epigastric/RUQ pain (2)
present with Charcot's triad.
fever (3)jaundice
What comprises the triad?
Failure of copper to circulate
in what form causes Wilson's ceruloplasmin
disease?

How can one distinguish Rotor's syndrome presents


between Dubin-Johnson similarly, except less severely
syndrome and Rotor's and no black liver(as seen in
syndrome? Dubin-Johnson)

How does Budd-Chiari


syndrome progress(in the Congestive liver disease
liver)?

How does cirrhosis/portal diffuse fibrosis, destroying


hypertension(HTN) affect liver normal structure, with
histology? nodular regeneration
How does Hirschsprung's Chronic constipation early in
disease first present? life

How does one diagnose


ultrasound
gallstones?

How does one differentiate micronodular(&lt;3mm and


liver nodules, in the case of uniform) macronodular(&gt;
cirrhosis? 3mm and varied)

How does one treat


cholecystectomy
gallstones?
Bird beak'--dilated
How is achalasia evidenced
esophagus with an area of
on a Barium swallow?
distal stenosis.

How is hepatocellular like renal cell carcinoma,


carcinoma spread? hematogenously

How is the hyperbilirubinemia


Dubin-Johnson syndrome--
in Dubin-Johnson syndrome
conjugated
different than in the other 2
hyperbilirubinemia (due to
hyperbilirubinemias(Gilbert's
defective liver excretion)
or Crigler-Najjar(type 1))?

How would you expect a Asymptomatically, with an


Gilbert's syndrome patient to elevated unconjugated
present? bilirubin
triple therapy (metronidazole,
In PUD, how can H.pylori be bismuth salicylate, amoxicillin
treated? or tetracycline with or without
a proton pump inhibitor

In what fatal childhood


hepatoencephalopathy is
there an association with viral Reye's syndrome
infections(VZV,influ.B) and
salicylates?

In Wilson's disease, where


(1)Liver (2)Brain (3)cornea
does copper accumulate(3)?

Is chronic pancreatitis
strongly associated with YES
alcoholism?
Is the dilation proximal, at, or
Proximal (results in a
distal to the aganglionic
'transition zone')
segment?

Low-fiber diets are associated


diverticulosis
with which of the following?

To what cancer is a chronic


Gastric carcinoma
gastritic patient predisposed?

What 3 common findings are (1)fatty liver (2)hypoglycemia


evident in a Reye's patient? (3)coma
What are 2 anti-androgen gynecomastia loss of sexual
effects of liver cell failure? hair

What are 2 extraintestinal migratory polyarthritis


manifestations of Crohn's? erythema nodosum

What are 2 main symptoms (1)pain Decreases with meals,


and a histological sign of (2)weight gain, hypertrophy
Duodenal ulcers? of Brunner's glands

What are 2 main symptoms of pain Greater with meals,


Gastric ulcers? weight loss
What are 2 signs of Crigler-
jaundice kernicterus(bilirubin
Najjar syndrome(other than
deposition in the brain)
hyperbilirubinemia)?

What are 2 types of Crohn's disease Ulcerative


Inflammatory Bowel Disease? colitis

What are 2 types of peptic


gastric ulcer duodenal ulcer
ulcer disease(PUD)?

(1)pregnancy (2)polycythemia
What are 3 associations of
rubra vera (3)hepatocellular
Budd-Chiari Syndrome?
carcinoma
What are 3 neuro effects of (1)asterixis, (2)scleral icterus,
liver cell failure? (3)coma

(1)Cholesterol stones (2)


What are 3 types of
Mixed stones (3)Pigment
gallstones?
stones

What are 4 potential (1)bleeding, (2)penetration,


complications of PUD? (3)perforation, (4)obstruction

(1)hepatomegaly (2)ascites (3)


What are 4 signs of
abdominal pain (4)eventual
congestive liver disease?
liver failure
(1)esophageal varices(-
&gt;hematemesis), (2)melena,
What are 5 GI and 2 GU (3)splenomegaly, (4)caput
effects of portal HTN? medusae, (5)ascites and (1)
testicular atrophy, (2)
hemorrhoids

What are 5 possible (1)DIC (2)ARDS (3)Diffuse fat


consequences of acute necrosis (4)hypocalcemia (5)
pancreatitis? pseudocyst formation

Gallstones Ethanol Trauma


What are possible causes of Steroids Mumps
acute pancreatitis(GET Autoimmune disease
SMASHeD)? Scorpion sting
Hyperlipidemia Drugs

primary(autos. Recessive)
What are possible etiologies
secondary to chronic
of hemochromatosis?
transfusion therapy
Achalasia, Barrett's
What are risk factors for esophagus, Corrosive
esophageal cancer(ABCDEF)? esophagitis, Diverticuli,
Esophageal web, Familial

(1)strictures, (2)fistulas, (3)


What are some complications perianal disease, (4)
of Crohn's(4)? malabsorption-nutritional
depletion

(1)severe stenosis, (2)toxic


What are some complications
megacolon, (3)colorectal
of ulcerative colitis(3)?
carcinoma

What are the 2 types of


Type A(fundal) Type B(antral)
chronic gastritis?
What are the 3 forms of (1)diverticulum, (2)
Diverticular disease? diverticulosis, (3)diverticulitis

What are the 4 risk factors for (1)Female (2)Fat (3)Fertile (4)
gallstone development? Forty

Asterixis Basal ganglia


degeneration Cirrhosis,
What are the ABCD Ceruloplasmin decrease,
characteristics of Wilson's? Corneal deposits(Kayser-
Fleischer rings), Carcinoma
(hepatocell.), Choreiform
movements Dementia

Autoimmune disorder
characterized by
What are the characteristic 4
A's of type A gastritis? Autoantibodies to parietal
cells, pernicious Anemia,
Achlorhydria
What are the respective
infectious(Crohn's)
etiologies of Crohn's and
autoimmune(UC)
Ulcerative colitis(UC)?

A secondary form can arise


What can achalasia arise from from Chagas' disease; can
and lead to? lead to progressive
dysphagia.

What can hemochromatosis (1)CHF (2)hepatocellular


lead to(2)? carcinoma

What cell tumor marker is


elevated in hepatocellular Alpha FetoProtein(AFP)
carcinoma?
What disorder is
characterized by increased
hemochromatosis
iron deposition in many
organs(up to 50g)?

What happens to hepatocytes


they become swollen and
as a result of alcoholic
necrotic
hepatitis?

What happens when


cholesterol and bilirubin
Gallstones
overwhelm solubilizing bile
acids and lecithin?

What histological changes, neutrophil infiltration,


other than to hepatocytes, Mallory bodies(hyaline),
does one see in liver increased fat, and sclerosis
hepatitis? around the central vein
BARRett's = Becomes
What is a mnemonic for
Adenocarcinoma, Results
Barrett's esophagus?
from Reflux.

The replacement of glandular


stratified squamous with
What is Barrett's esophagus?
gastric columnar epithelium
in distal esophagus.

What is failure of lower


Achalasia is due to the loss of
esophageal sphincter
the myenteric plexus.
relaxation due to?

What is hepatic vein or IVC


occlusion with centrilobular Budd-Chiari syndrome
congestion and necrosis?
What is recommended for
screen these patients over 50
patients over 50, in terms of
with stool occult blood test
CRC screening?

increased intraluminal
What is the cause of
pressure and focal weakness
diverticulosis?
in the colonic wall

(1)micronodular pigment
What is the classic triad of
cirrhosis (2)'bronze' diabetes
hemochromatosis?
(3)skin pigmentation

What is the discerning


Type B is caused by a Bug(H.
characteristic for type B
pylori)
gastritis?
What is the embryonic cause failure of neural crest
of Hirschsprung's disease? migration

H.pylori(100%)--lower
What is the etiology of mucosal protection or
duodenal ulcers? increased gastric acid
secretion

H.pylori(70%), NSAIDS both


What is the etiology of gastric
lower mucosal protection vs.
ulcers?
gastric acid

transmural inflamm.
COBBLESTONE mucosa,
What is the gross morphology
of Crohn's? creeping FAT, bowel wall
thickening(string sign on x-
ray), linear ulcers, fissures
mucosal inflamm. Friable
What is the gross morphology
mucosal pseudopolyps with
of ulcerative colitis?
freely hanging mesentery

What is the incidence of


over 60y/o, 50%
diverticulosis in the elderly?

What is the microscopic


noncaseating granulomas
morphology of Crohn's?

What is the microscopic


morphology of ulcerative crypt abscesses and ulcers
colitis?
For Crohn's, think of a FAT
What is the mnemonic for
old CRONE SKIPping down a
Crohn's?
COBBLESTONE road.

What is the most common


epigastric abdominal pain
clinical sign of acute
radiating to the back
pancreatitis?

What is the most common


primary malignant tumor of hepatocellular carcinoma
the liver in adults?

What is the most common


mixed stones
type of gallstone?
significant liver injury leading
What is the primary cause of
to hepatic necrosis(e.g.
macronodules in the liver?
Infections, drug-induced)

What is the primary cause of


metabolic(e.g. Alcohol)
micronodules in the liver?

~6months (very aggressive--


What is the prognosis for
often already spread at
pancreatic adenocarcinoma?
presentation)

What is the term for inflamm.


diverticulitis
of diverticula?
What is the term used for
congenital megacolon
characterized by loss of Hirschsprung's disease
parasympathetic ganglion
cells?

What is the term used to


describe breath that smells
fetor hepaticus
like a freshly opened corpse,
as seen in liver cell failure?

What is the term used to


describe having many diverticulosis
diverticula?

What is the treatment for


plasmaphoresis phototherapy
Crigler-Najjar syndrome?
What is the treatment of repeated phlebotomy
hemochromatosis? deferoxamine

What is the treatment of


penicillamine
Wilson's disease(1 drug)?

(1)Abdominal pain radiating


to the back (2)Weight loss (3)
What is the typical
Anorexia (4)Migratory
presentation of a patient with
thrombophlebitis(Trousseau's
pancreatic adenocarcinoma(5
Sd) (5)Pancreatic duct
signs)?
obstruction(palpable
gallbladder)

Terminal ileum, small


What is the usual location of intestine, colon (but any part
Crohn's? can be affected)-- often SKIP
lesions, rectal sparing
colon (with continuous
What is the usual location of
lesions and rectal
ulcerative colitis?
involvement)

What lab values are SGOT(AST)/SGPT(ALT) &gt;


characteristic in alcoholic 1.5,usually (A Scotch and
hepatitis? Tonic=AST elevation)

What lab values are


increased ferritin and
characteristic in
transferrin saturation
hemochromatosis?

What labs are elevated in amylase lipase(higher


acute pancreatitis? specificity)
What part of the GI tract is
most frequently involved in sigmoid colon
diverticulosis?

What risk group has an


increased incidence of PUD by smoking
2X?

What type of stones are seen


in patients with RBC
pigment stones
hemolysis,alcoholic
cirrhosis,biliary infection?

When do patients with


early in life(often die within a
Crigler-Najjar syndrome(type
few years)
I) usually present?
Where are pancreatic tumors pancreatic head (with
most often located? obstructive jaundice)

Where is the pain associated


left lower quadrant
with diverticulitis?

Which of the following are


effects of liver cell failure?
Anemia,
hypercoagulation,spider nevi, all, except hyper coagulation
jaundice,
gynecomastia,bleeding
tendency, ankle edema

Which of the following are


risk factors for colorectal
cancer(CRC):
age,smoking,personal and all, except for smoking
family history of colon
cancer, low-fiber diet,
hereditary non-polyposis
CRC?
Which of the following are
risk factors for colorectal
cancer: colorectal villous
adenomas, chronic all, except for Peutz-Jeghers
inflamm.bowel disease,
familial adenomatous
polyposis,Peutz-Jeghers?

Which of the following is a


blind pouch leading off the GI
tract lined by mucosa, diverticulum
muscularis, serosa?
Diverticulosis, diverticulum,
diverticulitis

Which of the following is a


common association between
cholesterol stones and
pigment stones: advanced age
obesity,Crohn's,cystic
fibrosis,age,clofibrate,
estrogens,multiparity,rapid
weight loss?

Which of the following is


associated with perforation,
Diverticulitis
peritonitis, abscesses, or
bowel stenosis?
Which of the following is
associated with
stress:Gilbert's, Dubin- Gilbert's
Johnson syndrome, or
Crigler-Najjar(type 1)?

Which of the following


syndromes have a mildly Gilbert's(Crigler-Najjar has an
decreased UDP-glucuronyl absence of UDP-glucuronyl
transferase: Gilbert's or transferase)
Crigler-Najjar(type 1)?

Which type of liver nodule is


associated with an increased
macronodules
risk of hepatocellular
carcinoma?

(1)Hepatitis B (2)Hepatitis C
With what 6 diseases does (3)Wilson's (4)
hepatocellular carcinoma Hemochromatosis (5)alpha 1
have an association? antitrypsin deficiency (6)
alcoholic cirrhosis
With what cancer is achalasia Increased risk for esophageal
associated? cancer.

An FEV1/FVC ratio greater


than 80% indicates what form Restrictive lung disease
of pulmonary disease?

Are bronchogenic carcinoma


Yes, very common
metastases common?

Decreased FEV1/FVC ratio are


the hallmark of what kind of COPD
pulmonary disease?
It causes increased recoil
How does interstitial fibrosis
(decreased compliance),
create a restrictive lung
thereby limiting alveolar
disease?
expansion.

- Cough - Hemoptysis -
How does lung cancer Bronchial obstruction -
commonly present? (5) Wheezing - Pneumonic 'coin'
lesion on x-ray

It leads to an increase in
How does surfactant
surface tension, resulting in
deficiency cause NRDS?
alveolar collapse

- Maternal steroids before


How to you treat NRDS? birth - Artificial surfactant for
infant
Identify: ivory-white pleural
Ferruginous bodies
plaques in the lung.

In COPD, are lung volumes


Increased (increasedTLC,
increased, decreased, or
increased FRC, increased RV)
normal?

In restrictive lung disease, are


lung volumes increased, Decreased
decreased, or normal?

In what occupations is
asbestosis most commonly Shipbuilders and plumbers
seen? (2)
Name three 'triggers' of - Viral URIs - Allergens -
asthma. Stress

Name three characteristics of


- Ptosis - Miosis - Anhidrosis
Horner's syndrome?

Name two extrapulmonary


- Poor muscular effort: polio,
(poor breathing mechanics)
myasthenia gravis - Poor
causes of restrictive lung
apparatus: scoliosis
disease.

Name two pulmonary (poor - Defective alveolar filling:


lung expansion) causes of pneumonia, ARDS, pulmonary
restrictive lung disease. edema - Interstitial fibrosis
Patients with asbestosis are at Pleural mesothelioma and
increased risk for what? (2) bronchogenic carcinoma

T/F Bronchiectasis is
associated with bronchial
True
obstruction, cystic fibrosis,
and poor ciliary motility.

T/F In obstructive (not


False, FEV1 and FVC are
restrictive) lung disease, FEV1
reduced in both
and FVC are reduced.

T/F Restricted lung expansion


causes decreased total lung
False, decreased VC and TLC
capacity and increased vital
capacity.
T/F Smokers with asbestosis
False, it increases
have a decreased risk of
synergistically
developing cancer.

What are asbestos fibers


coated with hemosiderin in Ferruginous bodies
the lung?

Intra-alveolar exudate -&gt;


What are the characteristics
consolidation; may involve
of lobar pneumonia?
entire lung

What are the clinical findings - Wheezing - Crackles -


of chronic bronchitis? (3) Cyanosis
- Dyspnea - Decreased
What are the clinical findings
breath sounds - Tachycardia
of emphysema? (4)
- Decreased I/E ratio

- Symptoms: gradual
progressive dyspnea and
What are the symptoms and cough - Complications
complications of interstitial include cor pulmonale (can be
lung fibrosis? seen in diffuse interstitial
pulmonary fibrosis and
bleomycin toxicity)

What bronchogenic carcinoma


is associated with ectopic
hormone production (ADH, Small cell carcinoma
ACTH) and may lead to
Lambert-Eaton syndrome?

What bronchogenic carcinoma


is associated with ectopic
Squamous cell carcinoma
PTH-related peptide
production?
What bronchogenic carcinoma
Adenocarcinoma
is most common?

What bronchogenic carcinoma


is thought not to be related Bronchioalveolar carcinoma
to smoking?

What bronchogenic
SSquamous cell carcinoma
carcinomas are clearly linked
and SSmall cell carcinoma
to SSmoking?

What bronchogenic
carcinomas usually express - Squamous cell carcinoma -
tumors that arise centrally? Small cell carcinoma
(2)
What bronchogenic - Adenocarcinoma -
carcinomas usually express Bronchioalveolar carcinoma -
tumors that arise Large cell carcinoma--
peripherally? (3) undifferentiated

What carcinoma occurs in the


apex of the lung and may
affect the cervical Pancoast's tumor
sympathetic, causing Horner's
syndrome?

Chronic necrotizing infection


What causes bronchiectasis?
of bronchi

What causes neonatal


respiratory distress syndrome Surfactant deficiency
(NRDS)?
What cells make surfactant Type II pneumocytes most
and when is it made most abundantly after 35th week of
abundantly in fetuses? gestation

What COPD has a productive


cough for greater than three
months in two years and
hypertrophy of mucus- Chronic bronchitis
secreting glands in the
bronchioles (Reid index
greater than 50%)?

What COPD is characterized


by dilated airways, purulent
Bronchiectasis
sputum, recurrent infections,
and hemoptysis?

What COPD is due to an


enlargement of air spaces and
decreased recoil resulting Emphysema
from destruction of alveolar
walls?
What COPD is due to
bronchial
hyperresponsiveness which Asthma
causes reversible
bronchoconstriction?

What does inhaled asbestos It causes diffuse pulmonary


do to the lungs? interstitial fibrosis

What is the composition of Dipalmitoyl


surfactant: phosphatidylcholine

FEV1 and FVC are reduced in


What is the difference in
both, but in obstructive the
FEV1/FVC ratios between
FEV1 is more dramatically
obstructive and restrictive
reduced, resulting in a
lungs diseases?
decreased FEV1/FVC ratio
What is the leading cause of
Lung cancer
cancer death?

Superior vena caval syndrome


Pancoast's tumor Horner's
What is the SPHERE (acronym) syndrome Endocrine
of complications associated (paraneoplastic) Recurrent
with lung cancer? laryngeal symptoms
(hoarseness) Effusions
(pleural or pericardial)

What kind of pulmonary


diseases are caused by a Obstructive lung diseases
inhibition of air flow resulting (COPD)
in air trapping in the lungs?

What organisms are is the


most frequent cause of lobar Pneumococcus
pnuemonia?
What organisms are the most
- S. aureus - H. flu -
frequent cause of
Klebsiella - S. pyogenes
bronchopnuemonia? (4)

What organisms are the most


- Viruses (RSV, adenoviruses)
frequent cause of interstitial
- Mycoplasma - Legionella
(atypical) pnuemonia? (3)

What specific type of


emphysema is caused by Panacinar emphysema (and
alpha-1-antitrypsin liver cirrhosis)
deficiency?

What specific type of


emphysema is caused by Centriacinar emphysema
smoking?
The lecithin-to-
sphingomyelin ratio in the
What test is used to measure
amniotic fluid, usually less
in utero lung maturity?
than 1.5 in neonatal
respiratory distress syndrome

What type of lung cancer can


Carcinoid tumor
cause carcinoid syndrome?

What type of pneumonia is


characterized by acute
inflammatory infiltrates from
bronchioles into adjacent Bronchopneumonia
alveoli with a patchy
distribution affecting more
than one lobe?

What type of pneumonia is


characterized by diffuse
patchy inflammation localized Interstitial (atypical)
to interstial areas at alveolar pneumonia
walls and involves more than
one lobe?
Where does bronchogenic
- Brain (epilepsy) - Bone
carcinoma commonly
(pathologic fracture) - Liver
metastasize and how does it
(jaundice, hepatomegaly)
present? (3)

Why does alpha-1-antitrypsin


Increased elastase activity
deficiency cause emphysema?

Epilepsy is a disorder of
Define epilepsy.
recurrent seizures.

Define syrinx. Tube, as in syringe


Describe a myoclonic seizure. Quick,repetitive jerks

Describe a tonic-clonic Alternating stiffening and


seizure. movement (grand mal)

Describe a tonic seizure. Stiffening

A blank stare (petit mal- it's


Describe an absence seizure.
in 1st aid this way!!)
Describe an atonic seizure. drop' seizures

Broca's is nonfluent aphasia


Describe Broca's aphasia. with intact comprehension.
BROca's is BROken speech.

Sympathectomy of face
(lesion above T1).
Describe Horner's syndrome.
Interruption of the 3-neuron
oculosympathetic pathway.

Wernicke's is fluent aphasia


with impaired
Describe Wernicke's aphasia.
comprehension. Wernicke's is
Wordy but makes no sense.
How do patients present with
Worst headache of my life'
a subarachnoid hemorrhage?

-Optic neuritis (sudden loss


of vision) - MLF syndrome
(internuclear
How do pts present with MS? ophthalmoplegia) -
Hemiparesis -Hemisensory
symptoms -Bladder/bowel
incontinence

Through the bloodstream to


How does it spread?
the CNS

How does Werdnig-Hoffman


At birth as a 'floppy baby'
disease present?
How is Huntington's disease
Autosomal dominant
inherited?

How is the polio virus


Fecal-oral route
transmitted?

Higher prevalence with


How is the prevalence of MS
greater distance from the
geographically distributed?
Equator

In what persons is subdural Elderly individuals, alcoholics,


hemorrhage often seen? and blunt trauma
T/F. Partial seizures can not False- Partial seizures can
generalize. generalize.

What are 2 common


1. Toxo!Toxo!Toxo! 2.
organisms that target the
Cryptococcus
brain in AIDS pts?

What are 2 degenerative


1. Alzheimer's 2. Pick's
diseases of the cerebral
disease
cortex?

What are 2 degenerative


1. Huntington's disease 2.
diseases that affect the basal
Parkinson's disease
ganglia and brain stem?
What are 3 degenerative 1. Amyotrophic lateral
disorders of the motor sclerosis (ALS) 2. Werdnig-
neuron? Hoffman disease 3. Polio

1. Infections (herpesvirus or
What are associated with
C. jejuni) 2. Inoculations 3.
Guillain-Barre?
Stress

What are neurofibrillary Abnormally phosphorylated


tangles? tau protein

1. Multiple sclerosis (MS) 2.


Progressive multifocal
leukoencephalopathy (PML) 3.
What are some demyelinating Postinfectious
and dysmyelinating diseases? encephalomyelitis 4.
Metachromatic
Leukodystrophy 5. Guillain-
Barre syndrome
1. Epidural hematoma 2.
What are the 4 types of Subdural Hematoma 3.
intracranial hemorrhages? Subarachnoid hemorrhage 4.
Parenchymal hematoma

1. Absence 2. Myoclonic 3.
What are the 5 types of
Tonic-clonic 4. Tonic 5.
generalized seizures?
Atonic

What are the clinical


symptoms of Huntington's Dementia, chorea
disease?

TRAP= Tremor (at rest)


What are the clinical
cogwheel Rigidity Akinesia
symptoms of Parkinson's
Postural instability (you are
disease?
TRAPped in your body)
-Charcot joints -Shooting
What are the clinical pain -Argyll-Robertson Pupils
symptoms of Tabes dorsalis? -Absence of deep tendon
reflexes

What are the common causes -Tumors -Trauma -Stroke -


of seizures in adults? Infection

What are the common causes -Genetic -Infection -Trauma


of seizures in children? -Congenital -Metabolic

What are the common causes -Stroke -Tumor -Trauma -


of seizures in the elderly? Metabolic -Infection
Elevated CSF protein with
What are the lab findings in
normal cell count ('albumino-
Guillain-Barre syndrome?
cytologic dissociation')

-CSF with lymphocytic


pleocytosis with slight
What are the lab findings in
elevation of protein -Virus
poliomyelitis?
recovered from stool or
throat

What are the pathological


-Necrosis -Hemorrhage -
signs of glioblastoma
Pseudo-palisading
multiforme (GBM)?

-Muscle weakness and


What are the signs of LMN
atrophy -Fasciculations -
lesions seen in poliomyelitis?
Fibrillation -Hyporeflexia
1. Ptosis 2. Miosis 3.
What are the symptoms of
Anhidrosis and flushing of
Horner's?
affected side of face

-Malaise -Headache -Fever -


What are the symptoms of
Nausea -Abdominal pain -
poliomyelitis?
sore throat

What area of the brain is


affected by generalized Diffuse area
seizures?

What artery is compromised


Middle meningeal artery
in an epidural hematoma?
What blood vessels are
affected in subdural Rupture of bridging veins
hemorrhages?

What causes a parenchymal -HTN -Amyloid angiopathy -


hematoma? Diabetes Mellitus -Tumor

What causes poliomyelitis? Poliovirus

What chemical can


MPTP, a contaminant in illicit
Parkinson's disease be linked
street drugs
to?
Bilateral pain and
What clinical symptoms are temperature loss in the upper
present with syringomyelia? extremities with preservation
of touch sensation

-Symmetric ascending muscle


weakness beginning in the
What clinical symptoms are distal lower extremities -
present? Facial diplegia in 50% of cases
-Autonomic fx may be
severely affected

What congenital malformation


is often associated with Arnold Chiari Malformation
syringomyelia?

What damage does


cryptococcus cause in the Periventricular calcifications
brain?
What damage does
Diffuse (intracerebral)
toxoplasma cause in the
calcifications
brain?

-Adult polycystic kidney


What diseases are berry disease -Ehlers-Danlos
aneurysms associated with? syndrome -Marfan's
syndrome

What do partial seizures


One area of the brain
affect?

What does rupture of a berry


Stroke
aneurysm lead to?
What does the spinal tap
show in a subarachnoid Bloody or xanthochromic
hemorrhage?

What event is the rupture of


the middle meningeal artery Temporal bone fracture
secondary to?

What genes is the familial


Genes are chromosomes 1,
form of Alzheimer's
14, 19 and 21
associated with?

What is a complex partial


Impaired awareness
seizure?
What is a degenerative
Friedrich's ataxia
disorder of the
(olivopontocerebellar atrophy)
Spinocerebellar tract?

What is anhidrosis? Absence of sweating

What is another name for


Acute idiopathic polyneuritis
Guillain-Barre syndrome?

What is another symptom of


Tongue fasciculations
Werdnig-Hoffman disease?
What is another term for
Expressive aphasia
Broca's aphasia?

What is another term for


Receptive aphasia
Wernicke's aphasia?

What is miosis? Pupil constriction

What is PML associated with? JC virus


What is ptosis? Slight drooping of the eyelids

SIN 1. Scanning speech 2.


What is the classic triad of
Intention Tremor 3.
MS?
Nystagmus

What is the common name for


Lou Gehrig's disease
ALS?

Venous bleeding (less


What is the course of a
pressure) with delayed onset
subdural hemorrhage?
of symptoms
In most pts, the course is
What is the course of MS?
remitting and relapsing

Metastases&gt; Astrocytoma
What is the incidence of brain
(including glioblastoma)&gt;
tumors in adults?
Meningioma

Astrocytoma&gt;
What is the incidence of brain
Medulloblastoma&gt;
tumors in children?
Ependymoma

What is the most common


cause of dementia in the Alzheimer's disease
elderly?
What is the most common
complication of a berry Rupture of the aneurysm
aneurysm?

The bifurcation of the


What is the most common
anterior communicating
site for a berry aneurysm?
artery

Inflammation and
What is the pathogenesis of demyelination of peripheral
Guillain-Barre syndrome? nerves and motor fibers of
ventral roots (sensory effect
less severe than motor)

Degeneration of the dorsal


What is the pathogenesis of
columns and dorsal roots due
Tabes dorsalis?
to tertiary syphilis.
Associated with senile
What is the pathology of
plaques (beta-amyloid core)
Alzheimer's?
and neurofibrillary tangles

What is the pathology of Atrophy of the caudate


Huntington's disease? nucleus

-Periventricular plaques -
Preservation of axons -Loss
What is the pathology of MS? of oligodendrocytes -Reactive
astrocytic gliosis -Increased
protein (IgA) in CSF

Associated with Lewy bodies


What is the pathology of
and depigmentation of the
Parkinson's disease?
substantia nigra
Associated with Pick bodies,
What is the pathology of
intracytoplasmic inclusion
Pick's disease?
bodies

Destruction of anterior horn


What is the pathology of
cells, leading to LMN
poliomyelitis?
destruction

Softening and cavitation


What is the pathology of
around the central canal of
syringomyelia?
the spinal cord.

What is the prognosis for a


Very poor
pts diagnosed with a GBM?
What is the second most
common cause of dementia in Multi-infarct dementia
the elderly?

What is the shape of GBMs? Butterfly' glioma

What neural deficits are Impaired proprioception and


present in Tabes dorsalis? locomotor ataxia

What neural tracts are Crossing fibers of the


damaged? spinothalamic tract
What neurons are affected in Both the upper and lower
ALS? motor neurons

What neurons are affected in


Lower motor neurons only
Polio?

What seizures are categorized Awareness intact -Motor -


as simple partial? Sensory -Autonomic -Psychic

What tumor is Horner's


Pancoast's tumor
syndrome associated with?
Where are most brain tumors 70% are supratentorial
located in adults? (cerebral hemispheres)

Where are most childhood 70% below tentorium


brain tumors located? (cerebellum)

Where do berry aneurysms At the bifurcations in the


occur? Circle of Willis

Where does it initially The oropharynx and small


replicate? intestine
Where does the 3 neuron
oculosympathetic pathway The hypothalamus
project from?

1. Interomediolateral column
of the spinal cord 2. Superior
Where does the 3 neuron
cervical (sympathetic)
oculosympathetic pathway
ganglion 3. To the pupil,
project to?
smooth muscles of the
eyelids and the sweat glands

Where is Broca's area located? Inferior frontal gyrus

Where is Pick's disease The frontal and temporal


specific for? lobes
Where is the aopE-4 allele
Chromosome 19
located?

Where is the most common


C8-T1
site of syringomyelia?

Where is the p-App gene


21
located?

Where is Wernicke's area


Superior Temporal Gyrus
located?
Which demyelinating disease
is seen in 2-4% of AIDS PML
patients?

Chronic inflammatory disease


Define Ankylosing of spine &amp; large joints,
spondylitis? sacroilitis, uveitis, &amp;
aortic regurgitation

Chronic inflammatory disease


Define Ankylosing of spine &amp; large joints,
spondylitis? sacroilitis, uveitis, &amp;
aortic regurgitation

Autoimmune-mediated
Define Celiac sprue intolerance of gliadin (wheat)
leading to steatorrhea.
Autoimmune-mediated
Define Celiac sprue intolerance of gliadin (wheat)
leading to steatorrhea.

Precipitation of monosodium
Define Gout. urate crystals into joints due
to hyperuricemia.

Precipitation of monosodium
Define Gout. urate crystals into joints due
to hyperuricemia.

Excessive fibrosis &amp;


collagen deposition
Define Scleroderma throughout the body;
commonly sclerosis of the
skin, but also of CV &amp; GI
systems &amp; kidney
Excessive fibrosis &amp;
collagen deposition
throughout the body;
Define Scleroderma
commonly sclerosis of the
skin, but also of CV &amp; GI
systems &amp; kidney

dry eyes, dry mouth, nasal


&amp; vaginal dryness,
Define Sicca syndrome.
chronic bronchitis, reflux
esophagitis

dry eyes, dry mouth, nasal


&amp; vaginal dryness,
Define Sicca syndrome.
chronic bronchitis, reflux
esophagitis

In what population is
ankylosing sponsylitis more males (10-30 year old)
commonly found?
In what population is
ankylosing sponsylitis more males (10-30 year old)
commonly found?

In what population is Celiac Assoc. w/ people of northern


sprue more commonly found? European descent

In what population is Celiac Assoc. w/ people of northern


sprue more commonly found? European descent

In what population is
Goodpasture's syndrome Men 20-40 y/o
more commonly found?
In what population is
Goodpasture's syndrome Men 20-40 y/o
more commonly found?

In what population is gout


Men
more commonly found?

In what population is gout


Men
more commonly found?

In what population is
Osteoarthritis more Common in older patients
commonly found?
In what population is
Osteoarthritis more Common in older patients
commonly found?

In what population is
&gt; 50 y/o, both sexes
pseudogout more commonly
equally
found?

In what population is
&gt; 50 y/o, both sexes
pseudogout more commonly
equally
found?

In what population is Reiter's


syndrome more commonly Strong predilection for males
found?
In what population is Reiter's
syndrome more commonly Strong predilection for males
found?

In what population is
- Common in females - 80%
Rheumatoid arthritis more
of RA pt's have positive
commonly found &amp; what
rheumatoid factor (anti-IgG
the common autoimmune
Ab)
factor present?

In what population is
- Common in females - 80%
Rheumatoid arthritis more
of RA pt's have positive
commonly found &amp; what
rheumatoid factor (anti-IgG
the common autoimmune
Ab)
factor present?

In what population is
sarcoidosis more commonly black females
found?
In what population is
sarcoidosis more commonly black females
found?

In what population is
scleroderma more commonly 75% female
found?

In what population is
scleroderma more commonly 75% female
found?

In what population is
females between the ages of
Sjogren's syndrome more
40 &amp; 60
commonly found?
In what population is
females between the ages of
Sjogren's syndrome more
40 &amp; 60
commonly found?

90% are female &amp;


In what population is SLE between ages 14 &amp; 45.
more commonly found? More common &amp; severe
in black females

90% are female &amp;


In what population is SLE between ages 14 &amp; 45.
more commonly found? More common &amp; severe
in black females

Diffuse scleroderma: widespread skin


involvement, rapid progression, early
visceral involvement. Assoc. w/ anti-
What are the 2 major
Scl-70 Ab CREST syndrome:
categories of scleroderma Calcinosis, Raynaud's phenomenon,
&amp; what findings are they Esophageal dysmotility, Sclerodactyly
assoc w/? &amp; Telangiectasia; limited skin
involvement, often confined to fingers
&amp; face. More benign clinical
course - assoc w/ anticentromere Ab
Diffuse scleroderma: widespread skin
involvement, rapid progression, early
visceral involvement. Assoc. w/ anti-
What are the 2 major
Scl-70 Ab CREST syndrome:
categories of scleroderma Calcinosis, Raynaud's phenomenon,
&amp; what findings are they Esophageal dysmotility, Sclerodactyly
assoc w/? &amp; Telangiectasia; limited skin
involvement, often confined to fingers
&amp; face. More benign clinical
course - assoc w/ anticentromere Ab

What are the associated sx's - Parotid enlargement - incr


&amp; risks for Sjogren's risk of B-cell lymphoma -
syndrome? Assoc. w/ RA

What are the associated sx's - Parotid enlargement - incr


&amp; risks for Sjogren's risk of B-cell lymphoma -
syndrome? Assoc. w/ RA

Blunting of villi, lymphocytes


What are the characteristic
in the lamina propria, &amp;
findings in Celiac sprue?
abnormal D-xylose test
Blunting of villi, lymphocytes
What are the characteristic
in the lamina propria, &amp;
findings in Celiac sprue?
abnormal D-xylose test

immune-mediated,
What are the common widespread noncaseating
characteristics of Sarcoidosis? granulomas &amp; elevated
serum ACE levels

immune-mediated,
What are the common widespread noncaseating
characteristics of Sarcoidosis? granulomas &amp; elevated
serum ACE levels

pulmonary hemorrhages,
What are the common gross
renal lesions, hemoptysis,
findings in Goodpasture's
hematuria, crescentic
syndrome?
glomerulonephritis
pulmonary hemorrhages,
What are the common gross
renal lesions, hemoptysis,
findings in Goodpasture's
hematuria, crescentic
syndrome?
glomerulonephritis

Lesch-Nyan disease, PRPP


excess, decreased excretion
of uric acid, or G6PD
What can cause gout? deficiency. Also assoc. w/ the
use of thiazide diuretics
which competitively ingibit
the secretion of uric acid.

Lesch-Nyan disease, PRPP


excess, decreased excretion
of uric acid, or G6PD
What can cause gout? deficiency. Also assoc. w/ the
use of thiazide diuretics
which competitively ingibit
the secretion of uric acid.

deposition of calcium
What causes pseudogout? pyrophosphate crystals w/in
the joint space
deposition of calcium
What causes pseudogout? pyrophosphate crystals w/in
the joint space

SLE causes LSE (Libman-Sacks


Endocarditis): vavular
What CV disease state can be vegetations found on both
caused by SLE? sides of valve (mitral valve
stenosis) &amp; do not
embolize

SLE causes LSE (Libman-Sacks


Endocarditis): vavular
What CV disease state can be vegetations found on both
caused by SLE? sides of valve (mitral valve
stenosis) &amp; do not
embolize

What drugs can induce a


- procainamide - INH -
commonly reversible SLE-like
phenytoin - hydralazine
syndrome?
What drugs can induce a
- procainamide - INH -
commonly reversible SLE-like
phenytoin - hydralazine
syndrome?

90% of cases are assoc w/


What immune marker aids in
B27 (gene which codes for
dx?
HLA MHC-I)

90% of cases are assoc w/


What immune marker aids in
B27 (gene which codes for
dx?
HLA MHC-I)

What is characteristic about needle-shaped &amp;


gout crystals? negatively berefringent.
What is characteristic about needle-shaped &amp;
gout crystals? negatively berefringent.

What is characteristic about


basophilic, rhomboid crystals
pseudogout crystals?

What is characteristic about


basophilic, rhomboid crystals
pseudogout crystals?

a seronegative
What is Reiter's syndrome? spondyloarthropath w/ a
HLA-B27 link
a seronegative
What is Reiter's syndrome? spondyloarthropath w/ a
HLA-B27 link

1. Urethritis (Can't pee) 2.


What is the 'classic triad' for Conjunctivities &amp; ant.
Reiter's syndrome? uveitis (Can't see) 3. Arthritis
(Can't climb a tree)

1. Urethritis (Can't pee) 2.


What is the 'classic triad' for Conjunctivities &amp; ant.
Reiter's syndrome? uveitis (Can't see) 3. Arthritis
(Can't climb a tree)

1. dry eyes (conjunctivitis,


What is the 'classic triad' for xerophthalmia) 2. dry mouth
Sjogren's syndrome? (dysphagia, xerostomia) 3.
arthritis
1. dry eyes (conjunctivitis,
What is the 'classic triad' for xerophthalmia) 2. dry mouth
Sjogren's syndrome? (dysphagia, xerostomia) 3.
arthritis

Mechanical: wear &amp; tear


of joints leads to destruction
What is the classic pathology of articular cartilage,
for Osteoarthritis? subchondral bone formation,
sclerosis, osteophytes,
eburnation, &amp;
Heberden's nodes (DIP)

Mechanical: wear &amp; tear


of joints leads to destruction
What is the classic pathology of articular cartilage,
for Osteoarthritis? subchondral bone formation,
sclerosis, osteophytes,
eburnation, &amp;
Heberden's nodes (DIP)

Autoimmune: inflammatory
d/o affecting synovial joints,
What is the classic pathology w/ pannus formation in joints
for Rheumatoid arthritis? (MCP, PIP), subcutaneous
rheumatoid nodules, ulnar
deviation, subluxation.
Autoimmune: inflammatory
d/o affecting synovial joints,
What is the classic pathology w/ pannus formation in joints
for Rheumatoid arthritis? (MCP, PIP), subcutaneous
rheumatoid nodules, ulnar
deviation, subluxation.

pain in weight-bearing joints


What is the Classic
after use (e.g.- at the end of
presentation for
the day), improving w/ rest.
Osteoarthritis?
No systemic sx's

pain in weight-bearing joints


What is the Classic
after use (e.g.- at the end of
presentation for
the day), improving w/ rest.
Osteoarthritis?
No systemic sx's

morning stiffness improving


What is the classic w/ use, symmetric joint
presentation for Rheumatoid involvement &amp; systemic
arthritis? symptoms: fever, fatigue,
pleuritis, pericarditis
morning stiffness improving
What is the classic w/ use, symmetric joint
presentation for Rheumatoid involvement &amp; systemic
arthritis? symptoms: fever, fatigue,
pleuritis, pericarditis

Anti-glomerular basement
What is the common
membrane antibodies
immunologic finding for
produce linear staining on
Goodpasture's syndrome?
immunofluorescence

Anti-glomerular basement
What is the common
membrane antibodies
immunologic finding for
produce linear staining on
Goodpasture's syndrome?
immunofluorescence

What is the common tx for allopurinol, probenecid,


gout? colchicine, &amp; NSAID's.
What is the common tx for allopurinol, probenecid,
gout? colchicine, &amp; NSAID's.

What is the common tx for


no tx
pseudogout?

What is the common tx for


no tx
pseudogout?

GRAIN Gammaglobulinemia
What is the descriptive Rheumatoid arthritis ACE
acrynym for Sarcoidosis? incr. Interstitial fibrosis
Noncaseating granulomas
GRAIN Gammaglobulinemia
What is the descriptive Rheumatoid arthritis ACE
acrynym for Sarcoidosis? incr. Interstitial fibrosis
Noncaseating granulomas

there are TWO Good Pastures


What is the useful memory for this disease: Glomerulus
tool for Goodpasture's &amp; Pulmonary. Also, a
syndrome? type II (TWO) hypersensitivity
disease

there are TWO Good Pastures


What is the useful memory for this disease: Glomerulus
tool for Goodpasture's &amp; Pulmonary. Also, a
syndrome? type II (TWO) hypersensitivity
disease

restrictive lung disease, bilateral


hilar lypmphadenopathy, erythema
nodosum, Bell's palsy, epithelial
What sx's is sarcoidosis granulomas containing microscopic
Schaumann &amp; asteroid
commonly associated w/?
mobies, uveoparotitis, &amp;
hypercalcemia (due to elevated
conversion of vit. D to its active
form in epithelioid macrophages)
restrictive lung disease, bilateral
hilar lypmphadenopathy, erythema
nodosum, Bell's palsy, epithelial
What sx's is sarcoidosis granulomas containing microscopic
Schaumann &amp; asteroid
commonly associated w/?
mobies, uveoparotitis, &amp;
hypercalcemia (due to elevated
conversion of vit. D to its active
form in epithelioid macrophages)

What the common pattern of Usually affects large joints


psudogout presentation? (classically the knee)

What the common pattern of Usually affects large joints


psudogout presentation? (classically the knee)

Asymmetric joint distribution.


Favored manifestation is
Where is gout commonly painful MTP joint in the big
manifested? toe (podagra). Tophus
formation (often on external
ear or Achilles tendon)
Asymmetric joint distribution.
Favored manifestation is
Where is gout commonly painful MTP joint in the big
manifested? toe (podagra). Tophus
formation (often on external
ear or Achilles tendon)

Adrenal Atrophy and Absence


Addison's disease is
of hormone production;
characterized by what
involves All three cortical
(remember 4 A's)?
divisions

All MEN syndromes have what


Autosomal Dominant
mode of inheritance?

An increased risk of
carcinoma with atypical cells
Epithelial hyperplasia
is seen in which type of
fibrocystic breast disease?
Blood filled, 'chocolate cysts'
Endometriosis
are seen in what condition?

- diarrhea - cutaneous
Carcinoid tumors result in flushing - asthmatic
what recurrent symptoms? (4) wheezing - right-sided
valvular disease

Those affected were


Cretin means 'Christ-like,' considered so mentally
why is that name used? retarded as to be incapable of
sinning.

Excess fat breakdown (usually


due to an increase in insulin
Diabetic ketoacidosis (DKA) is requirements) and increased
caused by what? ketogenesis from the increase
in free fatty acids, which are
then made into ketone bodies
Do leiomyosarcomas derive
No, they usually arise de novo
from other known tumors?

From where do
leiomyosarcomas often Cervix
protrude?

How do you treat carcinoid


Treat with octreotide
syndrome?

How do you treat


postmenopausal Estrogen replacement therapy
osteoporosis?
Hydatiform moles result in an
Beta-HCG
increase in what hormone?

Increase in what substance is


seen in the urine due to 5-HIAA
carcinoid tumors?

Is ACTH increased or
decreased when increased
cortisol is due to a primary Decreased
adrenal hyperplasia or
neoplasia?

Is ACTH increased or
decreased when increased
Decreased
cortisol is due to an
iatrogenic etiology?
Is ACTH increased or
decreased when increased
Increased
cortisol is due to Cushing's
disease?

Is ACTH increased or
decreased when increased
cortisol is due to ectopic Increased
ACTH production (e.g.,
carcinoid)?

Is plasma renin low or high in


Low
primary hyperaldosteronism?

Is plasma renin low or high in


secondary High
hyperaldosteronism?
Leiomyomas are sensitive to
Estrogen
what?

- Fasting serum glucose -


Name 3 common tests for
Glucose tolerance test -
evaluating DM?
HbA1c

- Coronary artery disease -


Name 3 specific example of
Peripheral vascular occlusive
large vessel atherosclerosis
disease and gangrene -
due to DM?
Cerebrovascular disease

1. Renal artery stenosis 2.


Name five possible causes of
Chronic renal failure 3. CHF
secondary
4. Cirrhosis 5. Nephritic
hyperaldosteronism?
syndrome
Ectopic ADH CNS disorders/
Name four causes of SIADH. head trauma Pulmonary
disease Drugs

1. Increased frequency of
Name four common urination 2. Nocturia 3.
presenting symptoms of Difficulty starting and
benign prostatic hyperplasia. stopping the stream of urine
4. Dysuria

- Small vessel disease - Large


Name four important chronic vessel atherosclerosis -
manifestations of DM. Neuropathy - Cataracts,
glaucoma

Name four risk factors of 1. Prolonged estrogen use 2.


endometrial carcinoma. Obesity 3. Diabetes 4. HTN
Name four treatment - Weight loss - OCPs -
modalities for polycystic Gonadotropin analogs -
ovarian syndrome. Surgery

1. Gender 2. Age 3. Early first


menarche (under 12) 4.
Name six risk factors of delayed first pregnancy (over
breast disease. 30) 5. Late menopause (over
50) 6. Family history of first
degree relative with breast
cancer at a young age.

- Polydipsia - Polyuria -
Polyphagia - Weight loss -
Name some of the acute
DKA (type1) - Hyperosmolar
manifestations of Diabetes
coma (type 2) - Unopposed
Mellitus (DM)? (7)
secretion of GH and Epi
(exacerbating hyperglycemia)

Name syndrome: increased


LH due to peripheral estrogen
production leads to Polycystic ovarian syndrome
anovulation and may manifest
in amenorrhea, infertility,
obesity, and hirsutism.
Name the autoimmune
hyperthyroidism with TSH
receptor antibodies, Graves' disease
opthalmopathy, pretibial
myxedema, and diffuse
goiter.

Name the benign breast


tumor: most common tumor
under 25 years; small, Fibroadenoma
mobile, firm mass with sharp
edges.

Name the benign breast


tumor: tumor of lactiferous
Intraductal papilloma
ducts; presents with nipple
discharge.

Name the benign breast


tumor: large, bulky mass of
connective tissue and cysts; Cystosarcoma phyllodes
breast surface has 'leaflike'
appearance.
Name the disease caused by
primary deficiency of
Addison's disease
aldosterone and cortisol due
to adrenal atrophy?

Name the histologic type of


fibrocystic breast disease: Cystic
fluid-filled.

Name the histologic type of


fibrocystic breast disease: Fibrosis
hyperplasia of breast stroma.

Name the histologic type of


fibrocystic breast disease:
increase in number of Epithelial hyperplasia
epithelial cell layers in
terminal duct lobule.
Name the histologic type of
fibrocystic breast disease:
Sclerosing
increased acini and
intralobular fibrosis.

Name the histologic type of


malignant breast disease:
Comedocarcinoma
cheesy consistency of tumor
tissue due to central necrosis.

Name the histologic type of


malignant breast disease:
Paget's disease
eczematous patches on
nipple.

Name the histologic type of


malignant breast disease:
Inflammatory
lymphatic involvement; poor
prognosis.
Name the histologic type of
malignant breast disease:
Infiltrating ductal
most common carcinoma;
firm, fibrous mass.

1. Excessive water retention


Name three characteristics of 2. Hyponatremia 3. Serum
inappropriate ADH secretion hypo-osmolarity with urine
(SIADH). osmolarity &gt; serum
osmolarity

1. Vertebral crush fractures 2.


Name three fractures
Distal radius (Colle's)
commonly seen due to
fractures 3. Vertebral wedge
osteoporosis.
fractures

Pheochromocytomas may be
1. Neurofibromatosis 2. MEN
associated with what 3
type II 3. MEN type III
diseases?
T/F A partial hydatiform mole
is commonly triploid or True
tetraploid.

T/F DKA is common in Type 1


True
DM.

T/F DKA is common in Type 2


False, rare
DM.

T/F Endometriosis often


manifests with severe
True
menstrual-related pain and
often with infertility?
T/F Fibrocystic breast disease
usually does not indicate True
increased risk of carcinoma.

T/F Genotype of a complete


hydatiform mole is 46, XX False, it is 46, XX and Purely
and is purely maternal in Paternal in origin
origin.

T/F Glucose intolerance in


True
Type 1 DM is severe.

T/F Insulin is always


True
necessary to treat Type 1 DM.
T/F Insulin is always
False, sometimes
necessary to treat Type 2 DM.

T/F Leiomyomas often


True
present with multiple tumors.

T/F Leiomyomas often


transform into malignant False, it is rare
tumors.

T/F Leiomyomas progress to


False
leiomyosarcomas.
T/F Leiomyosarcomas: highly
aggressive, have a tendancy
True
to recur, and have an
increased incidence in blacks.

T/F Risk of breast disease is


increased by fibroadenoma False
and nonhyperplastic cysts.

T/F Type 1 diabetes is often


False
associated with obesity.

False. It is polygenic and only


T/F Type 1 diabetes is
weakly associated with
polygenic and strongly due to
genetic disposition, whereas
genetic disposition?
Type 2 is strongly associated.
T/F Women with endometrial
hyperplasia are at increased
risk for endometrial True
carcinoma which tends to
manifest with vaginal
bleeding?

Urinary VMA levels and


plasma catecholamines are
Pheochromocytoma
elevated due to what
neoplasms?

- hypertension - weight gain


- moon facies - truncal
obesity - buffalo hump -
What are the clinical effects of hyperglycemia (insulin
increased cortisol? (9) resistance) - skin changes
(thinning striae) -
osteoporosis - immune
suppression

What are the clinical effects of - Hypertension -


primary hyperaldosteronism? Hypokalemia - Metabolic
(4) alkalosis - Low plasma renin
What are the episodic
- Pressure - Pain (headache)
hyperadrenergic symptoms (5
- Perspiration - Palpitations -
P's) due to
Pallor/diaphoresis
pheochromocytomas?

Insulin deficiency (or


What are the primary
inefficiency) and glucagon
hormonal causes of DM?
excess

- 10% Malignant - 10%


What are the six 'Rule of 10's'
bilateral - 10% extraadrenal -
associated with
10% calcify - 10% kids - 10%
pheochromocytomas?
familial

Type 1 - viral or immune


What are theorized causes for destruction of pancreatic beta
Types 1 and 2 DM? cells Type 2 - Increased
resistance to insulin
What benign breast tumor
increases in size and Fibroadenoma
tenderness with pregnancy?

What breast disease is


common in postmenopausal
women and arises from Malignant tumors (carcinoma)
mammary duct epithilium or
lobular glands?

What causes Cushing's


Increased cortisol
Syndrome?

Defect in T4 formation or
What causes sporadic
developmental failure in
cretinism?
thyroid formation.
What condition can produce all
these symptoms: cold
intolerance, hypoactivity, weight
gain, fatigue, lethargy, decreased Hypothyroidism
appetite, constipation, weakness,
decreased reflexes, myxedema
(facial/periorbital), dry, cool skin,
and coarse, brittle hair?

What condition can produce


all these symptoms: heat
intolerance, hyperactivity,
weight loss, chest pain/ Hyperthyroidism
palpitations, arrhythmias,
diarrhea, increased reflexes,
warm, moist skin, and fine
hair?

What condition is associated


with the expressions
'honeycombed uterus' and Hydatiform mole
'cluster of grapes'
appearance?

What condition is caused by


increased bone resorption
due to decreased estrogen Type 1 Osteoporosis
levels (seen postmenopausal
by 10-15 years)?
What condition is
characterized by non-
neoplastic endometrial Endometriosis
glands/stroma in abnormal
locations outside the uterus?

What disease is characterized


by intense thirst and polyuria
together with an inability to
concentrate urine with fluid diabetes insipidus
restriction owing to lack of
ADH or to a lack of renal
response to ADH. Caused by
lithium demeclocycline.
What disease may be due to an
age-related increase in estradiol
with sensitization of the prostate
to the growth promoting effects
of DHT? It is characterized by Benign prostatic hyperplasia
nodular enlargement of the
periurethral lobes of the prostate
gland compressing the urethra
into a vertical slit?

What diuretic acts as an


aldosterone antagonist used
Spironolactone
to treat primary
hyperaldosteronism?
What endocrine pathology
may produce these findings:
pot-bellied, pale, puffy-faced Cretinism
child with protruding
umbilicus and protuberant
tongue?

What fractures cause acute


back pain, loss of height, and Vertebral crush fractures
kyphosis?

What gynecologic tumor is


often bulky with areas of Leiomyosarcoma
necrosis and hemorrhage?

What is a pathologic ovum


resulting in cystic swelling of
chorionic villi and Hydatiform mole
proliferation of chorionic
epithelium?
What is an abnormal
endometrial gland
proliferation usually caused Endometrial hyperplasia
by excess estrogen
stimulation?

1/3 metastasize 1/3 present


What is the 'Rule of 1/3s' for
with second malignancy 1/3
carcinoid tumors?
multiple

What is the etiology of


Cushing's Syndrome caused Primary pituitary adenoma
by Cushing's disease?

What is the etiology of


An aldosterone-secreting
primary hyperaldosteronism
tumor
(Conn's syndrome)?
What is the etiology of small Diffuse thickening of the
vessel disease due to DM? basement membrane

What is the most common


gynecologic malignancy, with Endometrial carcinoma
a peak age of 55-65 y/o?

What is the most common of


Leiomyoma
all tumors in females?

What is the most common


site of ectopic endometrial Ovary
tissue?
What is the most common
tumor of the adrenal medulla Pheochromocytoma
in adults?

What is the most common


tumor of the adrenal medulla Neuroblastoma
in children?

What is the most common


Carcinoid tumor
tumor of the appendix?

Kidney perception of low


What is the pathophysiology
intravascular volume results
of secondary
in an overactive renin-
hyperaldosteronism?
angiotensin system.
What metabolic reaction is
responsible for the chronic Nonenzymatic glycosylation
manifestations of DM?

What neoplasms secrete high


levels of serotonin (5HT) that Carcinoid tumors
does not get metabolized by (neuroendocrine cells)
the liver due to liver especially of the small bowel
metastases?

What non-selective,
irreversible alpha blocker is
Phenoxybenzamine
used to treat pts with
pheochromocytomas?

What phenotypic difference


Primary insufficiency results
can distinguish between
in hyperpigmentation due to
primary and secondary
increased MSH.
causes of Addison's disease?
What substance causes
cataract formation in DM Sorbitol accumulation
patients?

What syndrome is caused by a


gastrin-secreting tumor that
is usually located in the Zollinger-Ellison syndrome
pancreas, causes recurrent
ulcers, and may be associated
with MEN type 1?

What three organs (3 P's) are - Pancreas - Pituitary -


involved in MEN type I? Parathyroid

What type of respirations are


Kussmaul respiration
seen in diabetic ketoacidosis?
- Retinopathy - hemorrhage,
exudates, and
Where (and in what forms) is microaneurysms -
small vessel disease from DM Nephropathy - nodular
seen most prominently? sclerosis, progressive
proteinuria, chronic renal
failure, arteriosclerosis
leading to HTN

Wherever endemic goiter is


Where does endemic
prevalent (lack of dietary
cretinism occur?
iodine).

life threatening
mucormycosis, Rhizopus
Whether the complications of
infection, cerebral edema,
diabetes Q. as it does is
cardiac arrhythmias, heart
failure

Which type of Multiple


Endocrine Neoplasia (MEN) is
associated with medullary
carcinoma of the thyroid, Type III (formerly MEN IIb)
pheochromocytoma, and oral
and intestinal
ganglioneuromatosis
(mucosal neuromas)?
Which type of Multiple
Endocrine Neoplasia (MEN) is
associated with medullary
carcinoma of the thyroid, Type II (Sipple's syndrome)
pheochromocytoma,
parathyroid tumor, or
adenoma?

Which type of Multiple


Endocrine Neoplasia (MEN) is
associated with pancreas (e.g. Type I (Wermer's syndrome)
ZE syndrome, insulinomas,
VIPomas), parathyroid and
pituitary tumors?

Which type of osteoporosis


affects men and women over Type 2 (Senile) Osteoporosis
70 y/o?

Hyperglycemia increases
intracellular sorbitol (which is
Why is intracellular
associated with depletion)
myoinositol depleted in DM?
and may also directly inhibit
myoinositol uptake
Will total T4, free T4, and T3
All increased - Increased total
uptake be increased or
T4 - Increased free T4 -
decreased (respectively) in
Increased T3 uptake
primary hyperthyroidism?

Will total T4, free T4, and T3 All decreased (remember: TSH
uptake be increased or is increased) - Decreased
decreased (respectively) in total T4 - Decreased free T4
primary hypothyroidism? - Decreased T3 uptake

Will TSH be increased or


decreased in primary Increased
hyperthyroidism?

Will TSH be increased or


decreased in primary Increased
hypothyroidism?
Are most pericardial effusions
serous
serous or hemorrhagic?

Bacterial endocarditis of
which valve is associated with Tricuspid
IV drug abuse?

ST elevation (transmural
Characterize EKG changes in
ischemia) and Q waves
an MI
(transmural infarct)

elevated 1-3 days post MI.


characterize the AST levels in Nonspecific enzyme found in
an MI heart, liver, and skeletal
muscle
Elevated between 4 hrs. and
Characterize the cardiac
7-10 days post MI most
troponin I levels in an MI.
specific protein marker for MI

Characterize the CK-MB elevated in the first 24 hrs.


levels in an MI post MI

Characterize the LDH1 levels elevated from 2 to 7 days


in an MI post MI

Most frequent valvular lesion,


Describe a mitral prolapse esp. in young women. Late
murmur? systolic murmur ending with
2nd heart sound
High pitched holosystolic
Describe a mitral
(continuous sound
regurgitation murmur?
throughout systole)

Rumbling late diastolic


Describe a mitral stenosis murmur when LA&gt;&gt;LV
murmur during diastole. Begins in late
diastole

holosystolic murmur
Describe a vent. Septal defect
(continuous throughout
(VSD) murmur.
systole)

high-pitched 'blowing'
Describe an aortic murmur, beginning
regurgitation murmur immediately in diastole. Wide
pulse pressure
Crescendo-decrescendo
systolic murmur, with
Describe an aortic stenosis LV&gt;&gt;aortic pressure
murmur during systole. Follows an
'ejection click,' and ends
before 2nd heart sound

Continuous machine-like
Describe an patent ductus
murmur. Loudest at the time
arteriosus (PDA) murmur.
of 2nd heart sound

Describe the bacterial


Small vegetations on
growths in subacute bact.
congentially abnormal valves
Endocarditis.

Describe the bacterial


Large vegetations on
growths of acute bact.
previously normal valves
endocarditis?
Describe the onset of Staph.
rapid, acute onset
Aureus endocarditis.

Describe the onset of


Streptoccus viridans Insidious, subacute onset
endocarditis.

During what weeks of


20 weeks gestation to 6
pregnancy does preeclampsia
weeks postpartum
present?

It can resolve without scarring


How can pericarditis or it can lead to chronic
progress? adhesive or chronic
constrictive pericarditis
1. Fatty streaks in arteries 2.
How does atherosclerosis
Proliferative plaques 3.
progress?
Complex atheromas

How does Prinzmental's


chest pain at rest
variant angina present?

How does stable angina


chest pain with exertion
present?

Causes dilation of the aorta


How does syphilis change the and valve ring. Can result in
aorta? aortic aneurysm or aortic
valve incompetence
How does unstable/crescendo
Worsening chest pain
angina present?

Coronary heart dz, CVA, CHF,


To what does HTN predispose
renal dz, and aortic
one?(5)
dissection

1. Hemolysis 2. Elevated LFT


What are associations of
(liver fxn test) 3. Low
preeclampsia?(3)
platelets

1. Headache 2. Blurred vision


3. Abdominal pain 4. Edema
What are clinical features of
preeclampsia?(6) of face and extremities 5.
Altered mentation 6.
Hyperreflexia
1. Card. Arrhythmia(90%) 2.
LV failure and pul. Edema
(60%) 3. Thromboembolism:
What are complications from mural thrombus 4.
an MI?(7) Cardiogenic shock 5. Physical
trauma 6. Fibrinous
pericarditis 7. Dressler's
syndrome

1. Chordae rupture 2.
What are complications of Glomerulonephritis 3.
bacterial endocarditis?(4) Suppurative pericarditis 4.
Emboli

1. Vent wall rupture 2.


Interventricular. Septum
What are examples traumatic rupture 3. Papillary muscle
MI complications?(4) rupture (4-10 days post-MI)
4. Cardiac tamponade (heart
compression)

What are fat emboli Long bone fractures and


associated with?(2) liposuction
Small erythematous lesions
What are Janeway lesions?
on palms or soles

Secondary to metastasis or
What are nonbacterial causes
renal failure (marantic/
of endocarditis?(2)
thrombotic endocarditis)

tender raised lesions on


What are olser nodes?
finger or toe pads

1. Angina(CAD narrowing&gt;
What are possible 75%) 2. Myocardial infarction
manifestations of ischemic 3. Sudden cardiac death 4.
heart disease?(4) Chronic ischemic heart
disease
Increased age, obesity,
What are risk factors for diabetes, smoking, genetics,
hypertension?(6) race
(black&gt;white&gt;asian)

round white spots on retina


What are Roth's spots?
surrounded by hemorrhage

What are some Agina and claudication. Can


atherosclerosis symptoms? be asymptomatic

1. Preexisting HTN 2.
What are some risk factors for
Diabetes 3. Chronic renal dz
preecalmpsia?(4)
4. Autoimmune dz
1.Primary (essential) HTN,
What are the 2 major causes related to ?CO and ?TPR 2.
of HTN? Secondary HTN, usually
related to renal dz

What are the 3 most common


LAD&gt;RCA&gt;circumflex
sites of an MI?

stable angina, prinzmetal's


What are the 3 types of
variant, and unstable/
angina in ischemic heart dz?
crescendo

1. Aortic stenosis 2. Aortic


regurgitation 3. Mitral
What are the 7 types of heart stenosis 4. Mitral
murmurs? regurgitation 5. Mitral
prolapse 6. Vent. Septal
defect 7. Patent ductus
arteriosus
What are the causes/
A large infarct with a high
associations of cardiogenic
incidence of mortality
shock?

aneurisms, ischemia, infarcts,


What are the complications of
peripheral vasc dz thrombus,
atherosclerosis?(6)
and emboli

1. Chronic alcohol abuse 2.


What are the etiologies of Beriberi (wet) 3. Coxacke
dilated cardiomyopthy? (6) virus B postviral myocarditis
4. Cocaine use 5. Chagas dz.
6. Doxirubicin toxicity

1. Unilateral headache 2. Jaw


claudication 3. Impaired
What are the finding in vision 4. Systemic
temporal arteritis? involvement with polymyalgia
rheumatica (in 50% of
patients)
Intermittent claudication,
superficial nodular phlebitis,
What are the findings in cold sensitivity (Raynauld's
Buerger's dz? phenom.), severe pain in
affected part; may lead to
gangrene.

1. Pericardial pain 2. Friction


What are the findings in
rub 3. EKG changes 4. Pulsus
pericarditis?(4)
paradoxicus

1. C-ANCA positive 2. CXR


What are the findings of reveals large nodular lesions
Wegener's granulomatosis?(3) 3. Hematuria and red cell
casts

1. Sarcoidosis 2. Amyloidosis
What are the major causes of
3. Endocardial fibroelastosis
restrictive/obliterative
4. Endomyocardial fibrosis
cardiomyopathy?(4)
(Loffler's)
What are the possible lab thrombocytopenia and
findings in preeclampsia?(2) hyperuricemia

What are the risk factors of smoking, HTN, diabetes


atherosclerosis?(4) mellitus, and hyperlipidemia

1. Fever 2. Erythema
What are the signs and symp marginatum 3. Valvular
of rheumatic fever or damage 4. ESR increase 5.
rheumatic heart dz?(7) Polyarthritis 6. Subcutaneous
nodules 7. Chorea

JR= NO FAME 1. Janeway


What are the signs and symp. lesions 2. Roth's spots 3. Nail
of bacterial endocarditis?(8) bed hemorrhages 4. Osler
nodes 5. Fever 6. Anemia 7.
Murmur 8. Emobli
1. Cotton wool spots 2.
Microaneurysms 3.
What are the signs of Pericarditis 4. Myocarditis 5.
polyarteritis nodosa?(7) Palpable purpura 6. Elevated
ESR 7. P-ANCA positive
serum

What are the symptoms of a Chest pain, tachypnea, and


pulmonary embolus? (3) dyspnea

Severe retrosternal pain, pain


What are the symptoms of an in left arm or jaw, shortness
MI?(5) of breath, fatigue, and
adrenergic symptoms

fever, weight loss, malaise,


What are the symptoms of
abdominal pain, myalgia, and
polyarteritis nodosa?(6)
HTN
What are the symptoms of Fever, arthritis, night sweats,
Takaysu's arteritis?(6) myalgia, and skin nodules

1. Perforation of nasal
septum 2. Chronic sinusitis 3.
What are the symptoms of
Wegeners granulomatosis? (7) Otitis media 4. Mastoiditis 5.
Cough 6. Dyspnea 7.
Hemoptysis

1. Fat 2. Air 3. Thrombus 4.


What are the types of emboli?
Bacteria 5. Amniotic fluid 6.
(6)
Tumor

1. Infection 2. Ischemic heart


What can cause pericarditis? dz 3. Chronic renal failure
(4) leading to uremia 4.
Connective tissue dz
What causes acute bact.
Staphyloccus aureus
Endocarditis?

What causes cardiac dilation greater ventricle end-


in CHF? diastolic volume

What causes dyspnea on failure of LV output to


exertion in CHF? increase during exercise

increased central venous


What causes hepatomegaly in press.?increased resistance
CHF? to portal flow. Rarely leads to
'cardiac cirrhosis.'
Pooling of blood in lungs
when supine adds volume to
What causes othopenea
congested pul. Vasculature
(dyspnea when supine) in
system; increased venous
CHF?
return not put out by left
ventricle.

Failure of left heart to keep


What causes Paroxysmal up with rt. Heart output ?
nocturnal dyspnea and acute rise pul. Venous and
pulmonary edema in CHF? capillary press. ? transudation
of fluid

What causes prinzmental's


coronary artery spasm
variant angina?

LV failure?increased pul.
Venous press.? pul. Venous
What causes pulmonary distention and transudation
congestion in CHF? of fluid. Presence of
hemosiderin-laden
macrophages (heart failure
cells).
What causes pulmonary 95% of pulmonary emboli
emboli? arise from deep leg veins

Pharyngeal infection with


group A, ? hemolytic
What causes rheumatic fever? streptococci leads to cross
reactivity with self (not due to
direct effects of bacteria)

What causes stable angina? atherosclerosis

What causes sudden cardiac Most commonly from lethal


death? arrhythmia
What causes the edema seen RV failure?increased venous
in CHF? press.? fluid transudation

What causes unstable/ thrombosis in a branch of the


crescendo angina? coronary artery

1. Infarct appears pale 2.


Tissue surrounding infarct
What happens 2-4 days after shows acute inflammation 3.
an MI?(5) Dilated vessels (hyperemia) in
infarct 4. Neutrophil
emigration 5. Extensive
coagulative necrosis

1. A hyperemic boarder forms


around the infarct 2. The infarct
shows central softening with
What happens 5-10 days brown/yellow color 3. An outer
after an MI?(4) zone (ingrowth of granulation
tissue) forms around infarct 4.
Neutrophils and macrophages
infiltrate infarcted tissue
1. The Occluded artery
causing the MI is recanalized
What happens by 7 weeks 2. The infarct area is gray/
post-MI?(3) white 3. The infarcted tissue
shows contracted, complete
scarring

What happens to contractility,


cardiac output, and effective all decrease
atrial blood volume in CHF?

What happens to renal blood


decreases
flow in CHF?

What happens to renin,


angiotensin II, and all increase
aldosterone in CHF
What happens to sympathetic
increases
nervous activity in CHF?

1. Appearance of a pale
infarcted area 2. Coagulative
necrosis in the infarct 3.
What happens to the heart 1 Release of necrotic cells in
day after an MI?(5) the blood 4. Beginning of
neutrophil emigration 5.
Artery supplying infarcted
tissue is occluded

What happens to urinary


excretion of water and Na in decrease
CHF?

What happens to venous


increases
pressure in CHF?
What histologic part of the
vasa vasorum
aorta is affected by syphilis?

What is 'pulseless disease'? Takayasu's arteritis

A hemorrhagic infarct
What is a red infarct? associated with reperfusion
of infarcted tissue

What is an association of Hepatitis B infection (30% of


polyarteritis nodosa? patients)
Known as smoker's disease
and thromboangitis
obliterans; idiopathic,
What is Buerger's disease? segmental, thrombosing
vasculitis of intermediate and
small peripheral arteries and
veins.

Progressive onset of CHF over


What is chronic ischemic
several years due to chronic
heart dz?
ischemic myocardial damage

an autoimmune phenomenon
resulting in fibrinous
What is Dressler's syndrome?
pericarditis several weeks
post-MI

The addition of seizures to


What is eclampsia?
the preeclampsia triad
A friction rub of the
What is fibrinous pericarditis? pericardium usually 3-5 days
post-MI

What is hypertrophic
Familial hypertrophy, usually
cardiomyopathy (formerly
asymmetric, involving the
IHSS: idiopathic hypertrophic
interventricular septum
subaortic stenosis)?

Necrotizing immune complex


inflammation of small or
What is polyarteritis nodosa? medium-sized muscular
arteries, typically involving
renal or visceral vessels.

A triad of HTN, protenuria,


What is preeclampsia? and edema that occurs in
pregnancy
death from cardiac causes
What is sudden cardiac death? within 1 hr. of onset of
symptoms

Thickening of aortic arch and


proximal great vessels
What is Takayasu's arteritis? causing weak pulses in
extremities and ocular
disturbances.

Vasculitis that affects medium


and small arteries, usually
What is temporal arteritis?
branches of the carotid
artery.

What is the appearance of a Walls of LV are thickened,


heart with hypertrophic chamber becomes banana
cardiomyopathy? shaped on echocardiogram
What is the appearance of an
Tree bark appearance
aorta affected by syphilis?

What is the incidence of


7% of pregnant women
preeclampsia?

What is the inheritance


pattern of hypertrophic AD
cardiomyopathy?

What is the most common Dilated (congested)


cardiomyopathy? cardiomyopathy (90%)
What is the most common
metastases
heart tumor

What is the most common


primary cardiac tumor in Myxoma
adults?

What is the most common


primary cardiac tumor in rhabdomyoma
children

What is the most common


temporal arteritis
vasculitis?
What is the treatment for
stop smoking
Buerger's dz?

What is the treatment for


Responds well to steroids
temporal arteritis?

IV Magnesium sulfate and


What is the treatment of
diazepam This is a medical
eclampsia?
emergency

What is the treatment of Corticosteroids, azathioprine,


polyarteritis nodosa? and/or cyclophosphamide
Deliver the fetus ASAP.
What is the treatment of
Otherwise rest, salt
preeclampsia?
restriction, treatment of HTN

cyclophosphamide,
What is the treatment of
corticosteroids, and/or
wegener's granulomatosis?
methotrexate

What is the x-ray appearance


of a heart with dilated Dilated: looks like a balloon
myopathy?

Focal necrotizing vasculitis


and granulomas in the lung
What is Wegner's
granulomatosis? and upper airway with
necrotizing
glomerulonephritis
What kind of effusions are
found in pericarditis
hemorrhagic
associated with TB or
malignancy?

What kind of effusions are


found in pericarditis Serous of fibrinous
associated with renal failure?

What lab finding is seen in


Takayasu's arteritis or elevated ESR
temporal arteritis?

What part of the heart/


Aortic root and ascending
vasculature can be damaged
aorta
by syphilis?
What percentage of HTN is
90%
primary?

What percentage of HTN is


10%
secondary?

What population is associated


with death from hypertrophic young athletes
cardiomyopathy?

Virchow's triad: 1. Stasis 2.


What predisposes one to deep
Hypercoagulability 3.
vein thromosis?(3)
Endothelial damage
What stage of syphilis can
tertiary
affect the heart?

1. Ankle and sacral edema 2.


Hepatomegaly (nutmeg liver)
3. Pulmonary congestion 4.
What symptoms are Dyspnea on exertion 5.
associated with CHF?(8) Paroxysmal nocturnal
dyspnea 6. Pulmonary edema
7. Orthopnea (dyspnea when
supine) 8. Cardiac dilation

1. EKG (the gold standard) 2.


What tests are used to
Cardiac troponin I 3. CK-MB
diagnose an MI?(5)
4. LDH1 5. AST

What type of bacterial


Subacute endocarditis from
endocarditis is associated
Strep. Viridans infection
with dental procedures?
What type of embolus is amniotic fluid, especially
associated with DIC? postpartum

Viruses, TB, pyogenic


What types of infections bacteria; often by direct
cause pericarditis? spread from lung or
mediastinal lymph node

What visual complication can occlusion of ophthalmic


temporal arteritis cause? artery leading to blindness

90% occur in the atria, mostly


Where do myxomas occur? LA. Myxomas are described
as a 'ball valve' obstruction.
Solid tissues with single
Where do pale infarcts occur? blood supply: brain, heart,
kidneys, and spleen

Loose tissue with collaterals:


Where do red infarcts occur?
lungs or intestine

Where does a MI usually In the left anterior


occur? descending coronary artery

The elastic arteries and


Where histologically does
medium to large muscular
atherosclerosis occur?
arteries
Where, anatomically, does abdominal aorta&gt;coronary
atherosclerosis most occur? art&gt;popliteal art&gt;carotid
(4) art

Which valve is most


frequently involved in Mitral
bacterial endocarditis?

Which valves are most mitral&gt;aortic&gt;&gt;tricus


affected by rheumatic heart pid (high pressure valves
dz? most affected)

Primarily affects young Asian


Who gets Takayasu's arteritis?
females
Who gets temporal arteritis? Affects elderly females

Failure to make urine and


Define renal failure.
excrete nitrogenous wastes

How do you calculate anion Na-(Cl + HCO3) = 8-12 mEq/


gap? L

How do you treat minimal


Responds well to steroids
change disease?
How does acute
poststreptococcal Spontaneously
glomerulonephritis resolve?

How does renal cell


Invades the IVC and spreads
carcinoma spread
hematogenously
metastically?

How does transitional cell


Hematuria
carcinoma present?

How does Wilms' tumor


Huge, palpable flank mass
present?
In what epidemiological
group is renal cell carcinoma Men ages 50-70
most common?

T/F: Ammonium magnesium


phosphate kidney stones are TRUE
radiopaque

T/F: Calcium kidney stones


TRUE
are radiopaque.

T/F: Calcium kidney stones


FALSE
do not recur.
T/F: Cystine kidney stones FALSE, cystine stones are
are radiopaque. radiolucent

T/F: Transitional cell False, transitional cell


carcinoma is cured by carcinoma often recurs after
surgical removal. removal

T/F: Uric acid kidney stones FALSE, uric acid stones are
are radiopaque radiolucent

What additional sx are seen in


Peripheral and periorbital
a pt with acute streptococcal
edema
glomerulonephritis?
What age group is
poststreptococcal
Children
glomerulonephritis most
common?

1. Acute lung disease 2.


Chronic lung disease 3.
What are 4 causes of
Opioids, narcotics, sedatives
hypoventilation?
4. Weakening of respiratory
muscles

What are the 2 forms of renal


Acute and chronic
failure?

What are the 2 main


symptoms present in Hemoptysis, hematuria
Goodpasture's syndrome?
1. Calcium 2. Ammonium
What are the 4 major types of
magnesium phosphate 3. Uric
kidney stones?
acid 4. Cystine

Acute poststreptococcal
glomerulonephritis Rapidly
progressive (crescentic)
What are the 5 nephritic glomerulonephritis
syndromes? Goodpasture's syndrome
Membranoproliferative
glomerulonephritis Berger's
disease

1. Membranous
glomerulonephritis 2. Minimal
What are the 5 nephrotic change disease (lipoid
syndromes? nephrosis) 3. Focal segmental
glomerular sclerosis 4.
Diabetic nephropathy 5. SLE

What are the causes and -Kids- rickets -Adults-


signs of calcium ion osteomalacia -Contributes to
deficiency? osteoporosis -Tetany
What are the causes and -Low serum calcium ion -can
signs of phosphate toxicity? cause bone loss -renal stones

What are the causes of Secondary to emesis,


chloride ion deficiency? diuretics, renal disease

-Diabetic ketoacidosis -
What are the causes of Diarrhea -Lactic Acidosis -
metabolic acidosis? Salicylate OD -Acetazolamide
OD

What are the causes of


-COPD -Airway obstruction
respiratory acidosis?
What are the causes of -High altitude -
respiratory alkalosis? Hyperventilation

What are the characteristics


of acute poststreptococcal
Granular pattern
glomerulonephritis seen with
immunofluorescence?

What are the characteristics


of acute poststreptococcal
Subepithelial humps
glomerulonephritis seen with
the electron microscope?

What are the characteristics


Glomeruli enlarged and
of acute poststreptococcal
hypercellular neutrophils
glomerulonephritis seen with
'lumpy-bumpy'
the light microscope?
What are the characteristics
of rapidly progressive
(crescentic) Crescent-moon shape
glomerulonephritis seen on
LM and IF?

-Hematuria -Palpable mass -


What are the clinical features
Secondary polycythemia -
of renal cell carcinoma?
Flank pain -Fever

What are the clinical I' = inflammation; hematuria,


symptoms of a nephritic hypertension, oligouria,
syndrome? azotemia

What are the clinical O = proteinuria


symptoms of nephrotic Hypoalbuminuria Generalized
syndromes? edema Hyperlipidemia
1. Anemia 2. Renal
osteodystrophy 3.
What are the consequences of Hyperkalemia 4. Metabolic
renal failure? acidosis 5. Uremia 6. Sodium
and water excess 7. Chronic
pyelonephritis 8. HTN

What are the factors


-Increased pH -Increased
associated metabolic
PCO2 -Increased HCO3-
alkalosis?

What are the factors


-Decreased pH -Decreased
associated with metabolic
PCO2 -Decreased HCO3-
acidosis?

What are the factors


-Decreased pH -Increased
associated with respiratory
PCO2 -Increased HCO3-
acidosis?
What are the factors
-Increased pH -Decreased
associated with respiratory
PCO2 -Decreased HCO3-
alkalosis?

-Muscle contraction -
What are the functions of
Neurotransmitter release -
calcium ion?
Bones, teeth

-Extracellular fluid -Maintains


What are the functions of
plasma volume -Nerve/
sodium ion?
muscle function

-Fluid/electrolyte balance -
What are the functions of the
Gastric acid -HCO3/Cl shift in
chloride ion?
RBC
What are the functions of the -Bones, teeth -Enzyme
magnesium ion? cofactor

-ATP -nucleic acids -


What are the functions of the
Phosphorylation -Bones,
phosphate ion?
teeth

What are the functions of the -Intracellular fluid -Nerve/


potassium ion? muscle function

What are the signs of


-Diarrhea -Alcoholism
magnesium ion deficiency?
What are the signs of -Decreased reflexes -
magnesium ion toxicity? Decreased respirations

What are the signs of -Kids- rickets -Adults-


phosphate deficiency? osteomalacia

What are the signs of


-EKG changes -Arrhythmia
potassium ion toxicity?

What bugs cause ammonium Urease-positive bugs such as


magnesium phosphate kidney Proteus vulgaris or
stones? Staphylococcus
What calcium molecules form Calcium oxalate or calcium
calcium kidney stones? phosphate or both

What can excess Na and


CHF and pulmonary edema
water cause?

What can the hyperkalemia


associated with renal failure Cardiac arrhythmias
lead to?

1. Vomiting 2. Diuretic use 3.


What causes metabolic
Antacid use 4.
alkalosis?
Hyperaldosteronism
What causes renal Failure of active vitamin D
osteodystrophy? production

What characteristics of
Berger's disease are seen with Mesangial deposits of IgA
IF and EM?

What characteristics of focal


segmental glomerular Segmental sclerosis and
sclerosis are seen with the hyalinosis
LM?

What characteristics of Linear pattern Anti-


Goodpasture's syndrome are glomerular basement
seen with IF? membrane antibodies
What characteristics of
Membranoproliferative subendothelial humps 'tram
glomerulonephritis are seen track'
with the EM?

What characteristics of
membranous
Granular pattern
glomerulonephritis are seen
with IF?

What characteristics of
membranous
Spike and Dome'
glomerulonephritis are seen
with the EM?

What characteristics of
Diffuse capillary and
membranous
basement membrane
glomerulonephritis are seen
thickening
with the LM?
What characteristics of
minimal change disease are Foot process effacement
seen with the EM?

What characteristics of
minimal change disease are Normal glomeruli
seen with the LM?

Wire-loop appearance with


extensive granular
What characteristics of SLE subendothelial basement-
are seen with the LM? membrane deposits in
membranous
glomerulonephritis pattern

What defines metabolic -pH less than 7.4 -PCO2 less


acidosis? than 40 mm Hg
What defines metabolic -pH greater than 7.4 -PCO2
alkalosis with compensation? greater than 40 mm Hg

What defines respiratory -pH less than 7.4 -PCO2


acidosis? greater than 40mm Hg

What defines respiratory -pH greater than 7.4 -PCO2


alkalosis? less than 40 mm Hg

Diseases with increased cell


What diseases often cause proliferation and turnover,
uric acid kidney stones? such as leukemia and
myeloproliferative disorders
What disorders can lead to 1. Cancer 2. Increased PTH 3.
hypercalcemia and thus Increased vitamin D 4. Milk-
kidney stones? alkali syndrome

1. Renal failure 2. Lactic


What disorders cause an
acidosis 3. Ketoacidosis (DM)
increased anion gap?
4. Aspirin ingestion

What disorders cause 1. Diarrhea 2. Glue sniffing 3.


metabolic acidosis and Renal tubular acidosis 4.
normal anion gap? Hyperchloremia

Wilms' tumor Aniridia


What disorders make up the
Genitourinary malformation
WAGR complex?
mental-motor Retardation
What does potassium -Weakness -Paralysis -
deficiency cause? Confusion

What factors are associated Exposure to


with transitional cell cyclophosphamide, smoking,
carcinoma? phenacetin, and aniline dyes

Renal cell carcinoma is


What genetic disorder and
associated with von Hippel-
mutation are associated with
Lindau and gene deletion in
renal cell carcinoma?
chromosome 3

Deletion of tumor
What genetic disorder is
suppression gene WT-1 on
associated with Wilms' tumor?
chromosome 11
What is a common cause of Membranous
adult nephrotic syndrome? glomerulonephritis

What is acute renal failure


Hypoxia
often due to?

IgA nephropathy -Mild


What is Berger's disease?
disease -Often postinfectious

What is chronic failure due to? HTN and diabetes


What is the 2nd most
Ammonium magnesium
common type of kidney
phosphate
stone?

What is the cause of


Secondary to malabsorption
magnesium ion deficiency?

What is the cause of


Vomiting
metabolic alkalosis?

What is the cause of Secondary to injury, illness or


potassium ion deficiency? diuretics
What is the cause of sodium
Secondary to injury or illness
deficiency?

What is the compensatory


mechanism of metabolic Hypoventilation
alkalosis?

What is the compensatory


mechanism of respiratory Renal HCO3- secretion
alkalosis?

What is the compensatory


response to metabolic Hyperventilation
acidosis?
What is the compensatory
response to respiratory Renal HCO3- reabsorption
acidosis?

What is the course of


Slowly progresses to renal
membranoproliferative
failure
glomerulonephritis?

What is the course of rapidly


Rapid course to renal failure
progressive (crescentic)
from one of many causes
glomerulonephritis?

What is the Henderson- pH = pKa + log [(HCO3-)/


Hasselbalch equation? (0.03*PCO2)]
What is the most common
Minimal change disease
cause of childhood nephrotic
(lipoid nephrosis)
syndrome?

What is the most common


renal malignancy of early Wilms' tumor
childhood (ages 2-4)?

What is the most common


Renal cell carcinoma
renal malignancy?

What is the most common


tumor of the urinary tract Transitional cell carcinoma
system?
What is the primary
disturbance in respiratory Increased PCO2
acidosis?

What is the primary


disturbance of metabolic HCO3- decrease
acidosis?

What is the primary


disturbance of metabolic Increased HCO3-
alkalosis?

What is the primary


disturbance of respiratory Decreased PCO2
alkalosis?
What is the sign of calcium
Delirium
ion toxicity?

What is the sign of sodium


Delirium
ion toxicity?

What lesions are seen on the


Kimmelstiel-Wilson lesions
LM in diabetic nephropathy?

MUD PILES 1. Methanol 2.


Uremia (chronic renal failure)
3. Diabetic ketoacidosis 4.
What might an elevated anion Paraldehyde or Phenformin 5.
gap indicate? Iron tablets or INH 6. Lactic
acidosis (CN-, CO, shock) 7.
Ethanol or Ethylene glycol 8.
Salicylates
What paraneoplastic
Ectopic EPO, ACTH, PTHrP,
syndromes are associated
and prolactin
with renal cell carcinoma?

What severe complications Hydronephrosis


may kidney stones lead to? Pyelonephritis

What social factor increases


the incidence of renal cell Smoking
carcinoma?

What type of hypersensitivity


contributes to the
Type II hypersensitivity
pathogenesis of
Goodpasture's syndrome?
Where can transitional cell -Renal calyces -Renal pelvis -
carcinoma occur? Ureters -Bladder

Where does renal cell Renal tubule cells, polygonal


carcinoma originate? clear cells

Which kidney stone is often


Cystine
secondary to cystinuria?

Which kidney stone is


strongly associated with Uric acid kidney stones
gout?
Which of the nephrotic
Focal segmental glomerular
syndromes are worse in HIV
sclerosis
pts?

Which type of kidney stones


constitute the majority of Calcium
kidney stones (80-85%)?

Why are ammonium Ammonium magnesium


magnesium phosphate kidney phosphate stones can form
stones often associated with large struvite calculi that can
UTIs? be a nidus for UTIs

Why does renal failure cause


Failure of EPO production
anemia?
Due to decreased acid
Why does renal failure cause
secretion and decreased
metabolic acidosis?
generation of HCO3-

-Physiologic tolerance and


dependence with syptoms of
withdrawal when intake is
Define/Describe Alcoholism: interrupted. -Continued
drinking despite medical and
social contraindications and
life disruptions.

In alcoholics, what causes


Vitamin B1 (thiamine)
Wernicke-Korsakoff
deficiency
syndrome?

Is Korsakoff's syndrome
NO
reversible?
Wernicke-Korsakoff
syndrome is associated with
periventricular hemorrage/ Mamillary bodies
necrosis in which part of
brain?

-Jaundice -Hypoalbuminemia
-Coagulation factor
What are the accompanying deficiencies -Portal
symptomes of Alcoholic hypertension -Peripheral
cirrhosis? edema and ascites -
Encephalopathy -Neurologic
manifestations (e.g., asterixis,
flapping tremor of the hands)

-Toxicity (especially in the


What are the bodily effects of
brain) -Fatty liver -Increased
ethanol? (3)
NADH/HAD

-Increases lactate/pyrubate -
Inhibits gluconeogenesis -
What are the effects of
Inhibits fatty acid oxidation -
increased NADH/NAD (from
Inhibits glycerophosphate
ethanol use)? (4)
dehydrogenase leading to
elevated glycerophosphate
-Alcoholoic hepatitis and
cirrhosis -Pancreatitis -Dilated
cardiomyopathy -Peripheral
What are the long term neuropathy -Cerebellar
consequences of alcohol use? degeneration -Wernicke-
Korsakoff syndrome -Testicular
atrophy and hypertension -
Mallory-Weiss syndrome

-tremor -tachycardia -
What are the symptoms of
hypertension -malaise -
alcohol withdrawel?
nausea -delerium tremens

Progression of Wernicke's
What is Korsakoff's encephalopathy to memory
syndrome? loss, confabulation, and
confusion

Longitudinal lacerations at
the gastroesophageal
What is Mallory-Weiss junction caused by excessive
syndrome? vomitting with failure of
Lower Esophageal Sphincter
relaxation that could lead to
fatal hematemesis.
What is the treatment for
Wernicke-Korsakoff IV vitamin B1 (thiamine)
syndrome?

What is the triad of symptoms


1. Psychosis 2.
for Wernicke's
Ophthalmoplegia 3. Ataxia
encephalopathy?

What supportive group has


Alcoholics Anonymous
been mose successful in
(sorry… it was in the book :)
sustaining abstinence?

What treatment is used to


condition the patient
Disulfiram
negatively against alcohol
use?
Often precedes squamous cell
Actinic keratosis
carcinoma

Autoimmune (infection is the


Addison's
second most common cause)

Guillain-Barre (increased
Albumino-cytologic
protein in CSF with only
dissociation
modest increase in cell count)

Aneurysm, dissecting HTN


Anti-basement membrane Goodpasture's syndrome

Anti-centromere antibodies Scleroderma (CREST)

Anti-double-stranded-DNA
SLE (type III hypersensitivity)
antibodies (ANA antibodies)

Anti-epithelial cell Pemphigus vulgaris


Anti-gliadin antibodies Celiac disease

Anti-histone Antibodies Drug-induced SLE (cf. SLE)

Anti-IgM antibodies Rheumatoid arthritis

Anti-mitochondrial
Primary biliary cirrhosis
antibodies
Anti-neutrophil antibodies Vasculitis

Idiopathic thrombocytopenic
Antiplatelet antibodies
purpura

Aortic aneurysm, abdominal


Atherosclerosis
&amp; descending aorta

Aortic aneurysm, ascending Tertiary syphilis


Arachnodactyly Marfan's syndrome

Argyll-Robertson pupil Neurosyphilis

Aschoff bodies Rheumatic fever

Atrophy of the mamillary


Wernicke's encephalopathy
bodies
Acute myelogenous leukemia
Auer rods (especially the promyelocytic
type)

Autosplenectomy Sockle cell anemia

Babinski sign Upper motor neuron lesion

Bacteremia/pneumonia (IVDA) Staphylococcus aureus


Bacteria associated with
Helicobacter pylori
cancer

Bacteroides (second most


Bacteria found in GI tract
common is Escherichia coli)

Bacterial meningitis (adults) Neisseria meningitidis

Bacterial meningitis (elderly) Streptococcus pneumoniae


Bacterial meningitis (kids) Haemophilus influenza type B

Bacterial meningitis
Escherichia coli
(newborns)

Baker's cyst in popliteal fossa Rheumatoid arthritis

Bamboo spine' on xray Ankylosing spondylitis


Basophilic stippling of RBC's Lead poisoning

Multiple myeloma (kappa or


lambda Ig light chains in
Bence-Jones proteins
urine) Waldenstrom's
macroglobinemia (IgM)

Bilateral hilar adenopathy,


Sarcoidosis
uveitis

Histiocytosis X (eosinophilic
Birbeck granules on EM
granuloma)
Bloody tap on LP Subarachnoid hemorrhage

Fibrocystic change of the


Blue-domed cysts
breast

Blue bloater' Chronic bronchitis

Blue sclera Osteogenesis imperfecta


Tetralogy of Fallot; RV
Boot-shaped heart on x-ray
hypertrophy

Osteoarthritis (PIP swelling


Bouchard's nodes
secondary to osteophytes)

Boutonniere's deformity Rheumatoid arthritis

Brain tumor - supratentorial


Craniopharyngioma
(kids)
Astrocytoma (including
glioblastoma multiforme) &gt;
Brain tumor (adults)
mets &gt; meningioma &gt;
Schwannoma

Brain tumor (kids) Medullobastoma (cerebellum)

Branching rods in oral


Actinomyces israelii
infection

Infultrating ductal carcinoma


Breast cancer (in the US, one in nine women
will develop breast cancer)
Fibrocystic change (in post-
menopausal women,
Breast mass
carcinoma is the most
common)

Breast tumor (benign) Fibroadenoma

Hemorrhage causes brown


color of osteolytic cysts:
Brown tumor'of bone Hyperparathyroidism; Osteitis
fibrosa cystica (von
Recklinghausen's disease)

Brushfield's spots Down syndrome


Bruton's lines Lead poisoning

Bug in debilitated,
hospitalized pneumonia Klebsiella
patient

C-ANCA Wegerner's granulomatosis

Café au lait spots on skin Neurofibromatosis


Duchenne's muscular
Calf pseudohypertrophy
dystrophy

Granulosa/thecal cell tumor


Call-Exner bodies
of the ovary

Cancer associated with AIDS Kaposi's sarcoma

Cardiac primary tumor Myxoma (4:1 left to right


(adults) atrium; 'ball &amp; valve')
Cardiac primary tumor (kids) Rhabdomyoma

Cardic tumor (adults) Mets

Cardiomegaly with apical


Cagas' disease
atrophy

Cardiomyopathy Dilated cardiomyopathy


Mycosis fungoides (cutaneous
Cerebriform nuclei
T-cell lymphoma)

Chancre Primary syphilis (not painful)

Chancroid Haemophilus ducreyi (painful)

Multiple sclerosis (nystagmus,


intention tremor,scanning
Charcot's triad
speech); Cholangitis
(jaundice, RUQ, fever)
Bronchial asthma (eosinophil
Charcot-Leyden crystals
membranes)

Tay-Sachs, Niemann-Pick
Cherry-red spot on macula disease, central retinal artery
occlusion

Hypocalcemia (facial muscle


Chevostek's sign
spasm upon tapping)

Central apnea in CHF &amp;


Cheyne-Stokes respirations
increased ICP
Endometriosis (frequently
Chocolate cysts'
involve both ovaries)

Down syndrome (associated


with ALL, Alzheimer's
Chromosomal disorder
dementia, &amp; endocardial
cushion defects)

Atrial fibrillation (associated


Chronic arrhythmia
w/ high risk of emboli)

Predisposition to gastric
Chronic atrophic gastritis
carcinoma
Clue cells Gardnerella vaginitis

Codman's triangle on x-ray Osteoasarcoma

Mycoplasma pneumoniae;
Cold agglutinins
Infectious mononeucleosis

Cold intolerance Myxedema


Condyloma lata Secondary syphilis

Congenital adrenal
21-Hydroxylase deficiency
hyperplasia

Congenital cardiac anomaly VSD

Constrictive pericarditis Tuberculosis


Continuous machinery
Patent ductus arteriosus
murmur

Coronary artery involved in


LAD &gt; RCA &gt; LCA
thrombosis

Cotton wool spots Chronic hypertension

Cough, conjunctivitis, coryza


Measles
+ fever
Councilman bodies Toxic or viral hepatitis

Cowdry type A bodies Herpes virus

Crescents in Bowman's Rapidly progressive crescentic


capsule glomerulonephritis

Cretinism Hypothyroidism/iodine deficit


Currant-jelly sputum Klebsiella

Bronchial asthma (whorled


Curschmann's spirals
mucous plugs)

Corticosteroid therapy
(second most common cause
Cushing's syndrome
is excess ACTH secretion by
pituitary)

Tetralogy of Fallot,
Cyanosis (early; less common) transposition of great vessels,
truncus arteriosus
D-dimers DIC

Death in CML Blast crisis

Death in SLE Lupus nephropathy

Alzheimer's (second most


Dementia
common is multi-infarct)
Demyelinating disease Multiple sclerosis

Parkinson's disease (basal


Depigmentation of neurons in
ganglia disorder -- rigidity,
substantia nigra
resting tremor, bradykinesia)

Dermatitis, dementia, Pellagra (Niacin, vitamin B3


diarrhea deficiency)

Diabetes insipidus +
Hand-Schuller-Christian
exopthalmos + lesions of
disease
skull
Gram-negative sepsis,
DIC obstetric complications,
cancer, burns trauma

Dietary deficit Iron

Dog or cat bite Pasteurella multocida

Donovan bodies Granuloma inguinale


Ejection click Aortic/pulmonic stenosis

Elastic skin Ehlers-Danlos syndrome

Epiglottitis Haemophilus influenza type B

Erythema chronicum migrans Lyme disease


Esophageal cancer Squamous cell carcinoma

Fat, female, forty, &amp;


Acute cholecystitis
fertile'

Fatty liver Alcoholism

Ferruginous bodies Asbestosis


Food poisoning Staphylococcus aureus

Ghon complex Secondary TB

Ghon focus Primary TB

IgA nephropathy (Berger's


Glomerularnephritis (adults)
disease)
Duchenne's (use of patient's
Gower's maneuver arms to help legs pick self off
the floor)

Caucasians (fat-soluble
Group affected by cystic
vitamin deficiencies, mucous
fibrosis
plugs/lung infections)

Gynecologic malignancy Endometrial carcinoma

Hair on end' appearance on


Beta-thalassemia
x-ray
Hampton's hump on x-ray Pulmonary embolism

HbS Sickle cell anemia

Choriocarcinoma;
HCG elevated Hyadatidiform mole (occurs
with &amp; without embryo)

Heart murmur Mitral valve prolapse


Heart valve (rheumatic fever) Mitral valve (aotric is second)

Heart valve in bacterial


Mitral
endocarditis

Heart valve in bacterial


Tricuspid
endocarditis in IVDA

Osteoarthritis (DIP swelling


Heberden's nodes
secondary to osteophytes)
Heinz bodies G6PD deficiency

Enterobius vermicularis
Helminth infection (US) (Ascaris lumbricoides is
second most common)

Rupture of middle meningeal


Hematoma - epidural artery (arterial bleeding is
fast)

Rupture of bridging veins


Hematoma - subdural (trauma; venous bleeding is
slow)
Multiple blood transfusions
(can result in CHF, and
Hemochromocytosis
increases risk of
hepatocellular carcinoma)

Hepatic cirrhosis EtOH

Cirrhotic liver (often


Hepatocellular carcinoma associated with hepatitis B
&amp; C)

Hereditary bleeding disorder Von Willebrand's


Infectious mononucleosis
Heterophil antibodies
(EBV)

Hgb F Thalassemia major

High output cardiac failure Wet beriberi (thiamine,


(dilated cardiomyopathy) vitamin B1 deficiency)

Reiter's syndrome, ankylosing


HLA-B27
spondylitis
DM type 1 (caused by
HLA-DR3 or DR4 autoimmune destruction of
beta cells)

Holosystolic murmur VSD, tricuspid regurgitation

Homer-Wright rosettes Neuroblastoma

Honeycomb lung on x-ray Interstitial fibrosis


Splenectomy (or non-
Howell Jolly bodies
functional spleen)

Hyperphagia + hypersexuality Kluver-Bucy syndrome


+ hyperorality + hyperdocile (amygdala)

Primary adrenal insufficiency


Hyperpigmentation of skin
(Addison's disease)

Hypersegmented neutrophils Macrocytic anemia


Cushing &amp; Conn
Hypertension + hypokalemia
syndromes

Hypertension, secondary Renal disease

Hypochromic microcytosis Iron-deficiency anemia

Hypoparathyroidism Thyroidectomy
Hypopituitarism Adenoma

Increase alpha-fetoprotein in
amniotic fluid/maternal Anencephaly; Spina bifida
serum

Gout; Lesch-Nyhan;
Increased uric acid levels Myeloproliferative disorders;
Loop &amp; thiazide diuretics

Infection in blood transfusion Hepatitis C


Infection in burn victims Klebsiella

Adenovirus (cause
Intussesception hyperplasia of Peyer's
patches)

Janeway lesions Endocarditis

Syphilis; over-aggressive
treatment of an symptomatic
Jarisch-Herxheimer reaction
patient that causes symptoms
due to rapid lysis
Homosexual AIDS patients
Kaposi's sarcoma (not associated with IVDA
acquired HIV/AIDS)

Kayser-Fleischer rings Wilson's disease

Keratin pearls Squamous cell carcinoma

Calcium = radiopaque (2nd


most common is ammonium
Kidney stones = radiolucent; formed by
urease positive organisms
like Proteus vulgaris or
Staphylococcus)
Kimmelstiel-Wilson nodules Diabetic nephropathy

Koilocytes HPV

Koplik spots Measles

Kussmaul hyperpnea Diabetic ketoacidosis


Lens dislocation + aortic
Marfan's disease (fibrillin
dissection + joint
deficit)
hyperflexibility

Leukemia (adults) AML

Lewy bodies Parkinson's disease

Lines of Zahn Arterial thrombus


Neurofibromatosis (von
Lisch nodules
Recklinghausen's disease)

Liver disease Alcoholic liver disease

Location of brain tumors


Supratentorial
(adults)

Location of brain tumors


Infratentorial
(kids)
Low serum ceruloplasmin Wilson's disease

Lucid interval Epidural hematoma

Lumpy Bumpy' appearance of


Poststreptococal
glomeruli on
glomerulonephritis
immunoflourescence

Lysosomal storage disease


Gaucher's
disorder
Lytic bone lesions on x-ray Multiple myeloma

Machine-like' murmur PDA

Male cancer Prostatic carcinoma

Malignancy associated with


Hodgkin's
infectious fever
Basal cell carcinoma (rarely
Malignant skin tumor
metastasizes)

Mallory bodies Alcoholic liver disease

McBurney's sign Appendicitis

Down syndrome (Fragile X is


Mental retardation the second most common
cause)
Breast, lung, thyroid, testes,
Mets to bone
prostate

Lung, breast, skin


Mets to brain (melanoma), kidney (renal cell
carcinoma), GI

Colon, gastric, pancreatic,


Mets to liver breast, &amp; lung
carcinomas

MI Atherosclerosis
Mitral valve stenosis Rheumatic heart disease

MLF syndrome (INO) Multiple sclerosis

Multiple myeloma (called the


M protein; usually IgG or IgA);
Monoclonal antibody-spike MGUG; Waldenstrom's (M
Protein = IgM)
macroglobulinemia

Motor neuron disease ALS


Myocarditis Coxsackie B

Myxedema Hypothyroidism

Necrotizing vasculitis (lungs) Wegener's &amp;


&amp; necrotizing Goodpasture's (hemoptysis
glomerulitis &amp; glomerular disease)

Needle-shaped, negatively
Gout
bifringent crystals
Negri bodies Rabies

ALL (2nd most common is


Neoplasm (kids)
cerebellar medulloblastoma)

Nephritis + cataracts +
Alport syndrome
hearing loss

Minimal change disease


(associated with infections/
Nephrotic syndome (kids)
vaccinations; treat with
corticosteroids)
Membranous
Nephrotic syndrome
glomerulonephritis

Membranous
Nephrotic syndrome (adults)
glomerulonephritis

Neurofibrillary tangles Alzheimer disease

No lactation postpartum Sheehan's syndrome


Nutmeg liver Congestive heart failure

Obstruction of male urinary


BPH
tract

Occupational exposure to
Malignant mesothelioma
asbestos

Oncogene involved in cancer p53 Suppressor


Opening snap Mitral stenosis

Opportunistic infection in
PCP
AIDS

Adrenal glands (due to rich


Organ receiving mets
blood supply)

Organ sending mets Lung &gt; breast, stomach


Papillary carcinoma of the
Orphan Annie cells
ovary

Osler's nodes Endocarditis

Osteomyelitis Staphylococcus aureus

Osteomyelitis in patients with


Salmonella
sickle cell disease
Osteomyelitis with IVDA Pseudomonas

Ovarian tumor (benign) Hamartoma

Ovarian tumor (malignant) Serous cystadenoma

Owl's eye CMV


P-ANCA Polyarteritis nodosa

Painless jaundice Pancreatic cancer (head)

Palpable purpura on legs


Henoch-Schonlein purpura
&amp; buttocks

Adenocarcinoma (head of
Pancreatic tumor
pancreas)
Pancreatitus (acute) EtOH and gallstones

EtOH (adults), cystic fibrosis


Pancreatitus (chronic)
(kids)

Pannus Rheumatiod arthritis

ALL - Child / CLL - Adult over


Patient with ALL/CLL/AML/
60 / AML - Adult over 60 /
CML
CML - Adult 35-50
Young male (except nodular
Patient with Hodgkin's
sclerosis type - female)

Patient with minimal change


Young child
disease

Patient with Reiter's Male

Peau d'orange Carcinoma of the breast


Neisseria gonorrhoeae
Pelvic inflammatory disease
(monoarticular arthritis)

Periosteal elevation on x-ray Pyogenic osteomyelitis

Philadelphia chromosome CML (may sometimes be


(bcr;abl) associated with AML)

Pick bodies Pick's disease


Emphysema (centroacinar
(smoking), panacinar
Pink puffer'
(alpha1-antitrypsin
deficiency))

Prolactinoma (2nd -
Pituitary tumor somatotropic 'acidophilic'
adenoma)

Pneumonia, hospital-acquired Klebsiella

Pneumonia, in CF, burn


Pseudomonas aeruginosa
infection
Podagra Gout (MP joint of hallux)

Podocyte fusion Minimal change disease

Polyneuropathy preceded by
Guillian-Barre syndrome
GI or respiratory infection

Polyneuropathy, cardiac Dry beriberi (thiamine,


pathology, &amp; edema vitamine B1 deficiency)
Port-wine stain Hemangioma

Posterior anterior 'drawer Anterior cruciate ligament


sign' injury

Preventable blindness Chlamydia

Preventable cancer Lung cancer


Primary amenorrhea Turner's (XO)

Primary bone tumor (adults) Multiple myeloma

Primary hyperaldosteronism Adenoma of adrenal cortex

Adenomas (followed by:


Primary hyperparathyroidism
hyperplasia, then carcinoma)
Primary hyperparathyroidism Adenoma

Primary liver tumor Hepatoma

Caused by apoptosis of tumor


cells with dystrophic calcification
&amp; found in: Papillary
adenocarcinoma of the thyroid
Psammoma bodies
(most common cancer of the
thyroid); Serous papillary
cystadenocarcinoma of the ovary;
Meningioma, Mesothelioma

Pseudopalisade tumor cell


Glioblastoma multiforme
arrangement
Pseudorosettes Ewing's sarcoma

Horner's syndrome
Ptosis, miosis, anhidrosis
(Pancoast's tumor)

Pulmonary hypertension COPD

Pus, empyema, abscess Staphylococcus aureus


Secondary syphilis; Rocky
Rash on palms &amp; soles
Mountain Spotted Fever

RBC's in urine Bladder carcinoma

RBC casts in urine Acute glomerulonephritis

Recurrent pulmonary
Pseudomonas and
Cystic fibrosis
Staphylococcus aureus
infections
Paroxysmal nocturnal
Red urine in the morning
hemoglobinuria

Reed-Sternberg cells Hodgkin's lymphoma

Reid index (increased) Chronic bronchitis

Reinke crystals Leydig cell tumor


Renal cell carcinoma +
cavernous hemangiomas + Von Hippel - Lindau disease
adenomas

Renal epithelial casts in urine Acute toxic/viral nephrosis

Renal cell carcinoma -


associated with von Hippel-
Lindau &amp; acquired
Renal tumor polycystic kidney disease;
paraneoplastic syndromes
(erythropoietin, renin, PTH,
ACTH)

Rhomboid crystals, positively


Pseudogout
bifringent
Rib notching Coarctation of aorta

Right-sided heart faulure Left-sided heart failure

Right heart failure due to a


Cor pulmonale
pulmonary cause

Roth spots in retina Endocarditis


Rouleaux formation (RBC's) Multiple myeloma

Russell bodies Multiple myeloma

Left to right shunt (VSD, PDA,


S3 ASD); Mitral regurgitation; LV
failure (CHF)

Aortic stenosis, hypertrophic


S4
subaortic stenosis
Schiller-Duval bodies Yolk sac tumor

Schwarzman reaction Neisseria meningitidis

Secondary Hypocalcemia of chronic renal


hyperparathyroidism failure

Senile plaques Alzheimer's disease


Sexually transmitted disease Chlamydia

Postpartum pituitary
Sheehan's syndrome
infarction

Small cell carcinoma of the


SIADH
lung

Signet ring cells Gastric carcinoma


Simian crease Down syndrome

Site of diverticula Sigmoid colon

Site of metastasis Regional lymph nodes

Site of metastasis (2nd most


Liver
commond)
Abdominal aorta &gt;
Sites of atherosclerosis coronary &gt; popliteal &gt;
carotid

Skin cancer Basal cell carcinoma

Skip lesions Crohn's

Erythema infectiosum (fifth


Slapped cheeks
disease)'
Smith antigen SLE

Smudge' cell CLL

Soap bubble on x-ray Giant cell tumor or bone

Membranous
Spike &amp; dome on EM
glomerulonephritis
Splinter hemorrhages in
Endocarditis
fingernails

Starry-sky pattern Burkitt's lymphoma

Stomach cancer Adenocarcinoma

Strawberry tongue' Scarlet fever


Streaky ovaries Turner's syndrome

String sign on x-ray Crohn's disease

Poststreptococal
Subepithelial humps on EM
glomerulonephritis

Suboccipital
Rubella
lymphadenopathy
Sulfur granules Actinomyces israelii

Surgical wound Staphylococcus aureus

Scurvy (ascorbic acid, vitamin


Swollen gums, bruising, poor C deficiency) - vitamin C is
wound healing, anemia necessary for hydroxylation
of proline &amp; lysine in
collagen synthesis

Systolic ejection murmur


Aortic valve stenosis
(crescendo-decrescendo)
Follicular lymphomas (bcl-2
t(14; 18)
activation)

Burkitt's lymphoma (c-myc


t(8;14)
activation)

Philadelphia chromosome,
t(9;22)
CML (bcr-abl hybrid)

Tabes dorsalis Tertiary syphilis


Target cells Thalassemia

Tendon xanthomas
Familial hypercholesterolemia
(classically Achilles)

Testicular tumor Seminoma

Thumb sign on lateral x-ray Epiglottitis


Myasthenia gravis (present in
Thymoma
20% of those with MG)

Thyroid cancer Papillary carcinoma

Chronic bacterial
Thyroidization of kidney
pyelonephritis

Tophi Gout
Lower esophagus joins
Tracheoesophageal fistula trachea/upper esophagus -
blind pouch

Tram-track' appearance on Membranoproliferative


LM glomerulonephritis

Traumatic open wound Clostridium perfringens

Visceral cancer; pancreatic


adeneocarcinoma (migratory
Trousseau's sign
thrombophlebitis);
Hypocalcemia (carpal spasm)
Tumor in men Prostate carcinoma

Leiomyoma (estrogen
Tumor in women
dependent)

Tumor of infancy Hemangioma

Tumor of the adrenal medulla


Pheochromocytoma (benign)
(adults)
Tumor of the adrenal medulla
Neuroblastoma (malignant)
(kids)

Mixed cellularity (versus:


lymphocytic predominance,
Type of Hodgkin's
lymphocytic depletion,
nodular sclerosis)

Type of non-Hodgkin's Follicular, small cleaved

Type of pituitary adenoma Prolactinoma


UTI Escherichia coli

UTI (young women) Staphylococcus saprophyticus

Temporal arteritis (risk of


ipsilateral blindness due to
Vasculitis
thrombosis of ophthalmic
artery)

Viral encephalitis HSV


Left supraclavicular node
Virchow's node enlargement from metastatic
carcinoma of the stomach

Pulmonary embolism (triad =


Virchow's triad blood stasis, endothelial
damage, hypercoag.)

Folic acid (pregnant women


Vitamin deficiency (US) are at high risk; body stores
only 3-4 month supply)

Chronic end-stage renal


Waxy casts
disease
WBC's in urine Acute cystitis

WBC casts in urine Acute pyelonephritis

Wire loop' appearance on LM Lupus nephropathy

Berry aneurysm - associated


Worst headache of my life' with adult polycystic kidney
disease
Xanthochromia (CSF) Subacrachnoid hemorrhage

Xerostomia + arthritis +
Sjogren's syndrome
keratoconjunctivitis sicca
A common side effects of
Neutropenia
INF treatment is?

Antimicrobial prophylaxis
for a history of recurrent TMP-SMZ
UTIs

Antimicrobial prophylaxis
Ceftriaxone
for Gonorrhea

Antimicrobial prophylaxis
Rifampin (DOC),
for Meningococcal
minocycline
infection
Antimicrobial prophylaxis TMP-SMZ (DOC),
for PCP aerosolized pentamidine

Antimicrobial prophylaxis
Benzathine penicillin G
for Syphilis

Are Aminoglycosides
Yes
Teratogenic?

Are Ampicillin and


Amoxicillin penicillinase No
resistant?
Are Carbenicillin,
Piperacillin, and Ticarcillin No
penicillinase resistant?

No, but they are less


Are Cephalosporins
susceptible than the other
resistant to penicillinase?
Beta lactams

Are Methicillin, Nafcillin,


and Dicloxacillin Yes
penicillinase resistant?

Mycobacterium
Clinical use of Isoniazid tuberculosis, the only
(INH)? agent used as solo
prophylaxis against TB
Common side effects Pseudomembranous colitis
associated with (C. difficile), fever,
Clindamycin include? diarrhea

Common toxicities GI upset, Superinfections,


associated with Skin rashes, Headache,
Fluoroquinolones? Dizziness

Common toxicities
Teratogenic, Carcinogenic,
associated with
Confusion, Headaches
Griseofulvin are…...?

Glycoproteins from
Describe the MOA of leukocytes that block
Interferons (INF) various stages of viral RNA
and DNA synthesis
Do Tetracyclines penetrate
Only in limited amounts
the CNS?

Does Ampicillin or
AmOxicillin has greater
Amoxicillin have a greater
Oral bioavailability
oral bioavailability?

Does Amprotericin B cross


No
the BBB?

Does Foscarnet require


activation by a viral No
kinase?
Foscarnet toxicity? Nephrotoxicity

Leukopenia, Neutropenia,
Ganciclovir associated
Thrombocytopenia, Renal
toxicities?
toxicity

How are INFs used Chronic Hepatitis A and B,


clinically? Kaposi's Sarcoma

How are Sulfonamides Gram +, Gram -,


employed clinically? Norcardia, Chlamydia
Triple Therapy' 2
How are the HIV drugs
Nucleoside RT Inhibitors
used clinically?
with a Protease Inhibitor

How are the Latent


Hypnozoite (Liver) forms Primaquine
of Malaria (P. vivax,
P.ovale) treated?

How can Isoniazid (INH)-


Pyridoxine (B6)
induced neurotoxicity be
administration
prevented?

How can the t1/2 of INH


Fast vs. Slow Acetylators
be altered?
How can the toxic effects With supplemental Folic
fo TMP be ameliorated? Acid

How can Vancomycin- Pretreat with


induced 'Red Man antihistamines and a slow
Syndrome' be prevented? infusion rate

As PABA antimetabolites
How do Sulfonamides act that inhibit
on bacteria? Dihydropteroate Synthase,
Bacteriostatic

Inhibt Assembly of new


How do the Protease
virus by Blocking Protease
Inhibitors work?
Enzyme
How does Ganciclovir's
Ganciclovir is more toxic
toxicity relate to that of
to host enzymes
Acyclovir?

With an amino acid change


How does resistance to
of D-ala D-ala to D-ala D-
Vancomycin occur?
lac

HSV, VZV, EBV,


Mucocutaneous and
How is Acyclovir used
Genital Herpes Lesions,
clinically?
Prophylaxis in
Immunocompromised pts

Prophylaxis for Influenza


How is Amantadine used
A, Rubella ; Parkinson's
clinically?
disease
How is Amphotericin B
administered for fungal Intrathecally
meningitis?

Wide spectrum of systemic


mycoses: Cryptococcus,
How is Amphotericin B
Blastomyces, Coccidioides,
used clinically?
Aspergillus, Histoplasma,
Candida, Mucor

Meningitis (H. influenza,


How is Chloramphenical N. meningitidis, S.
used clinically? pneumoniae), Conserative
treatment due to toxicities

How is Foscarnet used CMV Retinitis in IC pts


clinically? when Ganciclovir fails
How is Ganciclovir Phosphorylation by a Viral
activated? Kinase

CMV, esp in
How is Ganciclovir used
Immunocompromised
clinically?
patients

How is Griseofulvin used Oral treatment of


clinically? superficial infections

How is Leishmaniasis
Pentavalent Antimony
treated?
How is Ribavirin used
for RSV
clinically?

1. Mycobacterium
tuberculosis 2. Delays
How is Rifampin used resistance to Dapsone
clinically? when used of Leprosy 3.
Used in combination with
other drugs

Used in combination
How is Trimethoprim used therapy with SMZ to
clinically? sequentially block folate
synthesis

For serious, Gram +


How is Vancomycin used
multidrug-resistant
clinically?
organisms
How would you treat
African Trypanosomiasis Suramin
(sleeping sickness)?

In what population does Premature infants, because


Gray Baby Syndrome they lack UDP-glucuronyl
occur? Why? transferase

Is Aztreonam cross-
No
allergenic with penicillins?

Is Aztreonam resistant to
Yes
penicillinase?
Is Aztreonam usually
No
toxic?

Is Imipenem resistant to
Yes
penicillinase?

Is Penicillin penicillinase
No - duh
resistant?

IV Penicillin G
Foscarnet = pyroFosphate
Mnemonic for Foscarnet?
analog

1)Binds penicillin-binding
proteins 2) Blocks
MOA for Penicillin (3 transpeptidase cross-
answers)? linking of cell wall 3)
Activates autolytic
enzymes

Penicillin, Cephalosporins,
Vancomycin,
MOA: Bactericidal
Aminoglycosides,
antibiotics
Fluoroquinolones,
Metronidazole

MOA: Block cell wall Penicillin, Ampicillin,


synthesis by inhib. Ticarcillin, Pipercillin,
Peptidoglycan cross- Imipenem, Aztreonam,
linking (7) Cephalosporins
MOA: Block DNA
Quinolones
topoisomerases

MOA: Block mRNA


Rifampin
synthesis

MOA: Block nucleotide Sulfonamides,


synthesis Trimethoprim

MOA: Block peptidoglycan


Bacitracin, Vancomycin
synthesis
MOA: Block protein Aminoglycosides,
synthesis at 30s subunit Tetracyclines

Chloramphenicol,
Erythromycin/macrolides,
MOA: Block protein
Lincomycin, Clindamycin,
synthesis at 50s subunit
Streptogramins
(quinupristin, dalfopristin)

MOA: Disrupt bacterial/


Polymyxins
fungal cell membranes

MOA: Unkown Pentamidine


MOA:Disrupt fungal cell Amphotericin B, Nystatin,
membranes Fluconazole/azoles

Name common Polymyxins Polymyxin B, Polymyxin E

Erythromycin,
Name several common
Azithromycin,
Macrolides (3)
Clarithromycin

Sulfamethoxazole (SMZ),
Name some common
Sulfisoxazole, Triple
Sulfonamides (4)
sulfas, Sulfadiazine
Tetracycline, Doxycycline,
Name some common
Demeclocycline,
Tetracyclines (4)
Minocycline

Gentamicin, Neomycin,
Name the common
Amikacin, Tobramycin,
Aminoglycosides (5)
Streptomycin

Fluconazole,
Ketoconazole,
Name the common Azoles
Clotrimazole, Miconazole,
Itraconazole

Ciprofloxacin, Norfloxacin,
Name the common
Ofloxacin, Grepafloxacin,
Fluoroquinolones (6)
Enoxacin, Nalidixic acid
Name the common Non-
Nucleoside Reverse Nevirapine, Delavirdine
Transcriptase Inhibitors

Zidovudine (AZT),
Name the common Didanosine (ddI),
Nucleoside Reverse Zalcitabine (ddC),
Transcriptase Inhibitors Stavudine (d4T),
Lamivudine (3TC)

Name the Protease Saquinavir, Ritonavir,


Inhibitors (4) Indinavir, Nelfinavir

Protease Inhibitors and


Name two classes of drugs
Reverse Transcriptase
for HIV therapy
Inhibitors
Staphlococcus aureus and
Name two organisms
Clostridium difficile
Vancomycin is commonly
(pseudomembranous
used for?
colitis)

Oral Penicillin V

Modification via
Resistance mechanisms for
Acetylation, Adenylation,
Aminoglycosides
or Phosphorylation

Resistance mechanisms for Beta-lactamase cleavage


Cephalosporins/Penicillins of Beta-lactam ring
Resistance mechanisms for Modification via
Chloramphenicol Acetylation

Methylation of rRNA near


Resistance mechanisms for
Erythromycin's ribosome
Macrolides
binding site

Altered bacterial
Dihydropteroate
Resistance mechanisms for
Synthetase, Decreased
Sulfonamides
uptake, or Increased PABA
synthesis

Decreased uptake or
Resistance mechanisms for
Increased transport out of
Tetracycline
cell
Terminal D-ala of cell wall
Resistance mechanisms for
replaced with D-lac;
Vancomycin
Decreased affinity

Hemolysis (if G6PD


Side effects of Isoniazid deficient), Neurotoxicity,
(INH)? Hepatotoxicity, SLE-like
syndrome

Specifically, how does Binds to the


Foscarnet inhibit viral DNA Pyrophosphate Binding
pol? Site of the enzyme

The MOA for


Inhibition of 50S peptidyl
Chloramphenicol is …â
transferase, Bacteriostatic
€¦â€¦â€¦â€¦..?
Megaloblastic anemia,
Toxic effects of TMP
Leukopenia,
include………?
Granulocytopenia

Hormone synthesis
Toxic side effects of the inhibition (Gynecomastia),
Azoles? Liver dysfunction (Inhibits
CYP450), Fever, Chills

Toxicities associated with Delirium, Tremor,


Acyclovir? Nephrotoxicity

What additional side Rash, Pseudomembranous


effects exist for Ampicillin? colitis
What antimicrobial class is
Aztreonam syngergestic Aminoglycosides
with?

What are Amantadine- Ataxia, Dizziness, Slurred


associated side effects? speech

What are Aminoglycosides


Beta-lactam antibiotics
synergistic with?

What are Aminoglycosides Severe Gram - rod


used for clinically? infections.
What are common serious Nephrotoxicity (esp. with
side effects of Cephalosporins),
Aminoglycosides and what Ototoxicity (esp. with Loop
are these associated with? Diuretics)

Fever/Chills, Hypotension,
What are common side
Nephrotoxicity,
effects of Amphotericin B?
Arrhythmias

GI intolerance (nausea,
What are common side diarrhea), Hyperglycemia,
effects of Protease Lipid abnormalities,
Inhibitors? Thrombocytopenia
(Indinavir)

BM suppression
What are common side
(neutropenia, anemia),
effects of RT Inhibitors?
Peripheral neuropathy
-Hypersensitivity reactions
-Hemolysis -
What are common toxic Nephrotoxicity
side effects of (tubulointerstitial nephritis)
Sulfonamides? (5) -Kernicterus in infants
Displace other drugs from
albumin (e.g., warfarin)

What are common GI discomfort, Acute


toxicities associated with cholestatic hepatitis,
Macrolides? (4) Eosinophilia, Skin rashes

GI distress, Tooth
What are common discoloration and
toxicities associated with Inhibition of bone growth
Tetracyclines? in children, Fanconi's
syndrome, Photosensitivity

Well tolerated in general


but occasionally,
What are common Nephrotoxicity,
toxicities related to Ototoxicity,
Vancomycin therapy? Thrombophlebitis, diffuse
flushing='Red Man
Syndrome'
1.Gram - rods of the
Urinary and GI tracts
What are Fluoroquinolones
(including Pseudomonas)
indicated for? (3)
2.Neisseria 3. Some Gram
+ organisms

What are major side


effects of Methicillin, Hypersensitivity reactions
Nafcillin, and Dicloxacillin?

What are Methicillin,


Nafcillin, and Dicloxacillin Staphlococcus aureus
used for clinically?

What are Polymyxins used Resistant Gram -


for? infections
Rifampin, Ethambutol,
What are the Anti-TB Streptomycin,
drugs? Pyrazinamide, Isoniazid
(INH)

What are the clinical


indications for Azole Systemic mycoses
therapy?

What are the clinical uses Gram + cocci, Proteus


for 1st Generation mirabilis, E. coli, Klebsiella
Cephalosporins? pneumoniae (PEcK)

Gram + cocci,
Haemophilus influenza,
What are the clinical uses Enterobacter aerogenes,
for 2nd Generation Neisseria species, P.
Cephalosporins? mirabilis, E. coli, K.
pneumoniae, Serratia
marcescens ( HEN PEcKS )
1) Serious Gram -
What are the clinical uses infections resistant to
for 3rd Generation other Beta lactams 2)
Cephalosporins? Meningitis (most penetrate
the BBB)

Gram - rods: Klebsiella


What are the clinical uses
species, Pseudomonas
for Aztreonam?
species, Serratia species

What are the clinical uses Gram + cocci, Gram -


for Imipenem/cilastatin? rods, and Anerobes

-Upper respiratory tract


infections -pneumonias -
STDs: Gram+ cocci
What are the Macrolides (streptococcal infect in pts
used for clinically? allergic to penicillin) -
Mycoplasma,
Legionella,Chlamydia,
Neisseria
Cephalosporin: 1) has a 6
member ring attached to
What are the major
the Beta lactam instead of
structural differences
a 5 member ring 2)has an
between Penicillin and
extra functional group
Cephalosporin?
( attached to the 6
member ring)

What are the major toxic GI distress, Skin rash, and


side effects of Imipenem/ Seizures at high plasma
cilastatin? levels

1) Hypersensitivity reactions
2) Increased nephrotoxicity
What are the major toxic of Aminoglycosides 3)
side effects of the Disulfiram-like reaction
Cephalosporins? with ethanol (those with a
methylthiotetrazole group,
e.g., cefamandole)

What are the side effects Neurotoxicity, Acute renal


of Polymyxins? tubular necrosis
Minor hepatotoxicity, Drug
What are the side effects
interactions (activates
of Rifampin?
P450)

What are toxic side effects Disulfiram-like reaction


for Metronidazole? with EtOH, Headache

What are toxicities Aplastic anemia (dose


associated with independent), Gray Baby
Chloramphenicol? Syndrome

Giardiasis, Amoebic
What conditions are dysentery (E. histolytica),
treated with Bacterial vaginitis
Metronidazole? (Gardnerella vaginalis),
Trichomonas
What do Aminoglycosides
Oxygen
require for uptake?

What do you treat


Mebendazole/
Nematode/roundworm
Thiabendazole, Pyrantel
(pinworm, whipworm)
Pamoate
infections with?

What drug is given for


Pneumocystis carinii Pentamidine
prophylaxis?

What drug is used during


AZT, to reduce risk of Fetal
the pregnancy of an HIV +
Transmission
mother?, Why?
What drug is used to treat
Trematode/fluke (e.g.,
Schistosomes, Praziquantel
Paragonimus, Clonorchis)
or Cysticercosis

What is a common drug


Increases coumadin
interaction associated with
metabolism
Griseofulvin?

What is a mnemonic to Blocks Influenza A and


remember Amantadine's RubellA; causes problems
function? with the cerebellA

What is a prerequisite for It must be Phosphorylated


Acyclovir activation? by Viral Thymidine Kinase
What is a Ribavirin
Hemolytic anemia
toxicity?

What is an acronym to
RESPIre
remember Anti-TB drugs?

What is an additional side


Interstitial nephritis
effect of Methicillin?

What is an occasional side


GI upset
effect of Aztreonam?
What is Clindamycin used Anaerobic infections (e.g.,
for clinically? B. fragilis, C. perfringens)

What is clinical use for


Pseudomonas species and
Carbenicillin, Piperacillin,
Gram - rods
and Ticarcillin?

Recurrent UTIs, Shigella,


What is combination TMP-
Salmonella, Pneumocystis
SMZ used to treat?
carinii pneumonia

What is combined with


Ampicillin, Amoxicillin,
Carbenicillin, Piperacillin, Clavulanic acid
and Ticarcillin to enhance
their spectrum?
Cryptococcal meningitis in
What is Fluconazole
AIDS patients and Candidal
specifically used for?
infections of all types

What is Imipenem always


Cilastatin
administered with?

Blastomyces, Coccidioides,
What is Ketoconazole
Histoplasma, C. albicans;
specifically used for?
Hypercortisolism

What is Metronidazole
Bismuth and Amoxicillin or
combined with for 'triple
Tetracycline; against
therapy'? Against what
Helobacter pylori
organism?
Antiprotozoal: Giardia,
Entamoeba, Trichomonas,
What is Metronidazole
Gardnerella vaginalis
used for clinically?
Anaerobes: Bacteroides,
Clostridium

Cestode/tapeworm (e.g.,
What is Niclosamide used
D. latum, Taenia species
for?
Except Cysticercosis

Chagas' disease, American


What is Nifurtimox
Trypanosomiasis
administered for?
(Trypanosoma cruzi)

What is the chemical name DHPG (dihydroxy-2-


for Ganciclovir? propoxymethyl guanine)
Extended spectrum
What is the clinical use for
penicillin: certain Gram +
Ampicillin and Amoxicillin?
bacteria and Gram - rods

What is the clinical use for Topical and Oral, for Oral
Nystatin? Candidiasis (Thrush)

Bactericidal for: Gram +


What is the clinical use for
rod and cocci, Gram -
Penicillin?
cocci, and Spirochetes

What is the major side


effect for Ampicillin and Hypersensitivity reactions
Amoxicillin?
What is the major side
effect for Carbenicillin, Hypersensitivity reactions
Piperacillin, and Ticarcillin?

What is the major toxic


Hypersensitivity reactions
side effect of Penicillin?

What is the memory aid for


subunit distribution of Buy AT 30, CELL at 50'
ribosomal inhibitors?

What is the memory key INH: Injures Neurons and


for Isoniazid (INH) toxicity? Hepatocytes
What is the memory key
for Metronidazole's clinical GET on the Metro
uses?

What is the memory key


for organisms treated with VACUUM your Bed Room'
Tetracyclines?

1. RNA pol inhibitor 2.


What is the memory key Revs up P450 3. Red/
involving the '4 R's of orange body fluids 4.
Rifampin?' Rapid resistance if used
alone

What is the MOA for Inhibit viral DNA


Acyclovir? polymerase
Binds Ergosterol, forms
What is the MOA for
Membrane Pores that
Amphotericin B?
Disrupt Homeostatis

Same as penicillin.
What is the MOA for
Extended spectrum
Ampicillin and Amoxicillin?
antibiotics

What is the MOA for Same as penicillin.


Carbenicillin, Piperacillin, Extended spectrum
and Ticarcillin? antibiotics

Blocks Peptide Bond


What is the MOA for
formation at the 50S
Clindamycin?
subunit, Bacteriostatic
What is the MOA for Same as penicillin. Act as
Methicillin, Nafcillin, and narrow spectrum
Dicloxacillin? antibiotics

Forms toxic metabolites in


What is the MOA for
the bacterial cell,
Metronidazole?
Bactericidal

What is the MOA for Binds ergosterol, Disrupts


Nystatin? fungal membranes

What is the MOA for Inhibits DNA dependent


Rifampin? RNA polymerase
Inhibits formation of
What is the MOA for the Initiation Complex, causes
Aminoglycosides? misreading of mRNA,
Bactericidal

What is the MOA for the


Inhibit Ergosterol synthesis
Azoles?

What is the MOA for the Beta lactams - inhibit cell


Cephalosporins? wall synthesis, Bactericidal

Inhibit DNA Gyrase


What is the MOA for the
(topoisomerase II),
Fluoroquinolones?
Bactericidal
Blocks translocation, binds
What is the MOA for the
to the 23S rRNA of the 50S
Macrolides?
subunit, Bacteriostatic

Binds 30S subunit and


What is the MOA for the prevents attachment of
Tetracyclines? aminoacyl-tRNA,
Bacteriostatic

Inhibits bacterial
What is the MOA for
Dihydrofolate Reductase,
Trimethoprim (TMP)?
Bacteriostatic

Inhibits cell wall


What is the MOA for
mucopeptide formation,
Vancomycin?
Bactericidal
Blocks viral penetration/
What is the MOA of uncoating; may act to
Amantadine? buffer the pH of the
endosome

Inhibits cell wall synthesis


What is the MOA of
( binds to PBP3). A
Aztreonam?
monobactam

What is the MOA of Inhibits Viral DNA


Foscarnet? polymerase

What is the MOA of Inhibits CMV DNA


Ganciclovir? polymerase
Interferes with microtubule
What is the MOA of
function, disrupts mitosis,
Griseofulvin?
inhibits growth

What is the MOA of Acts as a wide spectrum


Imipenem? carbapenem

What is the MOA of Decreases synthesis of


Isoniazid (INH)? Mycolic Acid

Bind cell membrane,


disrupt osmotic
What is the MOA of
properties, Are Cationc,
Polymyxins?
Basic and act as
detergents
Inhibits IMP
Dehydrogenase
What is the MOA of
(competitively), and
Ribavirin?
therefore blocks Guanine
Nucleotide synthesis

Inhibit RT of HIV and


What is the MOA of the RT prevent the incorporation
Inhibitors? of viral genome into the
host DNA

What is the most common


cause of Pt noncompliance GI discomfort
with Macrolides?

What is treated with


Chloroquine, Quinine, Malaria (P. falciparum)
Mefloquine?
What microorganisms are
Aminoglycosides Anaerobes
ineffective against?

Vibrio cholerae Acne


Chlamydia Ureaplasma
What microorganisms are
Urealyticum Mycoplasma
clinical indications for
pneumoniae Borrelia
Tetracycline therapy?
burgdorferi (Lyme's)
Rickettsia Tularemia

What microorganisms is
Aztreonam not effective Gram + and Anerobes
against?

What musculo-skeletal
side effects in Adults are Tendonitis and Tendon
associated with rupture
Floroquinolones?
What neurotransmitter
Dopamine; causes its
does Amantadine affect?
release from intact nerve
How does it influence this
terminals
NT?

What organism is
Imipenem/cilastatin the Enterobacter
Drug of Choice for?

What organisms does Dermatophytes (tinea,


Griseofulvin target? ringworm)

Giant Roundworm
What parasites are treated
(Ascaris), Hookworm
with Pyrantel Pamoate
(Necator/Ancylostoma),
(more specific)?
Pinworm (Enterobius)
What parasitic condition is Onchocerciasis ('river
treated with Ivermectin? blindness'--rIVER-mectin)

What populations are Pregnant women, Children;


Floroquinolones because animal studies
contraindicated in? Why? show Damage to Cartilage

Milk or Antacids, because


What should not be taken divalent cations inhibit
with Tetracyclines? / Why? Tetracycline absorption in
the gut

What Sulfonamides are


Triple sulfas or SMZ
used for simple UTIs?
When pts have Low CD4+
When is HIV therapy
(&lt; 500 cells/cubic mm)
initiated?
or a High Viral Load

1. Meningococcal carrier
When is Rifampin not used
state 2. Chemoprophylaxis
in combination with other
in contacts of children
drugs?
with H. influenzae type B

Where does Griseofulvin Keratin containing tissues,


deposit? e.g., nails

Which Aminoglycoside is
Neomycin
used for Bowel Surgery ?
Which antimicrobial 1) Aminoglycosides =
classes inhibit protein bactericidal 2)
synthesis at the 30S Tetracyclines =
subunit? (2) bacteriostatic

1) Chloramphenical =
bacteriostatic 2)
Which antimicrobials Erythromycin =
inhibit protein synthesis at bacteriostatic 3)
the 50S subunit? (4) Lincomycin = bacteriostatic
4)cLindamycin =
bacteriostatic

Which individuals are


predisposed to G6PD deficient individuals
Sulfonamide-induced
hemolysis?

Which RT inhibitor causes


AZT
Megaloblastic Anemia?
Which RT inhibitors cause
Non-Nucleosides
a Rash?

Which RT inhibitors cause


Nucleosides
Lactic Acidosis?

Which Tetracycline is used


Doxycycline, because it is
in patients with renal
fecally eliminated
failure? / Why?

B.W. !!!, Ha. Good Luck on


Who's your daddy?
Boards
Why are Methicillin,
Due to the presence of a
Nafcillin, and Dicloxacillin
bulkier R group
penicillinase resistant?

To inhibit renal
Why is Cilastatin Dihydropeptidase I and
administered with decrease Imipenem
Imipenem? inactivation in the renal
tubules

-S-phase anti-metabolite
List the mechanism, Pyr analogue -Colon, solid
clinical use, &amp; toxicity tumors, &amp; BCC/ -
of 5 FU. Irreversible
myelosuppression

List the mechanism,


-inhibits HGPRT (pur. Syn.)
clinical use, &amp; toxicity
- Luk, Lymph,
of 6 MP.
-DNA intercalator -
List the mechanism, testicular &amp;
clinical use, &amp; toxicity lymphomas -Pulmonary
of Bleomycin. fibrosis mild
myelosuppression.

List the mechanism, -Alkalates DNA -CML -


clinical use, &amp; toxicity Pulmonary fibrosis
of Busulfan. hyperpigmentation

-Alkalating agent -
List the mechanism,
testicular,bladder,ovary,&a
clinical use, &amp; toxicity
mp;lung -Nephrotoxicity
of Cisplatin.
&amp; CN VIII damage.

-Alkalating agent -NHL,


List the mechanism, Breast, ovary, &amp; lung.
clinical use, &amp; toxicity - Myelosuppression,
of Cyclophosphamide. &amp; hemorrhagic
cystitis.
-DNA intercalator -
List the mechanism, Hodgkin's, myeloma,
clinical use, &amp; toxicity sarcoma, and solid tumors
of Doxorubicin. -Cardiotoxicity &amp;
alopecia

-Topo II inhibitor(GII
specific) -Oat cell of Lung
List the mechanism,
&amp; prostate, &amp;
clinical use, &amp; toxicity
testicular -
of Etoposide.
Myelosuppression &amp;
GI irritation.

-S-phase anti-metabolite
folate analogue -Luk,
List the mechanism,
Lymp, sarc, RA,
clinical use, &amp; toxicity
&amp;psoriasis / -
of Methotrexate.
Reversible
myelosuppression

List the mechanism,


-Alkalate DNA -Brain
clinical use, &amp; toxicity
tumors -CNS toxicity
of Nitrosureas.
-MT polymerization
List the mechanism, stabilizer -Ovarian &amp;
clinical use, &amp; toxicity breast CA -
of Paclitaxel. Myelosupperession &amp;
hypersensitivity.

List the mechanism, -Triggers apoptosis -CLL,


clinical use, &amp; toxicity Hodgkin's in MOPP -
of Prednisone. Cushing-like syndrome

-Estrogen receptor
List the mechanism,
antagonist -Breast CA -
clinical use, &amp; toxicity
increased endometrial CA
of Tamoxifen.
risk

-MT polymerization
inhibitor(M phase) -MOPP,
List the mechanism,
lymphoma, Willm's &amp;
clinical use, &amp; toxicity
choriocarcinoma -
of Vincristine.
neurotoxicity and
myelosuppression
-Alkalating agents
Which cancer drugs effect +cisplatin -Doxorubicin
nuclear DNA (4)? +Dactinomycin -Bleomycin
-Etoposide

Which cancer drugs inhibit - Methotrexate - 5 FU - 6


nucleotide synthesis(3)? mercaptopurine

Which cancer drugs work


-Steroids -Tamoxifen
at the level of mRNA(2)?

Which cancer drugs work -Vinca alkaloids(inhibit


at the level of proteins(2)? MT) -Paclitaxel
ACE inhibitors- clinical hypertension, CHF,
use? diabetic renal disease

reduce levels of
Angiotensin II, thereby
ACE inhibitors- preventing the inactivation
mechanism? of bradykinin (a potent
vasodilator); renin level is
increased

fetal renal damage,


hyperkalemia, Cough,
Angioedema, Proteinuria,
ACE inhibitors- toxicity? Taste changes, hypOtension,
Pregnancy problems, Rash,
Increased renin, Lower
Angiotensin II (CAPTOPRIL)

glaucoma, urinary
Acetazolamide- clinical
alkalinization, metabolic
uses?
alkalosis, altitude sickness
acts at the proximal
convoluted tubule to
inhibit carbonic anhydrase.
Acetazolamide-
Causes self-limited
mechanism?
sodium bicarb diuresis and
reduction of total body
bicarb stores.

acetazolamide- site of proximal convoluted


action? tubule

hyperchloremic metabolic
Acetazolamide- toxicity? acidosis, neuropathy, NH3
toxicity, sulfa allergy

ACIDazolamide' causes
Acetazolamide causesÉ?
acidosis
DOC in diagnosing and
Adenosine- clinical use? abolishing AV nodal
arrhythmias

ADH antagonists- site of


collecting ducts
action?

adverse effect of cyanide toxicity (releases


Nitroprusside? CN)

impotence, asthma, CV
effects (bradycardia, CHF,
adverse effects of beta-
AV block), CNS effects
blockers?
(sedation, sleep
alterations)
fetal renal toxicity,
hyperkalemia, Cough,
adverse effects of Angioedema, Proteinuria,
Captopril? Taste changes, hypOtension,
Pregnancy problems, Rash,
Increased renin, Lower
Angiotensin II (CAPTOPRIL)

dry mouth, sedation,


adverse effects of
severe rebound
Clonidine?
hypertension

severe orthostatic
adverse effects of hypotension, blurred
ganglionic blockers? vision, constipation,
sexual dysfunction

orthostatic and exercise


adverse effects of
hypotension, sexual
Guanethidine?
dysfunction, diarrhea
nausea, headache, lupus-
adverse effects of like syndrome, reflex
Hydralazine? tachycardia, angina, salt
retention

hypokalemia, slight
hyperlipidemia,
adverse effects of
hyperuricemia, lassitude,
Hydrochlorothiazide?
hypercalcemia,
hyperglycemia

K+ wasting, metabolic
adverse effects of Loop
alkalosis, hypotension,
Diuretics?
ototoxicity

adverse effects of fetal renal toxicity,


Losartan? hyperkalemia
adverse effects of sedation, positive Coombs'
Methyldopa? test

hypertrichosis, pericardial
adverse effects of effusion, reflex
Minoxidil? tachycardia, angina, salt
retention

dizziness, flushing,
adverse effects of
constipation (verapamil),
Nifedipine, verapamil?
nausea

first dose orthostatic


adverse effects of
hypotension, dizziness,
Prazosin?
headache
adverse effects of sedation, depression,
Reserpine? nasal stuffiness, diarrhea

pulmonary fibrosis, corneal


deposits, hepatotoxicity, skin
deposits resulting in
Amiodarone- toxicity? photodermatitis, neurologic
effects, consitpation, CV
(bradycardia, heart block,
CHF), and hypo- or
hyperthyroidism.

slowly normalize K+,


lidocaine, cardiac pacer,
antidote?
and anti-Dig Fab
fragments

Beta Blockers- CNS


sedation, sleep alterations
toxicity?
bradycardia, AV block,
Beta Blockers- CV toxicity?
CHF

Beta adrenergic receptors


Beta Blockers- site of
and Ca2+ channels
action?
(stimulatory)

BP? decrease

BP? decrease
new arrhythmias,
Bretyllium- toxicity?
hypotension

Ca2+ channel blockers- hypertension, angina,


clinical use? arrhythmias

block voltage dependent


L-type Ca2+ channels of
Ca2+ channel blockers-
cardiac and smooth
mechanism?
muscle- decreasing
contractility

Cell membrane Ca2+


Ca2+ channel blockers-
channels of cardiac
site of action?
sarcomere
cardiac depression,
Ca2+ channel blockers- peripheral edema,
toxicity? flushing, dizziness,
constipation

Ca2+ sensitizers'- site of troponin-tropomyosin


action? system

Cautions when using check PFTs, LFTs, and


Amiodarone? TFTs

increased AP duration,
increased ERP increased
class IA effects?
QT interval. Atrial and
ventricular.
post MI and digitalis
class IB- clinical uses?
induced arrhythmias

decrease AP duration,
affects ischemic or
class IB- effects?
depolarized Purkinje and
ventricular system

local anesthetic. CNS


class IB- toxicity? stimulation or depression.
CV depression.

NO AP duration effect.
useful in V-tach that
class IC- effects? progresses to V-fib and in
intractable SVT LAST
RESORT
class IC- toxicity? proarrhythmic

decrease the slope of


phase 4, increase PR
class II- effects?
interval (the AV node is
particularly sensitive)

blocking the beta


adrenergic receptor leads
class II- mechanism?
to decreased cAMP, and
decreased Ca2+ flux

impotence, exacerbation
of asthma, CV effects, CNS
class II- toxicity?
effects, may mask
hypoclycemia
increase AP duration,
increase ERP, increase QT
Class III- effects?
interval, for use when
other arrhythmics fail

prevention of nodal
class IV- clinical use?
arrhythmias (SVT)

decrease conduction
class IV- effects? velocity, increase ERP,
increase PR interval

class IV- primary site of


AV nodal cells
action?
constipation, flushing,
edema, CV effects (CHF,
class IV- toxicity? AV block, sinus node
depression), and torsade
de pointes (Bepridil)

diuretics,
sympathoplegics,
classes of antihypertensive
vasodilators, ACE
drugs?
inhibitors, Angiotensin II
receptor inhibitors

angina, pulmonary edema


clinical use?
(also, erection enhancer)

clinical use? CHF, atrial fibrillation


contractility? increase (reflex response)

contractility? decrease

renal failure, hypokalemia,


contraindications?
pt on quinidine

decrease Digitoxin dose in


NO
renal failure?
decrease Digoxin dose in
YES
renal failure?

Digitalis- site of action? Na/K ATPase

Digoxin v. Digitoxin: Digitoxin&gt;95% Digoxin


bioavailability? 75%

Digoxin v. Digitoxin: Digoxin=urinary


excretion? Digitoxin=biliary
Digoxin v. Digitoxin: half Digitoxin 168hrs Digoxin
life? 40 hrs

Digoxin v. Digitoxin: Digitoxin 70% Digoxin


protein binding? 20-40%

ejection time? decrease

ejection time? increase


inc PR, dec QT, scooping
EKG results? of ST, and T wave
inversion

end diastolic volume? decrease

end diastolic volume? increase

Esmolol- short or long


very short acting
acting?
Ethacrynic Acid- clinical Diuresis in pateints with
use? sulfa allergy

not a sulfonamide, but


Ethacrynic Acid-
action is the same as
mechanism?
furosemide

NO HYPERURICEMIA, NO
Ethacrynic Acid- toxicity? SULFA ALLERGY; same as
furosemide otherwise

Sulfonamide Loop Diuretic.


Inhibits ion co-transport
system of thick ascending
Furosemide- class and
loop. Abolishes
mechanism?
hypertonicity of the
medulla, thereby preventing
concentration of the urine.
edematous states (CHF,
cirrhosis, nephrotic
Furosemide- clinical use?
syndrome, pulm edema),
HTN, hypercalcemia

Ototoxicity, Hypokalemia,
Furosemide- toxicity? (OH Dehydration, Allergy
DANG) (sulfa), Nephritis
(interstitial), Gout

Furosemide increases the


Ca2+ (Loops Lose calcium)
excretion of what ion?

HDL effect? no effect


HDL effect? increase

HDL effect? moderate increase

HDL effect? increase

HDL effect? DECREASE


decrease myocardial O2
consumption by: 1-
decreasing end diastolic
how do we stop angina? volume 2- decreasing BP
3- decreasing HR 4-
decreasing contractility 5-
decreasing ejection time

HR? increase (reflex response)

HR? decrease

vasodilator- increases
cGMP to induce smooth
Hydralazine- class and
muscle relaxation
mechanism?
(arterioles&gt;veins;
afterload reduction)
Hydralazine- clinical use? severe hypertension, CHF

compensatory tachycardia,
Hydralazine- toxicity? fluid retention, lupus-like
syndrome

Hydrochlorothiazide- HTN, CHF, calcium stone


clinical use? formation, nephrogenic DI.

Inhibits NaCl reabsorption


Hydrochlorothiazide-
in the early distal tubule.
mechanism?
Decreases Ca2+ excretion.
Hypokalemic metabolic
alkalosis, hyponatremia,
Hydrochlorothiazide- hyperGlycemia,
toxicity? (hyperGLUC, plus hyperLipidemia,
others) hyperUricemia,
hyperCalcemia, sulfa
allergy.

Ibutilide- toxicity? torsade de pointes

depresses ectopic
K+- clinical use? pacemakers, especially in
digoxin toxicity

K+ sparing diuretics- hyperaldosteronism, K+


clinical use? depletion, CHF
K+ sparing diuretics- site
cortical collecting tubule
of action?

hyperkalemia, endocrine
K+ sparing diuretics-
effects (gynecomastia,
toxicity?
anti-androgen)

LDL effect? moderate decrease

LDL effect? large decrease


LDL effect? moderate decrease

LDL effect? decrease

LDL effect? decrease

loop diuretics
(furosemide)- site of thick ascending limb
action?
ARF, shock, drug
overdose, decrease
Mannitol- clinical use?
intracranial/intraocular
pressure

Mannitol-
anuria, CHF
contraindications?

osmotic diuretic- increase


tubular fluid osmolarity,
Mannitol- mechanism?
thereby increasing urine
flow

proximal convoluted
mannitol- site of action? tubule, thin descending
limb, and collecting duct
pulmonary edema,
Mannitol- toxicity?
dehydration

vasodilate by releasing NO
in smooth muscle, causing
mechanism? and increase in cGMP and
smooth muscle relaxation
(veins&gt;&gt;arteries)

inhibits the Na/K ATPase,


increasing intracellular Na
mechanism? + decreasing the function
of the Na/Ca antiport
causing an increase in
intracellular Ca2+

Na+ channel blockers.


Slow or block conduction.
mechanism? Decreased slope in phase
4 and increased threshold
for firing in abnormal
pacemaker cells.
effective in torsade de
Mg+- clinical use? pointes and digoxin
toxicity

MVO2? decrease

MVO2? decrease

propanolol, esmolol,
name five in class II? metoprolol, atenolol,
timolol
name four HMG-CoA Lovastatin, Pravastatin,
reductase inhibitors. Simvastatin, Atorvastatin

Quinidine, Amiodarone,
name four in class IA. Procainamide,
Disopyramide

Sotalol, Ibutilide,
name four in class III.
Bretylium, Amiodarone

Captopril, Enalapril,
name three ACE inhibitors?
Lisinopril
name three calcium Nifedipine, Verapamil,
channel blockers? Diltiazem

Lidocaine, Mexiletine,
name three in class IB.
Tocainide

Flecainide, Encainide,
name three in class IC.
Propafenone

Verapamil, Diltiazem,
name three in class IV.
Bepridil
Spironolactone,
name three K+ sparing
Triamterene, Amiloride
diuretics?
(the K+ STAys)

name two bile acid resins. cholestyramine, colestipol

name two LPL stimulators. Gemfibrozil, Clofibrate

Nifedipine has similar


Nitrates
action to?
preferential action of the cardiac muscle:
Ca2+ channel blockers at Verapamil&gt;Diltiazem&gt
cardiac muscle? ;Nifedipine

preferential action of the vascular sm. Mus.:


Ca2+ channel blockers at Nifedipine&gt;Diltiazem&g
vascular smooth muscle? t;Verapamil

reversible SLE-like
Procainamide- toxicity?
syndrome

cinchonism: HA, tinnitus,


thrombocytopenia, torsade
Quinidine- toxicity?
de pointes due to
increased QT interval
Ryanodine- stie of action? blocks SR Ca2+ channels

slectively depress tissue


that is frequently
selectivity?
depolarized (fast
tachycardia)

tastes bad and causes GI


side effects/problems?
discomfort

expensive, reversible
side effects/problems? increase in LFTs, and
myositis
red, flushed face which is
side effects/problems? decreased by ASA or long
term use

side effects/problems? myositis, increased LFTs

side effects/problems? DECREASED HDL

torsade de pointes,
Sotalol- toxicity?
excessive Beta block
competitive inhibirot of
Spironolactone-
aldosterone in the cortical
mechanism?
collecting tubule

TG effect? slight increase

TG effect? decrease

TG effect? decrease
TG effect? large decrease

TG effect? no effect

distal convoluted tubule


thiazides- site of action?
(early)

tachycardia, hypotension,
toxicity? headache - 'Monday
disease'
nausea, vomiting,
toxicity? diarrhea, blurred vision,
arrhythmia

Triamterene and block Na+ channels in the


amiloride- mechanism? cortical collecting tubule

Verapamil has similar


Beta Blockers
action to?

what two vasodilators


require simultaneous
treatment with beta Hydralazine and Minoxidil
blockers to prevent reflex
tachycardia and diuretics
to prevent salt retention?
carbonic anhydrase
which diuretics cause
inhibitors, K+ sparing
acidosis?
diuretics

which diuretics cause


loop diuretics, thiazides
alkalosis?

which diuretics decrease


thiazides, amiloride
urine Ca2+?

which diuretics increase loop diuretics,


urine Ca2+? spironolactone
all except the K+ sparing
which diuretics increase
diuretics Spironolactone,
urine K+?
Triamterene, Amiloride

which diuretics increase


all of them
urine NaCl?

Acetaminophen has what Acetaminophen has


two clinical uses and lacks antipyretic and analgesic
what one clinical use of properties, but lacks anti-
the NSAIDs? inflammatory properties.

Can Heparin be used Yes, it does not cross the


during pregnancy? placenta.
No, warfarin, unlike
Can Warfarin be used
heparin, can cross the
during pregnancy?
placenta.

Does Heparin have a long,


Short.
medium, or short half life?

Does Warfarin have a long,


Long.
medium, or short half life?

For Heparin what is the 1. Heparin 1. Structure - Large anionic


Structure 2. Route of polymer, acidic 2. Route of administration
- Paranteral (IV, SC) 3. Onset of action -
administration 3. Onset of action Rapid (seconds) 4. Mechanism of action -
4. Mechanism of action 5. Activates antithrombin III 5. Duration of
Duration of action 6. Ability to action - Acute (hours) 6. Ability to inhibit
inhibit coagulation in vitro 7. coagulation in vitro - Yes 7. Treatment for
overdose - Protamine sulfate 8. Lab value
Treatment for overdose 8. Lab to monitor-aPTT (intrinsic pathway) 9.
value to monitor 9. Site of action Site of action - Blood
For Warfarin what is the 1. Warfarin 1. Structure - Small lipid-
soluble molecule 2. Route of
Structure 2. Route of
administration -Oral 3. Onset of action
administration 3. Onset of action - Slow, limited by half lives of clotting
4. Mechanism of action 5. factors 4. Mechanism of action -
Duration of action 6. Ability to Impairs the synthesis of vitamin K-
inhibit coagulation in vitro 7. dependent clotting factors 5. Duration
Treatment for overdose 8. Lab of action - Chronic (weeks or months)
6. Ability to inhibit coagulation in vitro
value to monitor 9. Site of action - No

7. Treatment for overdose


- IV vitamin K and fresh
For Warfarin what is the
frozen plasma 8. Lab value
(continued):
to monitor - PT 9. Site of
action - Liver

Is toxicity rare or common


whith Cromolyn used in Rare.
Asthma prevention?

1. Hydrocortisone 2.
Predisone 3.
List five common
Triamcinolone 4.
glucocorticoids.
Dexamethasone 5.
Beclomethasone
Secretion of what drug is
inhibited by Probenacid Penicillin.
used to treat chronic gout?

The COX-2 inhibitors The COX-2 inhibitors


(celecoxib, rofecoxib) have should not have the
similar side effects to the corrosive effects of other
NSAIDs with what one NSAIDs on the
exception? gastrointestinal lining.

Sulfonylureas are oral


What are are the
hypoglycemic agents, they
Sulfonylureas (general
are used to stimulate
description) and what is
release of endogenous
their use?
insulin in NIDDM (type-2).

1. Reliable (&lt;1% failure) 2.


Lowers risk of endometrial
What are five advantages
and ovarian cancer 3.
of Oral Contraceptives
Decreased incidence of
(synthetic progestins,
ectopic pregnancy 4. Lower
estrogen)?
risk of pelvic infections 5.
Regulation of menses
1. Taken daily 2. No
What are five protection against STDs 3.
disadvantages of Oral Raises triglycerides 4.
Contraceptives (synthetic Depression, weight gain,
progestins, estrogen)? nausea, HTN 5.
Hypercoagulable state

1. Gastric ulceration 2.
What are five possible Bleeding 3.
toxic effects of Aspirin Hyperventilation 4. Reye's
therapy? syndrome 5. Tinnitus (CN
VIII)

1. Significant:
What are five toxicities nephrotoxicity 2.
associated with Tacrolimus Peripheral neuropathy 3.
(FK506)? Hypertension 4. Pleural
effusion 5. Hyperglycemia.

1. Better bioavailability 2.
What are four advantages 2 to 4 times longer half
of newer low-molecular- life 3. Can be administered
weight heparins subcutaneously 4. Does
(Enoxaparin)? not require laboratory
monitoring
1. Antipyretic 2. Analgesic
What are four clinical
3. Anti-inflammatory 4.
activities of Aspirin?
Antiplatelet drug.

1. Addison's disease 2.
What are four clinical uses
Inflammation 3. Immune
of glucocorticoids?
suppression 4. Asthma

What are four conditions in 1. Peptic ulcer 2. Gastritis


which H2 Blockers are 3. Esophageal reflux 4.
used clinically? Zollinger-Ellison syndrome

1. Cimetadine 2. Ranitidine
What are four H2 Blockers?
3. Famotidine 4. Nizatidine
1. Tolbutamide 2.
What are four
Chlorpropamide 3.
Sulfonylureas?
Glyburide 4. Glipizide

1. Streptokinase 2.
What are four Urokinase 3. tPA
thrombolytics? (alteplase), APSAC
(anistreplase)

1. Hot flashes 2. Ovarian


What are four unwanted enlargement 3. Multiple
effects of Clomiphene use? simultaneous pregnancies
4. Visual disturbances

1. Buffalo hump 2. Moon


facies 3. Truncal obesity 4.
What are nine findings of
Muscle wasting 5. Thin
Iatrogenic Cushing's
skin 6. Easy bruisability 7.
syndrome caused by
Osteoporosis 8.
glucocorticoid therapy?
Adrenocortical atrophy 9.
Peptic ulcers
Headache, flushing ,
What are signs of Sildenafil
dyspepsia, blue-green
(Viagra) toxicity?
color vision.

Acute coronary syndrome;


What are the clinical uses coronary stenting.
for Ticlopidine, Decreases the incidence or
Clopidogrel? recurrence of thrombotic
stroke.

What are the four


1. Peptic ulcer 2. Gastritis
conditions in which
3. Esophageal reflux 4.
Omeprazole, Lansoprazole
Zollinger-Ellison syndrome
is used?

1. Infertility (pulsatile) 2.
Prostate cancer
What are three clinical
(continuous: use with
uses of the Leuprolide?
flutamide) 3. Uterine
fibroids
What are three clinical 1. Antipyretic 2. Analgesic
uses of the NSAIDs? 3. Anti-inflammatory

What are three common Ibuprofen, Naproxen, and


NSAIDS other than Aspirin? Indomethacin

What are three 1. Bleeding 2.


complications of Warfarin Teratogenicity 3. Drug-
usage? drug interactions

What are three possible 1. Bleeding 2.


complications of Heparin Thrombocytopenia 3.
therapy? Drug-drug interactions
1. Renal damage 2.
What are three possible
Aplastic anemia 3. GI
toxicities of NSAID usage?
distress

What are three toxicities of 1. Antiandrogen 2. Nausea


Leuprolied? 3. Vomiting

1. Skin rash 2.
What are three toxicities of
Agranulocytosis (rare) 3.
Propylthiouracil?
Aplastic anemia

1. Aluminum hydroxide:
constipation and
What are three types of hypophosphatemia 2.
antacids and the problems Magnesium hydroxide:
that can result from their diarrhea 3. Calcium
overuse? carbonate: Hypercalcemia,
rebound acid increase - All
may cause hypokalemia
1. Heavy bleeding 2. GI
What are three unwanted
effects (n/v, anorexia) 3.
effects of Mifepristone?
Abdominal pain

What are two Alpha-


1. Acarbose 2. Miglitol
glucosidase inhibitors?

1. Kidney transplantation
2. Autoimmune disorders
What are two clinical uses
(including
of Azathioprine?
glomerulonephritis and
hemolytic anemia)

What are two conditions in


Rheumatoid and
which COX-2 inhibitors
osteoarthritis.
might be used?
1. Pioglitazone 2.
What are two Glitazones?
Rosiglitazone.

Inhibits organification and


What are two mechanisms coupling of thyroid
of action of hormone synthesis. Also
Propythiouracil? decreases peripheral
conversion of T4 to T3.

1. Phospholipase A2 is
prevented from releasing
What are two processes
arachidonic acid 2.
Corticosteroids inhibit
Decreases protein
leading to decreased
synthesis thus lowering
inflammation?
amount of Cyclooxygenase
enzymes

1. Predisposes to viral
What are two toxicities infections and lymphoma
associated with 2. Nephrotoxic
Cyclosporine? (preventable with mannitol
diuresis)
1. Weight gain 2.
What are two toxicities of
Hepatotoxicity
the Glitazones?
(troglitazone)

1. Hypoglycemia (more
common with 2nd-
generation drugs:
What are two toxicities of
glyburide, glipizide) 2.
the Sulfonylureas?
Disulfiram-like effects (not
seen with 2nd-generation
drugs).

Sucralfate cannot work in


What are two types of the presence of antacids
drugs that interfere with or H2 blockers because it
the action of Sucralfate requires an acidic
and why? environment to
polymerize.

Can affect absorption,


bioavailability, or urinary
What can result due to excretion of other drugs
antacid overuse? by altering gastric and
urinary pH or by delaying
gastric emptying.
What enzyme does
Lipoxygenase
Zileuton inhibit?

What enzymes are


inhibited by NSAIDs, Cyclooxygenases (COX I,
acetaminophen and COX II COX II).
inhibitors?

What is a common side


effect of Colchicine used GI side effects. (Note:
to treat acute gout, Indomethacin is less toxic,
especially when given more commonly used.)
orally?

What is a common side


Diarrhea
effect of Misoprostol?
Overdose produces
hepatic necrosis;
What is a possible result of acetaminophen
overdose of metablolite depletes
Acetaminophen? glutathione and forms
toxic tissue adducts in
liver.

What is a possible toxicity


of Alpha-glucosidase GI disturbances.
inhibitors used in type-2
diabetes?

What is a possible toxicity Neutropenia (ticlopidine);


of Ticlopidine, Clopidogrel reserved for those who
usage? cannot tolerate aspirin.

What is a sign of toxicity


with the use of Bleeding.
thrombolytics?
1. In liver, increases storage
of glucose as glycogen. 2. In
What is action of insulin in muscle, stimulates glycogen
the liver, in muscle, and in and protein synthesis, and K
adipose tissue? + uptake. 3. In adipose
tissue, facilitates
triglyceride storage.

1. Suppresses organ
What is are two clinical rejection after
uses of Cyclosporine? transplantation 2. Selected
autoimmune disorders.

What is the category and


mechanism of action of Antileukotriene; blocks
Zafirlukast in Asthma leukotriene receptors.
treatment?

What is the category and


mechanism of action of Antileukotriene; blocks
Zileuton in Asthma synthesis by lipoxygenase.
treatment?
What is the category of
Inhalational general
drug names ending in -
anesthetic.
ane (e.g. Halothane)

What is the category of


drug names ending in - Benzodiazepine.
azepam (e.g. Diazepam)

What is the category of


drug names ending in - Phenothiazine
azine (e.g. (neuroleptic, antiemetic).
Chlorpromazine)

What is the category of


drug names ending in - Antifungal.
azol (e.g. Ketoconazole)
What is the category of
drug names ending in - Babiturate.
barbital (e.g.
Phenobarbital)

What is the category of


drug names ending in - Local anesthetic.
caine (e.g. Lidocaine)

What is the category of


drug names ending in - Penicillin.
cillin (e.g. Methicillin)

What is the category of


Antibiotic, protein
drug names ending in -
synthesis inhibitor.
cycline (e.g. Tetracycline)
What is the category of
drug names ending in - Tricyclic antidepressant.
ipramine (e.g. Imipramine)

What is the category of


drug names ending in - Protease inhibitor.
navir (e.g. Saquinavir)

What is the category of


drug names ending in - Beta antagonist.
olol (e.g. Propranolol)

What is the category of


Butyrophenone
drug names ending in -
(neuroleptic).
operidol (e.g. Haloperidol)
What is the category of
Cardiac glycoside
drug names ending in -
(inotropic agent).
oxin (e.g. Digoxin)

What is the category of


drug names ending in - Methylxanthine.
phylline (e.g. Theophylline)

What is the category of


drug names ending in -pril ACE inhibitor.
(e.g. Captopril)

What is the category of


drug names ending in - Beta-2 agonist.
terol (e.g. Albuterol)
What is the category of
drug names ending in - H2 antagonist
tidine (e.g. Cimetidine)

What is the category of


drug names ending in - Tricyclic antidepressant.
triptyline (e.g.
Amitriptyline)

What is the category of


drug names ending in - Pituitary hormone.
tropin (e.g. Somatotropin)

What is the category of


drug names ending in - Alpha-1 antagonist
zosin (e.g. Prazosin)
Nonspecific beta-agonist;
What is the category, desired effect is the
desired effect, and adverse relaxation of bronchial
effect of Isoproterenol in smooth muscle (Beta 2).
the treatment of Asthma? Adverse effect is
tachycardia (Beta 1).

Beta 2 agonist; desired


What is the category,
effect is the relaxation of
desired effect, and period
bronchial smooth muscle
of use of albuterol in the
(Beta 2). Use during acute
treatment of Asthma?
exacerbation.

Methylzanthine; desired
What is the category, effect is bronchodilation,
desired effect, and may cause bronchodilation
possible mechanism of by inhibiting
Theophylline in treating phosphodiesterase, enzyme
Asthma? involved in degrading cAMP
(controversial).

Muscarinic antagonist;
What is the category,
competatively blocks
mechanism of action, and
muscarinic receptors,
effect of Ipratroprium in
preventing
Asthma treatment?
bronchoconstriction.
Corticosteroids; prevent
What is the category,
production of leukotrienes
mechanism of action, and
from arachodonic acid by
particular use of
blocking phospholipase A2.
beclomethasone and
Drugs of choice in a patient
prednisone in Asthma
with status asthmaticus (in
treatment?
combination with albuterol.)

What is the category, Beta 2 agonist; used as a


method of use, and long-acting agent for
adverse effects of prophylaxis. Adverse
Salmeterol in Asthma effects are tremor and
treatment? arrhythmia.

Prevention of NSAID-
What is the clincial use for
induced peptic ulcers,
Misoprostol?
maintains a PDA.

What is the clinical use for


Treatment of infertility.
Clomiphene?
Immediate anticoagulation
What is the clinical use for
for PE, stroke, angina, MI,
Heparin?
DVT.

What is the clinical use for


Erectile dysfunction.
Sildenafil (Viagra)?

What is the clinical use for


Peptic ulcer disease.
Sucralfate?

What is the clinical use for


Chronic anticoagulation.
Warfarin?
What is the clinical use of
Abortifacient.
Mifepristone (RU486)?

Potent immunosuppressive
What is the clinical use of
used in organ transplant
Tacrolimus (FK506)?
recipients.

What is the effect of the


Increase target cell
Glitazones in diabetes
response to insulin.
treatment?

Finasteride inhibits 5
What is the enzyme
Alpha-reductase, this
inhibited, the effect of this
decreases the conversion
inhibition, and the clinical
of testosterone to
use of the antiandrogren
dihydrotestosterone,
Finasteride?
useful in BPH
What is the lab value used
to monitor the The PTT.
effectiveness of Heparin
therapy?

What is the lab value used


to monitor the The PT.
effectiveness of Warfarin
therapy?

What is the main clinical Early myocardial


use for the thrombolytics? infarction.

Aluminum sucrose sulfate


polymerizes in the acid
environment of the
What is the mecanism of
stomach and selectively
action of Sucralfate?
binds necrotic peptic ulcer
tissue. Acts as a barrier to
acid, pepsin, and bile.
Selectively inhibit
cyclooxygenase (COX)
What is the mecanism of isoform 2, which is found in
action of the COX-2 inflammatory cells nad
inhibitors (celecoxib, mediates inflammation and
rofecoxib)? pain; spares COX-1 which
helps maintain the gastric
mucosa.

Prevents release of
What is the mecanism of
mediators from mast cells.
action, effective period,
Effective only for the
and ineffective period of
prophylaxis of asthma. Not
use for Cromolyn in
effective during an acute
treating Asthma?
attack.

Flutamide is a nonsteroidal
What is the mechanism of competitive inhibitor of
action and clinical use of androgens at the
the antiandrogen testosterone receptor,
Flutamide? used in prostate
carcinoma.

What is the mechanism of Inhibit steroid synthesis,


action and clinical use of used in the treatment of
the antiandrogens polycystic ovarian
Ketoconazole and syndrome to prevent
Spironolactone? hirsutism.
Reversibly inhibits
What is the mechanism of cyclooxygenase, mostly in
action of Acetaminophen? CNS. Inactivated
peripherally.

What is the mechanism of Inhibits xanthine oxidase,


action of Allopurinol used decresing conversion of
to treat chronic gout? xanthine to uric acid.

Acetylates and irreversibly


inhibits cyclooxygenase
What is the mechanism of (COX I and COX II) to
action of Aspirin? prevent the conversion of
arachidonic acid to
prostaglandins.

Clomiphene is a partial
agonist at estrogen receptors
in the pituitary gland.
What is the mechanism of Prevents normal feedback
action of Clomiphene? inhibition and increses
release of LH and FSHfrom
the pituitary, which
stimulates ovulation.
Depolymerizes
What is the mechanism of
microtubules, impairing
action of Colchicine used
leukocyte chemotaxis and
to treat acute gout?
degranulation.

Binds to cyclophilins
(peptidyl proline cis-trans
isomerase), blocking the
What is the mechanism of
differentiation and
action of Cyclosporine?
activation of T cells mainly
by inhibiting the production
of IL-2 and its receptor.

Heparin catalyzes the


What is the mechanism of
activation of antithrombin
action of Heparin?
III.

What is the mechanism of Competitive inibitor of


action of Mifepristone progestins at progesterone
(RU486)? receptors.
Misoprostol is a PGE1
analog that increases the
What is the mechanism of
production and secretion
action of Misoprostol?
of the gastic mucous
barrier.

Reversibly inhibit
What is the mechanism of
cyclooxygenase (COX I and
action of NSAIDs other
COX II). Block
than Aspirin?
prostaglandin synthesis.

What is the mechanism of Irreversibly inhibits H+/K+


action of Omeprazole, ATPase in stomach parietal
Lansoprazole? cells.

What is the mechanism of


Inhibits reabsorption of
action of Probenacid used
uric acid.
to treat chronic gout?
Inhibits cGMP
phosphodiesterase,
What is the mechanism of casuing increased cGMP,
action of Sildenafil smooth muscle relaxation
(Viagra)? in the corpus cavernosum,
increased blood flow, and
penile erection.
Inhibit intestinal bursh
border Alpha-glucosidases;
What is the mechanism of delayed hydrolysis of
action of the Alpha- sugars and absorption of
glucosidase inhibitors? sugars leading to decresed
postprandial
hyperglycemia.

Decrease the production of


What is the mechanism of leukotrienes and
action of the protaglandins by inhibiting
glucocorticoids? phospholipase A2 and
expression of COX-2.

What is the mechanism of Reversible block of


action of the H2 Blockers? histamine H2 receptors
Close K+ channels in
Beta-cell membrane
What is the mechanism of leading to cell
action of the depolarization causing
Sulfonylureas? insulin release triggered
by increase in Calcium ion
influx.
Directly of indirectly aid
conversion of plasminogen to
What is the mechanism of plasmin which cleaves
action of the thrombin and fibrin clots. (It
is claimed that tPA
thrombolytics?
specifically converts fibrin-
bound plasminogen to
plasmin.)

Inhibits platelet
What is the mechanism of aggregation by irreversibly
action of Ticlopidine, inhibiting the ADP pathway
Clopidogrel involved in the binding of
fibrinogen.

Warfarin interferes with


the normal synthesis and
What is the mechanism of gamma-carboxylation of
action of Warfarin vitamin K-dependent
(Coumadin)? clotting factors II, VII, IX,
and X, Protein C and S via
vitamin K antagonism.
Antimetabolite derivative
of 6-mercaptopurine that
What is the mechanism of
interferes with the
Azathioprine?
metablolism and synthesis
of nucleic acid.

GnRH analog with agonist


properties when used in
pulsatile fashion and
What is the mechanism of
antagonist properties when
Leuprolide?
used in continuous fashion,
causing a transient initial
burst of LH and FSH

Similar to cyclosporine;
binds to FK-binding
What is the mechanism of
protein, inhibiting
Tacrolimus (FK506)?
secretion of IL-2 and other
cytokines.

What is the memory key


for the action of Sildenafil Sildenafil fills the penis
(Viagra)?
What is the memory key
AluMINIMUM amount of
for the effect of aluminum
feces.
hydroxide overuse?

What is the memory key


for the effect of Mg = Must go to the
magnesium hydroxide bathroom.
overuse?

What is the memory key to


remember which pathway WEPT: Warfarin affects the
(extrinsic vs. intrinsic) and Extrinsic pathway and
which lab value Warfarin prolongs the PT.
affects?

Mechanism unknown;
What is the possible possibly inhibits
mechanism and effect of gluconeogenesis and
Metformin in treating increases glycolysis; effect
diabetes? is to decrease serum
glucose levels
What is the specific clinical Indomethacin is used to
use of Indomethacin in close a patent ductus
neonates? arteriosus.

Protamine Sulfate is used


for rapid reversal of
What is used to reverse the heparinization (positively
action of Heparin? charged molecule that
binds to negatively
charged heparin).

What patients are at risk


for life threatening Those patients who are
hypotension when taking taking nitrates.
Sildenafil (Viagra)?

What process does Leukotrienes increasing


Zafirlukast interfere with? bronchial tone.
What type of gout is
Chronic gout.
treated with Allopurinol?

What type of gout is


Acute gout.
treated with Colchicine?

What type of gout is


Chronic gout.
treated with Probenacid?

Misoprostol is
What type of patient contraindicated in women
should not take of childbearing potential
Misoprostol and why? because it is an
abortifacient.
Cimetidine is a potent
inhibitor of P450; it also has
Which H2 Blocker has the an antiandrogenic effect and
most toxic effects and decreases renal excretion of
what are they? creatinine. Other H2
blockers are relatively free
of these effects.

Why are the Sulfonylureas Because they require some


inactive in IDDM (type-1)? residual islet function.

Acetaldehyde is
metabolized by -Disulfram &amp; also
Acetaldehyde sulfonylureas,
dehydrogenase, which metronidazole
drug inhibs this enzyme?

-Weak Acids&gt;Alkinalize
urine(CO3) to remove
Explain pH dependent
more -Weak
urinary drug elimination?
bases&gt;acidify urine to
remove more
-Airway -Breathing -
How do you treat coma in Circulation -Dextrose
the ER (4)? (thiamine &amp;narcan) -
ABCD

-Infections -Trauma -
In coma situations you rule Seizures -CO -Overdose -
out what (7)? Metabolic -Alcohol (IT'S
COMA)

-A57Blue lines in
gingiva&amp; long bones
List some specifics of lead -Encephalopathy &amp;
poisoning(4)? Foot drop -Abdominal
colic / -Sideroblastic
anemia

List the specific antidote


for this toxin: -N-acetylcystine
Acetaminophen
List the specific antidote
for this toxin: -Ammonium Chloride
Amphetamine

List the specific antidote


for this toxin: -Atropine &amp;
Anticholinesterases pralidoxime
(organophosphate.)

List the specific antidote


for this toxin: -Physostigmine salicylate
Antimuscarinic
(anticholinergic)

List the specific antidote


for this toxin: Arsenic (all -Dimercaprol, succimer
heavy metals)
List the specific antidote
for this toxin: -Flumazenil
Benzodiazepines

List the specific antidote


for this toxin: Beta -Glucagon
Blockers

List the specific antidote


for this toxin: Carbon -100% oxygen, hyperbaric
monoxide

List the specific antidote


-Penicillamine
for this toxin: Copper
-Nitrate,
List the specific antidote
hydroxocobalamin
for this toxin: Cyanide
thiosulfate

List the specific antidote -Normalize K+, Lidocaine,


for this toxin: Digitalis &amp; Anti-dig Mab

List the specific antidote


-Protamine
for this toxin: Heparin

List the specific antidote


-Deferoxamine
for this toxin: Iron
-EDTA, dimercaprol,
List the specific antidote
succimer, &amp;
for this toxin: Lead
penicillamine

List the specific antidote


-Ethanol, dialysis, &amp;
for this toxin: Methanol
fomepizole
&amp; Ethylene glycol

List the specific antidote


for this toxin: -Methylene blue
Methemoglobin

List the specific antidote -B51Naloxone /


for this toxin: Opioids naltrexone (Narcan)
List the specific antidote -Alkalinize urine &amp;
for this toxin: Salicylates dialysis

List the specific antidote


for this toxin: TPA &amp; -Aminocaproic acid
Streptokinase

List the specific antidote


for this toxin: Tricyclic -NaHCO3
antidepressants

List the specific antidote -Vitamin K &amp; fresh


for this toxin: Warfarin frozen plasma
What are the products and
-Acetaldehyde -Nausea,
their toxicities of the
vomiting, headache,
metabolism of ethanol by
&amp; hypotension
/ alcohol dehydrogenase?

What are the products and


their toxicities of the
-Oxalic acid -Acidosis
metabolism of Ethylene
&amp; nephrotoxicity
Glycol by / alcohol
dehydrogenase?

What are the products and


-Formaldehyde &amp;
their toxicities of the
formic acid -severe
metabolism of Methanol
acidosis &amp; retinal
by / alcohol
damage
dehydrogenase?

Which drug(s) cause this


reaction: Adrenocortical -Glucocorticoid withdrawal
Insufficiency
Which drug(s) cause this -Cloazapine -
reaction: Agranulocytosis carbamazapine -colchicine
(3)? -PTU

Which drug(s) cause this


-Penicillin
reaction: Anaphylaxis?

Which drug(s) cause this -Chloramphenicol -


reaction: Aplastic anemia benzene -NSAIDS -PTU -
(5)? phenytoin

Which drug(s) cause this


reaction: Atropine-like -Tricyclic antidepressants
side effects?
Which drug(s) cause this -Daunorubicin &amp;
reaction: Cardiac toxicity? Doxorubicin

Which drug(s) cause this


-Quinidine -quinine
reaction: Cinchonism (2)?

Which drug(s) cause this -ACE inhibitors


reaction: Cough? (Losartan&gt;no cough)

Which drug(s) cause this -Niacin -Ca++ channel


reaction: Cutaneous blockers -adenosine -
flushing (4)? vancomycin
Which drug(s) cause this
reaction: Diabetes -Lithium
insipidus?

-Metronidazole -certain
Which drug(s) cause this
cephalosporins -
reaction: Disulfram-like
procarbazine -
reaction (4) ?
sulfonylureas

Which drug(s) cause this -Haloperidol -


reaction: Drug induced chlorpromazine -reserpine
Parkinson's (4) ? -MPTP

Which drug(s) cause this


-Chlorpromazine -
reaction: Extrapyramidal
thioridazine -haloperidol
side effects (3)?
Which drug(s) cause this
reaction: Fanconi's -Tetracycline
syndrome?

Which drug(s) cause this -Halothane -Valproic acid


reaction: Focal to massive -acetaminophen -Amantia
hepatic necrosis (4)? phalloides

-Sulfonamides -INH -ASA


Which drug(s) cause this -Ibuprofen -primaquine -
reaction: G6PD hemolysis nitrofurantoin /-
(8)? pyrimethamine -
chloramphenicol

Which drug(s) cause this


reaction: Gingival -Phenytoin
hyperplasia?
Which drug(s) cause this
reaction: Gray baby -Chloramphenicol
syndrome?

Which drug(s) cause this -Cimetidine -ketoconazole


reaction: Gynecomastia (6) -spironolactone -digitalis
? -EtOH -estrogens

Which drug(s) cause this


-Isoniazid
reaction: Hepatitis?

Which drug(s) cause this


-Tamoxifen
reaction: Hot flashes?
Which drug(s) cause this
reaction: Neuro and -polymyxins
Nephrotoxic?

Which drug(s) cause this


-Corticosteroids -heparin
reaction: Osteoporosis (2)?

Which drug(s) cause this


-aminoglycosides -loop
reaction: Oto and
diuretics -cisplatin
Nephrotoxicity (3)?

Which drug(s) cause this -Barbiturates -phenytoin -


reaction: P450 induction carbamazipine -rifampin -
(6)? griseofulvin -quinidine
-Cimetidine -ketoconazole
Which drug(s) cause this
-grapefruit juice -
reaction: P450 inhibition
erythromycin -INH -
(6)?
sulfonamides

Which drug(s) cause this


-Tetracycline -amiodarone
reaction: Photosensitivity
-sulfonamides
(3)?

Which drug(s) cause this


reaction: -Clindamycin
Pseudomembranous
colitis?

Which drug(s) cause this


-Bleomycin -amiodarone -
reaction: Pulmonary
busulfan
fibrosis(3)?
Which drug(s) cause this -Hydralazine -
reaction: SLE-like Procainamide -INH -
syndrome phenytoin

Which drug(s) cause this


-Ethosuxamide -
reaction: Stevens-Johnson
sulfonamides -lamotrigine
syn. (3) ?

Which drug(s) cause this


reaction: Tardive -Antipsychotics
dyskinesia?

Which drug(s) cause this


reaction: Tendonitis and -Fluoroquinolones
rupture?
Which drug(s) cause this
reaction: Thrombotic -Oral Contraceptives
complications?

Which drug(s) cause this -Class III antiarrhythmics


reaction: Torsade de (sotalol) -class IA
pointes (2) ? (quinidine)

-Sulfonamides -
Which drug(s) cause this
furosemide -methicillin -
reaction: Tubulointerstitial
rifampin -NSAIDS (ex.
Nephritis (5)?
ASA)

Constant FRACTION
Describe first-order
eliminated per unit time.
kinetics?
(exponential)
-reduction, oxy, &amp;
Describe Phase I
hydrolysis -H2O sol. Polar
metabolism in liver(3)?
product -P450

-acetylation,
Describe Phase II glucuron.,&amp; sulfation
metabolism in liver(3)? -Conjugation -Polar
product

Explain differences
- Act on same receptor -
between full and partial
Full has greater efficacy
agonists(2).

- partial agonist can have


Explain potency in relation increased, decreased, /
to full and partial agonists A21or equal potency as
(2). full agonist. - Potency is
an independent factor.
- ED 50 is less than the
How do spare receptors
Km (less than 50% of
effect the Km?
receptors)

How do you calculate Md= (CpxCL)/F Cp= plas.


maintenance dose? Conc. CL=clear. F=bioaval.

How does a competitive


-Shifts the curve to the
antagonist effect an
right -increases Km
agonist?

How does a
- Shifts the curve down -
noncompetitive antagonist
reduces Vmax
effect an agonist?
-Phase I (clinical tests) -
Name the steps in drug
Phase II -Phase III -PhaseIV
approval(4)?
(surveillance)

Steady state concentration In 4 half-lifes= (94%) T1/2


is reached in __#half-lifes = (0.7x Vd)/CL

What is the definition of -Constant AMOUNT


zero-order kinetics? eliminated per unit time. -
Example? Etoh &amp;ASA

What is the formula for CL= (rate of elimination of


Clearance (CL) drug/ Plasma drug conc.)
What is the formula for Vd= (Amt. of drug in
Volume of distribution (Vd) body/ Plasma drug conc.)

Ld= (CpxVd)/F
What is the loading dose
Cp=plasma conc. F=
formula?
Bioaval.

A 12yo patient was treated


for a reaction to a bee
sting, what drug provides Epinephirine(Alpha1,2 and
the best coverage of Beta 1,2)
sympathomimetic
receptors?

A 57 yo heart failure pt
develops cardiac
decompensation, what
drug will give you Dopamine
adequate perfusion of his
kidneys as well as tx for
his Hypotension
A fellow passenger on a
Carnival cruise ship looks
pale and diaphoretic, what scopolamine
antimuscarinic agent
would you give them?

A group of pts are rushed


into the ER complaining of Atropine pts are suffering
excessive sweating, tearing, from Cholinestrase
salivation, HA, N and V, inhibitor poisining(Nerve
muscle twitching, difficulty
gas/Organophosphate
breathing and diarrhea. What
drug would be the most
poisining)
effective immediate tx

As an Anes you want to


use a depolarizing
neuromuscular blocking Succinylcholine
drug on your pt, what do
you use

By what mechanism does Prevents the release of Ca


this drug help from SR of skeletal muscle
Centrally acting alpha
Clonidine is the preferred
agonist, thus causing a
sym pathomimetic tx of
decrease in central
HTN in pts with renal
adrenergic outflow,
disease, why??
spairing renal blood flow

Cocaine casues
vasoconstriction and local Indirect agonist, uptake
anesthesia by what inhibitor
mechanism

Cocaine shares is
mechanism of action with TCA
what antidepressant

Dobutamine used for the


tx of shock acts on which Beta1 more than B2
receptors
Guanethidine enhances the No, it inhibits the release
release of Norepi? of Nor Epi

How does angiotensin II It acts presynaptically to


affect NE release? increase NE release.

Prevents the release of


How does botulinum toxin
ACh, which results in
result in respiratory arrest?
muscle paralysis.

Prevents the release of


How does dantrolene calcium from the
work? sarcoplasmic reticulum of
skeletal muscle.
NE acts presynaptically on
alpha-2 receptors to
How does NE modulate its
inhibit its own release.
own release? What other
ACh also acts
neurotransmitter has this
presynaptically through
same effect?
M1 receptors to inhibit NE
release.

Hemicholinium inhibits the


How would hemicholinium
transport of choline into
treatment affect
the nerve, thus inhibiting
cholinergic neurons?
formation of ACh.

How would you reverse the Give an antichloinesterase


effect of a neuromuscular - neostigmine,
blocking agent? edrophonium, etc

It would increase to ~ 100


beats/min. Both sympathetic
If a patient is given and vagal stimulation would
hexamethonium, what be knocked out, but the SA
would happen to his/her node has an intrinsic pace of
heart rate? 100 beats/min, which is
normally checked by vagal
stimulation.
Isopoterenol was given to
Stimulates beta adrenergic
a patient with a developing
receptors
AV block, why?

Norepi feedbacks and


Binding to the presynaptic
inhibits the presynaptic
alpha 2 release
receptor by what
modulating receptors
mechanism

Reserpine will block the


Blocks Norepi, but not
syntheis of this drug and
Dopamine
but not its precursor.

These drugs acts indirectly


by releasing strored Amphetamine and
catecholamines in the Ephedrine
presynaptic terminal
What anticholinesterase
crosses the blood-brain- physostigmine
barrier?

What antimuscarinic agent


is used in asthma and Ipratropium
COPD?

What antimuscarinic drug


is useful for the tx of Ipratropium
asthma

Diarrhea, Urination,
What are the classic Miosis, Bronchospasm,
symptoms of Bradycardia, Excitation of
cholinesterase inhibitor skeletal muscle and CNS,
poisoning (parathion or Lacrimation, Sweating, and
other organophosphates)? Salivation = DUMBBELS;
also abdominal cramping
Activates cholinergic
receptors on bladder and
What are the clinical
bowel smooth muscle,
indications for
alleviating post-op and
bethanechol?
neurogenic ileus and
urinary retention.

Post-op and neurogenic


ileus and urinary retention,
What are the clinical myasthenia gravis, and
indications for reversal of neuromuscular
neostigmine? junction blockade (post-
op) through
anticholinesterase activity.

narcolepsy, obesity, and


What are the indications
attention deficit disorder (I
for using amphetamine?
wouldn't recommend this)

What are the


Tubocurarine, atracurium,
nondepolarizing
mivacurium, pancuronium,
neuromuscular blocking
vecuronium, rapacuronium
drugs?
Phase 1 = prolonged
depolarization, no antidote,
What are the phases of effect potentiated by
succinylcholine anticholinesterase; Phase 2
neuromuscular blockade? = repolarized but blocked,
an anticholinesterase is the
antidote for this phase.

What are two indirect


amphetamine and
acting adrenergic
ephedrine
agonists?

What beta 2 agonist will


Albuterol, tertbutaline
help your 21yo Astma pt?

What cholinergic inhibitor


acts by directly inhibiting Botulinum
Ach release at the
presynaptic terminal
What cholinomimetic is
useful in the diagnosis of Edrophonium
Myasthenia Gravis

What cholinomimetics Carbachol, pilocarpine,


might your pt be taking physostigmine,
for his glaucoma echothiophate

What class of drug is


anticholinesterase
echothiophate? What is its
glaucoma
indication?

In treatment of malignant
hyperthermia, due to
What conditions would you concomitant use of halothane
use dantrolene? and succinylcholine. Also in
neuroleptic malignant
syndrome, a toxicity of
antipsychotic drugs.
What drug is used to edrophonium (extremely
diagnose myasthenia short acting
gravis? anticholinesterase)

Neostigmine,
pyridostigmine
What drugs target this
edrophonium
enzyme
physostigmine
echothiophate

Theoretically it could be
What effect would atropine
used to block the cephalic
have on a patient with
phase of acid secretion
peptic ulcer disease?
(vagal stimulation).

What effect would atropine None. No, because


have on the preganglionic atropine would block the
sympathetic activation of postganglionic muscarinic
sweat glands? Would this receptors involved in
person sweat? sweat gland stimulation.
What enzyme is
Acetylcholinesterase; ACh
responsible for the
is broken down into
breakdown of ACh in the
choline and acetate.
synaptic cleft?

What enzyme is
responsible for the Acetylcholine esterase
degredation of Ach

What enzyme is
responsible for the Choline acetyltransferase
production of Ach from
Acetyl CoA and Choline

Treatment of
hypertension, especially
What is the clinical utility
with renal disease (lowers
of clonidine?
bp centrally, so flow is
maintained to kidney).
The only local anesthetic
What is the clinical utility
with vasoconstrictive
of cocaine?
properties.

Dobutamine has more of an


What is the difference affintiy for beta-1 than
between the affinity for beta-2, and is used for
beta receptors between treating heart failure and
shock. Albuterol and
albuterol/terbutaline and
terbutaline is the reverse, and
dantroline? is used in treatment of acute
asthma.

What is the difference in Prefers beta's at low


receptor affinity of doses, but at higher doses
epinephrine at low doses? alpha agonist effects are
High doses? predominantly seen.

Increased systolic and


What is the effect of pulse pressure, decreased
epinephrine infusion on bp diastolic pressure, and
and pulse pressure? little change in mean
pressure.
What is the effect of
guanethidine on It inhibits release of NE.
adrenergic NE release?

Increases mean, systolic,


What is the effect of
and diastolic bp, while
norepinephrine on bp and
there is little change in
pulse pressure?
pulse pressure.

They inhibit reuptake of


What is the effect of TCA's
NE at the nerve terminal
on the adrenergic nerve?
(as does cocaine).

What is the only


depolarizing Succinylcholine
neuromuscular blocking
agent?
What is the receptor It affects beta receptors
affinity and clinical use of equally and is used in AV
isoproterenol? heart block (rare).

It antagonizes Ach M
What makes this drug
receptors and decreases
effective
parasym (GI) rxn

What nondepolorizing
Tubocurarine, atra-, miv-,
agents could you have
pan-,ve-, rapacuronium
used

What other substances


regulate the Norepi nerve Ach, AngiotensinII
ending
What other syndrome can Neuroleptic malignant
this drug tx syndrome

SLUD (salivation,
What physiological effects Lacrimation, urination,
was the Anes using Defecation)as well as
Atropine to tx airway secretion, GI
motility, acid secretions

What reversal agent could


Bethanechol, Neostigmine,
a Anes give to reverse the
physostigmine
effects of Atropine

Atropine would also block


What side effect of using the receptors in the ciliary
atropine to induce muscle, causing an
pupillary dilation would impairment in
you expect? accommodation
(cycloplegia).
What sympathomimetic
would you not prescribe Norepinephrine (Alpha1,2
for hypotension in a pt and beta 1)
with renal artery sclerosis.

Hexamethonium is a
What type of neurological
nicotinic antagonist, and
blockade would
thus is a ganglionic
hexamethonium create?
blocker.

Initially vasoconstriction
would increase bp, but
What would be the effect
then it acts on central
on blood pressure with
alpha-2 receptors to
infusion of the alpha -2
decrease adrenergic
agonist clonidine?
outflow resulting in
decreased bp.

What would be the next


Pralidoxime, regenerates
drug that you would give
active cholinestrase
and why
Which antimuscarinic
agents are used in atropine, homatropine,
producing mydriasis and tropicamide
cycloplegia?

Which drug increases Sys


BP w/o affecting Pulse Epinephrine
Pressure

Which of epi, norepi, or


isoproterenol results in Norepinephrine
bradycardia?

Dry flushed skin, due to


Which of the following
inhibition of sympathetic
would atropine
post-ganglionic blockade
administration cause?
on muscarinic receptors of
Hypothermia, bradycardia,
sweat glands. All others
excess salivation, dry
are opposite of what
flushed skin, or diarrhea
would be expected.
Which of these three drugs
will cause a reflex
bradycardia in your pt Norepinephrine
(Norepi, Epi, or
Isoporterenol)

alpha-1 &gt; alpha-2;


Which receptors does used as a pupil dilator,
phenylephrine act upon? vasoconstrictor, and for
nasal decongestion

While at a tail gait party,


Epinephrine to treat
you bite into a sandwich
anaphylaxis. Also useful if
that a yellow jacket is also
you have open angle
enjoying. Knowing your
glaucoma, asthma, or
allergy to this creature,
hypotension.
what should you do?

Why are albuterol and These B-2 agonists cause


terbutaline effective in tx respiratory smooth muscle
of acute asthmatic attacks? to relax.
Blocking muscarinic
receptors in the circular
Why does atropine dilate
fibers of the eye, results in
the pupil?
unopposed action of radial
muscles to dilate.

NE increases bp, which


stimulates baroreceptors
Why does NE result in in the carotid sinus and
bradycardia? the aorta. The CNS signals
through vagal stimulation
to decrease heart rate.

They activate the ciliary


Why is carbachol and
muscle of the eye (open
pilocarpine useful in
angle) and pupillary
treatment of glaucoma?
sphincter (narrow angle).

As an anticholinesterase it
Why is pyridostigmine
increases endogenous ACh
effective in the treatment
and thus increases
of myasthenia gravis?
strength.
Reserpine inhibits
dopamine transport into
Why is reserpine effective vesicles, attenuating its
in treating HTN? conversion to NE by
dopamine beta-
hydroxylase.

Stimulating beta receptors


Why is there a drop in
stimulates heart rate, but
systolic, mean, and
beta receptor induced
diastolic bp with infusion
vasodilation reduces
of isoproterenol?
peripheral resistance.

Parkinson patients benefit


Why would a patient with
from antimuscarinic
cog-wheel rigidity and a
agents through its
shuffling gait be given
inhibitory action within the
benztropine?
indirect pathway.

Receptors =
D1=D2&gt;beta&gt;alpha,
Why would dopamine be thus increasing heart rate
useful in treating shock? (beta) and blood pressure
(alpha vasoconstriction) while
maintaining kidney perfusion
(dopamine receptors)
Why would you give a drug Useful in muscle paralysis
like pancuronium or during surgery or
succinylcholine? mechanical ventilation.

Why would you use


pralidoxime after Pralidoxime regenerates
exposure to an active cholinesterase.
organophosphate?

Will Hemicholinum affect


No, hemicholinum block
the release of stored Ach
the uptake of Choline and
during Cholinergic
thus Ach synthesis
Stimulation

No. Atropine is used to


Would blockade of
reduce urgency in mild
muscarininc receptors in
cystitis. So it would
the bladder be useful in
aggravate the urinary
treating urinary retention?
retention.
No, hexamethonium
targets Nicotinc receptors
Would Hexamethonium be
and will block Parasym,
an effective substitute
Sym, as well as Somatic
systems

You tx your pt with


halothane as well and he
has also developed Dantrolene
malignant hypothermia,
what drug can you give

Your patient develops a


No cholinesterase
marked arrythmia due to a
inhibitors will potentiate
prolonged depolarization,
the stimulating action of
can you tx this w/
Succinlycholine
Neostigmine

Your patient has acute


Yes, Scopolamine would
angle glaucoma, does this
antagonize his glaucoma
affect your tx
Your patient wants an
effective drug to treat his Scopolamine
motion sickness, what
would you prescribe
At what concentration is the
transport mechanism for 300 mg/dL
glucose saturated?

Define effective renal plasma ERPF = U (PAH) x V/P (PAH) =


flow. C (PAH)

Define filtration fraction. FF = GFR/ RPF

Define free water clearance. C(H2O) = V- C(osm)


GFR = U(inulin) x V/P (inulin) =
C (inulin) GFR also equals the
difference in (osmotic pressure
Define GFR. of the glomerular capillary
minus Bowman's space) and
(hydrostatic pressure of the
glomerular capsule minus
Bowman's space).

Define renal blood flow. RBF = RPF/1 - Hct

Cx = UxV/Px The volume of


plasma from which the
Define renal clearance.
substance is cleared
completely per unit time.

V = urine flow rate C (osm) =


Define urine flow rate.
U(osm)V/P(osm)
Reabsorption occurs by at
How are amino acids cleared least 3 distinct carrier
in the kidney? systems, with competitive
inhibition within each group.

By inhibiting the production


of prostaglandins which
How do NSAIDs cause renal
normally keep the afferent
failure?
arterioles vasodilated to
maintain GFR

How high can the osmolarity


1200-1400 mOsm
of the medulla reach?

How is ICF measured? ICF = TBW - ECF


How is interstitial volume
Interstitial volume = ECF - PV
measured?

How is PAH secreted? Via secondary active transport

How is PAH transport Mediated by a carrier system


mediated? for organic acids

How much of the ECF is


Three-fourths
interstitial fluid?
How much of the ECF is
One-fourth
plasma?

How much of the total body


water is part of intracellular Two-thirds
fluid?

How much of the total body


water is part of the One-third
extracellular fluid?

If clearance of substance X is There is no net secretion or


equal to GFR, what occurs? reabsorption
If clearance of substance X is
greater than GFR, what Net tubular secretion of X
occurs?

If clearance of substance X is
Net tubular reabsorption of X
less than GFR, what occurs?

T/F. Secondary active


transport of amino acids is TRUE
saturable.

1. Fenestrated capillary
endothelium 2. Fused
What 3 layers form the basement membrane with
glomerular filtration barrier? heparan sulfate 3. Epithelial
layer consisting of podocyte
foot processes
-Increase water permeability
of principle cells in collecting
What actions does ADH have ducts -Increase urea
on the kidney? absorption in CD -Increase
Na/K/2Cl transporter in the
thick ascending limb

-Contraction of efferent
arteriole increasing GFR -
What actions does AII have on
Increased Na and HCO3
the kidney?
reabsorption in proximal
tubule

-Increased Na reabsorption in
What actions does aldo have distal tubule -Increased K
on the kidneys? secretion in DT -Increased H
ion secretion in DT

What actions does ANP have -Decreased Na reabsorption -


on the kidney? Increased GFR
-Increased Ca reabsorption -
What actions does PTH have Decreased phosphate
on the kideny? reabsorption -Increase 1,25-
(OH)2 Vit D production

What activates 1 alpha-


PTH
hydroxylase?

1. Vasoconstriction 2. Release
of aldo from adrenal cortex 3.
What are the 4 actions of Release of ADH from
angiotensin II? posterior pituitary 4.
Stimulates hypothalamus to
increase thirst

1. EPO release 2. Vitamin D


What are the 4 endocrine
conversion 3. Renin release 4.
functions of the kidney?
Prostaglandins release
-Albuminuria -
What are the consequences of Hypoproteinemia -
a loss in the charge barrier? Generalized edema -
Hyperlipidemia

What competitively inhibits


Probenecid
the carrier system for PAH?

What constricts the efferent


Angiotensin II
arteriole?

What dilates the renal afferent


Prostaglandins
arteriole?
What do the collecting ducts
reabsorb in exchange for K or Na ions
H?

Cleave angiotensinogen into


What does renin do?
angiotensin I

What does the anterior -FSH and LH -ACTH -GH -


pituitary secrete? TSH -MSH -Prolactin

What does the beta subunit The beta subunit determines


do? hormone specificity
What does the early distal
convoluted tubule actively -Na ions -Cl ions
reabsorb?

What does the posterior


ADH and oxytocin
pituitary secrete?

What does the secretion of


Vasodilates the afferent
prostaglandins from the
arterioles to increase GFR
kidney do?

What does the thick


ascending loop of Henle -Na ions -K ions -Cl ions
actively reabsorb?
What does the thick
descending loop of Henle -Mg ion -Ca ions
indirectly reabsorb?

What effect does constriction -Decreased RPF -Increased


of the efferent arteriole have? GFR -FF increases

What effect does dilation of -Increased RPF -Increased


the afferent arteriole have? GFR - FF remains constant

What enzyme converts 25-OH


1alpha-hydroxylase
Vit D to 1,25-(OH)2 Vit D?
What happens to glucose in Glucose is completely
the kidneys when glucose is reabsorbed in the proximal
at a normal level? tubule.

1. ADH 2. Aldosterone 3.
What hormones act on the
Angiotensin II 4. Atrial
kidney?
natriurtic Peptide 5. PTH

What inhibits constriction of


ACE inhibitors
the efferent arteriole by AII?

What inhibits dilation of the


afferent arteriole by NSAIDS
prostaglandins?
What is an important clinical
Glucosuria
clue to diabetes?

To increase intravascular
What is angiotensin II's
volume and increase blood
overall function?
pressure

What is passively reabsorbed Water via medullary


in the thin descending loop of hypertonicity (impermeable to
Henle? sodium)

What is reabsorbed in the


early distal tubule under the Ca ions
control of PTH?
Reabsorbs all of the glucose
What is the function of the
and amino acids and most of
early proximal convoluted
the bicarbonate, sodium, and
tubule?
water

What is the oncotic pressure


Zero
of Bowman's space?

What is the thick ascending


loop of Henle impermeable Water
to?

What is the threshold for


glucose reabsorption in the 200 mg/dL
proximal tubule?
What may act as a 'check' on
the renin-angiotensin system ANP
in heart failure?

What part of the nephron Early proximal convoluted


secretes ammonia? tubule

What part of the pituitary is


Posterior pituitary
derived from neuroectoderm?

What percentage of the body


0.6
is water?
What regulates the
reabsorption of water in the ADH
collecting ducts?

What secretes renin? JG cells

-Increased plasma osmolarity


What stimulates ADH
-Greatly decreased blood
secretion?
volume

-Decreased blood volume (via


What stimulates aldosterone
AII) -Increased plasma K
secretion?
concentration
What stimulates angiotensin Decreased blood volume (via
secretion? renin)

What stimulates ANP


Increased atrial pressure
secretion?

What stimulates EPO release? Hypoxia

What stimulates PTH Decreased plasma ca


secretion? concentration
1. Decreased renal arterial
What stimulates renin pressure 2. Increased renal
release? nerve discharge (Beta 1
effect)

What subunit do TSH, LH, FSH


Alpha subunit
and hCG have in common?

What symptom is present


Glucosuria
once threshold is reached?

What type of tissue is the


anterior pituitary derived Oral ectoderm
from?
What value is used clinically
Creatinine clearance
to represent GFR?

What variables are needed to -Urine flow rate -Urine


calculate free water osmolarity -Plasma
clearance? Osmolarity

Where does ACE convert AI to


Primarily the lung capillaries
AII?

Where does secondary active


transport of amino acids In the proximal tubule
occur?
Endothelial cells of the
Where is EPO secreted? peritubular capillaries
(kidney)

Where is paraaminohippuric
Proximal tubule
acid secreted?

Which barrier is lost in


Charge barrier
nephrotic syndrome?

Which layer filters by negative


Fused basement membrane
charge?
Fenestrated capillary
Which layer filters by size?
endothelium

Why does the nephron Acts As a buffer for secreted


secrete ammonia? H ions

Because it is freely filtered


Why is inulin sued to measure
and is neither absorbed or
GFR?
secreted

Why is PAH used to calculate


PAH is secreted and filtered.
RPF?
Decreased cortisol and
A 21-Beta-hydroxylase
mineralocorticoids
deficiency will result in what
(hypotension, hyperkalemia)
hormone deficiencies/
Increased sex hormones
excesses?
(masculinization)

A deficiency of 17-alpha
Decreased sex hormones and
hydroxylase will result in an
cortisol
decrease in what hormone(s)?

A deficiency of 17-alpha
Aldosterone Produces
hydroxylase will result in an
hypertension, hypokalemia
increase in what hormone(s)?

A dopaminergic antagonist
would be expected to have
Stimulates prolactin secretion
what effect prolactin
secretion?
A maturing graafian follicule
During the proliferative phase
can be found at what stage of
(Around Day 7)
the menstrual cycle?

Stimulates aldosterone
Angiotensin II has what effect production by enhancing the
on the adrenal cortex? activity of aldosterone
synthase

Calcitonin's actions Oppose. Calcitonin acts faster


(synergize/oppose) the than PTH to decrease serum
actions of PTH. Ca2+ levels.

Decreased cortisol levels as in


ACTH levels will be increased
any of the congenital adrenal
contributing to increased skin
hyperplasias will have what
pigmentation
effect on ACTH?
Decreased phosphate will
Increased activated Vit D.
have what effect on Vit D?

Degenerated. Shortly after


During the 2nd and 3rd
the first trimester, the
trimester, one would expect
placenta makes estriol and
the corpus luteum to be?
progesterone.

Estradiol is converted from


Aromatase converts
what precursor by what
Testosterone to Estradiol.
enzyme?

Estrogen is produced in what Corpus luteum, placenta,


4 locations in the body? adrenal cortex, and testes
Estrogen levels are low/med/ High. Estrogen switches to
high during the just before positive feedback of LH from
the peak of the LH surge? negative so both increase.

Complex effects. Early on


Estrogens have what effect of estrogen has a negative effect
LH secretion? that switches to positve just
before the LH surge.

Estrogens have what effect of Estrogens stimulate growth of


the follicle? the follicle

Failure of brain maturation


due to lack of thyroid Cretinism
hormone is known as?
Finasteride inhibits what step Converstion of testosterone
in testosterone metabolism? to DHT by 5-alpha reductase

Follicular growth is fastest During the second week od


during what part of the the proliferative phase (Days
menstrual cycle? 7-14)

FSH stimulates what cells in Sertoli cells


the male? (spermatogenesis)

Decreased Ca2+ will increase


Hypocalcemia will have what PTH which will stimulate the
effect on Vit D metabolism? kidney to produce more
activated Vit D.
In addition to peripheral
conversion, DHT is also Prostate
produced in the?

In what organ is Vitamin D3 The skin. Vit D requires sun


produced? exposure (UV light and heat)

Is testosterone considered to
be anabolic or catabolic Anabolic
overall?

LH levels would be low/med/


Low. The LH surge has
high at the time of ovulation
already declined
(Day 14)
LH stimulates what cells in Leydig cells (testosterone
the male? synthesis)

Brain and RBC's take up


Name the two primary insulin
glucose independent of
independent organs?
insulin

Order the following with the


most potent first: DHT &gt; testosterone &gt;
testosterone, androstenedione
androstenedione. DHT

Order the following with the


Estradiol &gt; estrone &gt;
most potent first: estrone,
estriol
estradiol, estriol.
Phosphate reabsortion in the
kidneys is inhibited by what PTH
hormone?

Progesterone has what effect


Increases body temperature
on body temperature?

To decrease the risk of


Progestorone is used in
endometrial cancer
combination with estrogen
associated with unopposed
for what reason?
estrogen therapy

Prolactin inhibits ovulation by


Prolactin has what effect on inhibiting the release/
ovulation? synthess of GnRH from the
hypothalamus
PTH causes increased calcium
reabsorption in what part of DCT
the kidney?

PTH is produced by what cell Chief cells of the parathyroid


type? glands

Androgen-binding protein
Sertoli cells stimulate (ABP) - concentrates
spermatogenesis by testosterone in the
producing what 2 factors in seminiferous tubules Inhibin
response to FSH? - inhibits FSH secretion fro
the ant pit

False. Thyroid hormones


T/F - Glycolisis is promoted increase blood glucose levels
by the thyroid hormones by stimulating glycolgenolysis
and gluconeogenesis.
T/F - PTH stimulates both
True
osteoclasts and osteoblasts?

False. 5-alpha reductase


T/F - Testosterone is the
activates testosterone to DHT
most active androgen in
which is the most active
males and females?
androgen.

Testosterone acts as a
negative inhibitor on what GnRH
hormone from the brain?

Testosterone is synthesized
Testis Adrenal Cortex
in what two locations?
The hormone with the highest
concentration during the Progesterone
secretory phase is?

Dopamine secreted from the


The key inhibitor of prolactin hypothalamus Bromocriptine
release is? (Dopamine agonist has the
same effect)

The parathyroid glands come


The 3rd and 4th pharyngeal
from what embryonic
pouches
structures?

The primary estrogen


Estradiol
produced by the ovary is?
The primary estrogen
Estriol
produced by the placenta is?

Thick mucous production is


Progesterone Decreases
the result of what sex
sperm entry into the uterus
hormone?

Throid Stimulating
Graves Disease
Immunoglobulin results in
(hyperthroidism)
what disease?

Thyroid hormones acts


synergistically with what
GH
hormone with respect to bone
growth?
TRH is produced in what
Hypothalamus
region of the brain?

TSH levels in a hypothroid Elevated TSH Decreased free


patient would be? Free T4? T4

Progesterone decreases
Unlike estrogen, what effect
myometrial excitability to
does progesterone have on
help maintain the pregnancy/
the myometrium?
facilitate fertilization

Vit D deficiency in kids cause Rickets in kids Osteomalacia


what disease? Adults? in adults
What 2 conditions other than Hydatidiform moles in women
pregnancy increase hCG? or choriocarcinoma

What adrenergic effects do


Beta-adrenergic effects
the thyroid hormones have?

HAVOC H = Hot flashes A V =


What are the symptoms of Atrophy of the Vagina O =
menopause? Osteoporosis C = Coronary
Artery Disease

Parafollicular cells (C cells) of


What cells produce calcitonin?
the thyroid
What does an elevated
Ovulation
progesterone level indicate?

During puberty, testosterone


What effect do androgens
stimulates bone growth but
have on growth of long
eventually causes closure of
bones.
the ephyseal plates

What effect do estrogens have Stimulate endometrial


on the endometrium? proliferation Increase
Myometrium? myometrial excitability

What effect do estrogens have Increase hepatic synthesis of


on the liver? transport proteins
What effect do the thyroid
hormones have on cardiac Thyroid hormones increase:
output? Heart rate? CO HR SV contractility and RR
Contractility? Stroke Volume?
Respiratory Rate?

What effect does Ca2+ have Stimulates bone resorption of


on bone? calcium.

What effect does


Progesterone is inhibitory to
progesterone have on FSH?
both gonadotrophins
On LH?

Progesterone stimulates the


What effect does endometrial glands to
progesterone have on the become secretory and
endometrium? increases spiral artery
development
What effect does PTH have on Increases bone resorption of
bone? Ca2+ and phosphate

What effect does thyroid


Lipolysis is stimulated
hormone have on lipolysis?

The kidney will produce more


1-25-OH2 Vit D which will
What effect will low serum increase phosphate release
phosphate have the kidney? from bone matrix and
increase Ca2+ and phosphate
absorption in the GIT

What enzyme deficiency will 11-Beta hydroxylase


produce BOTH hypertension deficiency 11-
and masculinization of deoxycorticosterone will act
females? as a mineralocorticoid
PTH stimulates 1-alpha-
What enzyme in the kidney is
hydroxylase cause increased
stimulated that affects
production of 1,25-(OH)2
vitamin D metabolism?
vitamin D.

What happens to the corpus


The corpus luteum regresses
lutem if progesterone levels
and menstration occurs
fall without fertilization?

The corpus luteum is


What happens to the corpus maintained by hCG acting like
lutem if progesterone levels LH which maintains both
with fertilization? estrogen and progesterone
levels.

What hormonal changes are


Decreased estrogen Increased
seen with untreated
FSH (Greatly) Increased LH
menopause with respect to
(No surge) Increased GnRH
estrogen, FSH, LH, GnRH?
What hormone predominates
during the secretory phase of Progesterone
the menstrual cycle?

Decrease in free serum Ca2+


What is the key regulator of increases PTH secretion.
PTH secretion? Increased Ca2+ feedback
inhibits PTH secretion.

What is the key regulator that


increases Calcitonin Increased serum Ca2+
secretion?

What is the most common


21-Beta hydroxylase
cause of congenital adrenal
deficiency
hyperplasia?
What is the physiologic The syncytiotrophoblasts of
source of hCG? the placenta

What is the primary organ


that converts Vit D to 25-OH Liver
Vit D?

What is the primary source of


Adrenal glands
androstenedione?

What is the role of calcitonin Probably not important as


in normal calcium PTH is the primary regulator
homeostasis? of calcium homeostasis.
What is the VERY first
molecule in the pathway for
Cholesterol
the synthesis of Aldosterone?
Cortisol? Adrenal androgens?

Cessation of estrogen
What is thought to be the
production due to decline in
cause of menopause?
the number of follicles

PTH increases serum Ca 2+,


What overall effects does PTH
decreases serum phosphates,
have on body electolytes?
increases urine phosphates

What signal from the body


Thyroid hormones, T3
decreases TRH secretion?
What substance is used by
the brain for energy during Ketone bodies
starvation?

What will the levels of Ca2+,


Increased Ca2+, decreased
phosphate, and alkaline
phosphate, increased alkaline
phosphatase be in
phosphatase
hyperparathyroidism?

What will the levels of Ca2+,


No changes in Ca2+,
phosphate, and alkaline
phosphate, or alkaline
phosphatase be in
phosphatase
osteoporosis?

What will the levels of Ca2+,


Alkaline phosphatase
phosphate, and alkaline
increased with normal Ca2+
phosphatase be in Paget's
and phosphate
disease of bone?
What will the levels of Ca2+,
Decreased Ca2+, increased
phosphate, and alkaline
phosphate, and alkaline
phosphatase be in renal
phosphates WNL
insufficiency?

What will the levels of Ca2+,


Increased Ca2+ and
phosphate, and alkaline
phosphate with alkaline
phosphatase be in Vit D
phosphatase WNL
intoxication?

Which ducts (Mullerian or Wolfian ducts are


Wolfian) are promoted by differentiated into the internal
androgens? gonadal structures.

It is detectable in the blood


Why is hCG so useful for
and urine 8 days after
detecting pregnancy?
successful fertilization.
Decrease hot flashes and
decrease bone loss.
Why is hormone replacement
Decreased risk of heart
therapy used in
disease could be on the
postmenopausal women?
boards but is no longer true
(2001).

Bound to specific binding


Will most steroids in the
globulins Steroids are
blood be bound or unbound?
lipophilic

Elevated Thyroid hormone


You would expect the body increases Na/K ATPase
temperature of a patient with activity =&gt; increased
hyperthroidism to be? consumption of O2 =&gt;
increased temp

Causes hypoxic
A decrease in PA O2 will have
vasoconstriction that shifts
what effect on the pulmonary
blood awayfrom poorly
vasculature?
ventilated regions
A value of infinity for V/Q
Blood flow obstruction
indicates?

A ZERO value for V/Q


Airway obstruction
indicates?

Bicarbonate in the RBC is


transported out of the cell in Cl- by a HCO3-/Cl- antiport
exchange for what ion?

Cor pulmonale is the result


Pulmonary hypertension
of?
Right ventricular failure
Cor pulmonale will lead to
(jugular venous distention,
what condition of the heart?
edema, hepatomegaly)

Dissociation of CO2 from Hb


upon oxygenation in the The Haldane effect
lungs is known as?

Exercise (increased cardiac The V/Q will approach 1


output) will have what effect (from 3) as a result of dilation
on V/Q to the apex? of vessels in the apex.

In the apex of the lung, V/Q V/Q &gt; 1. NL = 3 which


should be &gt;1, =1, or &lt;1? indicates wasted ventilation.
In the base of the lung, V/Q V/Q &lt; 1. NL = 0.6 which
should be &gt;1, =1, or &lt;1? indicates wasted perfusion.

In the perpheral tissue what


factor helps unload oxygen Increased H+ (decreased pH)
by shifting the curve to the a.k.a. the Bohr effect
right?

Increased 2,3-DPG will cause


The curve will shift RIGHT.
a shift in what direction of
This allows Hb to release
the oxygen-Hb dissociation
more oxygen
curve?

Increased erythropoietin
levels as a response to high
Increase hematocrit and Hb
altitudes will have what affect
on the blood?
Neonatal respiratory distress
Surfactant (dipalmitoyl
syndrome is due to a
phosphatidylcholine, lecithin)
deficiency of what?

Both ventilation and perfusion


Perfusion is greatest in what
are greater at the base than
part of the lung?
at the apex.

Recurrent TB grows best in


Apex because of high O2.
what part of the lung? Why?

Surfactant role in the lungs is Decrease alceolar surface


to do what? tension
T/F - The pulmorary
circulation is a high F. It has low resistance and
resistance, low compliance high compliance.
system.

The conversion of CO2 to


H2CO3 (Carbonic acid) is
Carbonic Anhydrase
catalyzed by what RBC
enzyme?

The kidneys would do what to


compensate for respiratory
Excrete bicarbonate
alkalosis as a response to
high altitude?

HCO3- (bicarbonate)
The predominant form of
accounts for 90%, followed by
CO2 transport from the
Hb bound CO2 (5%) and
tissues to the lungs is?
dissolved CO2 (5%)
TV+IRV+ERV = ? TV = tidal
Vital capacity. VC is
volume, IRV = inspirartory
everything but the residual
reserve volume, ERV =
volume.
expiratory reserve volume

Both ventilation and perfusion


Ventilation is greatest in what
are greater at the base than
part of the lung?
at the apex.

Decrease metabolic needs,


What 6 factors decrease O2
dcr PCO2, dcr temperature,
affinity to Hb/decrease P50?
increased pH, dcr 2,3-DPG,
What direction does the O2-
and Fetal Hb The curve shifts
Hb dissociation curve shift?
LEFT.

What are some potential side Cough and angioedema due


effects of ACE inhibitors? to decreased bradykinin
What cellular change could
you expect as a response to Increased mitochondria
high altitude?

What enzyme in the lungs is a Angiotensin-converting


key enzyme in the renin- enzyme (ACE) which converts
angiotensin system? Ang I to Ang II

What is expiratory reserve Air that can still be breathed


volume? out after normal expiration

FRC is the flume in the lungs


What is FRC? How is it
after normal respiration and
calculated?
is the sum of RV +ERV.
Air in excess of the tidal
What is inspiratory reserve volume that moves into the
volume? lungs with maximum
inspiration

Air in the lung at maximal


What is residual volume?
expiration

What is the bodies acute


reponse to a change from low Increase in ventilation
to high altitude?

What is the difference


Capacities are the sum of
between capacites and
&gt;= 2 volumes.
volumes in the lung?
What is the Total Lung IRV + TV + ERV + RV or VC +
Capacity? Normal Value? RV Normal would be ~ 6.0 L

Air that moves into the lung


What is tidal volume? What is
with each quiet expiration.
a normal TV value?
500 mL is normal

What would be the effect on


the heart due to chronic
hypoxic pulmonary Right ventricular hypertrophy
vasoconstriction (High
altitude)?

Would you expect acidosis or


alkalosis due as a response to
Respiratory alakalosis
high altitude? Metabolic or
Respiratory?
Exocrine secretion of
zymogens by secretory acini -Acetylcholine -CCK
is stimulated by what?

1. Increase production of
saliva 2. Increase gastric H+
Five effects of secretion 3. Increases
Parasympathetic GI pancreatic enzyme and
Innervation: HCO3- secretion 4. Stimulates
evteric nervous system to
creat intestinal peristalsis 5.
Relaxes sphincters

-Mucus (Mucous cell) -


Five main components of Intrinsic factor (Parietal cell) -
gastric secretions and their H+ (Parietal cell) -Pepsinogen
sources? (Chief cell) -Gastrin (G cell in
antrum and duodenum)

1. Proton pump inhibitors


(omeprazole) 2. H2 receptor
Four categories of drugs that antagonists (Rantidine,
inhibit/decrease secretion of Cimetidine, Famotidine) 3.
gastric acid: Anticholinergics 4.
Prostaglandin receptor
antagonists (Misoprostol)
1. Increase production of
saliva 2. Decreases
Four effects of Sympathetic GI splanchnic blood flow in
Innervation: fight-or-flight response 3.
Decreases motility 4.
Constricts Sphincters

-Kills bacteria -Breaks down


food -Lowers pH to optimal
Four functions of H+ secreted
range for pepsin function
in the stomach?
(conversion of pepsinoget) -
Sterilizes chyme

1. Inhibits Gastric acid and


pepsinogen secretion 2.
Four functions of Inhibits pancreatic and small
Samatostatin? intestine fluid secretion 3.
Gallbladder contraction 4.
Release of both insulin and
glucagon

From what cells is bile


hepatocytes
secreted?
Stimulates secretion of HCl,
Function of Gastrin secreted
IF, and pepsinogen (also
in the stomach?
stimulates gastric motility)

Binding protein required for


Function of Intrinsic factor
vitamin B12 absorption (in
secreted in the stomach?
terminal ileum)

-Limit fat intake -Monitor for


How do you treat Pancreatic
signs of fat-soluble vitamin
Insufficiency?
(A,D,E,K) deficiency

How does jaundice manifest


yellow skin and sclerae
in the body?
How much urobilinogen is
4mg
secreted per day?

In what form is bilirubin


urobilirubin
secreted by the kidney?

In what form is bilirubin


stercobilin
secreted in the feces?

-alpha-amylase -lipase -
phospholipase A -colipase -
Name as many Pancreatic proteases (trypsin,
enzymes as you can: chymotrypsin, elastase,
carboxypeptidases) -
trypsinogen (trypsin)
Name the major product of
heme metabolism that is
Bilirubin
actively taken up ty
hepatocytes:

Name the organ and enzyme


Nonerythroid enzymes in the
family involved in the
liver
production of bilirubin?

Name the three salivary -Parotic -Submandibular -


secretory glands: Sublingual

Name two potent stimulators 1. Phenylalanine 2.


of Gastrin: Tryptophan
Secretin's nickname? Nature's antacid

SEE PICTURE ON LAST PAGE SEE PICTURE ON LAST PAGE


OF GI PHYSIOLOGY!!! OF GI PHYSIOLOGY!!!

1. Stimulates gallbladder
contraction 2. Stimulates
Three main functions of CCK?
pancreatic enzyme secretion
3. Inhibits gastric emptying

1. Stimulates pancreatic
Two functions of Secretin? HCO3 secretion 2. Inhibits
gastric acid secretion
Two functions of the mucus -Lubricant -protects surface
secreted in the stomach? from H+

What activates all the


trypsin
proteases?

What are the products of


Monosaccharides (glucose,
oligosaccharide hydrolase
galactose, fructose)
action?

What are the products of


Oligosaccharides, maltose
starch hydrolysis by
and maltotriose
pancreatic amylase?
What are the products of the
hydrolysis of carbohydrate maltose, maltotriose and
alpha-1,4 linkages by salivary alpha-limit dextrans
amylase?

1. Begin starch digestion 2.


What are the three main Neutralize oral bacterial acids
functions of saliva? which maintains dental health
3. Lubricate food

-Hepatic production by
nonerythroid enzymes -
What are the two main
Metabolism of heme from red
sources of bilirubin in the
blood cells (120 day life span)
body?
and incomplete or immature
erythroid cells

What causes pain to worsen Eating fatty foods which


in Cholelithiasis? cause CCK release
What component of GI
secretion is 'not essetial for Gastric acid
digestion?'

What condition results from


Jaundice
elevated bilirubin levels?

What disease is commonly


associated with pancreatic Cystic Fibrosis
insufficiency?

What do pancreatic ducts


secrete when stimulated by -mucus -alkaline fluid
secretin?
What does inadequate gastric Increased risk of Salmonella
acid cause? infections

What enzyme converts enterokinase (a duodenal


trypsinogen to trypsin? brushborder enzyme)

What enzyme hydrolyzes


Pancreatic amylase
starch?

What enzyme is involved in


the rate-limiting step in Oligosaccaride hydrolases
carbohydrate digestion?
What enzyme starts digestion
and hydrolyzes alpha-1,4 Salivary Amylase
linkages?

What form are the proteases


proenzyme form
secreted in?

What form is Alpha-amylase


active form
secreted in?

What hormone decreases


absorption of substances Somatostatin
needed for growth)
What inhibits the release of
Somatostatin
gastrin and secretin?

What is pancreatic amylase in


In the duodenal lumen
highest concentration?

What is the composition of -bile salts -phospholipids -


bile? (5) cholesterol -bilirubin -water

What is the fate of Broken down to pepsin (a


pepsinogen? protease) by H+
What is the function (fxn) of Begins protein digestion
Pepsin? (optimal pH = 1.0 - 3.0

What is the function of


starch digestion
Alpha-amylase?

What is the function of


protein digestion
proteases?

-pancreatic HCO3- secretion


What is the function of VIP? - intibition of gastric H+
secretion
What is the function on Causes smooth muscle
Nitrous Oxide? relaxation

What is the major stimulus


for secretion of enzyme-rich
Cholecystokinin
fluid by pancreatic acinar
cells?

What is the major stimulus


for zymogen release, but a
Acetylcholine
poor stimulus for bicarbonate
secretion?

What is the only types of


carbohydrate that is Monosacharides
absorbed?
What is the primary location
over bacterial conversion or
Colon
conjugated bilirubin to
urobilinogen?

What is Zollinger-Ellison
1. Hypersecretion of Gastrin
syndrome? What is the main
2. Peptic ulcers
manifestation?

What manifestations are seen -malabsorption -stratorrhea


in pancreatic insufficiency? (greasy, malodorous stool)

Stimulated by secretin,
What regulates bicarbonate
potentiated by vagal input
secretion?
and CCK
What regulates CCK Stimulated by fatty acids and
secretion? amino acids

-Stimulated by stomach
distension, amino acids,
What regulates Gastrin
peptides, and vagus -
secretion?
Inhibited by secretin and
stomach acid pH less than 1.5

What regulates secretion of Stimulated by acid and fatty


secretin? acids in lumen of duodenum

What regulates secretion of -Stimulated by acid -


Somatostatin? Inhibited by vagus
They are amphipathic
What special characteristic do
(contain both hydrophilic and
bile salts possess?
hydrophobic domains)

What special characteristic


does the conjugated form of It is water soluble.
bilirubin possess?

What substance stimulates


ductal cells to secrete Secretin
bicarbonate-rich fluid?

What three enzymes aid in fat 1. Lipase 2. Phospholipase A


digestion? 3. Colipase
What trasport is utilized in
Sodium-glucose-coupled
glucose absorption across
transporter
cell membrane?

What two conditions are


-Chronic Gastritis -Pernicious
caused be autoimmune
Anemia
destruction of parietal cells?

What type(s) of innervation BOTH Sympathetic and


stimulate salivary secretion? Parasympathetic

Where are the oligosaccharide At the brush border of the


hydrolase enzymes located? intestine
Where does bilirubin
Liver
conjugation take place?

Where does glucose Duodenum and proximal


absorption occur? Jejunum

Where does heme catabolism In the Reticuloendothelial


take place? System

-Surface mucosal cells of


Where is bicarbonate secreted stomach and duodenum -
and what does it do? Neutralizes acid -Present in
the unstirred layer preventing
autodigestion
Where is Cholecystokinin I cells of duodenum and
(CCK) secreted? jejunum

Where is Secretin secreted? S cells of duodenum

Where is Somatostatin D cells in pancreatic islets


secreted? and GI mucosa

Where is Vasoactive Intestinal Smooth muscle nerves of the


Peptide (VIP) secreted intestines
Which component of bile
makes up the greatest Water (97%)
percentage?

Which component of bile


solubilizes lipids in micelles Bile salts
for absorption?

Which component of saliva


Alpha-amylase (ptyalin)
begins starch digestion?

Which component of saliva


Mucins (glycoproteins)
lubricates food?
Why do we need alkaline To neutralize gastric acid,
pancreatic juice in the allowing pancreatic enzymes
duodenum? to function
Microbiology Vignettes p174

An alcoholic vomits gastric


contents and has foul
Anaerobes
smelling sputum. What
organisms are most likely?

Middle-age man has acute


monoarticular joint pain and
Lyme, Ixodes tick
bilateral Bell's palsy. What
disease and how?

Pt with Mycoplasma
pneumoniae, cryoagglutinins. IgM
What Ig types?
U/A with WBC casts. Dx? Pyelonephritis

Young child with tetany,


candidiasis, hypocalcemia,
Tcell (DiGeorge)
and immunosuppression.
What cell is deficient?

Pt with "rose
gardener's" (thorn prick,
Sporothrix schenckii
ulcers along lymph drainage).
What bug?

25yr old med student has


"burning" gut after meals.
H. pylori
Gram negative rods in Bx.
What organism?
32yr old man w/ "cauliflower"
skin lesions. Bx shows broad-
Blastomyces
based budding yeasts.
Organism?

Breast feeding woman w/


redness & swelling of right Mastitis by S. aureus
breast. Fluctuant mass. Dx?

Young child w/ recurrent lung


infxns & granulomatous NADPH oxidase (chronic
lesions. What defect in granulomatous disease)
neutrophils?

20yr old college student w/


lymphadenopathy, fever, B-cell (EBV; infectious
hepatosplenomegaly. Serum mononucleosis)
agglutinates sheep RBCs.
What cell infected?
Microbiology Vignettes p175

1hr after eating custard at a


picnic, whole family vomits. S. aureus(produces
After 10hrs all better. preformed enterotoxin)
Organism?

Infant eats honey -> flaccid. Clostridium botulinum;


Organism? Mechanism? inhibited release of Ach

Man w/ squamous cell


HPV
carcinoma, penis. what virus?
Pt w/ endocarditis 3wks after
prosthetic heart valve. S. aureus or S. epidermis
Organism?

Pathology Vignettes p220

Woman w/ previous C-
section has scar in lower
Placenta previa
uterus close to opening of the
os. Risk for what?

35yr old man w/ hi BP in


Coarctation of Aorta
arms. Low in legs. Dx?
Woman w/ diffuse goiter and
Low TSH, and high thyroid
hyperthyroidism. Values of
hormones
TSH and thyroid hormones?

Pt w/ extended expiratory
Obstructive Lung Disease
phase. Disease process?

Woman w/ headache, visual


disturbance, galactorrhea, Prolactinoma
amenorrhea. Dx?

Baby has foul smelling stool


Cystic fibrosis. Chloride sweat
and recurrent pulmonary
test.
infxns. Dx? Test to confirm?
Obese woman w/ hirsutism
and hi serum testosterone. Polycyctic ovarian syndrome
Dx?

Man w/ pain and swelling in


knees, subcutaneous nodules
around the joints and
Achilles, exquisite pain in the Gouty arthritis
metatarsophalangeal join of
right big toe. Bx shows
needle-like crystals. Dx?

48yr old female w/


progressive lethargy and
Hypothyroidism
extreme sensitivity to cold
temp. Dx?

Pt w/ elevated serum cortisol


levels undergoes
dexamethasone supp. test. Pituitary tumor.
1mg doesn't lower cortisol.
8mgs does. Dx?
During a football game,
player collapses/dies
Hypertrophic cardiomyopathy
immediately. Likely Cardiac
disease?

Pathology Vignettes p221

Child anemic since birth.


Splenectomy -> increased Spherocytosis
Hct in what disease.

43yr old man experiences


dizziness/tinnitus. CT shows
Schwannoma
enlarged internal acoustic
meatus. Dx?
Child exhibits weakness and Duchenne's muscular
enlarges calves. Disease? How dystrophy, X-linked
inherited? recessive.

25yr old female w/ sudden


uniocular vision loss and
slightly slurred speech. Hx of Multiple Sclerosis
weakness and pareshesias
that have resolved. Dx?

Teenager w/ nephritic
Alport's syndrome
syndrome & hearing loss. Dx?

Tall thin male teenager w/


abrupt-onset dyspnea & left-
sided chest pain.
Hyperresonant percussion on Pneumothorax
the affected side &
diminished breath sounds.
Dx?
Young man concerned about
wife's URIs and inability to
Dynein (Kartagener's)
conceive. Also dextrocardia.
What protein?

55yr old man who smokes &


drinks has cough & flu-like
Sx. Gram stain shows Legionellapneumonia
nothing. Silver shows gram-
neg rods. Dx?

Pt has stoke after incurring


multiple long bone fractures
Fat emboli.
in trauma stemming from
MVA. Cause?

25yr old woman w/ a low


grade fever, rash across nose
from sun, widespread edema. SLE
Should be concerned about
what disease?
Pathology Vignettes p222

50yr old man w/ diarrhea, PE


reveals plethoric face and Carcinoid syndrome
heart murmur. Dx?

Elderly woman w/ a headache


and jaw pain. Elevated ESR. Temporal arteritis.
Dx?

Pregnant woman @16wks w/


an atypically large abdomen.
High hCG, hydatidiform mole
Blood test abnormality?
Disorder?
80yr old man w/ a systolic
crescendo-decrescendo Aortic stenosis
murmur. Most likely cause?

Woman of short stature w/


Albright's hereditary
short 4th and 5th
osteodystrophy or
metacarpals. Endocrine
pseudohypoparathyroidism
disorder?

After stressful event, 30yr old


man w/ diarrhea and blood
per rectum. Intestinal bx Crohn's
shows transmural inflamm.
Dx?

Young man w' mental


deterioration and tremors. Penicillamine for Wilson's
Brown pigmentation in a ring disease.
around periphery of cornea
and altered LFTs. Tx?
Pt w/ fatigue, blood tests
Masks signs of neural
show macrocytic,
damage w/ vitamin B12
megaloblastic anemia. Why
deficiency
danger to give folate alone?

10yr old "spaces out" in class


(stops talking midsentence
and then continues as if Absence seizure.
nothing had happened).
During spells, quivering of
lips. Dx?

85yr old man w/ knee pain


and swelling. X-ray shows Pseudogout. Rhomboid
increased joint space w/o calcium pyrophosphate
erosion. Dx? What does crystals.
aspirate show?

Pathology Vignettes p223


X-ray shows bilateral hilar
Sarcoidosis.
lymphadenopathy. Dx?

5yr old boy w/ systolic


murmur and a wide, fixed ASD
split S2. Dx?

Pharmacology Vignettes p286

28yr old chemist w/ MPTP


exposure. Neurotransmitter Dopamine
depleted?
Woman taking tetracycline is
Rash on sun-exposed regions
photosensitive. Clinical
of body
manifestations?

Non-diabetic pt has
hypoglycemia but low C- Surreptitious insulin injection
peptide. Dx?

African-american man who


goes to Africa develops a Glucose-6-Phosphate
hemolytic anemia after taking Dehydrogenase
malarial prophylaxis. Enzyme
deficiency?

27yr old female w/ hx of


psych illness now has urinary
Bethanechol
retention due to a
neuroleptic. Tx?
Farmer presents w/ dyspnea,
salivation, miosis, diarrhea, Insecticide poisioning;
cramping, blurry vision. inhibition of AChE
Cause & mechanism?

Pt w/ recent kidney transplant


is on cyclosporine for
immunosuppression. Ketoconazole
Requires antifungal agent for
candidiasis. What antifungal -
> cyclosporine toxicity?

Man on several meds


including antidepressants and
antihypertensives, has TCA
mydriasis and becomes
constipated. Cause?

55yr old postmenopausal


woman is on tamoxifen Endometrial carcinoma
therapy. Risk?
Woman on MAO inh has
hypertensive crisis after a Tyramine (wine/cheese)
meal. What did she eat?

After taking clindamycin, pt


Clostridium
dev toxic megacolon and
difficileovergrowth
diarrhea. Mechanism?

Pharmacology Vignettes p287

Man starts meds for


hyperlipidemia. Then rash, Niacin
pruritus, and upset GI. Drug?
Pt on carbamazepine. What
routine workup should always LFTs
be done?

23yr old female who is on


rifampin for TB prophylaxis Rifampin augments estrogen
and on birth control metabolism in the liver,
(estrogen) gets pregnant. rendering it less effective.
Why?

Pt develops cough and must Losartan, an angiotensin II


discontinue captopril. What is receptor antagonist, does not
a good replacement? Why not increase bradykinin as
same side effects? captopril does.
Anatomy Clinical Vignettes p. 68, 69

Baby vomits milk when fed


blind esophagus with lower
and has gastric air bubble.
segment of esophagus
What kind of fistula is
attached to trachea
present?

20-year old dancer with


decreased plantar flexion and
decreased sensation over the Tibial (L4 to S3)
back of her thigh, calf, and
lateral half of foot. What
spinal nerve?

Patient with decreased pain


and temp sensation over
syringomyelia
lateral aspects of both arms.
Lesion?
Penlight in patient's right eye
produces bilateral pupillary
constriction. In left eye, there atrophy of left optic nerve
is paradoxical bilateral
pupillary dilation. Defect?

Patient with decreased prick


sensation on lateral aspect of dorsiflexion and eversion of
leg and foot. A deficit in what foot (common peroneal
muscular action can also be nerve)
expected?

Elderly woman with arthritis


Carpal tunnel syndrome,
and tingling over lateral digits
median nerve compression.
of right hand. Diagnosis?

Right CN XI (runs through


Woman in car accident cannot
jugular foramen with CN IX
turn her head to the left and
and X), innervating
has right shoulder droop.
sternocleidomastoid and
What structure is damaged?
trapezius muscles)
Man with one wild, flailing contralateral subthalamic
arm. Where is lesion? nucleus (hemiballismus)

Pregnant woman in 3rd


trimester has normal blood
pressure when standing and compression of IVC
sitting. When supine, BP
drops to 90/50. Diagnosis?

Soccer player was kicked in


anterior cruciate ligament
leg and suffered a damaged
(remember the "unhappy
medial meniscus. What else is
triad")
likely to be damaged?

Gymnast dislocates her


shoulder anteriorly. What
Axillary nerve (C5, C6)
nerve is most likely to have
been damaged?
Patient with cortical lesion
does not know that he has a Right parietal lobe.
disease. Where is lesion?

Child presents with cleft lip.


Fusion of maxillary and
Which embryologic process
medial nasal processes.
failed?

Patient cannot protrude


tongue toward left side and
left medulla, CN XII
has right sided spastic
paralysis. Where is lesion?

Teen falls while rollerblading


and hurts his elbow. He can't Ulnar nerve due to broken
feel the medial part of his medial condyle
palm. Which nerve and what
injury?
24-year-old male develops para-aortic lymph nodes
testicular cancer. Metastatic (recal descent of testes
spread occurs by what route? during development)

Field hockey player presents


to ER after falling on her arm
Radial nerve and deep
during practice. X-ray shows
brachial artery, which run
mid-shaft break of humerus.
together.
Which nerve and what artery
are most likely damaged?

Patient cannot blink his right


eye or seal his lips and has
mild ptosis on the right side. Bell's palsy; CNVII
What is diagnosis, and what
nerve is often affected?

Patient complains of pain,


numbness, and tingling
sensation. On exam, she has Carpal tunnel syndrome,
wasting of thenar eminence. median nerve
What is diagnosis, and what
nerve is often affected?
Behavioral Science Vignettes p. 114,115

Woman with anxiety about


gyn exam is told to relax and
imagine going through steps systematic desensitization
of exam. What process does
this exemplify?

65-year-old man is
diagnosed with incurable
Assess whether telling patient
metastatic pancreatic
will negatively affect his
adenocarcinoma. His family
health. If not, tell him.
asks you not to tell the
patient. What do you do?

Man admitted for chest pain


is medicated for ventricular
tachycardia. The next day he
jumps out of bed and does denial
50 pushups to show the
nurses he has not had a heart
attack. What defense
mechanism?
Nothing! Tone must be
professional. It is not
You are attracted to your 26- acceptable to have romantic
year-old patient. What do you relationship with patients.
say? Invite a chaperone into room
if you think your actions may
be misinterpreted.

Large group of people


followed over 10 years. Every
2 years, it is determined who cohort study
develops heart disease and
who does not. What type of
study?

Girl can groom herself, can


hop on one foot, and has an
Four years old
imaginary friend. How old is
she?

Man has flashbacks about his


girlfriend's death 2 months
ago following a hit-and-run normal bereavement
accident. He often cries and
wishes for the death of the
culprit. Diagnosis?
36-year-old woman with
Discuss risks and benefits of
strong family history of
not having mammogram.
breast cancer refuses a
Patients must give informed
mammogram because she
consent. If she refuses, you
heard it hurts. What do you
must abide by her wishes.
do?

During a particular stage of


sleep, man has variable blood
pressure, penile tumescence, REM sleep
and variable EEG. What stage
of sleep is he in?

15-year-old girl of normal


height and weight for her age
has enlarged parotid glands
but no other complaints. The bulimia
mother confides that she
found laxatives in the
daughter's closet. Diagnosis?

11-year-old girl exhibits


Tanner stage 4 development advanced stage, early
(almost full breasts and pubic development.
hair). Diagnosis?
4-year-old girl complains of
burning feeling in her
genitalia; otherwise she
behaves and sleeps normally. sexual abuse
Smear of discharge shows N.
gonorrhoeae. How was she
infected?

Although you want to


encourage him to take his
72-year-old man insists on
medication, the patient has
stopping treatment for his
the final say. You should
heart condition because it
makes him feel "funny." What investigate the "funny" feeling
and determine if there are
do you do?
drugs available that don't
elicit this side effect.

Person demands only the best


and most famous doctor in Narcissism
town. Personality disorder?

Nurse has episodes of


hypoglycemia; blood analysis Factitious disorder; self-
reveals no elevation in C scripted insulin.
protein. Diagnosis?
55-year-old businessman
complains of lack of
successful sexual contacts
with women and lack of Fear of sudden death during
ability to reach full erection. intercourse.
Two years ago he had a heart
attack. What might be the
cause of his problem?

Biochemistry Clinical
p. 138
Vignettes

Full-term neonate of
uneventful delivery becomes
mentally retarded and PKU
hyperactive and has a musty
odor. Diagnosis?

Stressed executive comes


home from work, consumes 7 NADH increase prevents
or 8 martinis in rapid gluconeogenesis by shunting
succession before dinner, and pyruvate and oxaloacetate to
becomes hypoglycemic. lactate and malate.
Mechanism?
2-year-old has increased
abdominal girth, failure to
Kwashiorkor
thrive, and skin and hair
depigmentation. Diagnosis?

Alcoholic develops rash,


diarrhea, and altered mental Vitamin B3 (pellagra)
status. Vitamin deficiency?

20-year-old man presents


with idiopathic
Gilbert's disease
hyperbilirubinemia. Most
common cause?

51-year-old man has black


spots in his sclera and has
noted that his urine turns Alkaptonuria
black when he is standing.
Diagnosis?
25-year-old male complains
of severe chest pain and has Familial
xanthomas of his Achilles hypercholesterolemia; LDL
tendons. What is the disease, receptor
and where is the defect?

You might also like